Está en la página 1de 664

ANATOPATO 20.

De las adaptaciones celulares cual es terreno fértil


sobre la cual puede surgir un cáncer: Hiperplasia
1. Las células atróficas mueren cuando el aporte
patológica
sanguíneo no es suficiente para mantener la vida,
21. Lesión por isquemia-reperfusión: Tiene efecto
estas son sustituidas por: tejido graso
paradójico ya que puede ser benéfica o
2. Los leucotrienos son generados por la activación
perjudicial
de acido araquidónico por la vía: lipooxigenasa
22. La respuesta inmune por tuberculosis está
3. Patrón morfológico característico de la
mediada por: Linfocitos TH1
tuberculosis: granuloma
23. En las lesiones precursoras de cáncer del epitelio
4. El útero atrófico en una mujer postmenopáusica
escamoso del cuello uterino, el ADN del virus del
es un ejemplo de: atrofia por pérdida de
Papiloma Humano, se encuentra: Dentro
estimulación endógena
24. El fosfato de Calcio es conocido también como:
5. El fragmento de FC de IgG, C3B y las colectinas
Hidroxiapatita
funcionan como: opsoninas
25. Se puede revertir la lesión celular secundaria a
6. La respuesta inmune contra la tuberculosis esta
hipoxia, transitoria, aportando a las células una
mediada por: Linfocitos TH1
cantidad de: Oxígeno
7. Son ejemplos de sustancias químicas de acción
26. Características morfológicas principales de la
directa: cloruro de mercurio, cianuro
lesión irreversible: Aumento de la eosinofilia y
8. Sistema bactericida más eficaz de los neutrófilos:
pérdida de la continuidad de la membrana
H2O2 + MPO + Cl
27. Según la clasificación del sistema de Bethesda, los
9. Al glucógeno lo podemos visualizar con: tinción
cambios reparativos se refieren: Inflamación
PAS
28. Enfermedad que cursa con reacción inflamatoria
10. Son causas de inflamación crónica por
crónica granulomatosa: Sífilis
microorganismos difíciles de erradicar,
29. Las células atróficas mueren cuando el aporte
hipersensibilidad retardada, reacción
sanguíneo no es suficiente para mantener la vida,
granulomatosa o abcesos pulmonares: Infecciones
también se llama: Atrofia parda
persistentes.
30. Patología. Ejemplo donde los leucocitos liberan
11. Se refiere al reemplazo y recuperación del estado
sustancias en el fagolisosoma, pero también en el
normal de algunos tejidos: Regeneración
espacio extracelular con el propósito de eliminar
12. Característica principal de las células madre
el antígeno, sin embargo, a veces el daño
adultas: Son pluripotenciales.
provocado puede ser más lesivo que la infección
13. ¿Cuál de los siguientes elementos no es correcto
original: Ateroesclerosis
respecto a la hiperplasia? Disminución de la
31. La realización de una autopsia es un
capacidad funcional de un tejido que lo necesita
procedimiento elaborado, complejo y cuidadoso,
14. Están relacionadas con una activación excesiva del
que propósitos cumple: La integración de los
complemento. Degeneración macular y síndrome
hallazgos macro y microscópicos para constituir
hemolítico urémico
los diagnósticos
15. Biopsia de elección para estómago: Sacabocado //
32. Se considera uno de los signos clínicos de la lesión
Endoscópica
celular irreversible: Restos de organelos dañados
16. Se ha demostrado que se presenta en el cáncer,
33. Cuáles son las formas de evolución de la
trastornos neurodegenerativos, enfermedades
inflamación aguda: Resolución completa,
infecciosas y enfermedades inflamatorias
curación mediante sustitución por tejido
intestinales: Autofagia
conjuntivo (fibrosis), formación de abscesos,
17. Es una antiproteasa que se encuentra en sueros y
progresión de la respuesta tisular
fluidos:
hacia inflamación crónica.
18. La inflamación fibrinosa se caracteriza por: Se
34. Permite la obtención de un cilindro de tejido
presenta en inflamación de cavidades corporales
mediante un trocar: Biopsia por sacabocado.
como pericardio meninges y pleura
35. Es la toma de solo una porción de la lesión, solo
19. La cicatriz se define como: Reparación por
con fines diagnósticos. Biopsia incisional.
depósito de tejido fibroso
36. Sustancias que actúan como antioxidantes 59. Para la formación se requiere: Reacción
naturales: Superóxido dismutasa, glutatión inmunitaria mediada por células T.
peroxidasa. 60. Tipo de metaplasia más frecuente: De epitelio
37. Los cuerpos de Russell son acúmulos de: ciliíndrico a plano estratificado.
Inmunoglobulinas en células plasmáticas. 61. Principal quimiotáctica endógeno para neutrofilos:
38. Calcificación patológica que se acompaña de CXC.
niveles elevados de calcio sérico: Metastásica. 62. Tipo de calcificación que se acompaña de
39. Principales citocinas de respuesta inflamatoria: IL1 hipercalcemia: Metastásica.
y TNF. 63. Metabolismo del AA, vía COX. Leucotrienos
40. Fijador de tejidos utilizado de forma rutinaria: 64. Proliferación exuberante de fibroblastos y otros
Formol 10% elementos de tejido conjuntivo: Lesiones
41. Reacciones de la fase aguda provocador por IL1: desmoides.
Fiebre y pérdida del apetito. 65. Induce inflamación con independencia de la causa
42. Sustancias que actúan como opsoninas: C3B, IGG de muerte celular, que puede ser isquemia,
y Colectinas. traumatismo, agresión física y química. Necrosis
43. Característica de la inflamación crónica: tisular
Infiltración de macrófagos. 66. Todas las células tienen receptores citosólicos que
44. Característica de calcificación metastásica. reconocen una amplia diversidad de moléculas
Presente en tejidos normales liberadas o alteradas como consecuencia del daño
45. Los cuerpos de Weibel palada de las células celular, estos receptores activan un complejo
endoteliales contienen: Selectina citosólico multiproteico llamado: Inflamasoma
46. Sustancias químicas que causan lesión: 67. Fenómeno de adaptación por irritación crónica.
Paracetamol y tetracloruro de carbono. Metaplasia.
47. Factores de adhesión que pertenecen a las 68. Inhibe la apertura del poro de transición de
inmunoglobulinas: ICAM, VECAM, PCAM. permeabilidad mitocondrial de esta forma, inhibe
48. La inflamación fibrinosa se caracteriza por: Se la liberación del citocromo C: Ciclosporina
presenta en inflamación de cavidades corporales 69. Acumulación intracelular de colesterol en
como pericardio y pleura. macrófagos de tejido conjuntivo: Xantoma
49. Característica histológica de órganos que sufren 70. Principales quimiotácticas endógenos: C5a y LTB4
atrofia: Formación de vacuolas autofágicas. 71. ¿Qué efecto tiene la disminución de ATP en el PH
50. Fenómenos leucocitarios en la luz vascular: citosólico? Disminuye por aumento de ácido
Marginación, rodamiento y adhesión. láctico
51. El cáncer in situ se localiza: Limitado al grosor del 72. Se producen por el ensamblaje y la activación
epitelio. rápidos de una oxidasa de múltiples
52. Vitamina necesaria para la hidroxilación del componentes: ERO
procolágeno: C 73. Proteína de la fase aguda puede causar
53. En qué día se forman los puentes de colágeno en amiloidosis secundaria: Proteína amiloide sérica.
curación de primera intención: Día 5 74. Es una respuesta inflamatoria de largo tiempo de
54. Característica de la calcificación distrófica: Se evolución: Inflamación crónica
presenta en áreas de necrosis. 75. Es la magnitud en que las células parenquimatosas
55. Factores de riesgo para colelitiasis, excepto: Dieta neoplásicas se asemejan en su función y forma a
rica en fibra. las células correspondientes del parénquima sano:
56. La lesión celular por tetracloruro de carbono se Diferenciación
caracteriza por: Su conversión en metabolitos 76. Componente proteico de la MEC Encargado de dar
activos muy tóxicos. resistencia a la tensión. Colágeno
57. El primer punto de ataque de la hipoxia en la 77. Se refiere al reemplazo y recuperación del estado
célula es: Disminución de Fosforilación oxidativa. normal de algunos tejidos: Regeneración
58. ¿Cuál de los siguientes es antioxidante no 78. Desde el punto de vista clínico la lesión celular
enzimático? Transferrina. más común es: Lesión isquémica
79. Son causas de inflamación crónica por 96. Vía de la apoptosis que más se desregula en
microorganismos difíciles de erradicar, situación de cáncer: Vía extrínseca
hipersensibilidad retardada, reacción 97. A qué gen induce tp53 para que dirija a la célula
granulomatosa o abscesos pulmonares: hacia la apoptosis: BAX o BAC.
Infecciones persistentes. 98. Es considerado como guardián del genoma. TP53.
80. Las acumulaciones intracelulares de proteínas 99. ¿Cuál no es una característica de la anaplasia?
aparecen como gotículas redondeadas eosinófilas, Falta de mitosis.
vacuolas o agregados en el citoplasma: 100. La insensibilidad a las señales inhibitorias del
Amiloidosis crecimiento está dado por la lesión a: RB y p53.
81. Característica principal de las células madre 101. Molécula de adhesión que contienen los gránulos
adultas: Son pluripotenciales. alfa de las plaquetas: Selectina P.
82. Es un conjunto de proteínas solubles y receptores 102. Son células que se caracterizan por
de membrana que funcionan fundamentalmente autorrenovación y proliferación asimétrica:
en defensa del anfitrión contra microbios y en Células madre
reacciones inflamatorias patológicas: Sistema de 103. Mediadores que son potentes vasodilatadores e
complemento. inhibidores de la agregación plaquetaria:
83. Factor que ejerce efectos mitógenos en la mayoría Prostaciclina y el óxido nítrico
de las células epiteliales incluyendo epitelio biliar, 104. Son características de la apoptosis: Tamaño
pulmonar, piel y otros tejidos: Factor de celular reducido, membrana plasmática intacta,
crecimiento epidérmico. sin inflamación adyacente y fragmentación
84. Efectos de las citocinas en células cerebrales celular
durante la inflamación. Aumento de temperatura 105. Tanto células como núcleos desarrollan
corporal variaciones en tamaño y forma: Pleomorfismo
85. ¿Cómo afecta la disminución de ATP en la 106. Neutrófilos, con actividad limitada sobre
osmolaridad de la célula? El citoplasma se vuelve monocitos y eosinófilos: C-X-C O alfa
hipertónico por ganancia de sodio. 107. Uno de los genes mutado en la mayor parte de los
86. La realización de una autopsia médico legal es un cánceres humanos ES: P53
procedimiento elaborado, complejo y cuidadoso 108. Las células madre progenitoras del hígado se
que incluye las siguientes etapas: Evisceración, albergan en: Conductos de Hering
disección y análisis macroscópico de los órganos. 109. Adaptación celular, el crecimiento puede ser
87. Sistema bactericida más eficaz de los neutrófilos: terreno fértil, donde se inicia una proliferación
H202 + MPO + CL cancerosa: Displasia
88. Al glucógeno lo podemos visualizar con: PAS 110. Adaptación celular de la diferenciación que
89. El Fragmento de FC de IgG, C3B y las colectinas consiste en un cambio reversible de una célula
funcionan como: Opsoninas adulta por una célula adulta: Metaplasia
90. La respuesta inmune contra la TB es mediada por: 111. Cuerpos de Russell son acúmulos de:
Linfocitos TH1. Inmunoglobulinas en células plasmáticas
91. Ejemplos de sustancia química de acción directa:
Cloruro de mercurio, cianuro.
92. Gen 9,12 (Está mal la pregunta de la plataforma). 112. Causa predisponente de la calcificación
Gen 9,22: Cromosoma Filadelphia y Leucemia metastásica: Medio alcalino
mieloide crónica 113. La biopsia de elección para una lesión
93. Tp53 activa a que gen para mandar a apoptosis: hiperpigmentada localizada en la piel y que mide
BAX un centímetro de diámetro es: Excisional
94. Las células que contienen lípidos en los ateromas 114. Es causa de la calcificación distrófica. Focos de
son: Macrófagos necrosis
95. ¿Cuál es el punto de restricción que deben de 115. Características de necrosis fibrinoide: Depósitos
pasar las células para garantizar la replicación de inmunocomplejos en pared de vasos
genética: G2/M sanguíneos
116. 2:55 histamina, prostaglandina 138. La fractalquina es una molécula de adhesión para:
117. Son sensores de lesión o estrés celular en la monocitos y Linfocitos T
apoptosis: Puma y Bid 139. Los cuerpos de Russell son acúmulos de:
118. La vía extrínseca de la apoptosis se activa por: Inmunoglobulinas en las células plasmáticas.
caspasa 8 140. Productos del metabolismo del ácido
119. De las causas de lesión celular cuál es la que araquidónico por la vía de la ciclooxigenasa:
afecta de manera directa a la fosforilación Prostaglandinas y protaciclina, Txa2
oxidativa aeróbica. Isquemia o hipoxia. 141. Grupo de mediadores químicos que producen
120. La telomerasa es un complejo: RNA proteína inhibición de la quimiotaxis y de la adherencia de
121. En la lesión celular reversible debida a neutrófilos: Lipoxinas
isquemia/hipoxia la primera activación bioquímica 142. Prostaglandina que produce hiperalgesia, que
se presenta en: disminución de la fosforilación hace que la piel presente hipersensibilidad, dolor
122. Proteínas de transporte y almacenamiento con y fiebre: PGE.
capacidad antioxidante: Lactoferrina, 143. Proteína que puede unirse a la proteína BCL2, BCL-
transferrina, ferritina. XL y realiza funciones antiapoptóticas: BH3
123. La calcificación distrófica se presenta en: Dependiente de calcio
ateroesclerosis: Metaplasia 144. En inflamación son pirógenos endógenos: IL-1 y
124. Efectos locales provocados por IL-1, FNT e IL-6 en FNT
la reacción inflamatoria: aumento de la adhesión 145. Causas más frecuentes de esteatosis hepática:
y activación leucocitaria. alcoholismo y desnutrición.
125. Moléculas antiapoptóticas: BCL2, BCL-XL 146. Ejemplos de inflamación serosa: infecciones
126. Son características morfológicas de la apoptosis virales y quemaduras de segundo grado.
excepto: Fallo de funciones dependiente de 147. 10 veces más potente que la histamina: FAP
energía 148. En la apoptosis ambas moléculas se unen a la
127. Tipo de metaplasia en el esófago de Barret: de membrana de la mitocondria, hacen poros y
epitelio plano a epitelio cilíndrico. liberan citocromo c: BAX y BAK
128. La acumulación intracelular de polvo de carbón 149. Factor de crecimiento relacionado con la
produce reacción fibroblástica recibe el nombre: hipertrófia relacionado con el aumento de la carga
neumoconiosis de trabajo: TGF-B
129. Pigmento del envejecimiento: Lipocromo o 150. El receptor de muerte más conocido en la
Lipofuscina apoptosis se llama: FAS
130. Componentes de la inflamación aguda: 151. Proteínas plasmáticas de fase aguda de la
Modificación en el calibre de los vasos y en la inflamación: Proteína C reactiva, fibrinógeno,
estructura de la microvasculatura. 152. Inhibe el apetito: FNT
131. El aumento de la permeabilidad vascular da como 153. Son factores que intervienen en la patogenia
resultado: salida de un líquido rico en proteínas molecular de la patogenia cardiaca: TGF BETA,
hacia el intersticio IGF-1
132. Selectinas, integrinas y glicoproteínas actúan 154. La presencia de estrías rojas en la piel cerca de
como: moléculas de adhesión. una herida indica que: tejido de granulación/la
133. La aspirina inhibe la PGE2 para disminuir la fiebre. herida se ha infectado
134. Peroxidación de los lípidos de la membrana, 155. Son características de lesión celular
lesiones del DNA son causados por: Radicales irreversibles: Lesión excesiva de las membranas
libres derivados de oxígeno. plasmáticas e hinchazón de lisosomas
135. IgG e C3b funcionan como: opsoninas 156. Pertenecen a los mediadores locales preformados:
136. Los principales quimiotácticos endógenos: C5a, histamina
C3a 157. Las células de Langhans se caracteriza por: ser
137. Tipo de necrosis en encéfalo: necrosis células multinucleadas provenientes de
licuefactiva. macrófagos
158. Método mas empleado para la detección limite the moviment of her fingers, and she unable to fully
oportuna de cáncer: citología exfoliativa extend them. What are these lesión called? Contractures
159. Características principales de la autofagia: recicla 4.Tambien conocido como factor de dispersión: HGF
nutrientes intracelulares
160. Características de la morfología de la apoptosis, 5.Which the following is the most common site for arteriolar
excepto: constricción celular y picnocitosis thromboembolization? Lower extremities
161. La anemia como causa de lesión celular se 6.Citocinas que participan en la curación de las heridas: INF e
relaciona con: Hipoxia Il-1
162. Biopsia de elección en LEIAG del cervix:
7.En la biología celular de las carcinogenesis química se
sacabocado
observan los siguientes pasos: Iniciacion y promoción
163. Tipo de necrosis en el pie diabético: necrosis
coagulativa 8.En más del 95% de los casos el origen de una
164. Inhiben la COX 1 y COX2 en consecuencia, la tromboembolia pulmonar es: Trombosis venosa profunda
síntesis de prostaglandinas: AINES 9.¿Es un producto de las celulas endoteliales normales y un
165. Biopsia en la que se hace resección de toda la cofactor esencial para la unión de las plaquetas a los
zona de transformación y del canal endocervical elementos de la MEC? Factor de Von Willebrand
para evaluar lesiones premaglinas y malignas del
10.La formación insuficiente del tejido de granulación puede
cuello uterino: biopsia en cono
causar: Deshiscencia de la herida y ulceración.
166. Patrones morfológicos de muerte celular: necrosis
y apoptosis 11.Which of the following subtances is an anticoagulant
167. Necrosis coagulativa con una infección: gangrena secreted by endotelial cells? Thrombomodulin
húmeda 12.Es una de las consecuencias importantes cuando el embolo
168. Cual es la forma de necrosis más frecuente: produce una oclusión vascular parcial o completa: Necrosis
necrosis coagulativa isquémica
169. Infección por vph no asociada a atipias, en el
13.Which of the following tumors occurs most often in the
sistema Bethesda: lesión intraepitelial escamosa
ovary? Cystadenoma
de alto grado
170. aine que inhibe de forma irreversible la 14.Son carcinógenos de acción indirecta: Hidrocarburos
ciclooxigenasa: Aspirina aromáticos
171. presencia de citocromo c en citoplasma: activa via 15.Es una alteración de escasa frecuencia que se debe a una
intrínseca de la apoptosis mediante caspasa 9 proliferación exuberante de fibroblastos y otros elementos de
172. las células atróficas mueren cuando el aporte tejido conjuntivo: Lesiones desmoides
sanguíneo no es suficiente para mantener la vida:
16.Mediadores que son potentes vasodilatadores e
Atrofia parda
inhibidores de la agregación plaquetaria: prostaciclina y el
173. mediador de la inflamación que provoca dolor:
oxido nítrico
PGE2
174. 17.En las endocrinopatías en los síndromes paraneoplasicos se
caracterizan por: Producción hormona ectópica

18.Wich growth factor is the most potent promotor of


angiogénesis in the granulation tissue? Vascular endotelial
growth factor
1.Es un tipo especializado de tejido, característico de la
curación, de aspecto rosado, blanco y de apariencia granular 19. 60 year old man had congestive heart failure and sodium
en la superficie de las heridas: Tejido de degranulacion retention. Pitting edema of the lower extremities in this
patientis most likely associated with n increased blood level
2.Son carcinógenos de acción directa: Agentes alquilantes y of: Aldosterone
acilantes
20. Nombre que recibe la formación de vasos durante el
3.A 20 year old woman suffered extensive burns and
desarrollo embrionario :Vasculogenia
developed large irregular scars over her hands. These scars
21. Son ejemplos de factores sistémicos que afectan la 38.Marcador tumoral relacionado con carcino de colon y
cicatrización: Diabetes o cáncer páncreas: Antigeno carcinoembrionario

22. Se produce cuando un embolo venoso atraviesa una 39.Es la perdida de grasa corporal , masa magra, debilidad
comunicación interauricular o interventricular para llegar a la profunda anorexia y anemia: Caquexia
circulación sistémica: Embolia paradójica
40.¿A qué se asocia el carcinoma hepatocelular? A la
23. Célula con capacidad de autorrenovarse y de generar ingestión del metabolito fúngico aflatoxina B2
estirpes de células diferenciadas: Celulas madre
41. La ciclina E/CDK2 controla la transición entre: G1-S
24. ¿Qué nombre recibe la mutacion especifica del factor V?
Mutacion de Leiden

25Calcificación de válvula mitral área funcional de menos de


1cm3, ocacionan en primera instancia: Congestión Pulmonar EXAMENES VASQUEZ GUERA
26.¿Cuál es la diferencia entre hematoma, petequia, purputa y 42.- La patología o la anytomia patológica es una rama de la
equimosis? El tamaño de las hemorragias medicina que estudia: Los cambios estructurales y funcionales
de la enfermedad.
27.¿El edema de origen renal se caracteriza por?
43.- Causa predisponente de la calcificación metastásica.
Localizaciones en tejido laxo
medio alcalino
28.Son patologías que producen edema por aumento de la 44.- La biopsia de elección para una lesión hiperpigmentada
presión hidrostática del vaso: Pericarditis constictiva y localizada en la piel y que mide 1cm de diámetro, es:
trombosis Excisional
45.- El fijador más utilizado para los tejidos obtenidos por
29.La fase progresiva del shock se caracteriza por: Glucolisis biopsia o necropsia es: Formol al 10%
anaeróbica y acidosis láctica metabólica 46.- Es causa de calcificación distrofiaca: Focos de necrosis
30.The most common cause of mitral stenosis in adults is: 47.- El apoptosoma está formado por las siguientes sustancias:
Rheumatic endocarditis Citocromo C se une APAF-1
48.- What chemical mediators are responsable for increasing
31.Tanto células como núcleos desarrollan variaciones en the blood flow to an are of inflamation? Histamine,
tamaño y forma: Pleomorfismo Prostaglandins and nitric oxide
49.- Forma por la cual la telomerasa inhibe el acortamiento de
32.La función de esta proteasa degrada al colágeno tipo IV y
los telomeros: Añade nucleótidos a los extremos del
…….. a la metástasis de celulas tumorales? Metaloproteinasas
cromosoma
33,Genes incluidos en el genoma humano que regula el 50.- ¿Cuál es la característica del nucleo celular en la
crecimiento y diferenciación celular: Protooncogenes apoptosis? Condensacion de la cromatina
51.- La via extrínseca de la apoptosis activa a: Caspasa 8 y 10
34.Son aquellos que se caracterizan por aplicar frenos a la 52- De las causas de lesión celular, ¿cuál de las siguientes es la
proliferación celular, formando una red de puntos de control que afecta de manera directa a la respiración oxidativa
que impiden el crecimiento incontrolado: Genes inhibidores aeróbica en la célula? La isquemia y la hipoxia
apoptosis 53.- La telomerasa es un complejo: ARN- Proteina
35.Correlacionar las siguientes columnas contestando en su 54.- En lesión celular reversible debida a isquemia / hipoxia la
hoja de respuesta: primera alteración bioquímica que se presenta en:
Disminución de la fosforilacion oxidativa mitocondrial
a) Angiosarcoma / mesenquimatosa maligna
b) Coriocarcinoma o Seminoma /Epitelio maligna (D) 55.- Son fenómenos que caracterizan e indican que la lesión
c) Hemangioma o Linfangioma /Mesenquima benigna celular es irreversible: La disfunción mitocondrial, profundos
d) Cistoadenoma o papiloma escamoso /epitelial trastornos en la función de las membranas celulares
benigna
e) Glaucoma/ No es neoplasia (C) 56.- ¿Cuáles de los siguientes son factores antioxidantes
enzimáticos? Catalasa, Glutation peroxidasa y superoxido
36.La caquexia en el cáncer se da por el efecto de la siguiente
dismutasa.
citosina. TNF
57.- La calcificación distrofica se presenta en: Ateromatosis
37.Celulas linfoides que son cpaces de destruir celulas
tumorales sin una sensibilización previa: Natural killer
58.- Adaptación celular del crecimiento que puede ser terreno 78.- Estructura formada por un acumulo de celulas
fértil en donde se inicie una proliferación maligna: Hiperplasia epiteloides, rodeadas por un collar de linfocitos y celulas
patológica (revisar) plasmáticas se presenta en: Brucelosis
59.- son efectos sistémicos provocadas por la IL-1, el TNF en la 79.- Quimioxina que da lugar a la activación dy quimiotaxis de
reacción inflamatoria: Fiebre, pérdida del apetito, sueño los neutrófilos , con actividad limitada sobre monocitos y
60.- La atrofia puede ser: Fisiológica y patológica eosinofilos: CXC o alfa
61.- De las referidas a continuación, señale en que muestras 80.- Prostaglandina que produce hiperalgesia, que hace que la
puede realizarse citología exfoliativa: orina, expectoración y piel presente hipersensibilidad (dolor y fiebre). PGE2
liquido de ascitis 81.- sistema bactericida más eficaz de los neutrófilos . H2O2 +
62.- Tipo de metaplasia más frecuente: De epitelio cilíndrico a MPO + Cl
epitelio plano estratificado 82.- Nitric oxide sinthesis is augmented in endotelial cells by a
63.- La acumulación intracelular de polvo de carbón recibe el calmodulin medianted influex of which element? Calcium
nombre de: Antracosis 83.- Pus o material purulento se encuentra formado por:
64.- ¿A cuál de los pigmentos endógenos que Deritus celulares, neutrófilos y liquido de edema
se puede acumular intracelularmente se le llama “Pigmento 84.- la causa mas frecuente de la esteatosis hepática son:
del envejecimiento”? Lipofuscina Alcoholismo, desnutrricion
65.- Son componentes de la inflamación aguda: Modificación 85.- Son ejemplos de inflamación serosa: Infecciones virales y
en el calibre de los vasos y en la estructura de la quemaduras de 2 grado
microvasculatura. 86.- Mediador químicos que produce vasodilatación y su
66.- El aumento de la permeabilidad vascular da como efecto es 10 000 veces mas potente que la histamina: Factor
resultado: Salida de un fluido rico en células y proteínas al Activado de las Plaquetas
intersticio. 87.- Factor del complemento C5a se activa por la via ¿? Para
67.- Selectinas, integrinas y glucoproteínas tipo mucina actúan activar netrofilos y monocitos para aumentar la leberacion de
como: Moléculas de adhesión mas mediadores inflamatorios: Via lipooxigenasa
68.- A 60-year-old obese man was admitted to the hospital for 88.- Factor de crecimiento relacionado con la hipertrofia por
treatment of alcoholism. He has diabetes mellitus. A liver aumento de la carga de trabajo:
biopsy ws performed, and the specimen showed that liver TGF beta
cells contain increased amounts of: Triglycerides 89.- la atrofia se produce por: aumento en la degradacion de
69.- Peroxidacion de los lípidos de la membrana, modificación proteinas
oxidativa de las proteínas y lesiones del DNA son efectos 90.- el receptor de muerte mas conocido en la apoptosis se
relevantes para la lesión celular causados por: Radicales libres llama: fas
derivados del oxigeno (ERO) 91.- son proteinas plasmaticas de la fase aguda en la
70.- los anticuerpos de la IgG, C3B y las colectinas funcionan inflamacion: proteina c reactiva, fibrinogeno proteina
como: Opsoninas amiloide
71.- Principales quimiotacticos endógenos: C5A y LTB4
92.- los cuerpos de mallory son acumulos de: proteinas hialino
72.- A 60 – year-old man who had generalized aterosclerosis
alcoholicas en hígado
died 24 hours after having a stroke. Acerebral infarct was
found at autopsy. Necrosis of the brain is classified as: 93.- pigmento endógeno, no derivado de la hemoglobina:
Liquefactive necrosis melanina
73.- Las quimiocinas CX3C son moléculas de adhesión para:
Monocitos y Linfocitos T 94.- calcificación patológica que se acompaña de niveles
74.- Los cuerpos de Russell son acumulos de: elevados de calcio sérico es: metastásica
inmunoglobulinas en células plasmáticas.
95.- la iniciación de la calcificación distrofica intracelular se
75.- Son productos del metabolismo del acido araquidónico
inicia en: MITOCONDRIAS
por la via de las ciclooxigenasas: Tromboxano y
prostaglandinas 96.- el aspecto vítreo homogéneo y rosado a los cortes
76.- Grupo de mediadores químicos que producen inhibición histológicos habituales teñido con hematoxilina y eosina se
de la quimiotaxis de neutrófilos y la adherencia de los llama: cambio hialino
monocitos. Lipoxinas
77.- Bradykinin is produced from a high-moecular weight 97.- el llamado pigmento de desgaste se forma con mucha
kinoinogen circulating in the blood. This reaction is mediated frecuencia en: atrofia
by: Kallikrein 98.- los macrófagos modificados o células epitelioides son
características de: inflamación CRÓNICA
99.- HIPOXIA: apendicitis aguda supurativa 118.- el patrón morfológico de necrosis en el que predomina
la degradación enzimática es la: necrosis licuefactiva
100.- se denomina biopsia al estudio de: un fragmento de
tejido 119.- factores de adhesion que pertenecen a las
inmunoglobulinas: icam, vecam, pcam
101.- son sustancias que actúan como antioxidantes naturales:
superóxido dismutasa, glutation peroxidasa 120.- es un mediador quimico que tiene actividad bactericiday
produce vasodilatacion por relajacion directa del musculo liso
102.- las principales citocinas que intervienen en la respuesta arteriolar: oxido nitrico
inflamatoria son: il1 y tnf
121.- son fenómenos que caracterizan e indican que la lesión
103.- los siguientes son efectos de la hipoxia en la célula:
celular es irreversible: la disfunción mitocondrial y la perdida
disminución de la fosforilación oxidativa
de la integridad de las membranas celulares
104.- el fijador de tejidos utilizado de forma rutinaria: formol
122.- entre los factores de riesgo para el ca cu se encuentra,
al 10%
excepto: inicio de actividad sexual después de los 30 años
105.- mediadores relacionados con el dolor en la inflamación
123.- which of the following mediators of inflammation has
aguda: prostaglandinas y bradicinina
chemotactic properties and is increased in persons taking
106.- son reacciones de la fase aguda provocadas por la il-1, el aspirin? leukotriene b4
tnf: fiebre, perdida del apetito
124.- nitric oxide synthesis is augmented in endothelial cells
by a calmodulin-mediated influx of which element? calcium
107.- tipo de necrosis que se sucede secundaria a la lesión
125.- órgano o tejido que aumenta de tamaño por aumento
hipoxica letal en la mayoría de los tejidos: coagulativa
de su numero de células: hiperplasia
18.- la lentificación de la circulación en la inflamación aguda se
126.- which of the following mediators of inflammation causes
debe a: el aumento de la permeabilidad de la
pain: bradykinin
microvasculatura
127.- la inflamación fibrinosa se caracteriza por: se presenta
109.- son sustancias que actúan como opsoninas: c3b, igg, y
en inflamación de cavidades corporales como pericardio y
colectinas
pleura
110.- las prolongaciones o pseudopodos de los leucocitos
128.- corazon anormalmente grande por obstrucción valvular
contienen en su interior filamentos de: actina y miosina
mitral o aortica produce: D) hipertrofia
111.- en la hemosiderosis sistémica, el pgimento no lesiona a
129.- una característica histológica de los organos que sufren
las células, pero si su acumulación es excesiva, produce una
atrofia es: formación de vacuolas autofagicas
enfermedad conocida como: hemocromatosis
130.- para la formación de granulomas inmunitarios se
112.- es característica de la inflamación crónica: infiltración de requiere: reaccion inmunitaria mediada por linfocitos t
células mononucleares (macrófagos)
131.- son causas de hipoxia excepto (duda): aneurismas
113.- es caracteristica de la calcificacion metastasica: se
presenta en tejidos normales 132.- son componentes de la inflamación aguda:
modificaciones en el calibre de los vasos y en la estructura
114.- las sustancias químicas pueden producir lesión celular
microvasculatura
por: acción directa o indirecta
133.- son fenómenos leucocitarios en la luz vascular:
115.- el desprendimiento de los ribosomas del rer en la lesión
marginación, rodamiento y adhesión
celular produce: disminución de la síntesis de proteínas
134.- dentro de los ejemplos específicos de apoptosis, está la
116.- los cuerpos de weibel palade de las celulas endoteliales
inducida por lesión del adn celular, señale cúal es el gen
contienen: selectina p
involucrado en este mecanismo: p53
117.- son ejemplo de substancias químicas que causan lesión
135.- el cancer in situ se localiza : limitado al grosor del
celular de manera indirecta al transformarse, por vía
epitelio
enzimática, en radicales libres: paracetamol y tetracloruro de
carbono 136. Para cada tipo de muerte celular elija la patología que sea
más característica.
46) apoptosis (e) 155.- los cuerpos de weibel palade de las celulas endoteliales
47) necrosis licuefactiva (d) contiene: Selectina P
48) necrosis coagulativa (c) 156.- las quimiocinas CX3C son moléculas de adhesión y
49) necrosis grasa (b) agentes quiotacticos para: Monocitos y Linfocito T
50) necrosis caseosa (a) 157.- El calibre de las agujas utilizadas en la biopsia de
aspiración es de : 21 – 22
A) tuberculosis 158.- La lisozima, lactoferrina, proteína básica mayor
B) pancreatitis aguda destruyen microorganismos por mecanismos: independientes
C) infarto de miocardio a oxigeno.
D) infecciones bacterianas focales 159.- Es la secuencia de la fagocitosis:Reconocimiento,
E) muerte celular por rechazo inmunitario / hepatitis viral ingestión, muerte y degradación.
160.- Grupo de mediadores químicos que produce la
137.- ¿A qué gen induce TP53 para que dirija a la célula hacia inhibición de las quimiotaxis y de la adhesión de los
la apoptosis? BAX monocitos: lipoxinas
138.- ¿Son proteínas que facilitan la entrada y progresión de 161.- Estudio de gabinete de elección para el Dx de colecistitis
las células en el ciclo celular? Ciclinas crónica litiasica: Ultrasonido
139.- Factor de crecimiento que promueve la cicatrización de 162.- Mediadores que participan en la adherencia plaquetaria
heridas, la angiogenia, la hematopoyesis y desarrollo de (no sabemos que): ecosanoides.
musculos. FGF 163.- El TNF y la IL-1 produce vasodilatación por: liberación de
140.- La ciclina E/CDK2 controla la transición entre: G1 – S histamina.
141.- si durante una colecistectomía se observan en el hígado 164.- Es una antiproteasa que inhibe a la elastasa y es
del paciente 3 nodulos de diámetro variable entre 10 y 12 cm abundante en el tubo digestivo y pulmón: Alfa 1 –antitripsina
la biopsia de elección a utilizar es: en cuña 165.- Biopsia de elección en pacientes jóvenes con lesión
142.- Es la causa de calcificación distrofica: focos de necrosis escamosa: cono cervical
143.- Que molécula se unen a PAF-1 para evitar que el 166.- Sustancia que participa en la hipertrofia fisiológica: IGF -
aumento de la permeabilidad mitocondrial y salida del b
citocromo c ocacione la apoptosis por la activación de la 167.- La biopsia de elección para una lesión para lesión en
procaspasa 9: BCL2 BCL- X1 MCL-1 estomago de 1 cm de diámetro es: Laparoscopia
144.- De las causas de lesión celular cual de las siguientes es la 168.- El fijador más utilizado para biopsia por aspiración es:
que afeca de manera direct a la respiración oxidativa aerobica Alcohol del 96%
en la celula: isquemia e hipoxia 169.- Es causa de la calcificación distrofica es: Ganglio linfático
145.- Dentro de los ejemplos específicos de apoptosis esta la tuberculoso (necrosis de tejido)
inducida por lesión de DNA celular, señale cual es el gen 170.- El apoptosoma esta formado por las siguientes
involucrado en este meanismo: P53 sustancias: Citocromo C y APAF-1
146.- Son enzimas que producen apoptosis celular: Caspasas 171.- Es el factor que incrementa el flujo sanguíneo en la
147.- El aumento de la permeabilidad vascular da como inflamación: factor activador de plaquetas, C5a
resultado: Exudado 172.- ¿Cuál es la característica del nucleo celular en la
148.- En el aumento de la permeabilidad vascular las citocinas, apoptosis? : condensación de cromatina
IL-1, TNF, INF gamma ocacionan : retracción endotelial 173.- La via intrínseca de la apoptosis activa a: Caspasa 9
149.- Potentes quimiotacticos y activadores leucocitarios: C5a 174.- Las causas de lesión celular cuales son las que afectan
y LTB4 directamente a la fosforilación oxidativa: isquemia e hipoxia
150.- Mediadores que participan en el dolor: PGE2 y 175.- En la necroptosis el necrosoma esta formado por: RIP 1
bradicinina RIP 3 y caspasa 8
151.- Son ejemplos de inflamación serosa: infecciones virales y 176.- La Piroptosis se caracteriza por su respectiva : Caspasa 1
quemaduras de segunda grado. y 11
152.- Prostaglandinas, leucotrienos, tromboxanos se producen 177.- Son características que indican que la lesión celular es
por el metabolismo del: acido araquidónico irreversible: Disfunción mitocondrial y daño a la membrana
153.- Estructura formada por acumulo de celulas epiteloides 178.- Cual de estos es un mecanismo antioxidante enzimático:
redeadas por un collar de linfocitos y cel plasmáticas: Catalasa , glutatión peroxidasa.
granuloma 179.- La sobre carga sistémica …. Se le conoce como:
154.- son ejemplos de sustancias químicas que causan lesión Hemosiderosis
de manera indirecta al transformrse en radicales libres:
paracetamol y tetracloruro de carbono. 180.-Terreno fértil para inductor de cáncer: Metaplasia
181.- Son efectos sistémicos provocados por la IL-1, TNF: 212.- son proteínas plasmáticas de la fase aguda en la
Fiebre, sueño, disminución del apetito. inflamación: Proteina C reactiva, SSA, fibrinógeno
182.- La atrofia puede ser: fisiológica y patológica 213.- los cuerpos de mallory son acumulos de: proteínas
185.- El acumulo intracelular de carbono se le denomina: alcoholicas en hígado. ++
Antracosis
187.- son componentes de la inflamación aguda: cambios en la
microvasculatura de los vasos.
188.- el aumeto de la permeabilidad da como resultado: salida 214.-) CAUSA PREDISPONENTE DE LA CALCIFICACION
de un liquido rico en proteínas METASTASICA: MEDIO ALCALINO
189.- selectinas, integrinas y actúan como: moléculas de
adhesión. 215.-) LA BIOPSIA DE ELECCIÓN PARA UNA LESIÓN
190.- (ingles) Tratamiento de alcoholismo biopsia de hígado): HIPERPIGMENTADA LOCALIZADA EN LA PIEL Y QUE MIDE 1CM
triglicéridos DE DIÁMETRO, ES: EXCISIONAL
191.- peroxidacion de los lípidos de membrana, lesiones de
216.-) EL FIJADOR MÁS UTILIZADO PARA LOS
DNA y desplegamiento de proteínas son efectos de: EROS
TEJIDOS OBTENIDOS POR BIOPSIA O NECROPSIA ES: FORMOL
192.- Los anticuerpos de la igG funcionan como: opsoninas
AL 10%
193.- Los principales quimiotacticos endógenos son: C5a y
LTB4 217.-) ES CAUSA DE LA CALCIFICACIÓN DISTRÓFICA: FOCOS DE
194.- Ingles …. Un hombre de 60 años que tenia una NECROSIS
hemosiderosis generalizada con fallo cerebral y: Necrosis
licuefactiva 218.-) EL APOPTOSOMA ESTA FORMADO POR LAS SIGUIENTES
195.- Quimioxinas CX3X son moléculas de adhesión para: SUSTANCIAS: CITOCROMO C SE UNE APAF-1
Monocitos y linfocitos T 219.-) WHAT CHEMICAL MEDIATORS ARE RESPONSIBLE FOR
196.- Los cuerpos de russell son acumulos de: INCREASING THE BLOOD FLOW TO AN ARE OF
inmunoglobulinas en Celulas plasmáticas INFLAMATION? HISTAMINE, PROSTAGLANDINS (PGI2, PGE,
197.- son productos del metabolismo del ácido araquidónico AND PGD2), AND NITRIC OXIDE
mediado por las ciclooxigenasas: Prostaglandinas y
tromboxano 220.-) FORMA POR LA CUAL LA TELOMERASA INHIBE EL
198.- son mediadores químicos que los producen: Lipoxinas ACORTAMIENTO DE LOS TELOMEROS: AÑADE NUCLEOTIDOS A
199.- La bradicinina es producida por: Calicreina LOS EXTREMOS DEL CROMOSOMA
200.- Granuloma formado por un cumulo de celulas
221.-) ¿CUAL ES LA CARACTERISTICA DEL
epiteloides con un cordon linfocitario y multicelulas: lepra
201.- Quimiocina que funciona como quimiotactica y NUCLEO CELULAR EN LA APOPTOSIS: CONDENSACION DE LA
activación de los neutrófilos: CXC o Alfa CROMATINA
201.- Prostaglandina que produce hiperalgesia: PGE2
202.- Respuesta deficiente: inmunidad innata 222.-) LA VIA EXTRÍNSECA DE LA APOPTOSIS ACTIVA A:
Respuesta excesiva: inmunidad adaptativa CASPASAS 8, 10
204.- La pus esta formado por: detritos celulares, liquido de 223.-) ¿DE LAS CAUSAS DE LESIÓN CELULAR, CUÁL DE LAS
edema, celulas necróticas SIGUIENTES ES LA QUE AFECTA DE MANERA DIRECTA A LA
205.- la causa mas frecuente de esteatosis hepática son: RESPIRACIÓN OXIDATIVA AÉROBICA EN LA CÉLULA? LA
alcoholismo y desnutrición. ISQUEMIA / HIPOXIA
206.- son efectos de inflamación serosa: infecciones virales y
quemaduras de 2do grado 224.-) LA TELOMERASA ES UN COMPLEJO: ARN-PROTEINA
207.- mediador que produce vasodilatación 10 000 veces mas
225.-) EN LESIÓN CELULAR REVERSIBLE DEBIDA A ISQUEMIA
potentes que la histamina: PAF
/HIPOXIA LA PRIMERA ALTERACIÓN BIOQUÍMICA QUE SE
208.- factor del complemento que activa la via lipooxigenasa
PRESENTA EN: DISMINUCIÓN DE LA FOSFORILACIÓN
en neutrófilo: C5a
OXIDATIVA MITOCONDRIAL
209.- factor de crecimiento en el que participa la hipertrofia
por aumento de la carga de trabajo: TGF beta 226.-) SON FENÓMENOS QUE CARACTERIZAN E INDICAN QUE
210.- la atrofia se produce por: aumento por la degradación LA LESIÓN CELULAR ES IRREVERSIBLE: LA DISFUNCIÓN
de proteínas MITOCONDRIAL , PROFUNDOS TRASTORNOS EN LA FUNCIÓN
211.- El receptor de muerte más conocido en la apoptosis se DE LAS MEMBRANAS CELULARES
llama: Fas (CD95)
227.- ) ¿CÚALES DE LOS SIGUIENTES SON FACTORES A CEREBRAL INFARCT WAS FOUND AT AUTOPSY. NECROSIS
ANTIOXIDANTES ENZIMÁTICOS? CATALASA, GLUTATIÓN OF THE BRAIN IS CLASSIFIED AS: LIQUEFACTIVE NECROSIS
PEROXIDASA Y SUPERÓXIDO DISMUTASAS
244,.) LAS QUIMIOCINAS CX3C SON MOLÉCULAS DE
228.-) LA CALCIFICACION DISTROFICA SE PRESENTA EN: ADHESIÓN PARA: MONOCITOS Y LINFOCITOS T
ATEROMATOSIS
245.-) LOS CUERPOS DE RUSSELL SON ACUMULOS DE:
229.- )ADAPTACIÓN CELULAR DEL CRECIMIENTO QUE PUEDE INMUNOGLOBULINAS EN CÉLULAS PLASMÁTICAS
SER TERRENO FÉRTIL EN DONDE SE INICIE UNA
246.-) SON PRODUCTOS DEL METABOLISMO DEL ÁCIDO
PROLIFERACIÓN MALIGNA: HIPERPLASIA PATOLÓGICA
ARAQUIDÓNICO POR LA VÍA DE LA CICLOOXIGENASA:
230.-) SON EFECTOS SISTEMICOS PROVOCADAS POR LA IL-I, EL TROMBOXANO Y PROSTAGLANDINAS
TNF EN LA REACCION INFLAMATORIA: FIEBRE, PÉRDIDA DEL
247.-) GRUPO DE MEDIADORES QUIMICOS QUE PRODUCEN
APETITO , SUEÑO
INHIBICION DE LA QUIMIOTAXIS DE NEUTROFILOS Y LA
231.-) LA ATROFIA PUEDE SER: FISIOLÓGICA Y PATOLÓGICA ADHERENCIA DE LOS MONOCITOS : LIPOXINAS

232.-) DE LAS REFERIDAS A CONTINUACIÓN, SEÑALE EN QUE 248.-) BRADYKININ IS PRODUCED FROM A HIGH-MOLECULAR
MUESTRAS PUEDE REALIZARSE CITOLOGIA EXFOLIATIVA: WEIGHT KINOINOGEN CIRCULATING IN THE BLOOD. THIS
ORINA, EXPECTORACIÓN Y LÍQUIDO DE ASCITIS REACTION IS MEDIATED BY: KALLIKREIN

233.-) TIPO DE METAPLASIA MÁS FRECUENTE: DE EPITELIO 249.-) ESTRUCTURA FORMADA POR UN ACUMULO DE
CILÍNDRÍCO A EPITELIO PLANO ESTRATIFICADO CÉLULAS EPITELIOIDES, RODEADAS POR UN COLLAR DE
LINFOCITOS Y CÉLULAS PLASMÁTICAS SE PRESENTA EN:
234.-) LA ACUMULACIÓN INTRACELULAR DE POLVO DE
BRUCELOSIS
CARBÓN RECIBE EL NOMBRE DE: ANTRACOSIS
250.-) QUIMIOCINA QUE DA LUGAR A ACTIVACIÓN Y
235.-) ¿A CÚAL DE LOS PIGMENTOS ENDÓGENOS QUE SE
QUIMIOTAXIS DE LOS NEUTROFILOS, CON ACTIVIDAD
PUEDEN ACUMULAR INTRACELULARMENTE SE LE LLAMA LIMITADA SOBRE MONOCITOS Y EOSINOFILOS: C-X-C O ALFA
“´PIGMENTO DE ENVEJECIMIENTO”? LIPOFUSCINA
251.-) PROSTAGLANDINA QUE PRODUCE HIPERALGESIA, QUE
236.-) SON COMPONENTES DE LA INFLAMACIÓN AGUDA:
HACE QUE LA PIEL PRESENTE HIPERSENSIBILIDAD (DOLOR Y
MODIFICACIÓN EN EL CALIBRE DE LOS VASOS Y EN LA
FIEBRE): PGE2
ESTRUCTURA DE LA MICROVASCULATURA
252.-) SISTEMA BACTERICIDA MAS EFICAS DE LOS
237.-)EL AUMENTO DE LA PERMEABILIDAD VASCULAR DA
NEUTROFILOS: H2O2 + MPO + Cl
COMO RESULTADO: SALIDA DE UN FLUIDO RICO EN CELULAS
Y PROTEÍNAS AL INTERSTICIO 253.-) NITRIC OXIDE SYNTHESIS IS AUGMENTED IN
ENDOTHELIAL CELLS BY A CALMODULIN MEDIATED INFLUX OF
238.-) SELECTINAS, INTEGRINAS Y GLUCOPROTEÍNAS TIPO WHICH ELEMENT? CALCIUM
MUCINA ACTÚAN COMO: MOLÉCULAS DE ADHESIÓN
254.-) PUS O MATERIAL PURULENTO SE ENCUENTRA
239-) A 60-YEAR-OLD OBESE MAN WAS ADMITTED TO THE
FORMADO POR: DETRITUS CELULARES, NEUTRÓFILOS Y
HOSPITAL FOR TREATMENT OF ALCOHOLISM. HE HAS
LIQUIDO DE EDEMA
DIABETES MELLITUS. A LIVER BIOPSY WAS PERFORMED, AND
THE SPECIMEN SHOWED THAT THE LIVER CELLS CONTAIN 255.-)LA CAUSAS MÁS FRECUENTES DE LA ESTEATOSIS
INCREASED AMOUNTS OF: TRIGLYCERIDES HEPATICA SON: ALCOHOLISMO, DESNUTRICIÓN

240.-)PEROXIDACIÓN DE LOS LÍPIDOS DE LA MEMBRANA, 256.-) SON EJEMPLOS DE INFLAMACIÓN SEROSA:


MODIFICACIÓN OXIDATIVA DE LAS PROTEÍNAS Y LESIONES DEL INFECCIONES VIRALES Y QUEMADURAS DE 2º GRADO
DNA SON EFECTOS RELEVANTES PARA LA LESIÓN CELULAR
257.-) MEDIADOR QUIMICO QUE PRODUCE VASODILATACION
CAUSADAS POR: RADICALES LIBRES DERIVADOS DEL OXÍGENO
Y SU EFECTO ES 10.000 VECES MAS POTENTE QUE LA
241.-) LOS ANTICUERPOS DE LA IGG, C3B Y LAS COLECTINAS HISTAMINA: FACTOR ACTIVADOR DE LAS PLAQUETAS
FUNCIONAN COMO: OPSONINAS
258.-)FACTOR DEL COMPLEMENTO C5a SE ACTIVA POR LA VIA
242.-) PRINCIPALES QUIMIOTACTICOS ENDOGENOS: C5A Y ¿ ? PARA ACTIVAR NEUTROFILOS Y MONICITOS PARA
LTB4 AUMENTAR LA LIBERACION DE MAS MEDIADORES
INFLAMATORIOS: VIA LIPOOXIOGENASA
243.) A 60-YEAR-OLD MAN WHO HAD GENERALIZED
ATHEROSCLEROSIS DIED 24 HOURS AFTER HAVING A STROKE.
259.-) FACTOR DE CRECIMIEMTO RELACIONADO CON LA 279.-) SU EPITELIZACIÓN COMPLETA ES MÁS LENTA:
HIPERTROFIA POR AUMENTO DE LA CARGA DE TRABAJO: TGF CICATRICES DE SEGUNDA INTENCIÓN
BETA
280.-) SON EJEMPLOS DE INFLAMACIÓN SEROSA:
260.-) LA ATROFIA SE PRODUCE POR: AUMENTO EN LA DERRAME PLEURAL, AMPOLLAS DE LAS QUEMADURAS,
DEGRADACION DE PROTEINAS VESÍCULAS CAUSADAS POR INFECCIÓN VIRAL (HERPES),
LIQUIDO DE ASCITIS
261.-) EL RECEPTOR DE MUERTE MAS CONOCIDO EN LA
APOPTOSIS SE LLAMA: FAS 281.-) ES UNA ALTERACIÓN LOCAL DE LA CICATRIZACIÓN.
BORDES CON TEJIDOS NECROZADOS
262.-) SON PROTEINAS PLASMATICAS DE LA FASE AGUDA EN
LA INFLAMACION: PROTEINA C REACTIVA, FIBRINOGENO 282.-) ES UN PROCESO ACTIVO EN EL CUAL LA DILATACIÓN
PROTEINA AMILOIDE ARTERIAL AUMENTA EL FLUJO DE SANGRE: HIPEREMIA

263.-) LOS CUERPOS DE MALLORY SON ACUMULOS DE: 283.-) ES UNA EXTRAVASACIÓN DE SANGRE HACIA EL
PROTEINAS HIALINO ALCOHOLICAS EN HIGADO ESPACIO EXTRAVASCULAR PUEDE SER EXTERNA O
QUEDAR CONTENIDA EN UN TEJIDO: HEMORRAGIA

284) These gallstones surgically removed along with the


gallbladder of a 50-year-old obese woman are composed
predominantly of: CHOLESTEROL
EXAMEN EXTRAORDINARIO DE
ANATOMIA PATOLOGICA

269.-) DEFECTOS EN ESTOS GENES PUEDEN CONVERTIRLOS


285.-) EL PROCESO INICIAL DE VASOCONSTRICCIÓN TRAS
EN ONCOGENES Y CONTRIBUIR AL CRECIMIENTO CÁNCER:
UNA LESIÓN ESTA MEDIADO POR: ENDOTELINA.
PROTOONCOGENES
286-) PROCESO PASIVO, DEBIDO A LA REDUCCIÓN DEL
270.-) ¿QUÉ VITAMINA ES NECESARIA PARA LA
FLUJO DE SALIDA DE UN TEJIDO, ACUMULA
HIDROXILACIÓN DEL PROCOLAGENO? C
HEMOGLOBINA DESOXIGENADA : CONGESTIÓN
271.-) PRECURSORES CELULARES ENDOTELIALES QUE
287.-) ¿CUÁLES SON LOS COMPONENTES DE LA
PARTICIPAN EN LA VASCULOGÉNESIS: ANGIOBLASTOS
HEMOSTASIA ? ENDOTELIO, PLAQUETAS Y CASCADA DE
272.-) LA CURACIÓN DE UNA HERIDA CON DAÑO CÉLULAR COAGULACIÓN
EXTENSO ES UN EJEMPLO DE: CURACIÓN POR SEGUNDA
288.-) ¿QUE FACILITA LA ADHERENCIA Y ACTIVACIÓN
INTENCIÓN
PLAQUETARIA? LA EXPOSICIÓN DE LA MATRIZ
273.-) ¿ALREDEDOR DE QUE DÍA SE FORMAN LOS PUENTES EXTRACELULAR MUY TROMBOGENICA
DE COLÁGENO EN LA CURACIÓN DE HERIDAS POR
289.-) EL CUADRO SE CARACTERIZA POR LA SÚBITA
PRIMERA INTENCIÓN? ALREDEDOR DEL DÍA 5
APARICIÓN DE DISNEA, CIANOSIS Y SHOCK, SEGUIDO DE
274.-) CAMBIO EN LA DIFERENCIACIÓN DE UNA CÉLULA DE ALTERACIONES NEUROLÓGICAS QUE VAN DESDE LA
UN TIPO CELULAR A OTRO: TRANSDIFERENCIACIÓN. CEFALEA A CONVULSIONES Y COMA: EMBOLIA DE LÍQUIDO
ANMIÓTICO
275.-) LA APOPTOSIS FISIOLOGICA SE PRESENTA EN:
MAMA LACTANTE DURANTE EL DESTETE 290.-) EL SÍNDROME POR DESCOMPRESIÓN SE DA
PRINCIPALMENTE EN: BUCEADORES
276.-) CARACTERÍSTICA DE LA CALCIFICACIÓN
DISTROFICA: SE PRESENTA EN ÁREAS DE NECROSIS 291.-) SE CONSIDERAN FACTORES DE RIESGO PARA
PRODUCIR COLELITIASIS, EXCEPTO: DIETA RICA EN FIBRA
277.-) SU FORMACIÓN EN EL MARGEN DE LA HERIDA
AYUDA A LA CONTRACCIÓN DE LA HERIDA: 292.-) ES LA PRINCIPAL COMPLICACIÓN DE LOS CÁLCULOS
MIOFIBROBLASTOS BILIARES POR OBSTRUCCIÓN DEL CUELLO DE LA VESÍCULA
O DEL CONDUCTO CÍSTICO: COLECISTITIS LITIASICA AGUDA
278.-) SON SUSTANCIAS QUE ACTÚAN COMO OPSONINAS:
C3B, IGG, Y COLECTINAS 293.-) LOS CASOS MÁS GRAVES DONDE LA VESÍCULA BILIAR
ESTÁ NECRÓTICA, VERDE NEGRUZCO Y PUEDE PRESENTAR
PERFORACIONES SE DENOMINA: COLECISTITIS 308.-) EL GRADO EN QUE LAS CÉLULAS
GANGRENOSA PARENQUIMATOSAS DE UNA NEOPLASIA SE PARECEN A LA
CONTRAPARTE NORMAL SE DENOMINA: DIFERENCIACIÓN
294.-) ENTRE LOS FACTORES DE RIESGO MÁS
IMPORTANTES PARA LA PRESENTACIÓN DE IAM 309.-) CÓMO SE DENOMINAN A LOS TROMBOS QUE
DESTACAN: TABAQUISMO OBESIDAD, HIPERTENSIÓN PUEDEN TENER LAMINACIONES PRODUCIDOS POR CAPAS
ARTERIAL, DIABETES MELLITUS, HIPERCOLESTEROLEMIA Y PÁLIDAS ALTERNANTES DE PLAQUETAS MEZCLADAS CON
ATEROESCLEROSIS FIBRINA Y HEMATÍES? LÍNEAS DE ZAHN

295.-) ¿CUÁL ES EL VIRUS ARN ONCÓGENO? VIRUS DE LA 310.-) ¿CÓMO SE CLASIFICAN LOS INFARTOS? ROJOS,
LEUCEMIA DE CÉLULAS T BLANCOS, SÉPTICOS

296.-) .- ES EJEMPLO DE TROMBOSIS POR ESTADO DE 311.-) EL PRIMER PUNTO DE ATAQUE DE LA HIPOXIA EN LA
HIPERCOAGULABILIDAD PRIMARIA: DEF. DE CELULA ES: FOSFORILACION OXIDATIVA
ANTITROMBINA III Y DE PROTEÍNA C Y S
312.-) ¿CUÁL DE LOS SIGUIENTES ES ANTIOXIDANTES NO
297.-) LA LESIÓN CELULAR POR TETRACLORURO DE ENZIMÁTICO? TRANSFERRINA
CARBONO Y TYLENOL SE CARACTERIZAN POR: SU
313.-) LOS SIGUIENTES SON PIGMENTOS DERIVADOS DE LA
CONVERSIÓN EN METABOLITOS ACTIVOS MUY TÓXICOS
HEMOGLOBINA: HEMOSIDERINA Y BILIRRUBINA
298.-) ADAPTACIÓN CELULAR QUE CONSISTE EN EL
315.-) CON RESPECTO A LA CARACTERÍSTICA MICROSCÓPICO
AUMENTO EN EL NÚMERO DE CÉLULAS DE UN ÓRGANO O
MÁS IMPORTANTE DEL LEIOMIOMA: ASPECTO
TEJIDO CON EL CONSECUENTE AUMENTO DE VOLÚMEN
ARREMOLINADO DE LOS HACES DEL MÚSCULO LISO
DEL MISMO: HIPERPLASIA
316.-) OCURRE CUANDO EL NÚCLEO DE UN FIBROBLASTO
299.-) A LA ACUMULACIÓN ANORMAL DE TRIGLICÉRIDOS
CUTÁNEO DE UN PACIENTE SE INTRODUCE EN UN OVOCITO
DENTRO DE LAS CÉLULAS PARENQUIMATOSAS SE LE
HUMANO ENUCLEADO PARA FORMAR CME QUE SE
LLAMA: ESTEATOSIS
CULTIVAN, SE INDUCE SU DIFERENCIACIÓN, Y SE
300.-) A CÚAL DE LOS PIGMENTOS ENDÓGENOS QUE SE TRASPLANTAN AL PACIENTE: CLONACIÓN TERAPÉUTICA.
PUEDEN ACUMULAR INTRACELULARMENTE SE LE LLAMA
317.-) CARACTERÍSTICAS MORFOLÓGICAS DE LA APENDICITIS
“PIGMENTO DEL ENVEJECIMIENTO”? LIPOFUSCINA
AGUDA SUPURADA: ABSCESO DE PARED ACOMPAÑADO POR
301.-) ¿QUÉ TIPO DE BIOPSIA SE UTILIZA PARA ULCERACIÓN
PADECIMIENTOS EN PROSTATA? BIOPSIA POR PUNCIÓN
318.-) .-LAS CÉLULAS MADRE/PROGENITORAS DEL HÍGADO SE
302.-) PROBLEMAS CLÍNICOS EN RIÑONES CAUSADOS POR ALBERGAN EN: CONDUCTOS DE HERING.
UN SHOCK: OLIGURIA, ANURIA .
319.-) FACTOR DE CRECIMIENTO QUE CONTRIBUYE A LA
303.-) SI SE INMOVILIZA UNA EXTREMIDAD CON UN REPARACIÓN DE HERIDAS, ANGIOGENIA, HEMATOPOYESIS Y
APARATO DE YESO POR TIEMPO PROLONGADO, LOS DESARROLLO DE MÚSCULOS CARDIACO Y ESQUELÉTICO, LA
MUSCULOS INMOVILIZADOS PRESENTARAN: ATROFIA MADURACIÓN DEL PULMÓN, Y LA ESPECIFICACIÓN DEL
HÍGADO: FGF
304.-) SE LLAMA PERDIDA DE LA POLARIDAD CELULAR
NORMAL CUANDO: HAY PERDIDA DE LA ORIENTACIÓN DE 320) SUN UV RADIATIONS ARE ASSOCIATED WITH THE
LAS CÉLULAS ANAPLÁSICAS FOLLOWING TYPES OF CANCER: BASOCELULAR CARCINOMA

305.-) EL RANGO PROMEDIO PARA LA FIJACIÓN ADECUADA 321.-) .- EL NIVEL SERICO DE LA CPK-MB, SE ELEVA POR
DE UN ESPÉCIMENQUIRÚRGICO EN FORMOL ES: DE 8 a 12 ENCIMA DE LOS VALORES BASALES A LAS: 2-4 HRS.
HORAS
322-) SON EJEMPLOS DEL GRUPO FISIOPATÓLOGICO DE
306.-) LA TROMBOSIS ARTERIAL GENERALMENTE ES EDEMA POR: DESNUTRICIÓN KWASHORKIOR Y CIRROSIS
SECUNDARIA A: ATEROESCLEROSIS HEPÁTICA

307.-)UN TUMOR ES CLÍNICAMENTE DETECTABLE CUANDO 323.-) INTERACTUA CON UN RECEPTOR TIE 2 PARA RECLUTAR
TIENE UN PESO DE 1 GRAMO, EL CUAL CONTIENE: 10 9 A LAS CÉLULAS PERIENDOTELIALES: ANGIOPOYETINA 1
CELULAS
324.-) EL PATRÓN MORFOLÓGICO DE NECROSIS QUE
PRESENTAN LAS CÉLULAS MUERTAS POR ISQUEMIA (SALVO
EN EL SNC) ES: COAGULATIVA
325-) TIPO DE METAPLASIA MÁS FRECUENTE EN TEJIDO 341.-) CRITERIO HISTOLOGICO PARA DIAGNOSTICO DE
EPITELIAL: DE EPITELIO CILÍNDRICO A PLANO ESTRATIFICADO APENDICITIS AGUDA: INFILTRACION DE NEUTROFILOS EN LA
CAPA MUSCULAR
326.-) SON ANORMALIDADES ENCONTRADAS EN UNA
CITOLOGÍA EXFOLIATIVA EXCEPTO: PRESENCIA DE 342.-) SON SUSTANCIAS CON CAPACIDAD ANTITROBOTICA:
ANTICUERPOS PLASMINOGENO TISULAR Y PROSTACICLINA ENDOTELIAL

327.-) EL COLOR AZUL VERDOSO QUE ADQUIEREN LOS 343.-) ES EJEMPLO DE TROMBOSIS POR ESTADOS DE
HEMATOMAS EN LA FASE DE RESOLUCIÓN ES PORQUE: LA HIPERCOAGULABILIDAD SECUNDARIA: SINDROME DE
HEMOGLOBINA SE TRANSFORMA EN HEMOSIDERINA ANTICOAGULANTE LÚPICO

328.-) ES UNA ALTERACIÓN DE ESCASA FRECUENCIA QUE SE 344.-) LOS INFARTOS ROJOS APARECEN EN LAS SIGUIENTES
DEBE A UNA PROLIFERACIÓN EXUBERANTE DE FIBROBLASTOS CONDICIONES EXCEPTO: EN OCLUSIONES ARTERIALES
Y OTROS ELEMENTOS DEL TEJIDO CONJUNTIVO: LESIONES
345.-) ES MITÓGENO PARA CÉLULAS EPITELIALES,
DESMOIDES
HEPATOCITOS Y FIBROBLASTOS, ES HOMOLOGO DEL TGF
329.- ) ADAPTACIÓN CELULAR DEL CRECIMIENTO QUE PUEDE ALFA, AL TENER ACTIVIDADES BIOLÓGICAS SIMILARES:
SER TERRENO FÉRTIL EN DONDE SE INICIA UNA TGFBETA
PROLIFERACIÓN CANCEROSA: DISPLASIA
346.-) SON GRANDES COLECCIONES DE SANGRE EN
330.-) WHICH ARN VIRUS HAS ONCOGEN ACTIVITY? T-CELLS CAVIDADES EXCEPTO: HEMOSTASIA
LEUKEMIA VIRUS
347.-) EL TEJIDO DE GRANULACIÓN SE CARACTERIZA POR:
331.-) ES EL COAGULO CONSTITUIDO POR AGREGACION PROLIFERACIÓN FIBROBLASTOS Y DE PEQUEÑOS VASOS
PLAQUETARIA Y DEPOSITO DE FIBRINA: TAPON HEMOSTATICO SANGUÍNEOS
SECUNDARIO
348.-) MOST CASES OF SEPTIC SHOCK ARE CAUSED BY
332.-) EN EL PROCESO DE REMODELACIÓN DE LA REPARACIÓN ENDOTOXIN-PRODUCING BACTERIA, WHICH ARE CLASSIFIED
DE LAS HERIDAS PARTICIPAN LAS SIGUIENTES ENZIMAS: AS: GRAM-POSITIVE
METALOPROTEÍNASAS, GELATINASAS
349.-) EN EL FIBROADENOMA MAMARIO SE
333.-) ADAPTACIÓN CELULAR DE LA DIFERENCIACIÓN, QUE CARACTERIZA POR TODO LO SIGUIENTE EXCEPTO:
CONSISTE EN UN CAMBIO REVERSIBLE DE UNA CÉLULA
A) ES UN NODULO ESFERICO DE GRAN TAMAÑO
ADULTA POR OTRA CÉLULA ADULTA DE DIFERENTE TIPO:
METAPLASIA B) ESTA FIJO A PLANOS PROFUNDOS Y NO SE PUEDE LIMITAR
SU TAMAÑO
334.-) PRINCIPAL QUIMIOTÁCTICO ENDÓGENO PARA
NEUTROFILOS: C-X-C C) ES MOVIL Y NO FIJO A PLANOS PROFUNDOS
335.-) PRINCIPAL QUIMIOCINA QUE INDUCE POTENTE 350 THE ABNORMAL TRIGLYCERIDE ACCUMULATION INSIDE
ADHESION PARA MONOCITOS Y LINFOCITOS T: CX3C PARENCHYMA CELLS IS CALLED: STEATOSIS OR CHANGE
336.-) COMPLICACION MAS FRECUENTE DEL INFARTO DE 351.-) SE PUEDEN ENCONTRAR GRANULOMAS EN LAS
MIOCARDIO: EDEMA PULMONAR SIGUIENTES ENFERMEDADES: LEPRA LEPROMATOSA Y
TUBERCULOIDE
337.-) .- SELECTINAS INTEGRINAS Y GLUCOPROTEÍNAS TIPO
MUCINA ACTÚAN COMO: MOLÉCULAS DE ADHESIÓN 352.-) SON METABOLITOS DEL ACIDO ARAQUIDONICO POR LA
VIA DE LA CICLOOXIGENASA: PROSTAGLANDINAS Y
338.-) LA LIZOSIMA, LACTOFERRINA, PROTEÍNA BÁSICA
TROMBOXANO
PRINCIPAL Y LAS DEFENSINAS, DESTRUYEN
MICROORGANISMOS POR MECANISMOS: INDEPENDIENTES 353 SON TIPOS DE EMBOLIAS EXCEPTO: EMBOLIA
DEL OXÍGENO LINFATICA
339.-) ES LA CAPACIDAD DE UNA CÉLULA DE
354.-) TIPO DE CALCIFICACION QUE SE ACOMPAÑA DE
TRANSDIFERENCIARSE A DISTINTOS ESTIRPES CELULARES:
HIPERCALCEMIA Y TRASTORNOS EN EL METABOLISMO
PLASTICIDAD DEL DESARROLLO
DEL CALCIO: METASTASICA
340.-) MANIFESTACION CLINICA INICIAL MÁS FRECUENTE DEL
CARCINOMA RENAL: HEMATURIA
355 LOS LEIOMIOMAS DE ACUERDO A SU LOCALIZACIÓN 7.- Es la capacidad de una célula de transdiferenciarse a
EN EL CUERPO UTERINO SE CLASIFICAN EN: distintos estirpes celulares: Plasticidad de desarrollo
SUBMUCOSOS, SUBSEROSOS E INTRAMURALES
8.- En esta fase de shock hay liberación de ADH y
356.-) PIGMENTO ENDÓGENO, NO DERIVADO DE LA activación de baroreceptores: Fase inicial progresiva
HEMOGLOBINA: MELANINA
9.- Manifestaciones clínicas presentes en shock séptico:
357.-) LOS CUERPOS DE RUSSELL SON ACUMULOS DE: Piel caliente con vasodilatación periférica, hiperemia.
INMUNOGLOBULINAS EN CÉLULAS PLASMÁTICAS
10.- Casi siempre oclusiva puente largo de la luz del
358.-) LOS CÁLCULOS BILIARES ESTÁN FORMADOS POR: vaso: Trombosis venosa profunda
COLESTEROL, CALCIO, PIGMENTOS BILIARES
11.- Tipo de necrosis encéfalo: Licuefactiva
359.-) COMPLICACIÓN MAS FRECUENTE DE LA
12.- Sintomas de la embolia grasa:
APENDICITIS: PERITONITIS
Taquicardia, bradicardia y disnea
360.-) LA ALFA FETO PROTEINA SE ELEVA
13.- Mantiene la integridad del epitelio externo de la
ANORMALMENTE EN: CANCER PRIMARIO DE HIGADO,
cornea: Celulas madre del limbo
DE CELULAS GERMINALES DE TESTICULO
14.- Una neoplasia compuesta se divide como: Dos capas
361) LA CITOMETRIA DE FLUJO PUEDE MEDIR: LOS
celulares y una capa embrionaria
ANTIGENOS DE MEMBRANA, CONTENIDO DE ADN EN
CELULAS TUMORALES 15.- Metastasis: Implantes tumorales sin continuidad
con el tumor primario
362) LA INICIACIÓN DE LA CALCIFICACIÓN DISTROFICA
EXTRACELULAR SE INICIA EN: LOS FOSFOLIPIDOS DE LA 16.- Fases de la cascada metastasica: Invasion de la MEC,
MEMBRANA CELULAR diseminación vascular, alojamiento y colonización
363) FENÓMENOS QUE CONSIDERAN DENTRO DEL 17.- Inhibicion de TP53 mediado por: BAX
DESTINO DEL TROMBO: INCORPORACION A LA PARED,
RECANALIZACIÓN, PROPAGACIÓN, EMBOLIZACION 18.-wich one of the following characteristics not
acompany the cancer: Fibrous capsule
364) EN CUAL DE LOS SIGUIENTES ÓRGANOS SE PUEDE
DESARROLLAR UN TERATOMA: OVARIO 19.- What are the organs most affected by the
hematogenous spread? Liver and lung

20.-Wich ARN virus has on oncogene activity? Leucemia


and cel T

21.-Carcinogeno natural hepático secretado por plantas


y hongos: Aflatoxina B1
1.- Patología en la que se pierde albumina y genera
edema: Sindrome Nefrotico 22.- Causas frecuentes de esteatosis: Alcoholismo y
desnutrición
2.- Causantes de edema: Enfermedad cardiaca y renal
23.- Medidador quimico que produce dilatación y
3.- hematoma mayor a 2cm: Equimosis
aumento de la permeabilidad y se libera de tipo físico:
4.- Causa fisiológica de hiperemia: Vasodilatacion por Histamina
ejercicio
24.- son células que participan en la reacción
5.- Factor que inhibe la diseminación del trombo: inflamatoria excepto: Pericitos
Activador tisular de plasminogeno
25.- Las CX3C son moléculas de adhesión de: Monocitos
6.- Capacidad de cambiar de un tipo celular a otro: y Cel T
Transdiferenciasion
26.- De que vitamina son dependientes las proteínas C y
S: Vitamina K
27.- Localizacion mas frecuente del carcinoma de células 46.- Para la formación de granulomas se requiere:
renales: Polos reacción inmunitaria mediada por células T

28.- El cáncer renal causa frecuentemente metástasis en: 47.- Pancreatitis aguda ------------ Necrosis Grasa
Pulmones y hueso
48.- Infeccion Bacteriana-----------N. Licuefactiva
29.- Causa mas frecuente de cáncer renal: Tabaquismo
49.- Muerte celular------------------ Apoptosis
30.- Cual es la proteína viral que induce una rápida
50.- Tuberculosis--------------------- N. caseosa
degradación de P53: E6
51.- Infarto al miocardio----------- N. Coagulativa
31.- Causa frecuente de metástasis en CACU:
Pielonefritis y Uremia

32.- Lesion primaria de mayor riesgo de producir cáncer


1.-) LA BIOPSIA DE ELECCIÓN PARA UNA LESION
de mama: Mastopatia proliferativa con atipia
PIGMENTADA EN LA CARA: EXCISIONAL EN HUSO
33.- Quimico que dara activación quimiotactica,
2.-) LA BIOPSIA DE ELECCIÓN EN UNA LESION CEREBRAL:
actividad sobre monocitos y eosinofilos: CXC
POR PUNCION ESTEREOTAXICA
34.- sustancias que ocacionan fiebre, sueño y perdida del
3.- ) LA CLASIFICACION DE BETHESDA INCLUYE EN LAS
apetito: Citocinas
LESIONES ESCAMOSAS DE ALTO GRADO A: DISPLASIA
35.- La inflamación fibrinosa se caracteriza por: Fluidos MODERADA Y SEVERA POR INFECCION DEL VPH
corporales en pericardio y pleura
4.-) EL FIJADOR MÁS UTILIZADO PARA LOS TEJIDOS
36.- Caracteristicas histológicas de órganos que sufren OBTENIDOS POR BIOPSIA O NECROPSIA ES: FORMOL AL
atrofia, hay presencia de: Formacion de vacuolas 10%
autofagicas
5.-) ES CAUSA DE LA CALCIFICACIÓN DISTRÓFICA: FOCOS
37.- Los leiomiomas se dividen en: Submucosos, DE NECROSIS
subserosos e intramurales
6.- LA DEPLECION DE OXIGENO EN LA CELULA PROVOCA:
38.- Las enfermedades crónicas granulomatosas abarca GLUCOLISIS ANAEROBIA
varios trastornos que tienen en común: La incapacidad
7.-) LOS MECANISMOS DE LA APOPTOSIS SON: VIA
de destruir agentes etiológicos
EXTRÍNSECA Y VIA INTRÍNSECA
40.- El infarto de miocardio corresponde a una:
8.-) SON PROTEINAS CITOPLASMICAS INHIBIDORAS
cardiopatía isquémica
FISIOLOGICAS DE LA APOPTOSIS: BCL2 Y BCLX
41.- Todos las siguientes sustancias pertenecen al tren
9.-)QUE MOLÉCULAS SE UNEN PARA FORMAR EL
de tincion, excepto: Violeta de Genciana
APOPTOSOMA? CITOCROMO C Y APAF 1
42.- La citología exfoliativa puede reportar todo,
10.-) DEFECTOS ADQUIRIDOS EN LA FUNCION
excepto: El estado nutricional de la células.
LEUCOCITARIA: DIABETES, MALNUTRICION
43.- Carcinoma in situ ductual consiste en: Poblacion
11.-) ¿DE LAS CAUSAS DE LESIÓN CELULAR, CUÁL DE LAS
maligna de células que no pueden atravesar la
SIGUIENTES ES LA QUE AFECTA DE MANERA DIRECTA A
Membrana Basal
LA RESPIRACIÓN OXIDATIVA AÉROBICA EN LA CÉLULA?
44.- La fosfofructocinasa y la fosforilaza en lesión, LA ISQUEMIA / HIPOXIA
activan a: Glucolisis Anaerobia
12.-)LOS XANTOMAS SON UN ACUMULO DE: LIPIDOS
45.- Que moléculas emiten la adhesión de plaquetas al
13.- SON FENÓMENOS QUE CARACTERIZAN E INDICAN
endotelio no lesionado? Prostacicilinas y Oxido Nitrico
QUE LA LESIÓN CELULAR ES IRREVERSIBLE: LA
DISFUNCIÓN MITOCONDRIAL Y LOS PROFUNDOS
TRASTORNOS EN LA FUNCIÓN DE LAS MEMBRANAS 28.- MEDIADORES RELACIONADOS CON EL DOLOR EN LA
CELULARES INFLAMACIÓN AGUDA: PROSTAGLANDINAS Y
BRADICININA
14.- ¿CÚALES DE LOS SIGUIENTES SON FACTORES
ANTIOXIDANTES ENZIMÁTICOS? CATALASA, GLUTATIÓN 29.- SON EJEMPLOS DE INFLAMACIÓN SEROSA:
PEROXIDASA Y SUPERÓXIDO DISMUTASAS INFECCIONES VIRALES Y QUEMADURAS DE 2° GRADO

15.- DENTRO DE LOS EJEMPLOS ESPECÍFICOS DE 30.- EL FRAGMENTO DE FC DE IgG, C3B Y LAS
APOPTOSIS, ESTÁ LA INDUCIDA POR LESIÓN DEL ADN COLECTINAS FUNCIONAN COMO: OPSONINAS
CELULAR, SEÑALE CÚAL ES EL GEN INVOLUCRADO EN
31.- SON MEDIADORES QUIMICOS INHIBIDORES DE LA
ESTE MECANISMO: P 53
RESPUESTA INFLAMATORIA: LIPOXINAS
16.- ADAPTACIÓN CELULAR DEL CRECIMIENTO QUE
32.-EL FACTOR XII DE LA COAGULACIÓN (HAGEMAN)
PUEDE SER TERRENO FÉRTIL EN DONDE SE INICIE UNA
ACTIVA 4 SISTEMAS EXCEPTO: SISTEMA GRANULOCITICO
PROLIFERACIÓN MALIGNA: HIPERPLASIA PATOLÓGICA
33.- ESTRUCTURA FORMADA POR UN ACUMULO DE
17.- SON REACCIONES DE LA FASE AGUDA PROVOCADAS
CÉLULAS EPITELIOIDES, RODEADAS POR UN COLLAR DE
POR LA IL-I, EL FNT: FIEBRE, PÉRDIDA DEL APETITO
LINFOCITOS Y CÉLULAS PLASMÁTICAS: GRANULOMA
18.- LA ATROFIA PUEDE SER: FISIOLÓGICA Y PATOLÓGICA
34.- SON EJEMPLO DE SUBSTANCIAS QUÍMICAS QUE
19.- DE LAS REFERIDAS A CONTINUACIÓN, SEÑALE EN CAUSAN LESIÓN CELULAR DE MANERA INDIRECTA AL
QUE MUESTRAS PUEDE REALIZARSE CITOLOGIA TRANSFORMARSE, POR VÍA ENZIMÁTICA, EN RADICALES
EXFOLIATIVA: ORINA, EXPECTORACIÓN Y LÍQUIDO DE LIBRES: PARACETAMOL Y TETRACLORURO DE CARBONO
ASCITIS
35.- EL PATRÓN MORFOLÓGICO DE NECROSIS EN EL QUE
20.- TIPO DE METAPLASIA MÁS FRECUENTE: DE EPITELIO PREDOMINA LA DEGRADACIÓN ENZIMÁTICA ES LA: N.
CILÍNDRÍCO A EPITELIO PLANO ESTRATIFICADO LICUEFACTIVA

21.- LA ACUMULACIÓN INTRACELULAR DE POLVO DE 36.-LOS CUERPOS DE WEIBEL PALADE DE LAS CELULAS
CARBÓN RECIBE EL NOMBRE DE: ANTRACOSIS ENDOTELIALES CONTIENEN: SELECTINA P

22.- ¿A CÚAL DE LOS PIGMENTOS ENDÓGENOS QUE SE 37.- LAS QUIMIOCINAS CX3C (FRACTALINA) SON
PUEDEN ACUMULAR INTRACELULARMENTE SE LE LLAMA AGENTES QUIMIOTÁCTICOS PARA: MONOCITOS Y
“´PIGMENTO DE ENVEJECIMIENTO”? LIPOFUSCINA LINFOCITOS T

23.- SON PROTEINAS QUE AUMENTAN LA LONGEVIDAD 38.- LA LISOZIMA, LACTOFERRINA, PROTEÍNA BÁSICA
POR RESTRICCION CALORICA ACTIVANDO GENES PRINCIPAL Y LAS DEFENSINAS, DESTRUYEN
ESPECIFICOS: SIRTUINAS MICROORGANISMOS POR MECANISMOS:
INDEPENDIENTES DEL OXÍGENO
24.- EL AUMENTO DE LA PERMEABILIDAD VASCULAR DA
COMO RESULTADO: SALIDA DE UN FLUIDO RICO EN 39.- QUIMIOCINA QUE DA LUGAR A ACTIVACIÓN Y
PROTEÍNAS (EXUDADO) QUIMIOTAXIS DE LOS NEUTRÓFILOS: C-X-C (ALFA)

25.- EN EL AUMENTO DE LA PERMEABILIDAD VASCULAR, 40.- LA PROSTACICLINA (PGI2) SE PRODUCE EN LAS


HISTAMINA, OXIDO NITRICO OCASIONAN: RETRACCIÓN CELULAS ENDOTELIALES DE MANERA
ENDOTELIAL CONSTITUTIVAPARA PROVOCAR : VASODILATACION E
INHIBICION DE LA agregación plaquetaria
26.- SELECTINAS, INTEGRINAS Y GLUCOPROTEÍNAS TIPO
MUCINA ACTÚAN COMO: MOLÉCULAS DE ADHESIÓN 41.-MEDIADOR QUIMICO QUE PRODUCE
VASODILATACIÓN Y SU EFECTO ES 10 000 VECES MÁS
27.- ES UNA INMUNOGLOBULINA DE ADHESION QUE
POTENTE QUE LA HISTAMINA: FACTOR ACTIVADOR DE
FAVORECE LA MIGRACION DE LOS LEUCOCITOS A
LAS PLAQUETAS
TRAVES DEL ENDOTELIO: PECAM
42.- MEDIADORES LOCALES DE NUEVA SINTESIS:
EICOSANOIDES
43.- ES UN MEDIADOR QUIMICO QUE TIENE ACTIVIDAD INFLAMATION? HISTAMINE, PROSTAGLANDINS (PGI2,
BACTERICIDA Y PRODUCE VASODILATACION POR PGE, AND PGD2), AND NITRIC OXIDE
RELAJACION DIRECTA DEL MUSCULO LISO ARTERIOLAR:
7.-) FORMA POR LA CUAL LA TELOMERASA INHIBE EL
OXIDO NITRICO
ACORTAMIENTO DE LOS TELOMEROS: AÑADE
44.- ESTUDIO DE GABINETE DE ELECCIÓN PARA EL NUCLEOTIDOS A LOS EXTREMOS DEL CROMOSOMA
DIAGNOSTICO DE COLECISTITIS CRÓNICA LITIASICA:
8.-) ¿CUAL ES LA CARACTERISTICA DEL NUCLEO CELULAR
ULTRASONIDO
EN LA APOPTOSIS: CONDENSACION DE LA CROMATINA
45.- LAS ANAFILOTOXINAS PRODUCEN VASODILATACION
9.-) LA VIA EXTRÍNSECA DE LA APOPTOSIS ACTIVA A:
POR: LIBERACION DE HISTAMINA
CASPASAS 8, 10
46.-ES UNA ANTIPROTEASA QUE INHIBE A LA ELASTASA
10.-) ¿DE LAS CAUSAS DE LESIÓN CELULAR, CUÁL DE LAS
NEUTRA Y ES MAS ABUNDANTE EN TUBO DIGESTIVO Y
SIGUIENTES ES LA QUE AFECTA DE MANERA DIRECTA A
PULMON: ALFA 1 ANTITRIPSINA
LA RESPIRACIÓN OXIDATIVA AÉROBICA EN LA CÉLULA?
47.- BIOPSIA DE ELECCION EN PACIENTES JOVENES CON LA ISQUEMIA / HIPOXIA
LESION ESCAMOSA INTRAEPITELIAL DE ALTO GRADO DEL
11.-) LA TELOMERASA ES UN COMPLEJO: ARN-PROTEINA
CERVIX: CONO CERVICAL
12.-) EN LESIÓN CELULAR REVERSIBLE DEBIDA A
48.-SON PROTEINAS DE LA FASE AGUDA EXCEPTO:
ISQUEMIA /HIPOXIA LA PRIMERA ALTERACIÓN
SINAPTOFISINA
BIOQUÍMICA QUE SE PRESENTA EN: DISMINUCIÓN DE
49.-LA ACTVACION DE LA NADH OXIDASA (FAGOCITO LA FOSFORILACIÓN OXIDATIVA MITOCONDRIAL
OXIDASA) OCASIONA LA FORMACION DE: ESPECIES
13.-) SON FENÓMENOS QUE CARACTERIZAN E INDICAN
REACTIVAS DEL OXIGENO
QUE LA LESIÓN CELULAR ES IRREVERSIBLE: LA
50.-SON CONSTITUYENTES DE LOS GRANULOS DISFUNCIÓN MITOCONDRIAL , PROFUNDOS
AZUROFILOS DE LOS LEUCOCITOS EXCEPTO: TRASTORNOS EN LA FUNCIÓN DE LAS MEMBRANAS
HISTAMINASA Y FOSFATASA ALCALINA CELULARES

14.- )¿CÚALES DE LOS SIGUIENTES SON FACTORES


ANTIOXIDANTES ENZIMÁTICOS? CATALASA, GLUTATIÓN
PEROXIDASA Y SUPERÓXIDO DISMUTASAS
1.-) CAUSA PREDISPONENTE DE LA CALCIFICACION
15.-) LA CALCIFICACION DISTROFICA SE PRESENTA EN:
METASTASICA: MEDIO ALCALINO
ATEROMATOSIS
2.-) LA BIOPSIA DE ELECCIÓN PARA UNA LESIÓN
16.- )ADAPTACIÓN CELULAR DEL CRECIMIENTO QUE
HIPERPIGMENTADA LOCALIZADA EN LA PIEL Y QUE MIDE
PUEDE SER TERRENO FÉRTIL EN DONDE SE INICIE UNA
1CM DE DIÁMETRO, ES: EXCISIONAL
PROLIFERACIÓN MALIGNA: HIPERPLASIA PATOLÓGICA
3.-) EL FIJADOR MÁS UTILIZADO PARA LOS
17.-) SON EFECTOS SISTEMICOS PROVOCADAS POR LA
TEJIDOS OBTENIDOS POR BIOPSIA O NECROPSIA ES:
IL-I, EL FNT EN LA REACCION INFLAMATORIA:
FORMOL AL 10%
A) FIEBRE, PÉRDIDA DEL APETITO , SUEÑO
4.-) ES CAUSA DE LA CALCIFICACIÓN DISTRÓFICA: FOCOS
DE NECROSIS 18.-) LA ATROFIA PUEDE SER: FISIOLÓGICA Y
PATOLÓGICA
5.-) EL APOPTOSOMA ESTA FORMADO POR LAS
SIGUIENTES SUSTANCIAS: CITOCROMO C SE UNE APAF- 19.-) DE LAS REFERIDAS A CONTINUACIÓN, SEÑALE EN
1 QUE MUESTRAS PUEDE REALIZARSE CITOLOGIA
EXFOLIATIVA: ORINA, EXPECTORACIÓN Y LÍQUIDO DE
6.-) WHAT CHEMICAL MEDIATORS ARE RESPONSIBLE
ASCITIS
FOR INCREASING THE BLOOD FLOW TO AN ARE OF
20.-) TIPO DE METAPLASIA MÁS FRECUENTE: DE 33.-) SON PRODUCTOS DEL METABOLISMO DEL ÁCIDO
EPITELIO CILÍNDRÍCO A EPITELIO PLANO ARAQUIDÓNICO POR LA VÍA DE LA CICLOOXIGENASA:
ESTRATIFICADO TROMBOXANO Y PROSTAGLANDINAS

21.-) LA ACUMULACIÓN INTRACELULAR DE POLVO DE 34.-) GRUPO DE MEDIADORES QUIMICOS QUE


CARBÓN RECIBE EL NOMBRE DE: ANTRACOSIS PRODUCEN INHIBICION DE LA QUIMIOTAXIS DE
NEUTROFILOS Y LA ADHERENCIA DE LOS MONOCITOS:
22.-) ¿A CÚAL DE LOS PIGMENTOS ENDÓGENOS QUE SE
LIPOXINAS
PUEDEN ACUMULAR INTRACELULARMENTE SE LE LLAMA
“´PIGMENTO DE ENVEJECIMIENTO”? LIPOFUSCINA 35.-) BRADYKININ IS PRODUCED FROM A HIGH-
MOLECULAR WEIGHT KINOINOGEN CIRCULATING IN
23.-) SON COMPONENTES DE LA INFLAMACIÓN AGUDA:
THE BLOOD. THIS REACTION IS MEDIATED BY:
MODIFICACIÓN EN EL CALIBRE DE LOS VASOS Y EN LA
KALLIKREIN
ESTRUCTURA DE LA MICROVASCULATURA
36.-) ESTRUCTURA FORMADA POR UN ACUMULO DE
24.-)EL AUMENTO DE LA PERMEABILIDAD VASCULAR DA
CÉLULAS EPITELIOIDES, RODEADAS POR UN COLLAR DE
COMO RESULTADO: SALIDA DE UN FLUIDO RICO EN
LINFOCITOS Y CÉLULAS PLASMÁTICAS SE PRESENTA EN:
CELULAS Y PROTEÍNAS AL INTERSTICIO
BRUCELOSIS
25.-) SELECTINAS, INTEGRINAS Y GLUCOPROTEÍNAS TIPO
37.-) QUIMIOCINA QUE DA LUGAR A ACTIVACIÓN Y
MUCINA ACTÚAN COMO: MOLÉCULAS DE ADHESIÓN
QUIMIOTAXIS DE LOS NEUTROFILOS, CON ACTIVIDAD
26-) A 60-YEAR-OLD OBESE MAN WAS ADMITTED TO LIMITADA SOBRE MONOCITOS Y EOSINOFILOS: C-X-C O
THE HOSPITAL FOR TREATMENT OF ALCOHOLISM. HE ALFA
HAS DIABETES MELLITUS. A LIVER BIOPSY WAS
38.-) PROSTAGLANDINA QUE PRODUCE HIPERALGESIA,
PERFORMED, AND THE SPECIMEN SHOWED THAT THE
QUE HACE QUE LA PIEL PRESENTE HIPERSENSIBILIDAD
LIVER CELLS CONTAIN INCREASED AMOUNTS OF:
(DOLOR Y FIEBRE): PGE2
TRIGLYCERIDES
39.-) SISTEMA BACTERICIDA MAS EFICAS DE LOS
27.-) PEROXIDACIÓN DE LOS LÍPIDOS DE LA MEMBRANA,
NEUTROFILOS: H2O2 + MPO + Cl
MODIFICACIÓN OXIDATIVA DE LAS PROTEÍNAS Y
LESIONES DEL DNA SON EFECTOS RELEVANTES PARA LA 40.-) NITRIC OXIDE SYNTHESIS IS AUGMENTED IN
LESIÓN CELULAR CAUSADAS POR: RADICALES LIBRES ENDOTHELIAL CELLS BY A CALMODULIN MEDIATED
DERIVADOS DEL OXÍGENO INFLUX OF WHICH ELEMENT? CALCIUM

28.-) LOS ANTICUERPOS DE LA IGG, C3B Y LAS 41.-) PUS O MATERIAL PURULENTO SE ENCUENTRA
COLECTINAS FUNCIONAN COMO: OPSONINAS FORMADO POR: DETRITUS CELULARES, NEUTRÓFILOS Y
LIQUIDO DE EDEMA
29.-) PRINCIPALES QUIMIOTACTICOS ENDOGENOS: C5A
Y LTB4 42.-)LA CAUSAS MÁS FRECUENTES DE LA ESTEATOSIS
HEPATICA SON: ALCOHOLISMO, DESNUTRICIÓN
30.) A 60-YEAR-OLD MAN WHO HAD GENERALIZED
ATHEROSCLEROSIS DIED 24 HOURS AFTER HAVING A 43.-) SON EJEMPLOS DE INFLAMACIÓN SEROSA:
STROKE. A CEREBRAL INFARCT WAS FOUND AT APENDICITIS SUPURATIVA AGUDA, FURUNCULOSIS
AUTOPSY. NECROSIS OF THE BRAIN IS CLASSIFIED AS:
44.-) MEDIADOR QUIMICO QUE PRODUCE
LIQUEFACTIVE NECROSIS
VASODILATACION Y SU EFECTO ES 10.000 VECES MAS
31,.) LAS QUIMIOCINAS CX3C SON MOLÉCULAS DE POTENTE QUE LA HISTAMINA: FACTOR ACTIVADOR DE
ADHESIÓN PARA: MONOCITOS Y LINFOCITOS T LAS PLAQUETAS

32.-) LOS CUERPOS DE RUSSELL SON ACUMULOS DE: 45.-)FACTOR DEL COMPLEMENTO C5a SE ACTIVA POR
INMUNOGLOBULINAS EN CÉLULAS PLASMÁTICAS LA VIA ¿ ? PARA ACTIVAR NEUTROFILOS Y MONICITOS
PARA AUMENTAR LA LIBERACION DE MAS MEDIADORES
INFLAMATORIOS: VIA DE LECTINA
46.-) FACTOR DE CRECIMIEMTO RELACIONADO CON LA
HIPERTROFIA POR AUMENTO DE LA CARGA DE
TRABAJO: TGF BETA

47.-) LA ATROFIA SE PRODUCE POR: AUMENTO EN LA


DEGRADACION DE PROTEINAS

48.-) EL RECEPTOR DE MUERTE MAS CONOCIDO EN LA


APOPTOSIS SE LLAMA: FAS

49.-) SON PROTEINAS PLASMATICAS DE LA FASE AGUDA


EN LA INFLAMACION: PROTEINA C REACTIVA,
FIBRINOGENO PROTEINA AMILOIDE

50.-) LOS CUERPOS DE MALLORY SON ACUMULOS DE:


PROTEINAS HIALINO ALCOHOLICAS EN HIGADO
Preguntas Anatomía Patológica

1. Deficiencia de la glucoproteína GPIB provoca la enfermedad:


● Bernad – Soslier (SRS) ó Distrofia trombocitopenica hemorrágica.

Se afecta la serie megacariocitica o plaquetaria y provoca un Sx hemorrágico ↓Plaquetas


en gran tamaño. Autosómica recesivo GPIb-V-IX

2. Factores de crecimiento implicados en la síntesis de tejido conjuntivo, excepto:


● PDGF SI
● FGF-2 SI
● TGF-ᵦ SI
3. Son proteínas compuestas por 3 cadenas polipeptídicas distintas trenzadas en triple hélice a
modo de cuerda
● Colágeno → Se han identificado 30 tipos de colágeno Fibrilares 1, 2, 3 y 5
4. Son neoplasias que tienen peor diferenciación o totalmente indiferenciadas, se refiere al
termino de:
● Anaplasia → Falta de diferenciación
5. Corresponde al evento principal de la hemostasia primaria:
● Formación del tapón plaquetario.
6. La estadificacion de los canceres solidos se basa en que sistema:
● Sistema TNM
● T → Tumor primario
● N → Afectacion ganglios linfáticos
● M → Metastasis
7. ¿Cuál de las siguientes causas es más probable que se acumule el líquido, si existe variación?
● Permeabilidad vascular → P/e: Inflamación, puede afectar a todo el organismo.
8. Neoplasia maligna de las células de los ductos seminales:
● Seminoma → Tumores de células germinales
9. Un estado inflamatorio crónico puede causar anemia ferropénica en respuesta al:
● Aumento de la Hepcidina → En el caso de enfermedades como anemia de tipo
inflamatoria, se da una alta produccion de moléculas proinflamatorias, elevando
niveles de hepcidina, lo que conlleva a una disminución en la absorción de Fe⁺
duodenal.
10. Su crecimiento se acompañan infiltración, invasión y destrucción progresiva del tejido
circulante, están poco delimitados y son de lenta expansión:
● Neoplasia maligna
11. Fase de shock en la que se activa el SRAA:
● Fase no progresiva → Activacion de mecanismos compensadores reflejos y se
mantiene la perfusión de los órganos vitales. Mecanismos neurohumorales ayudan a
mantener el GC Y PA. Reflejos barorreceptores, liberación de catecolaminas y ADH
activación de SRA.
12. ¿Cuáles son las características del tejido de granulación?
● Migración y proliferación de fibroblastos y el depósito de tejido conjuntivo laxo, junto
con los vasos y leucocitos entremezclados. Apariencia macro → Rosada, blanda y
granular.
13. Fase de shock donde fracasan mecanismos reflejos de compensación disminuye la perfusión
de organismos vitales:
● Fase progresiva
14. Vía más frecuente para la diseminación inicial de los carcinomas:
● Diseminación linfática
15. Es el colágeno que proporciona fibrinas de anclaje de las células epiteliales a la membrana
basal:
● Colágenos no fibrilares → Tipo IV
16. La falta de diferenciación, es decir, implica una inversión de la diferenciación hacia un plano
más primitivo:
● Anaplasia
17. Estos trombos se forman en la circulación venosa suelen contener más eritrocitos
incorporados y relativamente pocas plaquetas:
● Trombos rojos o venosos.
18. ¿Cuáles son las fases de la reparación de heridas?
● Angiogenia → Formación de tejido de granulación y remodelación de tejido
conjuntivo.
19. Es el resultado de múltiples mutaciones que se acumulan de forma independiente en
diferentes células y contribuyen así a la generación de subclones con características diferentes
es a lo que se le conoce como:
● Progresión tumoral
20. Patología donde las personas que sufren mutaciones en el gen TP53 y que tienen una
posibilidad 25 veces mayor de experimentar una neoplasia maligna antes de los 50 años que
la población general:
● Li-Fraumeni → Heredan una copia defectuosa de TP53
21. Si la alteración displásica es intensa y afecta a todo el espesor del epitelio, pero la lesión no
penetra a la membrana basal, se habla de:
● Carcinoma in situ
22. Los tipos de VPH de alto riesgo producen proteínas oncógenas que ocasionan lo siguiente
excepto:
● Inmunodeficiencia
23. El gen APC, inhibidor de las vías de señalización mitogena, se ha relacionado con las siguientes:
● Cáncer estómago, colon, páncreas y melanoma.
24. Es una característica del síndrome de Trousseau asociado a carcinomas de páncreas y
estomago se refiere a:
● Tromboflebitis migratoria → Coagulo patia adquirida, asociada a cáncer.
25. ¿Qué tan resistentes son las células musculares cardiacas a la hipoxia?
● Bastante sensibles mueren a los 20-30 min. Vulnerabilidad de hipoxia, neuronas
sufren lesión irreversible después de 3-4 min.
26. Son patologías que producen edema por aumento de la presión hidrostática del vaso:
● Pericarditis constrictiva y trombosis.
27. Se conoce así cuando el tejido cicatrizal crece más allá de los límites de la herida original,
además no se contrae:
● Cicatriz queloide
● (Cicatriz hipertrófica: Acumulación excesiva colágeno.
28. Factores sistémicos que influyen en la curación de heridas:
● Nutrición, situación circulatoria, estado metabólico y hormonas.
29. Marcador tumoral relacionado con el cáncer prostático:
● PSA → Las mediciones de PSA sérico se emplea con profusión en el diagnóstico y
tratamiento de cáncer de próstata. Es un producto del epitelio de la próstata y se
segrega normalmente en semen.
30. Morfológicamente las neoplasias malignas se caracterizan por:
● Pleomorfismo nuclear y celular.
31. Dentro de las funciones de P53 se encuentran, excepto:
● Inducir la proliferación celular.
● Funciones → Controla la expresión y actividad de proteínas que intervienen en la
detección del ciclo celular, reparación ADN, senescencia celular y apoptosis.
32. El cáncer es una enfermedad caracterizada por:
● Adquisición de mutaciones subletales.
33. Se puede producir choque por:
● Todas las anteriores (Sepsis, infarto, quemadura).
34. Se observa tras una destrucción sustancial de tejido, cuando la lesión inflamatoria afecta a
tejidos que no son capaces de regenerarse o cuando hay abundante exudación de fibrina en
tejidos o cavidades serosas, que no es posible eliminar adecuadamente:
● Curación por reposición de tejido conjuntivo.
35. El gen 9, 12 se asocia con:
● Cromosoma filadelfia.
36. Características de las señales sinópticas:
● Secreción de neurotransmisores en uniones celulares especiales.
37. Neoplasia maligna de la glándula tiroides:
● Adenocarcinoma
38. Se consideran con extensión de las neoplasias a sitios anatómicos separados del lugar de
origen:
● Metástasis.
39. Son sustancias que inhiben la función plaquetaria:
● Prostaciclina y ADPASA.
40. El gen RB inhibidor de la progresión celular, se ha relacionado con las siguientes neoplasias:
● Retinoblastoma, osteosarcoma, carcinoma de mama, colon y pulmón.
41. Trombo formado en cavidades cardiacas:
● Trombo mural

42. Tienen mayores probabilidades de sufrir infarto rojo excepto: RIÑÓN.


● Los infartos rojos se observan en: Oclusiones venosas P/e: Torsión ovárica, tejidos
laxos, tejidos con doble circulación → Pulmón, intestino delgado. Tejidos previamente
congestionados debido a un retorno venoso lento, sitios de oclusión y necrosis previa
cuando se restablece el flujo.
43. Es la inmortalidad de las células malignas es considerado como factor decisivo:
● Evasión de la senescencia.
44. Se caracterizan por degradar la MEC y permite la remodelación y extensión del tubo vascular:
● MPM metaloproteinasas de matriz → Las enzimas presentes en la MEC en especial las
metaloproteinasas de matriz (MMP) degradan la MEC a fin de permitirla
remodelación y la extensión de los conductos vasculares.
45. Células que están presentes en el tejido conectivo contiguos vasos sanguíneos por debajo de
superficies mucosas de vías respiratorias, la unión de antígeno alérgeno a anticuerpo IgE, se
refiere a:
● Mastocitos.
46. Son trastornos adquiridos predisponentes a cáncer:
● Se dividen en inflamaciones crónicas, lesiones precursores y estados de
inmunodeficiencia.
47. ¿Cuál enunciado acerca de las plaquetas es verdad?
● Precursor → Megacariocitos, gránulos α adhesión a selectiva P, proteínas de la
coagulación, fibrinógeno, factor V y Vwf.
48. Este tipo de células están presentes en la inflamación crónica, son abundantes en las
reacciones inmunitarias mediadas por inmunoglobulinas E y en infecciones parasitarias:
● Eosinófilos
49. ¿En qué se diferencia la hiperemia de la congestión?
● Hiperemia → Es un proceso activo en el que la dilatación arteriolar provoca un
aumento del flujo sanguíneo, los tejidos afectados se tornan rojos por el mayor aporte
de sangre oxigenada y la congestión es un proceso pasivo secundario a una menor
salida de sangre de un tejido. P/e: IC, OV.
50. El edema que ocurre en la filariasis se debe:
● Bloqueo circulación linfática.
51. Todo lo siguiente se pueden esperar en la congestión hepática excepto:
● Incremento de peso
52. Contienen una mayor cantidad de sangre oxigenada se caracteriza por el color rojo brillante y
dilatación de las arteriolas:
● Hiperemia activa
53. Neoplasia maligna del tejido adiposo:
● Liposarcoma
54. Neoplasia maligna del epitelio placentario:
● Coriocarcinoma

55. Es la cicatrización por primera intención:


● En qué tiempo la neovascularizacion alcanza su máximo y el tejido de granulación
llena la incisión.
● La reparación comienza en un plazo de 24 hrs desde la lesión. Transcurridos 3-5 días,
el tejido de granulación aparece.
56. La extravasación del líquido al intersticio cual es el término que se utilizaría:
● Edema
57. La estadificación de un tumor maligno se requiere:
● Tamaño de la lesión primaria y si ha realizado metástasis.
58. La congestión pasiva crónica del pulmón puede incluir todo lo siguiente excepto:
● Induración café, causada a menudo por insuficiencia cardiaca congestiva, paredes
engrosadas y fibroticas y los alveolos suelen contener numerosos macrófagos
cargadas de hemosiderina denominados células de la insuficiencia cardiaca.
59. El efecto Warburg consiste en:
● La neoplasia maligna fija la glucolisis anaerobia en su metabolismo.
60. La inactivación o deleccion de p16 adquirida por vía somática se ve en las familias propensas
a:
● Carcinoma de páncreas, glioblastomas y cáncer de esófago.
61. Neoplasias malignas de los vasos sanguíneos:
● Angiosarcoma.
62. Son estados de hipercoagulabilidad de bajo riesgo, excepto:
● Bajo riesgo → Miocardiopatia, Sx nefrótico, estados hiperestrogenicos, gestación y
posparto, uso de anticonceptivos orales, drepanocitosis y tabaquismo.
63. ¿Cuál es la definición de teratoma?
● Que se origine de más de 2 capas germinales (a veces las 3)
64. ¿Los estados de hipercoagulabilidad se dividen en?
● Primarios (genéticos) y secundarios (adquiridos).
65. El shock cardiogénico se produce cuando existe una pérdida de la masa muscular del
ventrículo izquierdo del:
● 45%
66. Una de las razones por las cuales las células neoplásicas son importantes:
● Potencial ilimitado de replicación. Evitan la senescencia celular y la catástrofe
mitótica.
67. Son acontecimientos plaquetarios excepto:
● Si → Adhesión, cambio de forma, secreción y agregación
● No → Migración
68. El efecto Warburg consiste en:
● Las células tumorales sufren una conmutación metabolica hacia la glucolisis aerobica
que facilita la síntesis de las macromoléculas y orgánulos requerida para un
crecimiento celular rápido.
69. Una lesión endotelial tendrá como consecuencia:
● Desarrollo de un ambiente procoagulante.
70. La mayor parte de la flebotrombosis se produce en:
● Venas de las extremidades inferiores >90%
71. Es el término que se aplica a la neoplasia epitelial benigna de rivada de las glándulas:
● Adenoma
72. La luz UV:
● Puede causar carcinoma basocelular (factor ambiental).
73. El virus de Epstein Barr produce:
● Mononucleosis infecciosa/ transtorno linfoproliferativo benigno/ patogenia de
diversos tumores/ linfomas/ carcinoma nasofaríngeo. Linfoma de Burkitt.
74. Es una causa de edema por disminución de la presión oncotica:
● Causas → Glomerulopatias con pérdida de proteínas (Sx nefrótico), Cirrosis hepática,
Malnutricion, Gastroenteropatia con pérdida de proteínas.
75. Procesos en la evolución de un trombo:
● Propagación, embolia, disolución y organización, y recanalización.
76. Especifique las variables que alteran la cicatrización:
● Infección, diabetes , estado nutricional, factores mecánicos, mala perfusión, tipo,
alcance y localización de la lesión.
77. Se media por las interacciones con el factor de Von Willebrand con la glucoproteína IB,
receptor de la superficie de las plaquetas y el colágeno expuesto:
● Adhesión plaquetaria.
78. ¿Qué gen supresor de tumores intervienen en la patogenia del carcinoma de mama?
● BCRA1 – BCRA2
79. En el estado conocido como shock séptico:
● Es producido por el paso de bacterias, hongos al torrente sanguíneo.
80. El edema por obstrucción linfática se da por las siguientes condiciones excepto:
● No → Postquimioterapia, inflamatoria, neoplásica, posquirúrgica.
● Si → Postirradiación.
81. Es un factor de crecimiento con efectos mitógenos sobre los hepatocitos:
● Factor de crecimiento de dispersión.
82. Su crecimiento se acompañan de infiltración, invasión y destrucción progresiva del tejido
circundante, están poco delimitados y son de lenta expansión:
● Neoplasia maligna
83. ¿Cuál de los siguientes mediadores causan vaso constricción y broncoconstricción?
● Tromboxano A₂
84. Los infartos pálidos se observan en:
● Órganos sólidos, corazón, bazo y riñón, circulaciones arteriales terminales con muy
pocas colaterales.
85. Son tumores constituidos por células inmaduras que se parecen a las que forman tejido
rudimentario fetal:
● Blastoma.
86. Los tumores más comunes en hombres se originan en:
● Próstata, pulmones, colon y recto.
87. La falta de diferenciación, es decir implica una inversión de la diferenciación hacia un plano
más primitivo:
● Anaplasia.

88. Mutaciones que contribuyen al fenotipo maligno se denomina:


● Mutaciones conductoras.
89. Es un ejemplo de edema localizando el que ocurre en casos de:
● Cirrosis hepática.
90. El choque séptico puede producir:
● Vasodilatación periférica y acumulación de sangre, activación, lesión endotelial, lesión
inducida por leucocitos, coagulación intravascular diseminada, activación de la
cascada de citosinas.
● Daño alveolar difuso.
91. ¿Cuáles son las características del tejido de granulación?
● Proliferación de fibroblastos y nuevos capilares, finos y pared delgada (Angiogenia)
● MEC laxa mezclada con macrófagos.
92. Un embolo gaseoso puede originarse:
● Por cambios bruscos en la presión.
93. La bipedestación provoca edema en las partes declives del organismo ¿Cuál de los siguientes
mecanismos explica mejor este fenómeno?
● Edema dependiente clásico en ICC.
● Aumento de la presión hidrostática
94. Se determina mediante la exploración quirúrgica o con técnicas de imagen, y dependen del
tamaño, la propagación a los ganglios linfáticos locales y regionales, y la presencia de
metástasis remota:
● Estatificación
95. Son ejemplos de shock cardiogénico:
● Infarto de miocardio, rotura ventricular, arritmia, taponamiento cardiaco, embolismo
pulmonar.
96. Describe el deposito extenso de colágeno que se registra en pulmones, hígado, riñón y otros
órganos como consecuencia de inflamación crónica.
● FIBROSIS EN ORGANOS PARENQUIMATOSOS
97. Tiene mayor probabilidad de infarto rojo:
● Oclusiones venosas → Torsion ovárico, tejidos laxos.
● Tejidos con doble circulación → Pulmon, intestino delgado.
● Tejidos congestionados previamente debido a un retorno venoso lento.
● Sitios de oclusión y necrosis previa cuando se resuelve el flujo.
98. Neoplasia benigna del nervio periférico:
● Schwannoma/Neurilemioma
99. Una de las maneras en las que un tumor escapa de la respuesta inmune es:
● Proliferacion selectiva de variantes sin antígenos, perdida o disminución de la
expresión de antígenos de histocompatibilidad y la inmunodepresión mediada por
TGF-ᵦ, ligando PD-1, galectinas. Los anticuerpos vencen estos mecanismos de
inmunoevasion.
100. La llamada “induración parda” es un ejemplo:
● Hiperemia, Congestión, Pulmón.
101. Estado inflamatorio crónico asociado a cáncer:
● Enfermedad inflamatoria intestinal, pancreatitis, colecistitis, esofagitis, gastriris,
hepatitis, cistitis.

102. Estado inflamatorio crónico que puede causar anemia ferropénica en respecto al:
● La anemia de la enfermedad crónica es la forma más adecuada de llamarla “Anemia
de la inflamación crónica”. Se debe en parte a los mediadores inflamatorios que
incrementan la producción hepática de hepcidina.
103. La edad, influye de manera determinante en el riesgo de cáncer por lo tanto, la
mayoría de los carcinomas se presentan en la edad de:
● >55 años
● Mujeres 40 – 79 años
● Hombres 60 – 79 años
104. Característica histológica predominante en un infarto es:
● Necrosis coagulativa isquémica.
105. Sustancia generada por sistema de cininas, cuando hay daño vascular, promueve
contracción de musculo liso, aparición de dolor y promueve la permeabilidad vascular:
● Bradicinina.
106. La asbestosis y silicosis está asociada con qué tipo de neoplasia:
● Mesotelinoma, carcinoma de pulmón. Agente etiológico → Fibras de amianto,
partículas de sílice.
107. Es considerado el guardián del genoma:
● Proteína p53 vigilante fundamental del estrés celular.
108. En la cicatrización por primera intención ¿Cuándo se comienza a observar el tejido de
granulación?
● 3 días
109. En qué etapa de la curación de heridas cutáneas se forman el tejido de granulación,
mismo que predice la cicatrización y recuperación:
● 5 días
110. Es un cancerígeno químico de origen natural:
● Asbesto, Arsénico, Benceno.
111. Se refiere así, a la pérdida progresiva de la grasa corporal y de la masa corporal magra,
acompañada de una debilidad profunda, anorexia y anemia:
● Caquexia
112. Las proteínas E6 y E7 del VPH de alto riesgo inhiben los genes:
● Se unen a la Rb y la P53.
113. La graduación histopronostica de una neoplasia maligna se refiere a:
● Se basa en el grado de diferenciación de las células tumorales y en algunos casos el
número de mitosis o rasgos arquitecturales. Aspecto citológico y se basa en la idea de
que el comportamiento y la diferenciación guarden relación entre sí.
114. Enfermedad por descompresión con la aparición de burbujas de nitrógeno:
● Síndrome de descompresión
115. El restablecimiento de la estructura tisular normal solo se produce maduración de las
células madre adultas:
● Regeneración

116. Marcador tumoral de tipo hormonal:


● Gonadotropina coriónica humana → Tumores trofoblasticos, testiculares no
semimocromatosos.
● Calcitonina → Carcinoma medular tiroides
● Catecolaminas y metabolitos → Feacromacitoma y tumores relacionados.
● Hormonas ectópicas
117. Los estados de hipercoagulabilidad se divide en:
● d
● Trastornos secundarios (adquiridos)
118. El carcinoma epidermoide de pulmón, carcinoma de mama, carcinoma de células
renales y la leucemia. Linfoma de linfocitos T en el adulto se han relacionado con el siguiente
síndrome paraneoplasico:
● Hipercalcemia
● Mecanismo causal → Proteína relacionado con la hormona paratiroidea, (PTMRP) TGF
– α TNF, IL-1.
119. Características de la congestión pulmonar crónica:
● Causada a menudo por insuficiencia cardiaca congestiva, las paredes están
engrosadas y fibróticas y los alveolos suelen contener numerosos macrófagos
cargados de hemosiderina denominados células de la insuficiencia cardiaca.
120. Características microscópicas importantes del leiomioma, también llamado mioma:
● Neoplasia benigna de musculo liso, cariotipo normal, 40% anomalía cromosómica
simple. Cromosomas 12q 14 y 6p, genes HMGIC y HMGIY. MED12 → 70%. Redondos,
color blanco/ gris, patrón en espiral, aspecto arremolinado del musculo.
121. Factor de crecimiento que contribuye a la reparación de heridas oncogenia,
hematopoyesis y desarrollo de musculo cardiaco y esquelético, maduración del pulmón y
especificación del hígado:
● Factor de crecimiento fibrostatico
122. Ejemplos del grupo patológico de edema por disminución de la presión osmótica:
● Desnutricion proteica, KwashiorKor.
123. Interactua con un receptor TIE2 para reclutar células periendoteliales:
● Angiopoyetina tipo 1
124. Color azul verdoso que adquieren los hematomas en la fase de resolución:
● Hemoglobina a hemosiderina
125. Coagulo constituido por agregación plaquetaria y depósito de fibrina:
● Tapón hemostático secundario
126. Alteración de escasa frecuencia que se debe a una proliferación exuberante de
fibroblastos y otros elementos de tejido conjuntivo:
● Lesiones desmoides
127. Proceso de remodelación de la reparación de las heridas participan enzimas:
● Metaloproteinasas y gelatinasas
128. La elevación de alfa feto proteína se eleva a normalmente en:
● Cáncer primario de hígado, células germinales de testículo.
129. La citometria de flujo puede medir:
● Antígenos de membrana, contenido de ADN en células tumorales
130. Leiomas de acuerdo a su localización en el cuerpo uterino se clasifica en:
● Submucosos, subserosos e intramurales.
131. En el fibroadenoma mamario se caracteriza sobre todo los siguiente excepto:
● Esta fijo a planos profundos y no se puede delimitar su tamaño
132. Tipos de embolias excepto:
● Embolia linfática
133. Sustancias con actividad antitrombotica:
● Plasminogeno tisular y prostaciclina endotelial
134. Patología en la que se pierde albumina y genera edema:
● Sx Nefrotico
135. Hematoma mayor a 2cm:
● Equimosis
136. Causa fisiológica de hiperemia:
● Ejercicio
137. Factor que inhibe la diseminación del trombo:
● Activador tisular de plasminógeno
138. Capacidad de cambiar de un tipo celular a otro:
● Transdiferenciación
139. En esta fase de shock hay liberación de ADH y activación de baroreceptores:
● Fase inicial progresiva
140. Manifestaciones clínicas presentes en shock séptico:
● Piel caliente, vasodilatación periférica, hiperemia
141. Casi siempre es oclusiva puente largo de la luz del vaso:
● Trombosis venosa profunda
142. Síntomas embolia grasa
● Taquicardia, bradicardia y disnea.
143. Una neoplasia completa se divide como:
● Dos capas celulares y una capa embrionaria
144. Metástasis:
● Implantes tumorales sin continuidad con el tumor primario
145. La inhibición de TP53 es mediada por:
● BAX
146. Fase de la cascada metatásica:
● Invasión de la MEC, diseminación vascular, alojamiento y colonización
147. ¿Cuáles son los órganos más afectados por la diseminación hematógena?
● Hígado y pulmón
148. ¿Qué virus ARN tiene sobre actividad oncogénica?
● Leucemia y células T
149. Carcinógeno natural hepático secretado por planes y hongos:
● Aflatoxina B1
150. Causa más frecuente de cáncer renal:
● Tabaquismo
151. Causas de alteración del retorno venoso
● Insuficiencia cardiaca congestiva, obstrucción y compresión venosa.
152. La asbestosis y silicosis está asociada con qué tipo de neoplasia:
● Mesotelioma, carcinoma de pulmón
153. Es el cambio biológico necesario para la progresión de las neoplasias malignas:
● Angiogenia
154. Etapa caracterizada por la hipoperfusión tisular y el inicio del empeoramiento
circulatorio y de los desequilibrios metabólicos incluyendo acidosis láctica:
● Fase progresiva
155. Un cistoadenoma corresponde con:
● Neoplasia Benigna quística glandular
156. Pertenece a una familia de factores de crecimiento con más de 20 miembros:
● FCF
157. Esta vía de diseminación es bastante característica de los carcinomas de ovario ya que
frecuentemente se diseminan por la superficie peritoneal:
● Siembra de cavidades y superficies corporales
158. Se produce por la hipoxia provoca la angiogenia y promueve la migración de células
endoteliales
● VEGF
159. ¿Cuál es la proteína más abundante en el ser humano?
● Colágeno
160. International normalized ratio (INR) valora principalmente en el laboratorio:
● Anticoagulación oral
161. Responden por lo común al tratamiento con anticuerpos o fármacos que bloquean la
actividad HER2:
● Cáncer de mama
162. La activación del activador de plasminógeno genera su conversión a plasmina para:
● Iniciar la cascada fibrinolítica
163. Estos trombos son más frecuentes en arterias coronarias cerebrales y femorales
conformadas por plaquetas, fibrina, eritrocitos y leucocitos generados:
● Trombos arteriales
164. Permiten determinar el diagnostico, pronostico, detección de enfermedad residual
mínima y el diagnóstico de la predisposición hereditaria al cáncer:
● Análisis molecular
165. Vía de la apoptosis que más se desregula en situación de cáncer:
● Vía intrínseca (vía mitocondrial)
166. Clínicamente el proceso de fibrosis es común en:
● Órganos parenquimatosos
167. ¿Qué proteína fibrilar suele regenerar mal en el tejido cicatricial?
● Colágeno
168. Factor de crecimiento quimiotáctico para neutrófilos, macrófagos, células musculo
liso, estimula la proliferación de fibroblastos y células endoteliales:
● Factor de crecimiento epidérmico
169. Es el componente proteico de la MEC encargado de dar resistencia a la tensión:
● Colágeno

170. Es el termino aplicado a un resto heterotopico de células p/e: Un nódulo de tejido


pancreático perfectamente desarrollado y con una organización normal en la submucosa
estomago:
● Coristoma
171. Factor de crecimiento placentario, pertenece a la familia de:
● FCVE ó VEGF
172. Son características de la curación por segunda intención:
● La reacción inflamatoria es más intensa y mayor cantidad de tejido de granulación
173. En la regeneración hepática, la progresión de los hepatocitos del estadio G1 a S
comienza con la formación del complejo:
● Ciclina D – CDK4
174. Trastornos adquiridos predisponentes de cáncer:
● Todas las anteriores → Mayor replicación celular, inmunodeficiencia, inflamación
crónica.
175. Se denomina sarcoma a:
● Neoplasia maligna de tejidos mesenquimatosos
176. El gen CDH1inhibidor de invasión y metástasis se ha relacionado con los siguientes
neoplasias:
● Carcinoma gástrico, carcinoma lobulillar familiar.
177. Factor que convierte el fibrinógeno a fibrina, activa el endotelio a linfocitos,
monocitos y produce agregación plaquetaria:
● Trombina
178. Principal familia de proteínas de adhesión, encargadas de la interacción con la MEC:
● Integrinas
179. Son neoplasias malignas su origen derivado del tejido conectivo o conjuntivo:
● Sarcoma
180. El tiempo parcial de tromboplastina (ITP) evalúa:
● Vía intrínseca (II,V, VIII, IX,X, XI, XII)
181. La técnica de inmunohistoquimica sirve para:
● Sirve para caracterizar tumores indiferenciados
182. Molécula de adhesión que contienen los gránulos de las plaquetas:
● Selectina P
183. Son ejemplos del grupo fisiopatológico de edema por disminución de presión
oncótica:
● Desnutrición, kwashiorkor y cirrosis hepática
184. El factor de Von Willebrand:
● Se encuentra en el tejido endotelial
185. La deficiencia de vitamina C retrasa la curación de heridas por:
● Inhibe síntesis de colágeno
186. El edema de origen renal se caracteriza por:
● Presentarse en tejido laxo
187. Las hemorragias diminutas de 1-2 mm se llaman:
● Petequias

188. El tiempo de protrombina evalúa la vía extrínseca de la coagulación que factores


comprende:
● VIII – X – V y Fibrinógeno
189. Son funciones de la trombina excepto:
● Secreción prostaciclinas
190. En los tejidos lábiles el tipo de reparación es:
● Regeneración
191. En una célula de reposo la proteína APC:
● Inhibe la proliferación por degradación de BETA CATENINA
192. Los agentes alquilantes son considerados:
● Carcinógenos acción directa
193. Dentro de las funciones de P53 se encuentran, excepto:
● Inducir proliferación celular
194. La trombositopenia y anemia pueden manifestarse clínicamente con:
● Petequias
195. Características generales de las neoplasias malignas:
● No presentan capsula, están poco diferenciadas, invasivas
196. En qué tiempo las neuronas sufren daños irreversibles ante la hipoxia:
● 3 – 4 min
197. H. Pylori se relaciona con:
● Adenocarcinoma gástrico
198. Causas del edema de pulmón:
● Insuficiencia del ventrículo izquierdo, insuficiencia renal e infecciones pulmonares
199. Neoplasia maligna de hueso:
● Osteosarcoma
200. Son genes celulares normales cuyos productos fomentan la proliferación celular:
● Protooncogen
201. Son tumores de origen terminal:
● Teratoma
202. Los gránulos densos de las plaquetas contienen excepto:
● Factor de Von Willebrand
203. La gastritis y las ulceras están relacionadas con qué tipo de neoplasias:
● Adenocarcinoma gástrico, linfoma MALT
204. Mecanismo observado en el edema secundario a retorno venoso alterado:
● Retención de Na⁺
205. Es un carcinógeno producido por el aspergillus flavus:
● Aflatoxina
206. Neoplasia maligna de tejido linfoide:
● Linfagiosarcoma
207. Son ejemplos clínicos de choque asociado a inflamación sistémica:
● Shock séptico - infecciones microbianas devastadoras y pancreatitis
208. ¿Cuál es el agente biológico implicado en el linfoma de Burkit?
● Epstein-Barr

209. Una de las maneras en las que un tumor escapa de la respuesta inmune es:
● Sobre expresan moléculas CPH1
210. No siempre responden a las moléculas que inhiben la proliferación de células
normales, habitualmente por la inactivación de genes supresores de tumores que modifican
componentes de estas vías inhibitorias del crecimiento:
● Evasión Apoptosis
211. Fin, vuelva pronto:D
Es un tipo especializado de tejido, característico de la curación, de aspecto rosado, blanco y de
apariencia granular en la superficie de las heridas:

a) Tejjido conectivo
b) Tejido epitelial
c) Tejido mesenquimatoso
d) Tejido conjuntivo
e) Tejido de degranulacion

Son carcinógenos de acción directa:

f) Hidrocarburos aromáticos
g) Aflatoxina B
h) Cloruro de vinilo
i) Griseofulvina
j) Agentes alquilantes y acilantes

A 20 year old woman suffered extensive burns and developed large irregular scars over her hans.
These scars limite the moviment of her fingers, and she unable to fully extend them. What are these
lesión called?

a) Desmoids
b) Aggressive fibromatoses
c) Cntractures
d) Keloids
e) Wound dehiscences

Tambien conocido como factor de dispersión:

a) FGF
b) VEGF
c) HGF
d) TGF-a
e) PDGF

Which the following is the most common site for arteriolar thromboembolization?

a) Brain
b) Heart
c) Kidneys
d) Lower extremities
e) Eyes

Citocinas que participan en la curación de las heridas:

a) Il-2 e il-4
b) Il-6 e il-1
c) Il-12 e il-2
d) INF e Il-1
e) TNF e Il-6
En la biología celular de las carcinogenisis química se observan los siguientes pasos

a) Mutaciones y angiogénesis
b) Mutacines y metástasis
c) Iniciacion y promoción
d) Apoptosis y emperipolesis
e) Expansión clonal y mitosis
En más del 95% de los casos el origen de una tromboembolia pulmonar es:

a) Necrosis isquémica
b) Embolia aérea
c) Embolia de líquido amniótico
d) Embolia grasa y medula ósea
e) Trombosis venosa profunda

¿Es un producto de las celulas endoteliales normales y un cofactor esencial para la unión de las
plaquetas a los elementos de la MEC?

a) Proteina C
b) Acetil Coenzima A
c) ATP
d) Factor de Von Willebrand
e) Fibrina

La formación insuficiente del tejido de granulación puede causar:

a) Tejido necrótico y cuerpos extraños


b) Diabetes y cáncer
c) Contracturas y cicatrización queloide
d) Movilización precoz de las heridas
e) Deshiscencia de la herida y ulceración.

Which of the following subtances is an anticoagulant secreted by endotelial cells?

a) Von willebrand factor


b) Prothrombin
c) Inhibitor of plasminogen activator
d) Thrombomodulin
e) Tissue factor

Es una de las consecuencias importantes cuando el embolo produce una oclusión vascular parcial o
completa:

a) Necrosis isquémica
b) Embolo aérea
c) Embolia pulmonar
d) Embolia grasa y medula osea
e) Embolo de líquido amniótico

Which of the following tumors occursmost often int he ovary?


a) Condroma
b) Hemangioma
c) Leiomyoma
d) Transitional cell papiloma
e) Cystadenoma

Son carcinógenos de acción indirecta:

a) Beta propiolactona
b) Dimetil sulfato, Ciclofosfamida
c) 1 Acetil-imidazol
d) Hidrocarburos aromáticos
e) Diepoxibutano

Es una alteración de escasa frecuencia que se debe a una proliferación exuberante de fibroblastos y
otros elementos de tejido conjuntivo

a) Lesiones desmoides
b) Hiperplasia beninga
c) Neoplasias
d) Carne orgullosa

Mediadores que son potentes vasodilatadores e inhibidores de la agregación plaquetaria

a) Especies reactivas de oxígeno y NO


b) prostaciclina y el oxido nítrico
c) Factores de coagulación
d) Glucoproteínas aunadas al endotelio

En las endocrinopatías en los síndromes paraneoplasicos se caracterizan por:

a) Producción hormona ectópica


b) Secreción de mucopolisacáridos
c) Producción de lípidos
d) Secreción de proteínas
e) Producción de polisacáridos

Wich growth factor is the most potent promotor of angiogénesis in the granulation tissue?

a) Angiotensin
b) Platelet derived growth factor
c) Tumor growth factor b
d) Vascular endotelial growth factor
e) Epidermal Growth factor

60 year old man had congestive heart failure and sodium retention. Pitting edema of the lower
extremities in this patientis most likely associated with n increased blood level of:

a) Albumin
b) Globulin
c) Aldosterone
d) Troponin
e) Plasminogen

Nombre que recibe la formación de vasos durante el desarrollo embrionario

a) Hematopoyesis
b) Vasculogenia
c) Angiogenia
d) Neovascularizacion

Son ejemplos de factores sistémicos que afectan la cicatrización

a) Hematoma y tejido necrótico


b) Infeccion local y cuerpos extraños
c) Alergia a suturas y denervación
d) Diabetes o cáncer
e) Contracturas y cicatrización queloide

Se produce cuando un embolo venoso atraviesa una comunicación interauricular o interventricular


para llegar a la circulación sistémica:

a) Embolia aérea
b) Embolia de grasa
c) Embolia paradójica (placas ateroescleróticas, vegetaciones valvulares y trombos venosos)
d) Embolia de medula osea

Célula con capacidad de autorrenoarse y de generar estirpes de células diferenciadas

a) Células estables
b) Celulas madre
c) Células pluripotenciales
d) Células madre embrionarias
e) Células quiescientes
¿Qué nombre recibe la mutacion especifica del factor V?

a) Mutacion de Leiden
b) Defecto de a fibrinólisis
c) Deficiencia de proteína COS
d) Déficit de antitrombina III
e) Homocistenemia

Calcificación de válvula mitral área funcional de menos de 1cm3, ocacionan en primera instancia:

a) Hiperemia
b) Hemorragia
c) Congestión Pulmonar
d) Hemotorax
e) Neumotorax

¿Cuál es la diferencia entre hematoma, petequia, purputa y equimosis?

a) La localización anatómica de cada una de ellas


b) La causa de la hemorragia
c) El tamaño de las hemorragias
d) La capa en donde ocurre , mucosa, submucosa y serosa
e) Por los órganos donde se presentan

¿El edema de origen renal se caracteriza por?

a) Postural
b) Presentarse en la región sacra
c) Localizaciones en tejido laxo

Son patologías que producen edema por aumento de la presión hidrostática del vaso:

a) Neoplasis y filariasis
b) Aumeto de la secreción renina-Angiotensina-Aldosterona
c) Glomerulopatias y malnutrición
d) Pericarditis constictiva y trombosis
e) Gastroenteropatias con perdida de proteínas

La fase progresiva del shock se caracteriza por:

a) Activación del sistema renina-angiotensina


b) Liberacion de catecolaminas y antidiuréticos
c) Sintesis de oxido nítrico y anuria
d) Glucolisis anaeróbica y acidosis láctica metabólica
e) Necrosis tubular aguda con insuficiencia renal
The most common cause of mitral stenosis in adults is:

a) Floppy mital valve


b) Rheumatic endocarditis
c) Bacterial endocarditis
d) Mitral anular calcification
e) Nonbacterial trombotic endocarditis

Tanto células como núcleos desarrollan variaciones en tamaño y forma:

a) Diferenciación
b) Anaplasia
c) Pleomorfismo
d) Perdida de polaridad
e) Hipercromatismo

La función de esta proteasa degrada al colágeno tipo IV y …….. a la metástasis de celulas tumorales?

a) Caspazas
b) Metaloproteinasas
c) Telomerasa
d) Anhidrasa carbonica

Genes incluidos en el genoma humano que regula el crecimiento y diferenciación celular

a) Genes implicados en la reparación del ADN


b) Protooncogenes
c) P53
d) Oncogenes (PP:84)
e) Genes inductores de apoptosis

Son aquellos que se caracterizan por aplicar frenos a la proliferación celular, formando una red de
puntos de control que impiden el crecimiento incontrolado

a) Genes inhibidores apoptosis


b) Genes de la apotosis
c) Genes supresores tumorales
d) Genes pre apoptosicos
e) Genes reparadores

Correlacionar las siguientes columnas contestando en su hoja de respuesta:

a) Angiosarcoma / mesenquimatosa maligna


b) Coriocarcinoma o Seminoma /Epitelio maligna (D)
c) Hemangioma o Linfangioma /Mesenquima benigna
d) Cistoadenoma o papiloma escamoso /epitelial benigna
e) Glaucoma/ No es neoplasia (C)
La caquexia en el cáncer se da por el efecto de la siguiente citosina.

a) PDFG
b) TGB
c) TNF
d) Adenalina
e) IL-8

Celulas linfoides que son cpaces de destruir celulas tumorales sin una sensibilización previa:

a) Netrofilos
b) Natural killer
c) Eosinofilos
d) Celulas CD
e) Macrófagos

a) cificaMarcador tumoral relacionado con carcino de colon y páncreas:


b) Alfa fetoproteina
c) Antígeno prostático
d) Antigeno carcinoembrionario
e) Enolasa neuronal espe
f) Inmunoglobulinas
Es la perdida de grasa corporal , masa magra, debilidad profunda anorexia y anemia:

a) Inanición
b) Caquexia
c) Marasmatica
d) Kwashorkor

¿A qué se asocia el carcinoma hepatocelular?


a) A la ingestión del metabolito fúngico aflatoxina B2
b) Mutacion del P450
c) Mutacion de G6PDH
d) Al efecto iniciador de fármacos con daño al hígado

La ciclina E/CDK2 controla la transición entre:

a) G1-S
b) G0-G1
c) G2-M
d) M-G0
e) M-G1

La acción carcinogénica se le atribuye a la formación de dimeros de pirimidina en el ADN:

a) Rayos UVA
b) Rayos UVB
c) Rayos UVC
d) Radiaciones térmicas
e) Rayos laser

Son ejemplos de factores locales que afectan la cicatizacion:

a) Edad, anemia
b) Desnutrición , obesidad
c) Tejido necrótico , cuerpos extraños
d) Deficiencia de vit. C zic y cobre
e) Síndrome de marfan , osteogenisis imperfecta

Los trombos arteriales que surgen de las cámaras cardiacas o de la aorta abdominal se adhieren
habitualmente a la pared de la estructura supreadyacente se denomina:

a) Trombosis supradyacente
b) Trombosis endocardica
c) Trombosis murales
d) Trombos miocárdicos

Capacidad de una celula de transdiferenciarse a distintos estripes celulares: Plasticidad del


desarrollo

Forma parte de la familia de proteínas del VEGF: PIGF: Factor de crecimiento placentario

Hemorragias diminutas de 1-2mm en la piel, mucosa y serosa: Petequias

Se llama equimosis a: hemorragias >1-2cm por traumatismos

Precursor circulante de la plasmina: Plasminogeno

Los infartos blancos se producen cuando hay: circulación arterial terminal


Son cambios que afectan casi todo el espesor del epitelio, pero aún se encuentran limitados por la
membrana basal: cáncer in situ

El VPH implicado en el carcinoma de celulas escamosas del cuello uterino y región del ano genital:
Tipo 16 y 18

Se basa en la lesión primaria , diseminación y persistencia de la metástasis usando el sistema TNM:


Estatificacion

En los carcinomas de mama originados en el cuadrante superior externo hacia que ganglios se
diseminan primero: Ganglios axilares

El cáncer se disemina principalmente por: via linfática

Gránulos específicos que contiene las plaquetas: Alfa y Beta

Tejido de capa blanca, rosado y granulado: Tejido de granulación.

Primeria sustancia que actua en el sitio de lesión: Endotelina

Consecuencia de la congestión pasiva crónica: hipoxia crónica

Factor que activa la via de la cascada extrínseca: Factor tisular

Sindrome de anticuerpos fosfolipidico en un estado de hipercoagubilidad inducido por anticuerpos


está asociado con enfermedad autoinmunitaria como: Lupus eritematoso

Los trombos arteriales que surgen de las arritmias cardiacas que surgen de las cámaras cardiacas se
adhieren habitualmente en la pared de la estructura subyacente se denominan: Trombos murales

La trombosis venosa superficial ocurre con más frecuencia en el: sistema safeno

Mujer de 40 años con Dx de cáncer avanzado, después del tratamiento y terapia desarrollo edema
en el brazo izquierdo. El edema se asocia a la obstrucción de: Via linfática

Características de cicatrización por segunda intención: Reaccion mas intensa con mayor cantidad de
tejido de granulación mas celulas musculares lisas.

Ejemplo de edema fisiopatológico por disminución de la presión oncotica: kwashorkor y cirrosis


hepática

La tumefacion intensa de tejido subcutáneo provoca: anasarca / edema generalizado

Calcificacion de valvula mitral: Congestion pulmonar

A que gen se une P53 para inducir a una celula a apoptosis: BAX

El endotelio intacto evita que las plaquetas y los actores de la coagulación alcancen la MEC, es un
efecto:

a) Primario
b) Antiplaquetario
c) Coagulante
d) Hemorrágico
e) Secundario

Factor anticuagulante que inhibe la formación del coagulo mediante insicion proteolítica de
los factores Va y VIIa: Proteina C

De donde se pueden aislar celulas madre: Blastocisto

Ejemplo de celulas con diferenciación terminal: Neurona

La fibrosis es sinónimo de: sustitución por tejido conjuntivo

Celulas que son progenitores bipotenciales capaces de diferenciarse tanto en hepatocitos como a
celulas biliares: Celulas ovales

Factor de transcripción que promueve la cicatizacion de heridas, angiogenia, hematopoyesis, y el


desarrollo del musculo: FGF

Son proteínas que facilitan la entrada y progresión de las células en el ciclo celular: Ciclinas

Inhibidor de la angiogénesis: Angiopoyetina 2 mas TIE2

Factores locales que afectan a la cicatrizcion: tejido necrótico y cuerpos extraños

Aspectos anormales durante la repacion de heridas: deficiencia de cicatriz, formación excesiva de


los componentes de la repacion y contracturas.

Formacion insuficiente de tejido de granulación puede causar: Dehiscencia de la herida y ulceración

Ateroesclerosis de notembarg (Respuesta C)

-Son acontecimientos plaquetarios, excepto:

Adhesión plaquetaria, cambio de forma, secreción y agregación.

Leiomiomas -> tumores miometrio

SE LE DENOMINA ASÍ CUANDO LAS CÉLULAS PARENQUIMATOSAS ESTIMULAN LA SÍNTESIS DE UN


ESTROMA CON COLÁGENO ABUNDANTE: DESMOPLASIA

ÁREAS DE NECROSIS ISQUÉMICA, CAUSADAS POR OCLUSIÓN ARTERIAL DEBIDA A TROMBOSIS O


EMBOLIA: INFARTO

CON RESPECTO A LA DIFERENCIACIÓN CELULAR EN LAS NEOPLASIAS MALIGNAS: PUEDEN SER BIEN,
MODERADAMENTE O POCO DIFERENCIADAS

LOS ESTADOS DE HIPERCOAGULABILIDAD SE DIVIDEN EN: GENETICOS Y ADQUIRIDOS O PRIMARIOS Y


SECUNDARIOS

EL FACTOR DE CRECIMIENTO TRANSFORMANTE BETA TIENE:


INHIBE LA PROLIFERACIÓN y MIGRACION ENDOTELIAL Y POTENCIA LA PRODUCCION DE PROTEINAS
DE LA MEC.

MARCADOR TUMORAL RELACIONADO CON FEOCROMOCITOMAS Y TUMORES RELACIONADOS:

CATECOLAMINAS Y METABOLITOS.

AL QUINTO DÍA SE OBSERVAN LOS SIGUIENTES CAMBIOS:

NEOVASCULARIZACION INTENSA Y EL TEJIDO DE GRANULACION INVADE LA HERIDA

MUTACIONES QUE CONTRIBUYEN AL FENOTIPO MALIGNO SE DENOMINAN: MUTACIONES


CONDUCTORAS.

MUTACIONES SIN RELEVANCIA FENOTIPICA: MUTACIONES PASAJERAS.

LAS NEOPLASIAS MALIGNAS MUESTRAN ALGUNOS ATRIBUTOS FENOTIPICOS, DENOMINADOS:


RASGOS CARACTERÍSTICOS DEL CÁNCER

EL DAÑO GENÉTICO NO LETAL, ESENCIAL EN LA CARCINOGENIA PUEDE DEBERSE A: MUTACIONES

SÍNDROME PARANEOPLÁSICO MÁS FRECUENTE, GENERALMENTE EL 50 % DE LOS PACIENTES QUE LO


PADECEN SUFREN UN CARCINOMA DE PULMÓN, SOBRE TODO DEL MICROCÍTICO: SÍNDROME
NERVIOSO

¿DÓNDE SE LOCALIZAN LAS CÉLULAS MADRE?: Estas células madre se encuentran en


pequeñas cantidades en la mayoría de los tejidos adultos, como la médula ósea o la grasa.
En comparación con las células madre embrionarias, las células madre adultas tienen una
capacidad más limitada para generar diferentes células del cuerpo.

P53 Y RB SON EJEMPLOS DE: GENES SUPRESORES DE TUMORES

¿CUÁL DE LAS SIGUIENTES AFIRMACIONES SOBRE LA DISPLASIA ES INCORRECTA?:

LA DISPLASIA ES EL CRECIMIENTO DESORDENADO PRINCIPALMENTE EN EPITELIOS Y SE CARACTERIZA


POR UNA CONSTELACION DE ALTERACIONES, PERDIDA DE LA UNIFORMIDAD DE CADA CÉLULA Y
DESORIENTACION ARQUITECTONICA, PLEOMORFISMOS Y MUCHOS NÚCLEOS HIPERCROMATICOS
CON UN ELEVADO INDICE NUCLEOCITOPLASMÁTICO.

LA CASCADA DE COAGULACIÓN REPRESENTA: LA FUNCION PLAQUETARIA

EN LOS TEJIDOS LÁBILES EL TIPO DE REPARACIÓN DE UNA LESIÓN LEVE ES: REGENERACION

NEOPLASIA MALIGNA DE LAS CELULAS PLASMATICAS: MIELOMA

SON ACONTECIMIENTOS PLAQUETARIOS EXCEPTO: MIGRACION

SE LE DENOMINA ASÍ CUANDO LAS CÉLULAS PARENQUIMATOSAS ESTIMULAN LA SÍNTESIS DE UN


ESTROMA CON COLÁGENO ABUNDANTE: DESMOPLASIA
CON RESPECTO A LA DIFERENCIACIÓN CELULAR EN LAS NEOPLASIAS MALIGNAS: PUEDEN SER BIEN,
MODERADAMENTE O POCO DIFERENCIADAS

AL LÍQUIDO DE EDEMA GENERALIZADO SE LE CONOCE COMO: ANASARCA

¿LOS INFARTOS BLANCOS, SE PRESENTAN?: CUANDO UN TEJIDO SOLIDO CON CIRCULACION


TERMINAL SE QUEDA SIN RIEGO ARTERIAL

POTENTE FARMACO ANTICOAGULANTE QUE ACTÚA A TRAVÉS DE LA ANTITROMBINA III: HEPARINA

ES UN FACTOR DE ALTO RIESGO DE HIPERCOAGULABILIDAD: REPOSO PROLONGADO EN CAMA

LOS FACTORES QUE INFLUYEN EN LA CURACIÓN DE HERIDAS SE CLASIFICAN EN: LOCALES Y


SISTEMICOS

MARCADOR TUMORAL RELACIONADO CON FEOCROMOCITOMAS Y TUMORES RELACIONADOS:


CATECOLAMINAS Y METABOLITOS

AL QUINTO DÍA SE OBSERVAN LOS SIGUIENTES CAMBIOS: NEOVASCULARIZACION INTENSA Y EL


TEJIDO DE GRANULACION INVADE LA HERIDA

P53 Y RB SON EJEMPLOS DE: GENES SUPRESORES DE TUMORES

Las proteínas E6 y E7 del VPH de alto riesgo inhiben los genes de: p21 y p53.

Puede presentarse en cirugías de derivación, en neurocirugía en posición sedente y en el descenso


brusco de la presión atmosférica: EMBOLIA GASEOSA

¿Qué puede producir un exceso de factores estimulantes del crecimiento celular?: Cáncer o
crecimiento celular incontrolado.

¿A que deben los tumores su potencial replicativo ilimitado?: Activación de telomerasa.

Ejemplo de tejidos quiescentes: Hígado y neuronas.

Las células madre pluripotenciales son capaces de: Pueden inducidas en múltiples linajes celulares.

Moléculas de adhesión que contienen los gránulos alfa de las plaquetas: Selectina P

La vía apoptósica intrínseca es la que más se deshabita en el cáncer.

En una célula en reposo la proteína APC: Inhibe la proliferación por la degradación de beta- catenina.

El hígado en nuez moscada es un patrón morfológico característico de: Congestión pasiva crónica.

Neoplasia benigna músculo liso: Leiomioma

Constituye el asiento más habitual de la invasión venosa en la diseminación: Higado y pulmones.

La diseminación vascular y alojamiento de las células tumorales, una vez alcanzada la circulación, las
células tumorales son vulnerables a la destrucción por una serie de mecanismos, como la tensión
mecánica por cizallamiento, la apoptosis estimulada por la pérdida de adhesión (Denominada anoikis)
y las defensas de la inmunidad innata y adaptativa.

Triada de Virchow: Lesión endotelial, estasis o flujo sanguíneo turbulento e hipercoagulabilidad de la


sangre. Excepto: Incremento de volemia.

La aparición de hematomas puede deberse a: Plaquetopenia/Trombocitopenia.

¿De dónde proceden principalmente los émbolos sistémicos?: Trombos murales cardiacos o
valvulares.

Características de las señales autocrinas: Secreción de neurotransmisores en uniones celulares


especiales.

Un defecto en la expresión del factor de Von Willebrand: PETEQUIAS

Las heridas pueden llegar a ulcerarse debido: Una vascularización inadecuada del proceso de curación.

Masa neoplásica denominada pseudomixoma peritoneal en ocasiones proviene de: Carcinoma


apendicular y de ovario.

Infartos del sistema nervioso central: Necrosis licuefactiva.

Pueden presentarse tras fracturas de huesos largos o rara vez en traumatismos de partes blandas y
quemaduras: Embolia grasa y medular.

Esta vía de diseminación es bastante característica de los carcinomas de ovario, ya que


frecuentemente se diseminan por la superficie peritoneal: Siembra de cavidades y superficies
corporales.

Una vez que las células tumorales rompen la membrana basal se habla de: Tumor infiltrante.

Conversión de fibrinógeno a fibrina: Trombina.

Proceso pasivo por reducción de flujo de salida de los tejidos: Congestión.

Un ateroma ulcerado, se produce: Lesión endotelial y alteraciones en el flujo sanguíneo.

En esta fase de choque se presenta insuficiencia renal por necrosis tubular aguda que la supervivencia
es imposible: FASE IRREVERSIBLE

Afirmación de las plaquetas: Contienen gránulos delta ricos en difosfato de adenosina.

Qué entidad se refiere a una forma de coagulopatia de consumo caracterizada por trombosis
microangiopatica de círculo vicioso y hemorragia: COAGULACION INTRAVASCULAR DISEMINADA

Características de las señales sinápticas: Secreción de neurotransmisores en uniones celulares


especiales.

Se media por interacciones con el factor de Von Willebrand con la glucoproteína Ib (GpIb) del receptor
de superficie de las plaquetas y el colágeno expuesto: Adhesión plaquetaria.
Estos genes se han relacionado con neoplasia endocrina múltiple 1, tumores endocrinos, hipofisiarios,
paratiroideos y pancreáticos: RET / MEN1

Los miofibroblastos son: Contribuyen a la contracción de la cicatriz con el tiempo.

Tras un episodio de síndrome coronario agudo con isquemia del tejido cardiaco es más factible
encontrar: NECROSIS COAGULATIVA ISQUEMICA/INFARTO BLANCO.

Neoplasia maligno de los vasos linfáticos: LINFANGIOSARCOMA

La formación de cantidades excesivas de tejido de granulación, que sobre sale por encima del nivel de
piel circundante y bloquea la reepitelización: Granulación exuberante.

La mayoría de las causas de shock endotóxico son causados por: Shock séptico, 70 %Bacilos Gram
negativos que muestran endotoxinas. También pueden desencadenar un shock séptico moléculas de
análogas de hongos y bacterias gram positivas.

Al líquido acumulado dentro de una cavidad se le denomina: Derrame

¿Cuál de las siguientes afirmaciones sobre el ganglio centinela es incorrecta?: TODO ESTO ES
CORRECTO: Se define como el primer ganglio de una región linfática que recibe el flujo linfático del
tumor primario. El mapeo del ganglio centinela se lleva a cabo inyectando trazadores reactivos o
colorantes; el estudio por cortes por congelación, se utiliza para detectar la propagación de los
melanomas, canceres de colon y otros tumores. Se utiliza a menudo la biopsia.

Factores sistémicos que ocurren en la degradación de heridas: Estado nutricional del huésped
(nutrición proteica, ingesta de vitamina C) Estado metabólico (Diabetes retrasa la curación) Estado
circulatorio o suficiencia vascular, hormonas (Glucocorticoides) pueden impedir el proceso
inflamatorio-reparador.

La principal causa de exudado es: Reacción inflamatoria aguda.

Condiciones predisponentes para una trombosis vascular incluyen: Todas las siguientes, excepto:

TROMBOCITOPENIA.

Este componente de la MEC que retiene agua y da resistencia a la comprensión: Ácido hialuronico.

Son causas de shock hipovolémico, excepto: ANAFILAXIA

¿Cuál es el papel de los macrófagos M2 en la reparación?: Desempeñan un papel esencial en la


reparación, eliminando a agentes causales y el tejido muerto, aportando factores del crecimiento para
la proliferación de distintas células y secretando citocinas, que estimulan la proliferación de
fibroblastos y la síntesis de tejido conjuntivo.

El desarrollo de nuevos canales de sangre a trasvés de los trombos oclusivos, se llama: ORGANIZACIÓN
Y RECANALIZACIÓN.

Se caracteriza por la exudación de líquido con bajo contenido de células en los espacios creados por
la lesión celular o en las cavidades corporales: INFLAMACIÓN SEROSA/DERRAME.
PIGMENTO ENDÓGENO, NO DERIVADO DE LA HEMOGLOBINA: Melanina

CALCIFICACIÓN PATOLÓGICA QUE SE ACOMPAÑA DE NIVELES ELEVADOS DE CALCIO SÉRICO ES:


Metatásica

LA INICIACIÓN DE LA CALCIFICACIÓN DISTROFICA INTRACELULAR SE INICIA EN: MITOCONDRIAS

La hemosiderina derivada de la hemoglobina es un pigmento de color amarillo dorado a pardo,


granular o cristalino, que contribuye una de las principales formas de almacenamiento de hierro.

EL ASPECTO VÍTREO HOMOGÉNEO Y ROSADO A LOS CORTES HISTOLÓGICOS HABITUALES TEÑIDO CON
HEMATOXILINA Y EOSINA SE LLAMA: cambio graso

Los cuerpos de Russell son acumulos de: INMUNOGLOBULINAS EN CÉLULAS PLASMÁTICAS

EL LLAMADO PIGMENTO DE DESGASTE (lipofiscina) SE FORMA CON MUCHA FRECUENCIA EN:


METAPLASIA.

LOS MACRÓFAGOS MODIFICADOS O CÉLULAS EPITELIOIDES SON CARACTERÍSTICAS DE:


INFLAMACION GRANULOMATOSA

ES EJEMPLO DE INFLAMACIÓN PURULENTA: APENDICITIS AGUDA SUPURATIVA

SE DENOMINA BIOPSIA AL ESTUDIO DE: UN FRAGMENTO DE TEJIDO

SON SUSTANCIAS QUE ACTÚAN COMO ANTIOXIDANTES NATURALES: SUPERÓXIDO DISMUTASA,


GLUTATION PEROXIDASA

LAS PRINCIPALES CITOCINAS QUE INTERVIENEN EN LA RESPUESTA INFLAMATORIA SON: IL1 Y FNT

LOS SIGUIENTES SON EFECTOS DE LA HIPOXIA EN LA CÉLULA: DISMINUCIÓN DE LA FOSFORILACIÓN


OXIDATIVA

EL FIJADOR DE TEJIDOS UTILIZADO DE FORMA RUTINARIA: FORMOL AL 10%

MEDIADORES RELACIONADOS CON EL DOLOR EN LA INFLAMACIÓN AGUDA: PROSTAGLANDINAS Y


BRADICININA

SON REACCIONES DE LA FASE AGUDA PROVOCADAS POR LA IL-1, EL TNF: FIEBRE, PERDIDA DEL
APETITO

TIPO DE NECROSIS QUE SE SUCEDE SECUNDARIA A LA LESIÓN HIPOXICA LETAL EN LA MAYORÍA DE LOS
TEJIDOS: COAGULATIVA

LA LENTIFICACIÓN DE LA CIRCULACIÓN EN LA INFLAMACIÓN AGUDA SE DEBE A: EL AUMENTO DE LA


PERMEABILIDAD DE LA MICROVASCULATURA

SON SUSTANCIAS QUE ACTÚAN COMO OPSONINAS: C3B, IGG, Y COLECTINAS

LAS PROLONGACIONES O PSEUDOPODOS DE LOS LEUCOCITOS CONTIENEN EN SU INTERIOR


FILAMENTOS DE: ACTINA Y MIOSINA
EN LA HEMOSIDEROSIS SISTÉMICA, EL PIGIMENTO NO LESIONA A LAS CÉLULAS, PERO SI SU
ACUMULACIÓN ES EXCESIVA, PRODUCE UNA ENFERMEDAD CONOCIDA COMO: HEMOCROMATOSIS

ES CARACTERÍSTICA DE LA INFLAMACIÓN CRÓNICA: INFILTRACIÓN DE CÉLULAS MONONUCLEARES


(MACRÓFAGOS)

ES CARACTERISTICA DE LA CALCIFICACION METASTASICA: PROVOCA DISFUNCION ORGANICA

LAS SUSTANCIAS QUÍMICAS PUEDEN PRODUCIR LESIÓN CELULAR POR: ACCIÓN DIRECTA O INDIRECTA

EL DESPRENDIMIENTO DE LOS RIBOSOMAS DEL RER EN LA LESIÓN CELULAR PRODUCE: DISMINUCIÓN


DE LA FOSFORILACIÓN OXIDATIVA

LOS CUERPOS DE WEIBEL PALADE DE LAS CELULAS ENDOTELIALES CONTIENEN: SELECTINA P

SON EJEMPLO DE SUBSTANCIAS QUÍMICAS QUE CAUSAN LESIÓN CELULAR DE MANERA INDIRECTA AL
TRANSFORMARSE, POR VÍA ENZIMÁTICA, EN RADICALES LIBRES: PARACETAMOL Y TETRACLORURO DE
CARBONO.

EL PATRÓN MORFOLÓGICO DE NECROSIS EN EL QUE PREDOMINA LA DEGRADACIÓN ENZIMÁTICA ES


LA: N. COAGULATIVA

FACTORES DE ADHESION QUE PERTENECEN A LAS INMUNOGLOBULINAS: ICAM, VECAM, PCAM

ES UN MEDIADOR QUIMICO QUE TIENE ACTIVIDAD BACTERICIDAY PRODUCE VASODILATACION POR


RELAJACION DIRECTA DEL MUSCULO LISO ARTERIOLAR: C3a Y c5a

SON FENÓMENOS QUE CARACTERIZAN E INDICAN QUE LA LESIÓN CELULAR ES IRREVERSIBLE: LA


DISFUNCIÓN MITOCONDRIAL Y LA PERDIDA DE LA INTEGRIDAD DE LAS MEMBRANAS CELULARES
ENTRE LOS FACTORES DE RIESGO PARA EL CA CU SE ENCUENTRA, EXCEPTO: INICIO DE ACTIVIDAD
SEXUAL DESPUÉS DE LOS 30 AÑOS

CUAL DE LOS SIGUIENTES MEDIADORES DE LA INFLAMACIÓN TIENE PROPIEDADES QUIMOTÁCTICAS


THROMBOXANE A2
¿Y AUMENTA A LAS PERSONAS QUE TOMAN ASPIRINA?:
¿AUMENTA EN CÉLULAS ENDOTELIALES POR UN INFLUJO CALMODULINMEDIADO DE CUÁL
ELEMENTO?: CALCIUM

ORGANO O TEJIDO QUE AUMENTA DE TAMAÑO POR AUMENTO DE SU NUMERO DE CELULAS:


Hiperplasia

CUAL DE LOS SIGUIENTES MEDIADORESDE INFLAMACIÓN CAUSA DOLOR: Bradicinina


LA INFLAMACIÓN FIBRINOSA SE CARACTERIZA POR: SE PRESENTA EN INFLAMACIÓN DE
CAVIDADES CORPORALES COMO PERICARDIO Y PLEURA
CORAZON ANORMALMENTE GRANDE POR OBSTRUCCIÓN VALVULAR MITRAL O AORTICA PRODUCE:
HIPERTROFIA

UNA CARACTERÍSTICA HISTOLÓGICA DE LOS ORGANOS QUE SUFREN ATROFIA ES: FORMACIÓN DE
VACUOLAS AUTOFAGICAS

PARA LA FORMACIÓN DE GRANULOMAS INMUNITARIOS SE REQUIERE: REACCION INMUNITARIA


MEDIADA POR LINFOCITOS T

SON CAUSAS DE HIPOXIA EXCEPTO: ANEURISMAS

SON COMPONENTES DE LA INFLAMACIÓN AGUDA: MODIFICACIONES EN EL CALIBRE DE LOS VASOS Y


EN LA ESTRUCTURA MICROVASCULATURA.

SON FENÓMENOS LEUCOCITARIOS EN LA LUZ VASCULAR: MARGINACIÓN, RODAMIENTO Y ADHESIÓN

PROTEINAS PROAPOPTOSICAS: CUATRO DOMINIOS BH. BAX-BAK

DENTRO DE LOS EJEMPLOS ESPECÍFICOS DE APOPTOSIS, ESTÁ LA INDUCIDA POR LESIÓN DEL ADN
CELULAR, SEÑALE CÚAL ES EL GEN INVOLUCRADO EN ESTE MECANISMO: CED 3

EL CANCER IN SITU SE LOCALIZA: IMITADO AL GROSOR DEL EPITELIO

APOPTOSIS------- MUERTE CELULAR POR RECHAZO INMUNITARIO

NECROSIS LICUEFACTIVA------- INFECCIONES BACTERIANAS FOCALES

NECROSIS COAGULATIVA------- INFARTO DE MIOCARDIO

NECROSIS GRASA--------- PANCREATITIS AGUDA

NECROSIS CASEOSA------------ TUBERCULOSIS

CON RESPECTO A LA CARACTERÍSTICA MICROSCÓPICO MÁS IMPORTANTE DEL LEIOMIOMA: ASPECTO


ARREMOLINADO DE LOS HACES DEL MÚSCULLISO O

CARACTERÍSTICAS MORFOLÓGICAS DE LA APENDICITIS AGUDA SUPURADA: ABSCESO DE PARED


ACOMPAÑADO POR ULCERACIÓN.
OCURRE CUANDO EL NÚCLEO DE UN FIBROBLASTO CUTÁNEO DE UN PACIENTE SE INTRODUCE EN UN
OVOCITO HUMANO ENUCLEADO PARA FORMAR CME QUE SE CULTIVAN, SE INDUCE SU
DIFERENCIACIÓN, Y SE TRASPLANTAN AL PACIENTE: CLONACIÓN REPRODUCTIVA.

LAS CÉLULAS MADRE/PROGENITORAS DEL HÍGADO SE ALBERGAN EN: CONDUCTOS DE HERING.

FACTOR DE CRECIMIENTO QUE CONTRIBUYE A LA REPARACIÓN DE HERIDAS, ANGIOGENIA,


HEMATOPOYESIS Y DESARROLLO DE MÚSCULOS CARDIACO Y ESQUELÉTICO, LA MADURACIÓN DEL
PULMÓN, Y LA ESPECIFICACIÓN DEL HÍGADO: FGF

EL PATRÓN MORFOLÓGICO DE NECROSIS QUE PRESENTAN LAS CÉLULAS MUERTAS POR ISQUEMIA
(SALVO EN EL SNC) ES: COAGULATIVA.

EL NIVEL SERICO DE LA CPK-MB, SE ELEVA POR ENCIMA DE LOS VALORES BASALES A LAS: 2-4 HRS.

TIPO DE METAPLASIA MÁS FRECUENTE EN TEJIDO EPITELIAL: DE EPITELIO CILÍNDRICO A PLANO


ESTRATIFICADO (Escamoso)

SON ANORMALIDADES ENCONTRADAS EN UNA CITOLOGÍA EXFOLIATIVA EXCEPTO: PRESENCIA DE


ANTICUERPOS

INTERACTUA CON UN RECEPTOR TIE 2 PARA RECLUTAR A LAS CÉLULAS PERIENDOTELIALES:


ANGIOPOYETINA 1

EL COLOR AZUL VERDOSO QUE ADQUIEREN LOS HEMATOMAS EN LA FASE DE RESOLUCIÓN ES


PORQUE: LA HEMOGLOBINA SE TRANSFORMA EN HEMOSIDERINA.

ES EL COAGULO CONSTITUIDO POR AGREGACION PLAQUETARIA Y DEPOSITO DE FIBRINA: TAPON


HEMOSTATICO SECUNDARIO.

PRINCIPAL QUIMIOTÁCTICO ENDÓGENO PARA NEUTROFILOS: FRAGMENTOS FC DE IGG Y


COLECTINAS.

LA LIZOSIMA, LACTOFERRINA, PROTEÍNA BÁSICA PRINCIPAL Y LAS DEFENSINAS, DESTRUYEN


MICROORGANISMOS POR MECANISMOS: INDEPENDIENTES DEL OXÍGENO

PRINCIPAL QUIMIOCINA QUE INDUCE POTENTE ADHESION PARA MONOCITOS Y LINFOCITOS T: CX3C

ES LA CAPmetastasisACIDAD DE UNA CÉLULA DE TRANSDIFERENCIARSE A DISTINTOS ESTIRPES


CELULARES: PLASTICIDAD DEL DESARROLLO.

COMPLICACION MAS FRECUENTE DEL INFARTO DE MIOCARDIO: ARRITMIAS CARDIACAS.

MANIFESTACION CLINICA INICIAL MÁS FRECUENTE DEL CARCINOMA RENAL: HEMATURIA

CRITERIO HISTOLOGICO PARA DIAGNOSTICO DE APENDICITIS AGUDA: infiltacion de neutrofilos en la


capa muscular

ES MITÓGENO PARA CÉLULAS EPITELIALES, HEPATOCITOS Y FIBROBLASTOS, ES HOMOLOGO DEL TGF


ALFA, AL TENER ACTIVIDADES BIOLÓGICAS SIMILARES: TGFBETA
SON GRANDES COLECCIONES DE SANGRE EN CAVIDADES EXCEPTO: HEMOSTASIA

SON SUSTANCIAS CON CAPACIDAD ANTITROBOTICA: PLASMINOGENO TISULAR Y PROSTACICLINA


ENDOTELIAL

La acumulación anormal de triglicéridos linside perenshma células llamadas: esteatosis

Son tipos de embolias excepto: embolia linfática

Los cuerpos psamoma : apariencia de cúmulos de granos de arena. Calcio

LA INICIACIÓN DE LA CALCIFICACIÓN DISTROFICA EXTRACELULAR SE INICIA EN: LOS FOSFOLIPIDOS


DE LA MEMBRANA CELULAR

1.- Patología en la que se pierde albumina y genera edema: Sindrome Nefrotico

2.- Causantes de edema: Enfermedad cardiaca y renal

3.- hematoma mayor a 2cm: Equimosis

4.- Causa fisiológica de hiperemia: Vasodilatacion por ejercicio

5.- Factor que inhibe la diseminación del trombo: Activador tisular de plasminogeno

6.- Capacidad de cambiar de un tipo celular a otro: Transdiferenciasion

7.- Es la capacidad de una célula de transdiferenciarse a distintos estirpes celulares: Plasticidad de


desarrollo

8.- En esta fase de shock hay liberación de ADH y activación de baroreceptores: Fase inicial
progresiva

9.- Manifestaciones clínicas presentes en shock séptico: Piel caliente con vasodilatación periférica,
hiperemia.

10.- Casi siempre oclusiva puente largo de la luz del vaso: Trombosis venosa profunda

11.- Tipo de necrosis encéfalo: Licuefactiva

12.- Sintomas de la embolia grasa: Taquicardia, bradicardia y disnea

13.- Mantiene la integridad del epitelio externo de la cornea: Celulas madre del limbo

14.- Una neoplasia compuesta se divide como: Dos capas celulares y una capa embrionaria

15.- Metastasis: Implantes tumorales sin continuidad con el tumor primario

16.- Fases de la cascada metastasica: Invasion de la MEC, diseminación vascular, alojamiento y


colonización

17.- Inhibicion de TP53 mediado por: BAX

18.- ¿Cuál de las siguientes características no acompaña al cáncer? Cápsula fibrosa


19.- ¿Cuáles son los órganos más afectados por la diseminación hematógena? Hígado y pulmón

21.-Carcinogeno natural hepático secretado por plantas y hongos: Aflatoxina B1

22.- Causas frecuentes de esteatosis: Alcoholismo y desnutrición

23.- Medidador quimico que produce dilatación y aumento de la permeabilidad y selibera de tipo
físico: Histamina

24.- son células que participan en la reacción inflamatoria excepto: Pericitos

25.- Las CX3C son moléculas de adhesión de: Monocitos y Cel T

26.- De que vitamina son dependientes las proteínas Cy S: Vitamina K

27.- Localizacion mas freuente del carcinoma de células renales: Polos

28.- El cáncer renal causa frecuentemente metástasis en: Pulmones y hueso

29.- Causa mas frecuente de cáncer renal: Tabaquismo

30.- Cual es la proteína viral que induce una rápida degradación de P53: E6

31.- Causa frecuente de metástasis en CACU: Pielonefritis y Uremia

32.- Lesion primaria de mayor riesgo de producir cáncer de mama: Mastopatia proliferativa con
atipia

33.- Quimico que dara activación quimiotactica, actividad sobre monocitos y eosinofilos: CXC

34.- sustancias que ocacionan fiebre, sueño y perdida del apetito: Citocinas

35.- La inflamación fibrinosa se caracteriza por: Fluidos corporales en pericardio y pleura

36.- Caracteristicas histológicas de órganos que sufren atrofia, hay presencia de: Formacion de
vacuolas autofagicas

37.- Los leiomiomas se dividen en: Submucosos, subserosos e intramurales

38.- Las enfermedades crónicas granulomatosas abarca varios trastornos que tienen en común: La
incapacidad de destruir agentes etiológicos

40.- El infarto de miocardio corresponde a una: cardiopatía isquémica

41.- Todos las siguientes sustancias pertenecen al tren de tincion, excepto: Violeta de Genciana

42.- La citología exfoliativa puede reportar todo, excepto: El estado nutricional de la células.

43.- Carcinoma in situ ductual consiste en: Poblacion maligna de células que no pueden atravesar la
Membrana Basal

44.- La fosfofructocinasa y la fosforilaza en lesión, activan a: Glucolisis Anaerobia


45.- Que moléculas emiten la adhesión de plaquetas al endotelio no lesionado?: Prostacicilinas y
Oxido Nitrico

46.- Para la formación de granulomas se requiere: reacción inmunitaria mediada por células T

47.- Pancreatitis aguda ------------------------ Necrosis Grasa

48.- Infeccion Bacteriana----------------------N. Licuefactiva

49.- Muerte celular----------------------------- Apoptosis

50.- Tuberculosis------------------------------- N. caseosa

51.- Infarto al miocardio--------------------- N. Coagulativa

1.-Características microscópicas importantes del Leiomioma


aspecto arremolinado del musculo

2.-Características morfológicas de la apendicitis aguda supurada


absceso de la pared acompañado con ulceración

3.-Ocurre cuando el núcleo de fibroblasto cutáneo de un paciente se introduce en un ovocito


humano enucleado para formar células madre embrionarias que se cultivan se inicia su
diferenciación y se trasplantan al paciente
clonación terapéutica

4.- las células madre progenitoras del hígado se albergan en


conductos de Hering

5.- factor de crecimiento que contribuyye a la reparación de heridas oncogenia, hematopoyesis y


desarrollo de musculo cardiaco y esquelético, la maduración del pulmon y la especificación del
hígado
factor de crecimiento fibroblastico

7.- patron morfologico de necrosis que presentan las celulas muertas por isquemia salvo en el
sistema nerviso central
coagulación o coagulativa

8.- nivel serico de las enzimas necestia de cuanto tiempo para dar el Dx seguro
2-4 hrs

9.- ejemplos del grupo sisiopatologico de edema por disminución de presión osmótica
desnutrición proteica, kwashiorkor

10.- tipo de metaplasia mas frecuente en tejido epitelial


epitelio cilíndrico a plano estratificado
11.- anormalidades encontradas en citología exfoliativa excepto
presencia de anticuerpos
12.- interactua con un receptor TIE2 para reclutar células periendoteliales
angiopoyetina tipo 1
13.- color azul verdoso que adquieren los hematomas en la fase de resolución xq
hemoglobina se transforma en hemosiderina

14.- RNA virus have oncogene activity


leucemia, cel T

15.- coagulo constituido por agregación plaquetaria y deposito de fibrina


tapon hemostático secundario

16.- alteración de escasa frecuencia que se debe a una proliferación exuberante de fibroblastos y
otros elementos de tejido conjuntivo
lesiones desmoides

17.- proceso de remodelación de la reparación de las heridas participan enzimas


metaloproteinasas y gelatinasas

18.- adaptación cel. Crecimiento que puede ser terreno fértil, donde se inicia una proliferación
cancerosa
displasia

*19.- adaptación celular de la diferenciacion que consiste en un cambio reversible de una celula
adulta por una celula adulta
metaplasia

*20.- selectinas integrinas y flicoproteinas tipo mucina atuan como


moléculas de adhesión

*21.- principal quimiotactico endógeno para …..


CXC o alfa

*22.- la lilsosima lactoferrina……


independientes del oxigeno

*23.-principal quimiocina que induce ……..


CX3C

*24.- capacidad de una celula de transdiferenciarse a distintos tipos celulares


plasticidad de desarrollo

*26.- manifestación clínica mas frecuente del canceer renal


hematuria
27.- criterio hitologico para Dx de apendicitis aguda
infiltacion de neutrofilos en la capa muscular

28.- es mitogeno para celulas epiteliales hepatocitos y fibroblastos, es homogeno de TGF alfa, al
neter actividades biológicas similares
TGF

29.- son grandes colecciones de sangre en cavidades excepto


hemostasia
30.- son sustancias con capacidad antitrombotica
plasminogeno tisular y prostaciclina endotelial

31.- tejido de granulación se carateriza por


proliferación de fibroblastos y pequeños vasos sanguíneos

32.- ejemplo de trombosis por estadios de hipercoagulabilidad secundaria


síndrome anticoagulante lupico

33.- infartos rojos aparecen en las siguientes condiciones excepto


en oclusiones arteriales

35.- en el fibroadenoma mamario se caracteriza sobre todo los siguientes excepto


esta fijo a planos profundos y no se puede delimitar su tamaño

36.- the abnormal triglyceride accumulation linside perenshma cells called


steatosis or change
37.- se pueden encontrar granulomas en las siguientes enfermedades
lepra peromatosa y tuberculosa

38.- son tipos de embolias excepto


embolia linfática

39.- tipo de calcificaicon que se acompaña de hipercalemia y trastornos en el metabolismo del calcio
metastasica

40.- leiomiomas de acuerdo a su localización en el cuerpo uterino se clasifican en


submucosos, subserosos e intramurales

41.- metabolitos del acido araquidonico por via de ………….


prostaglandinas y tromboxano
42.- pigmento endógeno, no derivado de la hemoglobina
melanina

43.- cuerpos de Russell son acumulos de


inmunoglobulinas en células plasmáticas
44.- los cálculos biliares están formados por
colesterol, calcio y pigmentos biliares

45.- complicaciones mas frecuentes de la apendicitis


peritonitis

46.- la alfa feto proteína se eleva anormalmente en


cáncer primario de hígado, de células germinales del testículo

47.- la citometria de flujo puede medir


los antígenos de membrana, contenido de ADN en células tumorales
48.- la iniciación de la calcificación distrofica extracelular se inicia en
los fosfolipidos de la membrana celular

49.- fenómeno que considera dentro del destino del trombo excepto
propagación

50.- en cual de los siguientes órganos se puede desarrolar un teratoma


ovario

1.- SON NEOPLASIAS EPITELIALES GLANDULARES MALIGNAS QUÍSTICAS: CISTOADENOCARCINOMAS

2.- NEOPLASIA MALIGNA DE LOS VASOS LINFÁTICOS: LINFANGIOSARCOMA

3.- neoplasia maligna de los vasos sanguineos R: angiosarcoma

ES EL CAMBIO FUNDAMENTAL PARA OBTENER POTECIAL REPLICATIVO ILIMITADO: ACTIVACION DE


LA TELOMERASA

LOS CARCINÓGENOS INDIRECTOS NECESITAN: ACTIVACIÓN METABÓLICA

LOS SÍNDROMES PARANEOPLÁSICOS PUEDEN PRODUCIR EXCEPTO: SEPSIS

LOS TEJIDOS QUE TIENE UN NIVEL DE REPLICACION BAJO PERO QUE PUEDEN SUFRIR DIVISIONES
RAPIDAS EN RESPUESTA A ESTIMULOS ESPECIFICOS SON: QUIESCENTES

LA CAQUEXIA DEL CANCER SE DEBE A: SECRECION DE FACTOR DE NECROSIS TUMORAL


(CAQUECTINA)

SON LESIONES SUBLETALES DEL DNA Y DIANAS GENETICAS PARA EL DESARROLLO DE NEOPLASIAS
MALIGNAS EXCEPTO: SÍ: ONCOGENES , GENES DE LA APOPTOSIS, GENES SUPRESORES DEL CANCER
(P53, P21, RB), GENES REPARADORES DEL DNA NO: GENES REPRESORES

LA INSENSIBILIDAD A LAS SENALES INHIBITORIAS DEL CRECIMIENTO ESTA DADO POR LESION A:
RB,P53

LOS MARCADORES TUMORALES PUEDEN SER EXCEPTO: CITOCINAS

SI: ISOENZIMAS, ANTIGENOS ONCOFETALES, HORMONAS

LAS CELULAS MADRE SE CARACTERIZAN POR: CAPACIDAD DE AUTORENOVACION Y REPLICACIÓN


ASIMETRICA

LA LESION SUBLETAL DEL DNA POR RADIACIONES ULTRAVIOLETA ES POR: FORMACION DE DIMEROS
DE PIRIMIDINA

EL VEGF Y LAS ANGIOPOYETINAS INTERVIENEN EN PROCESOS COMO: VASCULOGENESIS Y


ANGIOGENESIS

LAS PROTEINAS E6 Y E7 DEL VPH DE ALTO RIESGO SE UNEN AL GEN: RB Y P53

ENZIMAS ENCARGADAS DE LA DEGRADACIÓN DEL COLAGENO Y OTRAS PROTEÍNAS DE LA MATRIZ


EXTRACELULAR: METALOPROTEINASAS
ABCESOS, ÚLCERAS Y QUEMADURAS SON EJEMPLOS DE CICATRIZACIÓN POR: SEGUNDA INTENCIÓN

SON CAMBIOS CROMOSOMICOS EN LAS NEOPLASIAS EXCEPTO: A)AMPLIFICACION


B)TRASLOCACION } C) TRANSCRIPCION } D) DELECION

ES UNA CARACTERÍSTICA EXCLUSIVA DE LAS NEOPLASIAS MALIGNAS: OCASIONAR METASTASIS

EL PARECIDO QUE EXISTE ENTRE LA CÉLULA NEOPLÁSICA Y LA CONTRAPARTE NORMAL SE


DENOMINA GRADO DE: DIFERENCIACIÓN

SON CELULAS DIFERENCIADAS REPROGRAMADAS: CELULAS MADRE INDUCIDAS

ENTRE LAS ONCOPROTEÍNAS (PRODUCTOS PROTÉICOS DE LOS ONCOGENES) TENEMOS: BCL2

SON MARCADORES TUMORALES EXCEPTO: GONADOTROPINA CORIONICA HUMANA, INSULINA,


ALFA FETOPROTEINA, ANTIGENO PROSTÁTICO ESPECÍFICO.

CARCINÓGENOS QUÍMICOS EXCEPTO: A.AGENTES ALQUILANTES B. AFLATOXINA B C.AMINAS


AROMATICAS D.ANTIBIOTICOS

UNO DE LOS GENES MUTADO EN LA MAYOR PARTE DE LOS CÁNCERES HUMANOS ES: P53

SE DEFINE COMO AUSENCIA DE DIFERENCIACIÓN: ANAPLASIA

LAS ENDOCRINOPATÍAS DE SÍNDROMES PARANEOPLÁSICOS SE ASOCIAN CON MAYOR FRECUENCIA


A: CÁNCER DE PULMON

SON PATOLOGÍAS QUE PRODUCEN EDEMA POR DISMINUCION DE LA PRESIÓN OSMOTICA DEL
PLASMA: GLOMERULOPATÍAS Y MALNUTRICIÓN

ES UNA PROTEINA GRANDE DE ADHERENCIA CELULAR QUE LIGA MUCHAS MOLECULAS DE LA MEC Y
FORMA MATRIZ PROVISIONAL DURANTE LA CICATRIZACION: FIBRONECTINA

SON MEDIADORES QUIMICOS QUE INTERVIENEN EN LA ESTABILIZACION DE LOS VASOS RECIEN


FORMADO: ANGIOPOYETINAS Y FACTOR DE CRECIMIENTO DE PLAQUETAS (FCDP)

LA ASCITIS EN UN PACIENTE CON CIRROSIS ALCOHOLO NUTRICIONAL ES DEBIDO A: AUMENTO DE LA


PRESIÓN HIDROSTÁTICA Y OBSTRUCCIÓN LINFÁTICA

¿CUÁL DE LAS SIGUIENTES SUSTANCIAS INHIBEN LA AGREGACION PLAQUETARIA?: OXIDO NITRICO Y


PROSTACICLINA

SUSTANCIA QUE ACTUA COMO FACTOR PROCOAGULANTE: FACTOR TISULAR

- ES UN ESTADO DE HIPERCOAGULABILIDAD DE ALTO RIESGO: FIBRILACION AURICULAR

-ES UN ANTICOAGULANTE NATURAL: ANTITROMBINA

LA TROMBOSIS MESENTERICA ES UN EJEMPLO DE: INFARTO ROJO

EL ORIGEN MAS FRECUENTE DE LAS TROMBOEMBOLIAS PULMONARES ES: VENAS PROFUNDAS Y


SUPERFICIALES DE LOS MIEMBROS INFERIORES
LOS INFARTOS BLANCOS SE PRODUCEN CUANDO HAY: OBSTRUCCIÓN ARTERIAL EN ÓRGANOS
SÓLIDOS CON RIEGO ARTERIAL TERMINAL

ES UN POTENTE VASOCONSTRICTOR QUE SE PRESENTA AL INICIO DE LA LESIÓN VASCULAR:


ENDOTELINA

PERTENECEN AL GRUPO DE SHOCK DISTRIBUTIVO EXCEPTO: SHOCK ANAFILACTICO B.


SHOCKCARDIOGENICO C.SHOCK ENDOTOXICO D.SHOCK NEUROGENICO

- SON FACTORES SISTEMICOS QUE INFLUYEN EN LA REPARACIÓN DE LAS HERIDAS: INFECCIONES Y


CUERPOS EXTRAÑOS

SON TIPOS DE TROMBOS, EXCEPTO: A.ARTERIALES B. LIGAMENTOSOS, SUBSEROSOS


C.FLEBOTROMBOSIS D.MURALES

LAS NEOPLASIAS MALIGNAS PUEDEN SER RESISTENTES A LA APOPTOSIS POR DAÑO A: .BCL2

LA CAPACIDAD DE INVASION Y METASTASIS DE UNA NEOPLASIA MALIGNA ESTA DADA POR :


CAPACIDAD PARA DEGRADAR LA MEC

PROTEINAS DE LA MEC QUE DESESTABILIZAN LAS INTERACCIONES CELULA-MATRIZ PARA


PROMOVER LA ANGIOGENESIS: ESTROMIELISINAS

-NEOPLASIA MALIGNA DE LAS CELULAS GERMINALES DEL TESTICULO: SEMINOMA

SHOCK CARDIOGENICO-------------- TAPONAMIENTO CARDIACO

LINFOMA---------------------------------NEOPLASIA MALIGNA

INFARTO ROJO-------------------------- TORSION DE QUISTE DE OVARIO

ADENOMA PLEOMORFO----------------- NEOPLASIA BENIGNA MIXTA

SHOCK HIPOVOLÉMICO-------------------- QUEMADURAS

PREGUNTA 1

NEOPLASIA BENIGNA DE MÚSCULO ESTRIADO: RABDOMIOMA.

PREGUNTA 2

ES UNA CAUSA DE LA REDUCCIÓN DE LA PRESIÓN OSMÓTICA: SÍNDROME NEFRÓTICO

PREGUNTA 3

SON CONSIDERADAS CAUSA DE SHOCK CARDIÓGENO: INFARTO, ARRITMIAS VENTRICULARES

PREGUNTA 4

LA CICATRIZACION DE SEGUNDA INTENCION: SE PRESENTA EN HERIDAS ANFRACTUOSAS CON GRAN


PERDIDA DE TEJIDO

PREGUNTA 5
EL CARCINOMA OVÁRICO, EL FIBROSARCOMA Y OTROS SARCOMAS MESENQUIMATOSOS, SE HAN
RELACIONADO CON EL SIGUIENTE SÍNDROME PARANEOPLÁSICO: HIPOGLUCEMIA.

PREGUNTA 6

ES UNA CARACTERÍSTICA EXCLUSIVA DE LAS NEOPLASIAS MALIGNAS: METASTASIS

PREGUNTA 7

DESCRIBE EL DEPÓSITO EXTENSO DE COLÁGENO QUE SE REGISTRA EN PULMONES, HÍGADO, RIÑÓN


Y OTROS ÓRGANOS, COMO CONSECUENCIA DE LA INFLAMACIÓN CRÓNICA: FIBROSIS

PREGUNTA 8

SITIO DONDE FRECUENTEMENTE SE ORIGINA LA TROMBOSIS VENOSA PROFUNDA: EN LAS


EXTREMIDADES INFERIORES

PREGUNTA 9

UN ÉMBOLO GASEOSO PUEDE ORIGINARSE: POR CAMBIOS BRUSCOS EN LA PRESIÓN

PREGUNTA 10

SINDROME HEREDITARIO SE CARACTERIZA POR MULTIPLES POLIPOS ADENOMATOSOS DEL


INTESTINO GRUESO QUE SE MALIGNIZAN A ADENCARCINOMA: POLIPOSIS ADENOMATOSA FAMILIA

PREGUNTA 11

EN LA CIRROSIS HEPÁTICA AVANZADA SE PRODUCE EDEMA POR: DISMINUCIÓN DE LA PRESIÓN


ONCÓTICA DEL PLASMA

PREGUNTA 12

NEOPLASIA MALIGNA DE LA GLANDULA TIRODES: ADENOCARCINOMA

PREGUNTA 13

EL ORIGEN MÁS FRECUENTE DE LOS TROMBOS EN EMBOLIAS PULMONARES ES: VENAS PROFUNDAS
DE LAS PIERNAS

PREGUNTA 14

UNA DE LAS MANERAS EN LAS QUE UN TUMOR ESCAPA DE LA RESPUESTA INMUNE ES:
SOBREEXPRESANDO MOLÉCULAS CPH I

PREGUNTA 15

ES UN CARCINOGENO QUIMICO DE ORIGEN NATURAL: BENCIDINA

PREGUNTA 16

ES UN CARCINOGENO PRODUCIDO POR EL ASPERGILLUS FLAVUS: AFLATOXINA


PREGUNTA 17

LA PRODUCCIÓN DE ALFA FETOPROTEÍNA ES CARACTERÍSTICA DE: HEPATOCARCINOMAS

PREGUNTA 18

CONTIENE CÉLULAS O TEJIDOS MADUROS E INMADUROS RECONOCIBLES Y PERTENECIENTES A MÁS


DE UNA DE LAS CAPAS GERMINALES: TERATOMA

PREGUNTA 19

HELICOBACTER PYLORI SE HA RELACIONADO CON: ADENOCARCINOMAS Y LINFOMAS GÁSTRICOS.

PREGUNTA 20

CONSISTE EN ASPIRAR LAS CÉLULAS Y EL LÍQUIDO ASOCIADO CON UNA AGUJA DE PEQUEÑO
CALIBRE: PUNCIÓN ASPIRACIÓN CON AGUJA FINA.

PREGUNTA 21

LOS INFARTOS, CARACTERISTICAMENTE MUESTRAN: NECROSIS COAGULATIVA**

PREGUNTA 22

SE LE LLAMA ASÍ, A LOS TUMORES MALIGNOS QUE DERIVAN DE LOS VASOS SANGUÍNEOS:
ANGIOSARCOMA.

PREGUNTA 23

UN TROMBO CON LÍNEAS DE ZAHN: SE FORMÓ ANTE MORTEM (Contienen plaquetas, fibrina y
múltiples eritrocitos)

PREGUNTA 25

EL TIEMPO DE PROTROMBINA VALORA PRINCIPALMENTE EN EL LABORATORIO: FACTOR VII

PREGUNTA 28

ES UN RASGO CARACTERÍSTICO DE LAS NEOPLASIAS MALIGNAS. OCURRE EN CUATRO ETAPAS:


AFLOJAMIENTO DE LOS CONTACTOS INTERCELULARES, DEGRADACIÓN DE LA MEC, ADHESIÓN A
NUEVOS COMPONENTES DE LA MEC Y MIGRACIÓN DE LAS CÉLULAS TUMORALES: METÁSTASIS.

PREGUNTA 29

CONTRIBUYEN AL FENOTIPO MALIGNO DE LAS NEOPLASIAS: MUTACIONES CON PÉRDIDA DE LA


FUNCIÓN DE GENES.

PREGUNTA 30

LA CARACTERÍSTICA HISTOLÓGICA PREDOMINANTE DE UN INFARTO ES: NECROSIS COAGULATIVA


ISQUÉMICA.

PREGUNTA 31
LA DEFICIENCIA DE LA GLUCOPROTEINA GPIB PROVOCA LA ENFERMEDAD DE: BERNARD SOULIER

PREGUNTA 32

ENFERMEDAD QUE PUEDE CAUSAR PERDIDA HEMÁTICA: SANGRADO GENITAL ANORMAL

PREGUNTA 33

SON ESTADOS DE HIPERCOAGULABILIDAD DE ALTO RIESGO: CÁNCER, PROTESIS DE VÁLVULAS


CARDIACAS, INMOVILIZACIÓN PROLONGADA, INFARTO AL MIOCARDIO Y FIBRILACIÓN AURICULAR.

PREGUNTA 34

EL DAÑO GENÉTICO NO LETAL, ESENCIAL EN LA CARCINOGENIA PUEDE DEBERSE A: MUTACIONES.

PREGUNTA 35

SON EJEMPLOS DE CELULAS ESTABLES: PARÉNQUIMA DE ÓRGANOS SÓLIDOS, COMO HÍGADO,


RIÑÓN Y PÁNCREAS

PREGUNTA 36

NEOPLASIA MALIGNA DE LAS CELULAS MESOTELIALES: MESOTELIOMA

PREGUNTA 37

CON RESPECTO A LA DIFERENCIACIÓN CELULAR EN LAS NEOPLASIAS MALIGNAS: TODAS SON POCO
DIFERENCIADAS

PREGUNTA 38

CARACTERISTICA DE LAS SEÑALES AUTOCRINAS: SECRECIÓN DE NEUROTRANSMISORES EN UNIONES


CELULARES ESPECIALES

PREGUNTA 39

ESTOS GENES SE HAN RELACIONADO CON CARCINOMA FAMILIAR DE MAMA Y OVARIO; CARCINOMA
DE MAMA MASCULINO Y LEUCEMIA LINFOCÍTICA CRÓNICA: BRCA1 Y BRCA 2

PREGUNTA 41

NEOPLASIA MALIGNA DEL TEJIDO LINFOIDE: LINFANGIOSARCOMA

PREGUNTA 42

UN HOMBRE DE 60 AÑOS TIENE INSUFICIENCIA CARDIACA CONGESTIVA, RETENCION DE SODIO,


FOVEA, EDEMA DE MIEMBROS INFERIORES. EL PACIENTE PUEDE PROBABLEMENTE PRESENTAR
ELEVACION SÉRICA DE: ALDOSTERONA

PREGUNTA 43
TIPO DE SHOCK ASOCIADO A LIBERACIÓN MASIVA DE TOXINAS: SHOCK SÉPTICO

PREGUNTA 44

EN LA CICATRIZACIÓN POR PRIMERA INTENCIÓN, ¿EN QUÉ TIEMPO LA NEOVASCULARIZACIÓN


ALCANZA SU MÁXIMO Y EL TEJIDO DE GRANULACIÓN LLENA LA INCISIÓN?: 3 DIAS

PREGUNTA 45

QUE ALTERA EL FLUJO LAMINAR, EVITA LA DILUCIÓN DE LOS FACTORES DE COAGULACIÓN,


RETRASAN EL AFLUJO DE INHIBIDORES DE FACTORES DE COAGULACIÓN Y PERMITE LA FORMACIÓN
DE TROMBO: AUMENTO DE LA PERMEABILIDAD

PREGUNTA 46

UNA DE LAS RAZONES POR LAS CUALES LAS CÉLULAS NEOPLÁSICAS SON INMORTALES ES: EXPRESAN
TELOMERASA

PREGUNTA 47

ES LA FASE DEL SHOCK EN LA QUE SE ACTIVA EL EJE RENINA-ANGIOTENSINA: FASE NO PROGRESIVA

PREGUNTA 48

SON NEOPLASIA MIXTAS EXCEPTO: CONDROMA

PREGUNTA 49

ESTE TIPO DE CÉLULAS ESTÁN PRESENTES EN LA INFLAMACIÓN CRÓNICA, SON ABUNDANTES EN LAS
REACCIONES INMUNITARIAS MEDIADAS POR INMUNOGLOBULINA E Y EN LAS INFECCIONES
PARASITARIAS: EOSINÓFILOS

PREGUNTA 50

FACTOR DE CRECIMIENTO INVOLUCRADO EN LA REEPITELIZACION DE UNA HERIDA SUPERFICIAL:


FACTOR DE CRECIMIENTO EPIDERMICO

. fribroadenoma
A. ES ENCAPSULADO
B. ESTA FIJO A PLANOS PROFUNDOS
C. ES BLANCO GRISÁCEO
D. GENERALMENTE MIDE MÁS DE 4 CM

0.2 puntos
PREGUNTA 2
1. SE OBSERVA TRAS UNA DESTRUCCIÓN SUSTANCIAL DE TEJIDO, CUANDO LA LESIÓN INFLAMATORIA
AFECTA A TEJIDOS QUE NO SON CAPACES DE REGENERARSE O CUANDO HAY ABUNDANTE EXUDACIÓN
DE FIBRINA EN TEJIDOS O CAVIDADES SEROSAS, QUE NO ES POSIBLE ELIMINAR ADECUADAMENTE.
A. CURACIÓN POR REPOSICIÓN DE TEJIDO CONJUNTIVO

B. RESOLUCIÓN COMPLETA

C. REMISIÓN

D. INFLAMACIÓN CRÓNICA

0.2 puntos
PREGUNTA 3
1. SON LAS ETAPAS DE LA INVASIÓN Y METÁSTASIS DE UNA NEOPLASIA MALIGNA EXCEPTO:
A. FORMACIÓN DE NUEVA MATRIZ EXTRACELULAR

B. MIGRACIÓN DE CÉLULAS TUMORALES

C. DISMINUCIÓN DE CONTACTO INTERCELULAR POR CADHERINA E

D. DEGRADACIÓN DE LA MATRIZ EXTRACELULAR

0.2 puntos
PREGUNTA 4
1. ES UNA CAUSA DE LA REDUCCIÓN DE LA PRESIÓN OSMÓTICA:
A. SÍNDROME NEFRÓTICO

B. NEOPLASIAS.

C. ANEMIA FERROPÉNICA.

D. RADIACIÓN.

0.2 puntos
PREGUNTA 5
1. NEOPLASIA BENIGNA DEL CARTILAGO
A. ADENOMA PLEOMORFO

B. OSTEOSARCOMA
C. CONDROMA
D. LEIOMIOMA

0.2 puntos
PREGUNTA 6
1. SE REFIERE ASÍ, A LA PÉRDIDA PROGRESIVA DE LA GRASA CORPORAL Y DE LA MASA CORPORAL
MAGRA, ACOMPAÑADA DE UNA DEBILIDAD PROFUNDA, ANOREXIA Y ANEMIA:
A. EFECTOS LOCALES DEL CÁNCER.

B. SÍNDROMES PARANEOPLÁSICOS
C. EFECTOS HORMONALES DEL CÁNCER

D. CAQUEXIA CANCEROSA.
0.2 puntos
PREGUNTA 7
1. SUSTANCIAS QUE INHIBEN LA FUNCION PLAQUETARIA
A. ON Y PROSTACICLINA Y ADPASA
B. ANTITROMBINA
C. PLASMINOGENO
D. TROMBOPLASTINA

0.2 puntos
PREGUNTA 8
1. ES UN PROTEOGLUCANO:
A. SINDECANO

B. HEPARAN SULFATO
C. FIBRILINA
D. FIBRONECTINA

0.2 puntos
PREGUNTA 9
1. EL TEJIDO DE GRANULACION ESTA FORMADO POR:
A. TEJIDO CONECTIVO
B. VASOS DE NEOFORMACION Y DEPOSITO DE COLAGENO
C. FIBRILINA
D. REACCION INFLAMATORIA
0.2 puntos
PREGUNTA 10
1. SE CARACTERIZA POR EXUDACIÓN DE LÍQUIDO CON BAJO CONTENIDO DE CÉLULAS EN LOS ESPACIOS
CREADOS POR LA LESIÓN CELULAR O EN LAS CAVIDADES CORPORALES REVESTIDAS POR PERITONEO,
LA PLEURA O EL PERICARDIO.
A. INFLAMACIÓN PURULENTA.
B. INFLAMACIÓN SEROSA.

C. ABSCESO.

D. INFLAMACIÓN FIBRINOSA.

0.2 puntos
PREGUNTA 11
1. MORFOLÓGICAMENTE LAS NEOPLASIAS MALIGNAS SE CARACTERIZAN POR:
A. RELACIÓN NUCLEO CITOPLASMA NORMAL

B. ARQUITECTURA TISULAR INTACTA

C. PLEOMORFISMO NUCLEAR Y CELULAR

D. FIGURAS MITÓTICAS NORMALES

0.2 puntos
PREGUNTA 12
1. ENFERMEDAD GRANULOMATOSA NO NECROSANTE:
A. TUBERCULOSIS

B. SIFILIS

C. SARCOIDOSIS

D. ENFERMEDAD DE CROHN

0.2 puntos
PREGUNTA 13
1. NEOPLASIA MALIGNA DE LAS CELULAS MESOTELIALES
A. MIELOMA MULTIPLE

B. MELANOMA
C. MESOTELIOMA
D. EPITELIOMA

0.2 puntos
PREGUNTA 14
1. NEOPLASIA QUISTICA GLANDULAR MALIGNA
A. NEFROBLASTOMA

B. CISTOADENOMA
C. CISTOADENOCARCINOMA
D. CORISTOMA

0.2 puntos
PREGUNTA 15
1. LA CICATRIZACIÓN POR SEGUNDA INTENCIÓN:
A. SE CONTRAE LA CICATRIZ POR MIOFIBROBLASTOS

B. NO EXISTE CICATRIZ

C. LA RESPUESTA INFLAMATORIA ES BAJA

D. HAY POCA PÉRDIDA DE TEJIDO

0.2 puntos
PREGUNTA 16
1. SON GENES QUE SE DAÑAN EVENTUALMENTE EN LA MAYOR PARTE DE LAS NEOPLASIAS MALIGNAS
A. GENES REGULADORES DE LA APOPTOSIS.

B. GENES REPARADORES DEL ADN.

C. GENES SUPRESORES DE TUMORES.

D. PROTOONCOGENES.

0.2 puntos
PREGUNTA 17
1. EN LA TROMBOEMBOLIA SISTÉMICA, LA MAYORÍA DE LOS ÉMBOLOS SITÉMICOS PROVIENEN DE:
A. TROMBOSIS VENOSA SUPERFICIAL.
B. TROMBOS MURALES INTRACARDIACOS.

C. TROMBOSIS RENAL.

D. TROMBOSIS VENOSA PROFUNDA.

0.2 puntos
PREGUNTA 18
1. DENTRO DE LAS FUNCIONES DE P53 SE ENCUENTRAN, EXCEPTO:
A. PAUSAR EL CICLO CELULAR PARA REPARAR EL ADN

B. INDUCIR PROLIFERACIÓN CELULAR

C. INDUCIR APOPTOSIS

D. INDUCIR SENESCENCIA

0.2 puntos
PREGUNTA 19
1. EN LA EMBOLIA PULMONAR, LOS TROMBOS PROCEDEN PRINCIPALMENTE DE:
A. CAVIDADES CARDIACAS

B. RANDES VASOS ARTERIALES

C. VALVULAS CARDIACAS

D. VENAS PROFUNDAS DE LA PIERNA

0.2 puntos
PREGUNTA 20
1. LAS TURBULENCIAS PRODUCEN:
A. ATEROESCLEROSIS
B. TROMBOS ARTERIALES
C. TROMBOSIS VENOSA PROFUNDA
D. EDEMA

0.2 puntos
PREGUNTA 21
1. LA CICATRIZ DE PRIMERA INTENCIÓN ES DADA POR HERIDAS CON:
A. BORDES CON SEPSIS

B. BORDES NECROSADOS

C. BORDES ANFRACTUOSOS

D. BORDES ALINEADOS

0.2 puntos
PREGUNTA 22
1. CONTIENE CÉLULAS O TEJIDOS MADUROS O INMADUROS RECONOCIBLES Y PERTENECIENTES A MÁS
DE UNA DE LAS CAPAS GERMINALES (A VECES, INCLUSO, DE LAS TRES):
A. TERATOMA.

B. FIBROMA.
C. CONDROMA.

D. LINFOMA.

0.2 puntos
PREGUNTA 23
1. CARACTERÍSTICAS MORFOLÓGICAS DE UNA CICATRIZ:
A. NECROSIS Y CÉLULAS INFLAMATORIAS

B. CAPILARES DE NUEVA FORMACIÓN

C. MACRÓFAGOS DEL TIPO M2

D. ABUNDANTE COLÁGENO DENSO

0.2 puntos
PREGUNTA 24
1. TODAS LAS SIGUIENTES DESCRIBEN LAS FORMAS DE HEMORRAGIA EXCEPTO
A. HEMOPERICARDIO

B. PETEQUIAS

C. HEMOTORAX

D. HEMOSTASIA

0.2 puntos
PREGUNTA 25
1. DEL SHOCK ASOCIADO A INFLAMACIÓN SISTÉMICA, SON EJEMPLOS CLÍNICOS
A. DIARREA, TRAUMATISMOS Y QUEMADURAS.

B. NFARTO AL MIOCARDIO Y ROTURA VENTRICULAR.

C. INFECCIONES MICROBIANAS DEVASTADORAS .MICOSIS SISTEMICAS

D. HEMORRAGIA Y VÓMITOS.

0.2 puntos
PREGUNTA 26
1. EL VIRUS DE EPSTEIN BARR PRODUCE:
A. SARCOMA DE KAPOSI
B. HEPATOCARCINOMA

C. LINFOMA DE BURKITT
D. LEUCEMIA DE CELULAS PELUDAS

0.2 puntos
PREGUNTA 27
1. NEOPLASIA BENIGNA DE MÚSCULO ESTRIADO:
A. RABDOMIOMA.

B. LEIOMIOMA.

C. GLIOMA.

D. ADENOMIOMA.
0.2 puntos
PREGUNTA 28
1. SON EJEMPLOS DE SHOCK CARDIOGENICO EXCEPTO:
A. ARRITMIAS
B. TAPONAMIENTPO CARDIACO
C. TROMBOEMBOLIA PULMONAR
D. QUEMADURAS

0.2 puntos
PREGUNTA 29
1. CLÍNICAMENTE, EL PROCESO DE FIBROSIS ES COMÚN EN:
A. EPITELIOS

B. TEJIDOS LÁBILES

C. MUCOSAS

D. ÓRGANOS PARENQUIMATOSOS

0.2 puntos
PREGUNTA 30
1. H. PYLORI ESTÁ RELACIONADA CON EL:
A. ADENOMA GÁSTRICO

B. ADENOCARCINOMA GÁSTRICO

C. PÓLIPOS

D. LIPOMA

0.2 puntos
PREGUNTA 31
1. CÓMO SE CONOCE AL GASTO CARDIACO BAJO DEBIDO A UN VOLUMEN SANGUÍNEO REDUCIDO:
A. CHOQUE HIPOVOLÉMICO

B. CHOQUE TÓXICO

C. CHOQUE ASOCIADO A LA INFLAMACIÓN SISTÉMICA

D. CHOQUE CARDIOGÉNICO

0.2 puntos
PREGUNTA 32
1. SON ELEMENTOS DE LA TRIADA DE VIRCHOW, EXCEPTO:
A. ESTADOS DE HIPOCOAGULABILIDAD

B. LESIÓN ENDOTELIAL

C. ESTADOS DE HIPERCOAGULABILIDAD

D. TURBULENCIA DEL FLUJO

0.2 puntos
PREGUNTA 33
1. ES UN CARCINOGENO PRODUCIDO POR EL ASPERGILLUS FLAVUS:
A. CICLOFOSFAMIDA.

B. AFLATOXINA
C. HIDROCARBUROS AROMATICOS
D. CLORURO DE VINILO
0.2 puntos
PREGUNTA 34
1. EN ESTE TIPO DE SHOCK EXISTE VOLUMEN SANGUÍNEO O PLASMÁTICO INADECUADO:
A. SHOCK ASOCIADO A LA RESPUESTA INFLAMATORIA SISTÉMICA

B. SHOCK HIPOVOLEMICO

C. SHOCK CARDIÓGENO

D. SHOCK SÉPTICO

0.2 puntos
PREGUNTA 35
1. LA INDUCCIÓN DE FORMACIÓN DE ANTICUERPOS EN COMPLEJOS MOLECULARES DE HEPARINA Y A
PROTEÍNAS DEL FACTOR PLAQUETARIO, ES CARACTERÍSTICA DE:
A. SÍNDROME DE ANTICUERPO FOSFOLÍPIDO

B. SÍNDROME DE TROMBOCITOPENIA INDUCIDA POR HEPARINA

C. SÍNDROME DE CUSHING

D. MUTACIÓN DEL FACTOR V

0.2 puntos
PREGUNTA 36
1. SE DENOMINA SARCOMA A:
A. DNEOPLASIA MALIGNA DE TEJIDOS MESENQUIMATOSOS

B. NEOPLASIA MALIGNA DERIVADA DE TEJIDO LINFOIDE

C. NEOPLASIA MALIGNA EPITELIAL

D. NEOPLASIA MALIGNA DE LA SANGRE

0.2 puntos
PREGUNTA 37
1. CAUSA DE EDEMA EN LA DESNUTRICIÓN PROTÉICA
A. FILARIASIS

B. AUMENTO DE LA PRESIÓN ONCÓTICA

C. INSUFICIENCIA CARDIACA

D. DISMINUCIÓN DE LA PRESION ONCOTICA

0.2 puntos
PREGUNTA 38
1. SON UN ELEMENTO IMPORTANTE EN LA CONTRACCION DE LAS HERIDAS:
FIBROBLASTOS
MIOCITOS

MIOFIBROBLASTOS

MACROFAGOS

0.2 puntos
PREGUNTA 39
1. SON MOLECULAS DE ADHESION QUE UNEN LA SUPERFICIE CELULAR CON EL CITOESQUELETO Y LA
MEC
A. FIBRILINA CATENINAS
B. COLAGENO IV
C. CADHERINAS E INTEGRINAS
D. CATENINAS

0.2 puntos
PREGUNTA 40
1. SON CARACTERÍSTICAS DE LA CURACIÓN POR SEGUNDA INTENCIÓN
A. POCA PERDIDA DE TEJIDO CONJUNTIVO Y CELULAS EPITELIALES

B. SECUNDARIA A HERIDA QUIRÚRGICA LIMPIA Y ASÉPTICA

C. LA REACCIÓN INFLAMATORIA ES MAS INTENSA Y MAYOR CANTIDAD DE TEJIDO DE


GRANULACIÓN
D. BORDES APROXIMADOS POR SUTURA QUIRÚRGICA

0.2 puntos
PREGUNTA 41
1. . EL PROCESO DE REGENERACION SE LLEVA A CABO POR:
1.A. CUANDO TODO EL TEJIDO ESTÁ DAÑADO POR INFLAMACIÓN.

1.B. CUANDO ES PROMOVIDA POR FACTORES DE CRECIMIENTO Y LA INMTEGRIDAD DE


LA MATRIZ

1.C. CUANDO EL TEJIDO ES PARTE DE NEFRECTOMÍA UNILATERAL

D. CUANDO TODO EL TEJIDO ESTÁ DAÑADO POR INFECCIÓN

0.2 puntos
PREGUNTA 42
1. LOS INFARTOS ROJOS SE PRODUCEN EN CASOS DE:
A. ÓRGANOS CON DOBLE CIRCULACIÓN

B. EN LOS TEJIDOS DONDE LA DENSIDAD LIMITA LA SALIDA DE SANGRE DE CAPILARES


ADYACENTES AL ÁREA DE NECROSIS
C. ÓRGANOS SÓLIDOS CON IRRIGACIÓN TERMINAL
D. EN CORAZON Y RINON

0.2 puntos
PREGUNTA 43
1. LA MALIGNIZACIÓN DE UN PÓLIPO DEL APARATO DIGESTIVO VIENE
DETERMINADA POR LA INVASIÓN DE LAS CÉLULAS CANCEROSAS EN LA:
A. MUSCULAR DE LA MUCOSA**

B. MUCOSA
C. BASE DEL PEDÍCULO
D. SEROSA

0.2 puntos
PREGUNTA 44
1. ¿CUÁL ES EL PUNTO DE RESTRICCIÓN QUE DEBEN PASAR LAS CÉLULAS PARA GARANTIZAR LA
REPLICACIÓN GENÉTICA?
A. G1/S

B. G0/G
1
C. G0

D. G2/M

0.2 puntos
PREGUNTA 45
1. MARCADOR TUMORAL RELACIONADO CON CARCINOMAS DE COLON, PÁNCREAS, PULMÓN, ESTÓMAGO
Y CORAZÓN.
A. GONADOTROPINA CORIÓNICA HUMANA

B. ANTÍGENO CARCINOEMBRIONARIO

C. CALCITONINA.

D. ALFA FETOPROTEÍNA

0.2 puntos
PREGUNTA 46
1. SON PATOLOGÍAS QUE PRODUCEN EDEMA POR AUMENTO DE LA PRESIÓN HIDROSTATICA DEL VASO:
A. NEOPLASIAS Y FILARIAS

B. AUMENTO DE LA SECRECION RENINA-ANGIOTNSINA-ALDOSTERONA

C. GLOMERULOPATIAS Y MALNUTRICIÓN

D. PERICARDITIS CONSTRICTIVA Y TROMBOSIS

0.2 puntos
PREGUNTA 47
1. ES UNA MASA ANORMAL DE TEJIDO CUYO CRECIMIENTO EXCEDE Y NO ESTA COORDINADO CON EL DE
LOS TEJIDOS NORMALES Y PERSISTE DE LA MISMA MANERA EN EXCESO TRAS CESAR EL ESTÍMULO
QUE SUSCITÓ EL CAMBIO.
A. AGENESIA

B. NEOPLASIA

C. HIPOPLASIA

D. APLASIA

0.2 puntos
PREGUNTA 48
1. SON PROTEINAS COMPUESTAS POR TRES CADENAS POLIPEPTIDICAS DISTINTAS TRENZADAS EN
TRIPLE HELICE:
A. GLUCOPROTEINAS DE ADHESIVAS

B. PROTEOGLUCANOS

C. COLAGENOS

D. HIALURONATOS

0.2 puntos
PREGUNTA 49
1. ES EL COLAGENO QUE PROPORCIONA FIBRILLAS DE ANCLAJE DE LAS CELULAS EPITELIALES A LA
MEMBRANA BASAL
A. COLAGENO TIPO I

B. COLAGENO TIPO III


C. COLAGENO TIPO IV
D. COLAGENO TIPO VII

0.2 puntos
PREGUNTA 50
1. CONTRIBUYEN AL FENOTIPO MALIGNO DE LAS NEOPLASIAS:
A. MUTACIONES PASAJERAS.

B. MUTACIONES CON PÉRDIDA DE LA FUNCIÓN DE GENES.

C. MUTACIONES CONDUCTORAS.

D. MUTACIONES INICIADORAS.

polipo

DEFECTOS EN ESTOS GENES PUEDEN CONVERTIRLOS EN ONCOGENES Y CONTRIBUIR AL


CRECIMIENTO CÁNCER: PROTOONCOGENES

¿QUÉ VITAMINA ES NECESARIA PARA LA HIDROXILACIÓN DEL PROCOLAGENO?: VITAMINA C

PRECURSORES CELULARES ENDOTELIALES QUE PARTICIPAN EN LA VASCULOGÉNESIS:


ANGIOBLASTOS

LA CURACIÓN DE UNA HERIDA CON DAÑO CÉLULAR EXTENSO ES UN EJEMPLO DE: CURACIÓN POR
SEGUNDA INTENCIÓN
¿ALREDEDOR DE QUE DÍA SE FORMAN LOS PUENTES DE COLÁGENO EN LA CURACIÓN DE HERIDAS
POR PRIMERA INTENCIÓN?: ALREDEDOR DEL DÍA 5

CAMBIO EN LA DIFERENCIACIÓN DE UNA CÉLULA DE UN TIPO CELULAR A OTRO:


TRANSDIFERENCIACIÓN.

LA APOPTOSIS FISIOLOGICA SE PRESENTA EN: MAMA LACTANTE DURANTE EL DESTETE

CARACTERÍSTICA DE LA CALCIFICACIÓN DISTROFICA: SE PRESENTA EN ÁREAS DE NECROSIS

SU FORMACIÓN EN EL MARGEN DE LA HERIDA AYUDA A LA CONTRACCIÓN DE LA HERIDA:


MIOFIBROBLASTOS

SU EPITELIZACIÓN COMPLETA ES MÁS LENTA: CICATRICES DE SEGUNDA INTENCIÓN

SON EJEMPLOS DE INFLAMACIÓN SEROSA. EXCEPTO: APENDICITIS AGUDA SUPURATIVA

ES UNA ALTERACIÓN LOCAL DE LA CICATRIZACIÓN: BORDES CON TEJIDOS NECROZADOS

ES UN PROCESO ACTIVO EN EL CUAL LA DILATACIÓN ARTERIAL AUMENTA EL FLUJO DE SANGRE:


HIPEREMIA

ES UNA EXTRAVASACIÓN DE SANGRE HACIA EL ESPACIO EXTRAVASCULAR PUEDE SER EXTERNA O


QUEDAR CONTENIDA EN UN TEJIDO : HEMORRAGIA

Estos cálculos biliares se extirpan quirúrgicamente con la vesícula biliar de un obeso de 50 años.
Las mujeres se componen predominantemente de: Colesterol
EL PROCESO INICIAL DE VASOCONSTRICCIÓN TRAS UNA LESIÓN ESTA MEDIADO POR:
ENDOTELINA.

PROCESO PASIVO, DEBIDO A LA REDUCCIÓN DEL FLUJO DE SALIDA DE UN TEJIDO, ACUMULA


HEMOGLOBINA DESOXIGENADA: CONGESTIÓN

¿CUÁLES SON LOS COMPONENTES DE LA HEMOSTASIA?: ENDOTELIO, PLAQUETAS Y CASCADA DE


COAGULACIÓN

¿QUE FACILITA LA ADHERENCIA Y ACTIVACIÓN PLAQUETARIA? : LA EXPOSICIÓN DE LA MATRIZ


EXTRACELULAR MUY TROMBOGENICA

EMBOLO EN SILLA DE MONTAR: EMBOLIA PULMONAR

EL CUADRO SE CARACTERIZA POR LA SÚBITA APARICIÓN DE DISNEA, CIANOSIS Y SHOCK, SEGUIDO


DE ALTERACIONES NEUROLÓGICAS QUE VAN DESDE LA CEFALEA A CONVULSIONES Y COMA:
EMBOLIA DE LÍQUIDO ANMIÓTICO

EL SÍNDROME POR DESCOMPRESIÓN SE DA PRINCIPALMENTE EN: BUCEADORES

SE CONSIDERAN FACTORES DE RIESGO PARA PRODUCIR COLELITIASIS, EXCEPTO: DIETA RICA EN


FIBRA
ES LA PRINCIPAL COMPLICACIÓN DE LOS CÁLCULOS BILIARES POR OBSTRUCCIÓN DEL CUELLO DE LA
VESÍCULA O DEL CONDUCTO CÍSTICO: COLECISTITIS LITIASICA AGUDA

LOS CASOS MÁS GRAVES DONDE LA VESÍCULA BILIAR ESTÁ NECRÓTICA, VERDE NEGRUZCO Y PUEDE
PRESENTAR PERFORACIONES SE DENOMINA: COLECISTITIS GANGRENOSA

ENTRE LOS FACTORES DE RIESGO MÁS IMPORTANTES PARA LA PRESENTACIÓN DE IAM DESTACAN:
TABAQUISMO OBESIDAD, HIPERTENSIÓN ARTERIAL, DIABETES MELLITUS, HIPERCOLESTEROLEMIA Y
ATEROESCLEROSIS

ES EJEMPLO DE TROMBOSIS POR ESTADO DE HIPERCOAGULABILIDAD PRIMARIA: DEF. DE


ANTITROMBINA III Y DE PROTEÍNA C Y S

LA LESIÓN CELULAR POR TETRACLORURO DE CARBONO Y TYLENOL SE CARACTERIZAN POR: SU


CONVERSIÓN EN METABOLITOS ACTIVOS MUY TÓXICOS

A LA ACUMULACIÓN ANORMAL DE TRIGLICÉRIDOS DENTRO DE LAS CÉLULAS PARENQUIMATOSAS SE


LE LLAMA: ESTEATOSIS

ADAPTACIÓN CELULAR QUE CONSISTE EN EL AUMENTO EN EL NÚMERO DE CÉLULAS DE UN


ÓRGANO O TEJIDO CON EL CONSECUENTE AUMENTO DE VOLÚMEN DEL MISMO: HIPERPLASIA

A CÚAL DE LOS PIGMENTOS ENDÓGENOS QUE SE PUEDEN ACUMULAR INTRACELULARMENTE SE LE


LLAMA “PIGMENTO DEL ENVEJECIMIENTO”?: LIPOFUSCINA

¿QUÉ TIPO DE BIOPSIA SE UTILIZA PARA PADECIMIENTOS EN PROSTATA?: BIOPSIA POR PUNCIÓN

PROBLEMAS CLÍNICOS EN RIÑONES CAUSADOS POR UN SHOCK: OLIGURIA, ANURIA .

SI SE INMOVILIZA UNA EXTREMIDAD CON UN APARATO DE YESO POR TIEMPO PROLONGADO, LOS
MUSCULOS INMOVILIZADOS PRESENTARAN: ATROFIA

SE LLAMA PERDIDA DE LA POLARIDAD CELULAR NORMAL CUANDO: HAY PERDIDA DE LA


ORIENTACIÓN DE LAS CÉLULAS ANAPLÁSICAS

EL RANGO PROMEDIO PARA LA FIJACIÓN ADECUADA DE UN ESPÉCIMEN QUIRÚRGICO EN FORMOL


ES: DE 8 A 12 HORAS

LA TROMBOSIS ARTERIAL GENERALMENTE ES SECUNDARIA A: ATEROESCLEROSIS

EL GRADO EN QUE LAS CÉLULAS PARENQUIMATOSAS DE UNA NEOPLASIA SE PARECEN A LA


CONTRAPARTE NORMAL SE DENOMINA: DIFERENCIACIÓN

EL PRIMER PUNTO DE ATAQUE DE LA HIPOXIA EN LA CELULA ES: FOSFORILACION OXIDATIVA

¿CUÁL DE LOS SIGUIENTES ES ANTIOXIDANTES NO ENZIMÁTICO?: TRANSFERRINA

UN TUMOR ES CLÍNICAMENTE DETECTABLE CUANDO TIENE UN PESO DE 1 GRAMO, EL CUAL


CONTIENE: 10 9 CÉLULAS
LOS SIGUIENTES SON PIGMENTOS DERIVADOS DE LA HEMOGLOBINA: HEMOSIDERINA Y
BILIRRUBINA

CONDROSARCOMA…………….MESENQUIMATOSA

CORIOCARCINOMA……………..EPITELIAL MALIGNA

MOLA HIDATIFORME……………….MESENQUIMATOSA BENIGNA

PAPILOMA…………………………….EPITELIAL BENIGNA

GRANULOMA……………NO ES NEOPLASIA

1. Patología en la que se pierde albumina y genera edema:


Sindrome Nefrotico

2. Causantes de edema:
Enfermedad cardiaca y renal

3. Hematoma mayor a 2cm:


Equimosis

4. Causa fisiológica de hiperemia:


Vasodilatacion por ejercicio

5. Factor que inhibe la diseminación del trombo:


Activador tisular de plasminógeno

6. Capacidad de cambiar de un tipo celular a otro:


Transdiferenciasion

7. Es la capacidad de una célula de transdiferenciarse a distintas estirpes celulares:


Plasticidad de desarrollo

8. En esta fase de shock hay liberación de ADH y activación de baroreceptores:


Fase inicial no progresiva

9. Manifestaciones clínicas presentes en shock séptico:


Piel caliente con vasodilatación periférica, hiperemia.

10. Casi siempre oclusivo puente largo de la luz del vaso:


Trombosis venosa profunda

11. Tipo de necrosis encéfalo:


Licuefactiva

12. Síntomas de la embolia grasa:


Taquipnea, disnea y taquicardi, inquietud o coma.
13. Mantiene la integridad del epitelio externo de la cornea:
Celulas madre del limbo

14. Una neoplasia compuesta se divide como:


Dos capas celulares y una capa embrionaria

15. Metastasis:
Implantes tumorales sin continuidad con el tumor primario
16. Fases de la cascada metastasica:
Invasion de la MEC, diseminación vascular, alojamiento y colonización

17. Inhibicion de TP53 mediado por:


BAX

18. wich one of the following characteristics not acompany the cancer:
Fibrous capsule

19. What are the organs most affected by the hematogenous spread?
Liver and lung

20. Wich ARN virus has on oncogene activity?


Leucemia and cel T

21. Carcinogeno natural hepático secretado por plantas y hongos:


Aflatoxina B1

22. Causas frecuentes de esteatosis:


Alcoholismo y desnutrición

23. Medidador quimico que produce dilatación y aumento de la permeabilidad y se libera de


tipo físico:
Histamina

24. Son células que participan en la reacción inflamatoria excepto:


Pericitos

25. 25.- Las CX3C son moléculas de adhesión de:


Monocitos y Cel T

26. De que vitamina son dependientes las proteínas C y S:


Vitamina K

27. Localización más frecuente del carcinoma de células renales:


Polos
28. El cáncer renal causa frecuentemente metástasis en:
Pulmones y hueso

29. Causa más frecuente de cáncer renal:


Tabaquismo

30. Cual es la proteína viral que induce una rápida degradación de P53:
E6

31. Causa frecuente de metástasis en CACU:


Pielonefritis y Uremia

32. Lesión primaria de mayor riesgo de producir cáncer de mama:


Mastopatia proliferativa con atipia

33. Químico que dará activación quimiotáctica, actividad sobre monocitos y eosinófilos:
CXC

34. Sustancias que ocasionan fiebre, sueño y perdida del apetito:


Citocinas

35. La inflamación fibrinosa se caracteriza por:


Fluidos corporales en pericardio y pleura

36. Caracteristicas histológicas de órganos que sufren atrofia, hay presencia de:
Formacion de vacuolas autofagicas

37. Los leiomiomas se dividen en:


Submucosos, subserosos e intramurales

38. Las enfermedades crónicas granulomatosas abarcan varios trastornos que tienen en común:
La incapacidad de destruir agentes etiológicos

39. El infarto de miocardio corresponde a una:


cardiopatía isquémica

40. Todos las siguientes sustancias pertenecen al tren de tincion, excepto:

● Hematoxilina SI
● Orange G SI
● Eosina SI

41. La citología exfoliativa puede reportar todo, excepto:


El estado nutricional de la células.

42. Carcinoma in situ ductual consiste en:


Poblacion maligna de células que no pueden atravesar la Membrana Basal
43. La fosfofructocinasa y la fosforilaza en lesión, activan a:
Glucolisis Anaerobia

44. ¿Qué moléculas emiten la adhesión de plaquetas al endotelio no lesionado?:


Prostacicilinas y Oxido Nitrico
45. Para la formación de granulomas se requiere:
reacción inmunitaria mediada por células T

46. Pancreatitis aguda ------------------------ Necrosis Grasa

47. Infección Bacteriana----------------------N. Licuefactiva

48. Muerte celular----------------------------- Apoptosis

49. Tuberculosis------------------------------- N. caseosa

50. Infarto al miocardio--------------------- N. Coagulativa

51. Neoplasias epiteliales glandulares malignas:


Cistoadenocarcinomas

52. Neoplasia maligna de los vasos linfáticos:


Linfangiosarcoma

53. Es el cambio fundamental para obtener potencial replicativo ilimitado:


Activación de la telomerasa

54. Los carcinógenos indirectos necesitan:


Activación metabólica.

55. Los síndromes paraneoplásicos pueden producir excepto:

● Trastornos dermatológicos SI
● Endocrinopatías SI
● Síndromes nerviosos y muscular SI
● Cambios vasculares y hematológicos
● Cambios oseos
56. Los tejidos que tiene un nivel de replicación bajo pero que pueden sufrir divisiones rápidas
en respuesta a estímulos específicos son:
Quiescientes

57. Es el cambio fundamental para obtener potencial replicativo ilimitado:


Activación de la telomerasa.

58. La caquexia se debe a:


Secreción del factor de necrosis tumoral.

59. Son lesiones subletales del DNA y dianas genéticas para el desarrollo de neoplasias malignas
excepto.
● Oncogenes SI
● Genes de la apoptosis SI
● Genes supresores del cáncer (p53. P21, RB) SI
● Genes reparadores del DNA SI

60. La insensibilidad a las señales inhibitorias del crecimiento esta dado por:
RB, P53

61. Los marcadores tumorales pueden ser excepto:


● Isoenzima SI
● Antígenos oncofetales SI
● Hormonas SI

62. Las células madre se caracterizan por:


Capacidad de autorrenovación y replicación asimétrica.

63. La lesión subletal por radiación ultravioleta es por:


Formación de dímero de pirimidina.

64. El VEGF y las angiopoyetinas intervinen en procesos como:


Vasculogénesis y angiogénesis

65. Las proteínas E7 y E6 de VPH de alto grado se unen al gen:


RB y P53

66. Enzimas encargadas de la degradación del colágeno y otras proteínas de la matriz


extracelular:
Metaloproteinasas

67. Son cambios cromosómicos en las neoplasias, excepto:


● Amplificación SI
● Traslocación SI
● Deleción SI

68. Es una característica exclusiva de las neoplasias malignas:


Metástasis

69. El parecido entre una célula neoplásica y la contraparte normal se denomina grado de:
Diferenciación.

70. Son células diferenciadas reprogramadas:


Células madre inducidas

71. Entre las oncoproteínas (productos protéicos de los oncogenes) tenemos:


BCL2

72. Son marcadores tumorales:


Antígeno prostático, Antígeno carcinoembrinario, alfa fetoproteína.

73. Carcinógenos químicos excepto:


● Agentes alquilantes Si
● Aflatoxina B Si
● Aminas aromáticas Si
● Antibióticos NO

74. Uno de los genes mutado en la mayor parte de los cánceres humanos es:
P53

75. Se define como ausencia de diferenciación:


Anaplasia

76. Las endocrinopatías de síndromes paraneoplásicos se asocian en mayor frecuencia a:


Cáncer de pulmón.

77. Son patologías que producen edema por disminución de la presión osmótica en el plasma:
Glomerulopatías y mal nutrición.

78. Es una proteína grande de adherencia celular que liga muchas moléculas de la MEC y forma
cicatriz provisional durante la cicatrización:
Fibronectina

79. Son mediadores químicos que intervienen en la estabilización de los vasos recién formados.
Angiopoyetina y FCDP

80. La ascitis en un paciente con cirrosis alcohólica nutricional es debido a:


Disminución de la presión osmótica y aumento de la presión hidrostática.

81. ¿Cuál de las siguientes sustancias inhiben la agregación plaquetaria?


Prostaciclina y óxido nítrico.

82. Sustancias que actúan como factor coagulante


Factor tisular

83. Es un estado de hipercoagulabilidad de alto riesgo


Fibrilación auricular, reposo en cama, infarto al miocardio.
84. Es un anticoagulante natural:
Antitrombina

85. La trombosis mesentérica es un ejemplo de:


Infarto rojo

86. El origen más frecuente de las tromboembolias pulmonares:


Trombosis en venas profundas y superficiales de los miembros inferiores.

87. Los infartos blancos se producen cuando hay:


Obstrucción arterial en órganos sólidos con riego arterial terminal.

88. Es un potente vasoconstrictor que se presenta al inicio de la lesión endotelial:


Endotelina

89. Pertenecen al tipo de shock distributivo excepto:


● Anafiláctico SI
● Endotóxico Si
● Neurogénico Si
● Cardiogénico NO

90. Son factores sistémicos que influyen en la reparación de las heridas:


Infecciones y cuerpos extraños

91. Son tipos de trombos excepto:


● Ligamentosos y subserosos. NO
● Murales SI
● Flebotrombosis SI
● Arteriales SI

92. Las neoplasias malignas pueden ser resistentes a la apoptosis por daño a:
BCL 2

93. La capacidad de invasión y metástasis de una neoplasia maligna esta dada por:
Capacidad para degradar la MEC

94. Proteínas de la MEC que desestabilizan las interacciones célula – matriz para promover la
angiogénesis:
Estromelisinas (Metaloproteasa)

95. Neoplasia maligna de las células germinales del testículo:


Seminoma

96. Shock cardiogénico


Taponamiento cardiaco
97. Linfoma
Neoplasia maligna

98. Infarto rojo:


Torsión de quiste ovárico

99. Adenoma pleomorfo:


Neoplasia benigna mixta.

100. Shock hipovolémico


Quemaduras

1. El factor con menos probabilidad de causar trombosis:


2. A la hemorragia diminuta de 1-2mm en la piel mucosa o serosa asociada a trombocitopenia
o déficit de factores de coagulación se llama: Petequias
3. La carencia de protrombina origina: Incompatible con la vida
4. La carencia de V, VII, VIII, IX y X: Se asocian a unos trastorboa hemorrágicos de moderados
a graves.
5. ¿Qué es lo que mantiene a las neoplasias benignas como masas delimitadas, fácilmente
palpables y móviles? Ribete de tejido fibroso, llamada cápsula, compuesta por células
estromales, del tipo fibroblastos.
6. Factor de transcripción que regula la expresión de genes como ciclinas: RB
7. La estadificación de los cánceres sólidos se basa en que sistema:TNM del American Joint
Committe on Cancer
1. Se describe como la sustitución de un tipo de célula por otro, se asocia generalmente a
daño, reparación o regeneración del tejido: Metaplasia
1. Neoplasia benigna del músculo estriado: Rabdomioma
2. La trombocitopenia (plaquetopenia) y la uremia se pueden manifestar clínicamente con:
Diatésis hemorrágica
3. En el proceso de cicatrización hay una producción exuberante de colágeno, lo que da lugar a:
cicatrización queloide
4. Es un polisacárido de la familia de los glucosaminoglucanos:
5. ¿Cuál es la respuesta correcta que inhibe la cicatrización? Glucocorticoides
6. La activación plaquetaria está dada por: Adhesión y cambio de forma
7. El shock cardiogénico se produce cuando existe una pérdida de la masa muscular del
ventrículo izquierdo del: 45%
8. El plasminógeno realiza las siguientes actividades excepto: Convierte el fibrinógeno en
fibrina
9. LA VÍA DE PROPAGACIÓN MÁS COMÚN DE LOS CARCINOMAS ES:

A. SIEMBRA QUIRÚRGICA
B. VÍA HEMATÓGENA
C.SIEMBRA DIRECTA
D.VÍA LINFÁTICA
10. la infeccion por helicobacter pylori esta relacionada con que tipo de neoplasias: R-
adenocarcinoma gastrico, linfoma malt
11. LA CICATRIZACION DE SEGUNDA INTENCION:

A. SE OBSERVA EN HERIDAS DE OPERACIONES QUIRURGICAS

B. SE PRESENTA EN HERIDAS ANFRACTUOSAS CON GRAN PERDIDA DE TEJIDO

C. OCURRE EN FORMA RAPIDA, SIN DEJAR CICATRIZ

D. NO REQUIERE TRATAMIENTO
12. Son fases del shock: Fase no progresiva, fase progresiva y fase reversible
13. Son ejemplos de cánceres familiares excepto:Fibroadenoma mamario
14. Como se conoce al gasto cardíaco bajo debido a un volumen sanguíneo reducido: Shock
hipovolémico
15. Es una causa de edema por disminución de la presión oncótica: Desnutrición
16. Sufren una conmutación metabólica hacia la glucólisis aerobica(efecto warburg), que facilita
la síntesis de macromoléculas y orgánulos requeridos para un crecimiento celular rápido R
alteración del metabolismo celular.

20 CÉLULAS QUE SE DETIENEN EN FASE G 0 Y FORMAN LA MAYOR PARTE DE LOS ÓRGANOS


PARENQUIMATOSO COMO HÍGADO Y RIÑONES. R- CÉLULAS ESTABLES,+. (RECORDATORIO: LAS
LÁBILES SON LAS QUE CONSTANTEMENTE ENTRAN EN DIVISIÓN Y LAS PERMANENTES SON
TEJIDOS DIFERENCIADOS TERMINADOS , NEURONAS M, CARDIACO)

21 CUÁL ES EL AGENTE BIOLÓGICO IMPLICADO EN EL LINFOMA DE BURKITT: R- VIRUS DE EPSTEIN


BARR

22 SON CONSIDERADAS CAUSAS DE SHOCK CARDIOGÉNICO: R- INFARTO, ARRITMIAS


VENTRICULARES.

24 NEOPLASIA MALIGNA DE CÉLULAS MESOTELIALES- R. MESOTELIOMA

25 UNA LESIÓN ENDOTELIAL TENDRÁ COMO CONSECUENCIA -R: DESARROLLO DE UNA AMBIENTE
PROCOAGULANTE.

29 LOS GRÁNULOS DENSOS DE LAS PLAQUETAS CONTIENEN EXCEPTO - R: FACTOR DE VON


WILLEBRAND ( TIENE ADP, CALCIO, ADRENALINA Y SEROTONINA)

35 LA CÉLULAS DE ESTOS TEJIDO SE DESTRUYEN Y SE REPONEN CONTINUAMENTE POR


MADURACIÓN, A PARTIR DE CÉLULAS MADRE ADULTAS Y POR PROLIFERACIÓN DE CÉLULAS
MADURAS - R: TEJIDOS LÁBILES

Sustancia producida por las células endoteliales que produce vasoconstricción en caso de ruptura
vascular: Endotelina

La estasis de la sangre en capilares con dificultad para el flujo de sangre por el extremo venoso por
trombos o presión retrógrada causada por la insuficiencia cardiaca se asocia: Congestión crónica.
Es la enfermedad por compresión con aparición de burbujas de nitrógeno: ES TÍPICA DE
BUCEADORES

Son lesiones precursoras de cáncer: ESÓFAGO DE BARRETT , METAPLASIA ESCAMOSA DE LA


MUCOSA BRONQUIAL, HIPERPLASIA ENDOMETRIAL, Y LEUCOPLASIA.

Los genes de receptores con actividad tirosin cinasa intrínseca son ejemplos de: Protooncogenes
oncogenes

Se produce cuando la herida se abre parcialmente cicatrizada, el término correcto es:Dehiscencia

El carcinoma de células renales, el hemangioma cerebeloso, y el hepatocarcinoma , se han


relacionado con el siguiente síndrome paraneoplásico: Policitemia.

LAS CÉLULAS Th17 SECRETAN: IL-17

En los tejidos lábiles el tipo de reparación de una lesión leve es: Regeneración

Con respecto a la diferenciación celular en las neoplasias malignas: Pueden ser bien,
moderadamente o poco diferenciadas.

Líquido presente en el edema por aumento de la presión hidrostática dentro de los vasos
sanguíneos: Exudado.

En esta fase del choque se presenta insuficiencia renal por necrosis tubular aguda que la
supervivencia es imposible: Fase irreversible.

La respuesta inmune hacia los tumores esta mediada principalmente por: Linfocitos T

Marcador hormonal de tipo hormonal: calcitonina


SINDROMES PARANEOPLASICOS
MARCADORES TUMORALES
ESTADOS DE HIPERCOAGULABILIDAD
ENFERMEDADES CAUSADAS POR MAL PLEGAMIENTO DE PROTEINAS
APOPTOSIS
INFLAMACION AGUDA
MEDIADORES DE LA INFLAMACIÓN
EJEMPLOS DE ENFERMEDADES CON INFLAMACIÓN GRANULOMATOSA.
TUMORES MALIGNOS Y BENIGNOS
COMPARACIÓN ENTRE TUMORES BENIGNOS Y MALIGNOS
CANCER OCUPACIONAL
ESTADO DE INFLAMACION CRÓNICA Y CANCER
PROTONCOGENES Y ONCOGENES SELECTOS
COMPONENTES DEL CICLO CELULAR E INHIBIDORES QUE MUTAN EN CANCER
GENES SUPRESORES Y SINDROMES Y CANCERES FAMILIARES ASOCIADOS.
INHIBIDORES DE LAS VÍAS DE SEÑALIZACIÓN MITÓGENAS
ONCOGENES CREADOS POR TRASLOCACIONES
CARCINOGENOS QUIMICOS
ALQUILANTES, ACILADORES,
HIDROCARBUROS, AMINAS, HIERBAS
NATURALES
VACUNA VIRUS DEL PAPILOMA HUMANO VPH

Gardasil, Gardasil 9 y Cervarix. Las tres vacunas protegen de la infección por los
tipos 16 y 18 de VPH, dos de los virus del papiloma humano de alto riesgo que
causan cerca de 70 % de los cánceres de cuello uterino y un porcentaje todavía
mayor de algunos de los otros cánceres causados por el VPH (1, 2). Gardasil
protege también de la infección por los tipos 6 y 11 de VPH, los cuales causan 90 %
de las verrugas genitales (3). Gardasil 9 protege de la infección por los mismos
cuatro tipos de VPH y otros cinco tipos de VPH que causan cáncer (31, 33, 45, 52 y
58).
1. Son ejemplos clínicos de choque asociado a inflamación sistémica
a. Infarto al miocardio y rotura ventricular
b. Traumatismos y quemadoras
c. Hemorragia y Vómitos
d. Infecciones microbianas devastadoras y pancreatitis
2. ¿Cuáles son las características del tejido de granulación?
a. Rico en colágeno
b. Ricos en tejido conjuntivo provisional y nuevos capilares finos
c. Bajo en fibroblastos
d. Abundantes células inflamatorias
3. ¿Cómo se le llaman a las células que detienen la fase G0 y forman la mayor parte de los
órganos Parenquimatosos como hígado y riñón etc?
a. Células estables
b. Células postmitóticas
c. Células lábiles
d. Células permanentes
4. ¿Cuál es el agente biológico implicado en el linfoma de burkit?
a. Virus de Epstein-Barr
b. Virus del papiloma humano
c. Ninguna de las anteriores
d. Virus B y C de la hepatitis
5. El cáncer es una enfermedad caracterizada por
a. Lesión subletal al DNA
b. Adquisición de mutaciones letales
c. Senecencia celular
d. Daño genético letal
6. Neoplasia maligna de los vasos sanguíneos
a. Linfoma
b. Linfanngiosarcoma
c. Leucemia
d. Angiosarcoma
7. Mutación de gen supresor de tumores más frecuente encontrada en los cánceres
humanos
a. MYC
b. P53
c. RAS
d. RB
8. Neoplasia maligna de las células mesoteliales
a. Mieloma múltiple
b. Melanoma
c. Mesotelioma
d. Epitelioma
9. La infección por Helicobacter Pylori esta relacionada con qué tipo de neoplasias
a. Carcinoma pancreático
b. Hepatocarcinoma
c. Colangiocarcinoma
d. Adenocarcinoma gástrico, linfoma de MALT
10. Sufren una conmutación metabólica hacia la glucólisis aeróbica (efecto de warburg) Que
afecta la síntesis de macromoléculas y orgánulos requeridos para un crecimiento celular
rápido
a. Insensibilidad a las señales inhibitorias del crecimiento
b. Evasión de la apoptosis
c. Potencial ilimitado de la replicación
d. Alteración del metabolismo celular
11. Son consideradas causas de shock cardiogénico
a. Infarto, arritmias ventriculares
b. Accidentes anestésicos
c. Reacción inmunitaria severa
d. Hemorragias masivas
12. El virus de Epstein Barr produce
a. Leucemia de células peludas
b. Linfoma de Burkitt
c. Hepatocarcinoma
d. Sarcoma de Kaposi
13. Una lesión endotelial tendrá como consecuencia
a. Produce la disolución de un coagulo
b. Desarrollo de un ambiente procoagulante
c. No tiene relación con la coagulación
d. Desarrollo de un ambiente anticoagulante
14. Es el cambio biológico necesario para la progresión de las neoplasias malignas
a. Evasión de la apoptosis
b. Metástasis
c. Potencial replicativo ilimitado
d. Angiogenia
15. En la inmortalidad de las células malignas es considerado como un factor decisivo
a. Reordenamiento de cromosomas
b. Capacidad de regulación
c. Evasión de la senescencia
d. Mutación puntual
16. En este tipo de shock hay insuficiencia de la bomba miocárdica debido a lesiones
intrínsecas del miocárdico
a. Shock séptico
b. Shock Hipovolémico
c. Shock Cardiogénico
d. Shock hemorrágico
17. El edema por obstrucción linfática se da por las siguientes condiciones excepto:
a. Neoplásica
b. Postquirúrgica
c. Postirradiación
d. Postquimioterapia
18. Las células de estos tejidos se distribuyen y se reponen continuamente por maduración a
partir de células madre adultas y por proliferación de células maduras
a. Tejidos proliferativos
b. Tejidos permanentes
c. Tejidos lábiles
d. D tejidos estables
19. ¿Qué es lo que mantiene a las neoplasias benignas como masas delimitadas, fácilmente
palpables y móviles?
a. Sú parénquima y estroma
b. Cápsula fibrosa
c. La diferenciación de sus células
d. Su crecimiento lento
20. La llamada “induración parda” es un ejemplo, excepto
a. Congestión pasiva crónica esplénica
b. Hemorragia de tubo digestivo
c. Congestión pasiva crónica pulmonar
d. Congestión pasiva crónica pulmonar
21. Son neoplasias mixtas excepto
a. Fibroadenoma
b. Condroma
c. Adenoma pleomorfo
d. Tumor de Wilms
22. Una de las características principales del shock es
a. Hipertensión
b. Estado de hipoperfusión tisular
c. Aumento de la perfusión tisular
d. Aumento del volumen sanguíneo circulante
23. Son áreas de necrosis isquémicas causadas con más frecuencia por oclusión arterial debida
típicamente a trombosis o embolia:
a. Trombosis
b. Infarto
c. Isquemia
d. Coagulopatía por consumo
24. Es la enfermedad por descompresión con aparición de burbujas de nitrógeno
a. Tromboflebitis
b. Tromboembolia séptica
c. Es típica de la CID
d. Es típica de buceadores
25. ¿en qué órganos la regeneración no es posible?
a. Piel, cavidad, oral, cuello uterino
b. Vejiga, glándulas salivales
c. Músculo liso, hígado
d. Corazón, encéfalo
26. Sustancia producida por las células endoteliales que produce vasoconstricción en caso de
ruptura vascular
a. Histamina
b. Prostaciclina
c. Endotelina
d. Oxido nítrico
27. El Gen CDH1, inhibidor de invasión y metástasis se ha relacionado con las siguientes
neoplasias
a. Carcinoma gástrico, y carcinoma lobulillar de mama
b. Carcinomas de estómago, colon páncreas y melanoma
c. Carcinoma basocelular y meduloblastoma
d. Schwannoma y meningioma
28. La estasis de la sangre en capilares con dificultad para el flujo
a. Hiperemia activa
b. Hiperemia pasiva
c. Congestión crónica
d. Congestión pasiva crónica
29. A la hemorragia diminuta de 1-2mm en la piel, mucosas o serosas asociada a la
trombocitopenia o déficit de factores de la coagulación de le llama:
a. Petequia
b. Hematoma
c. Equimosis
d. Púrpura
30. Son lesiones precursoras de cáncer
a. Hepatitis y osteomelitis
b. Linfoma MALT y ácidos biliares
c. Úlceras trombos y helicobarcter pylori
d. Esófago de Barret, metaplasia escamosa de la mucosa bronquial, hiperplasia
endometrial y leucoplasia
31. Estos trombos se forman en la circulación venosa, suele contener más eritrocitos
incorporados y relativamente pocas plaquetas
a. Trombos arteriales
b. Trombos rojos
c. Trombos murales
d. Vegetaciones
32. En la formación de un trombo interviene
a. Estados de hipocoagulabilidad
b. Sólo a y b
c. Daño endotelial
d. Lentitud y turbulencia del flujo
33. La asbestosis y silicosis está asociada con qué tipo de neoplasia
a. Hepatocarcinoma
b. Carcinoma colorrectal
c. Mesotelioma, carcinoma de pulmón
d. Carcinoma pancreático
34. El gen Rb se encuentra en estado inactivo
a. Hipofosforilado unido a E2F
b. Hiperfosforilado separado de E2F
c. Inducido
d. Unido a E2F
35. Analiza la aparición de tumores en poblaciones humanas
a. Ninguna de las anteriores
b. Servicios coordinados
c. Salud pública
d. Epidemiología del cáncer
36. Es un tipo de tumor benigno
a. Linfoma
b. Leucemia
c. Adenoma
d. Sarcoma
37. La trombocitopenia (plaquetopenia) y la uremia se pueden manifestar clínicamente con;
a. Hematomas
b. Plaquetas
c. Hipermenorrea
d. Púrpura
38. Contiene células o tejidos maduros e inmaduros reconocibles y pertenecientes a más de
una de las capas germinales
a. Teratoma
b. Hamartoma
c. Adenoma pleomorfo
d. Desmoplasia
39. Cual enunciado acerca de las plaquetas es verdad
a. Contiene actina y miosina, microfilamentos y microtúbulos
b. Los precursores son monocitos
c. Contiene cuerpos densos ricos en fibrinógeno
d. Contiene gránulos alfa ricos en fosfato de adenosina
40. Neoplasia benigna mixta
a. Adenoma pleomorfo
b. Osteosarcoma
c. Condroma
d. Leiomioma
41. Tiene mayores probabilidades de sufrir infarto rojo, excepto
a. Intestino delgado
b. Pulmón
c. Riñón
d. Órganos con circulación doble
42. Los tumores tienen capacidad de proliferar sin estímulos externos, en general, como
consecuencia de la activación
a. Insensibilidad de las señales inhibidoras del crecimiento
b. Alteración del metabolismo celular
c. Autosuficiencia de las señales de crecimiento
d. Evasión de la apoptosis
43. El tiempo de protrombina valora principalmente en el laboratorio
a. Factor VII
b. Factor III
c. Anticoagulación con heparina
d. Factor VIII
44. El carcinoma ovárico, el fibrosarcoma y otros sarcomas mesenquimatosos, se han
relacionado con el siguiente síndrome paraneoplásico
a. Síndrome de secreción inadecuada de hormona antidiurética
b. Hipercalcemia
c. Síndrome de Cushing
d. Hipoglucemia
45. Los gránulos densos de las plaquetas contienen, excepto
a. ADP
b. Calcio
c. Factor de Von Willebrand
d. Adrenalina y serotonina
46. Los principales factores de crecimiento implicados en la síntesis de tejido conjuntivo
excepto:
a. TGF-Beta
b. FGF
c. PDGF
d. IL-1
47. En el choque séptico se puede producir
a. Todas las anteriores
b. Daño alveolar difuso
c. Enteropatía hemorrágica
d. Encefalopatía isquémica
48. Se puede producir choque por:
a. Sepsis
b. Todas las anteriores
c. Quemaduras
d. Infarto del miocardio
49. Describe cómo la sustitución de un tipo de célula por otro, se asocia generalmente con a
daño, reparación o regeneración del tejido:
a. Metaplasia
b. Desmoplasia
c. Pleomorfismo
d. Displasia
50. Dentro de las funciones de p53 se encuentran, excepto
a. Inducir senescencia
b. Pausar el ciclo celular para reparar el ADN
c. Inducir apoptosis
d. Inducir proliferación
51. SON NEOPLASIAS EPITELIALES GLANDULARES MALIGNAS QUÍSTICAS:
a. CARCINOMAS
b. SARCOMAS
c. ESCLEROMAS
d. CISTOADENOCARCINOMAS
52. NEOPLASIA MALIGNA DE LOS VASOS LINFÁTICOS:
a. A.HEMANGIOMA
b. B. ANGIOSARCOMA
c. C.LINFOMA
d. D.LINFANGIOSARCOMA
53. ES EL CAMBIO FUNDAMENTAL PARA OBTENER POTECIAL REPLICATIVO ILIMITADO
a. A.P53,P21,P16
b. B. ACTIVACION DE LA TELOMERASA
c. C.HIPERFOSFORILACION DE rb
d. D.EXPRESION DE ONCOGENES
54. LOS CARCINÓGENOS INDIRECTOS NECESITAN:
a. A.ACTIVACIÓN METABÓLICA
b. B. COCARCINÓGENOS
c. C.AGENTES ALQUILANTES
d. D.AGENTES ACILANTES
55. LOS SÍNDROMES PARANEOPLÁSICOS PUEDEN PRODUCIR EXCEPTO:
a. A. TRASTORNOS DERMATOLOGICOS
b. B. ENDOCRINOPATIAS
c. C. SEPSIS
d. D.CAMBIOS VASCULARES Y HEMATOLOGICOS
56. LOS TEJIDOS QUE TIENE UN NIVEL DE REPLICACION BAJO PERO QUE PUEDEN SUFRIR
DIVISIONES RAPIDAS EN RESPUESTA A ESTIMULOS ESPECIFICOS SON:
a. A. LABILES
b. B. CELULAS MADRE
c. C. PERMANENTES
d. D. QUIESCENTES
57. 7.-LA CAQUEXIA DEL CANCER SE DEBE A:
a. A. LA NEOPLASIA COMPITE POR LOS NUTRIENTES CON EL HUESPED
b. B. ANOREXIA
c. C. SECRECION DE FACTOR DE NECROSIS TUMORAL
d. D.DETERIORO GENERAL
58. SON LESIONES SUBLETALES DEL DNA Y DIANAS GENETICAS PARA EL DESARROLLO DE
NEOPLASIAS MALIGNAS EXCEPTO
a. A. ONCOGENES
b. B. GENES DE LA APOPTOSIS
c. C. GENES SUPRESORES DEL CANCER (P53, P21, RB)
d. D. GENES REPARADORES DEL DNA
e. E. GENES REPRESORES
59. LA INSENSIBILIDAD A LAS SENALES INHIBITORIAS DEL CRECIMIENTO ESTA DADO POR
LESION A :
a. A.BRCA-1
b. B. RB,P53
c. C.BCL2
d. D.BAX
60. LOS MARCADORES TUMORALES PUEDEN SER EXCEPTO:
a. A.ISOENZIMAS
b. B. ANTIGENOS ONCOFETALES
c. C.HORMONAS
d. D.CITOCINAS
61. LAS CELULAS MADRE SE CARACTERIZAN POR:
a. A. SINTETIZAR HORMONAS
b. B. CAPACIDAD DE AUTORENOVACION Y REPLICACIÓN ASIMETRICA
c. C. CAPACIDAD DE SÍNTESIS PROTEICA
d. D. RESPONDER A ESTIMULOS EXTERNOS
62. LA LESION SUBLETAL DEL DNA POR RADIACIONES ULTRAVIOLETA ES POR:
a. A.DAÑO AL RB
b. B. DAÑO A RECEPTORES DE FACTORES DE CRECIMIENTO
c. C.DAÑO AL RNA MENSAJERO
d. D.FORMACION DE DIMEROS DE PIRIMIDINA
63. EL VEGF Y LAS ANGIOPOYETINAS INTERVIENEN EN PROCESOS COMO:
a. A. REMODELACIÓN DE LA MATRIZ EXTRACELULAR
b. B. DEGRADACIÓN DEL COLAGENO DE LA MATRIZ EXTRACELULAR
c. C. SÍNTESIS DE COLAGENO POR LOS FIBROBLASTO
d. D. VASCULOGENESIS Y ANGIOGENESIS
64. 14.- LAS PROTEINAS E6 Y E7 DEL VPH DE ALTO RIESGO SE UNEN AL GEN:
a. A. P27
b. B. RAS
c. C.RB Y P53
d. D.MYC
e. E. FOS
65. ENZIMAS ENCARGADAS DE LA DEGRADACIÓN DEL COLAGENO Y OTRAS PROTEÍNAS DE LA
MATRIZ EXTRACELULAR:
a. A. ELASTASA DE LOS NEUTROFILOS
b. B. CININAS
c. C. METALOPROTEINASAS
d. D. CATEPSINA G
66. ABCESOS, ÚLCERAS Y QUEMADURAS SON EJEMPLOS DE CICATRIZACIÓN POR:
a. A.TEJIDO CONECTIVO
b. B. SEGUNDA INTENCIÓN
c. C. FIBROSIS
d. D.PRIMERA INTENCIÓN
67. SON CAMBIOS CROMOSOMICOS EN LAS NEOPLASIAS EXCEPTO:
a. A)AMPLIFICACION
b. B)TRASLOCACION
c. C) TRANSCRIPCION
d. D) DELECION
68. ES UNA CARACTERÍSTICA EXCLUSIVA DE LAS NEOPLASIAS MALIGNAS:
a. A) NECROSIS
b. B) ANGIOGENESIS
c. C) OCASIONAR METASTASIS
d. D) MITOSIS
69. EL PARECIDO QUE EXISTE ENTRE LA CÉLULA NEOPLÁSICA Y LA CONTRAPARTE NORMAL SE
DENOMINA GRADO DE:
a. A.PROGRESIÓN
b. B. DIFERENCIACIÓN
c. C.INDIFERENCIACIÓN
d. D.DESDIFERENCIACIÓN
70. SON CELULAS DIFERENCIADAS REPROGRAMADAS:
a. A.CELULAS MADRE INDUCIDAS
b. B. CELULAS MADRE TISULARES
c. C. CELULAS MADRE HEMATOPOYETICAS
d. D.CELULAS MADRE EMBRIONARIAS
71. ENTRE LAS ONCOPROTEÍNAS (PRODUCTOS PROTÉICOS DE LOS ONCOGENES) TENEMOS:
a. A.BCL2
b. B. TELOMERASA
c. C.FCF
d. D.P53
72. SON MARCADORES TUMORALES EXCEPTO:
a. A.GONADOTROPINA CORIONICA HUMANA
b. B. INSULINA
c. C.ALFA FETOPROTEINA
d. D.ANTIGENO PROSTÁTICO ESPECÍFICO
73. CARCINÓGENOS QUÍMICOS EXCEPTO:
a. A.AGENTES ALQUILANTES
b. B. AFLATOXINA B
c. C.AMINAS AROMATICAS
d. D.ANTIBIOTICOS
74. UNO DE LOS GENES MUTADO EN LA MAYOR PARTE DE LOS CÁNCERES HUMANOS ES:
a. A.BAX
b. B. MYC
c. C.FIS
d. D.P53
75. SE DEFINE COMO AUSENCIA DE DIFERENCIACIÓN:
a. A.METAPLASIA
b. B. DISPLASIA
c. C.HIPERPLASIA
d. D.ANAPLASIA
76. LAS ENDOCRINOPATÍAS DE SÍNDROMES PARANEOPLÁSICOS SE ASOCIAN CON MAYOR
FRECUENCIA A:
a. CA EMBRIONARIO TESTICULAR
b. B. SEMINOMA
c. C. ADENOCARCINOMA PROSTÁTICO
d. D. CÁNCER DE PULMON
77. SON PATOLOGÍAS QUE PRODUCEN EDEMA POR DISMINUCION DE LA PRESIÓN OSMOTICA
DEL PLASMA:
a. A. GLOMERULOPATÍAS Y MALNUTRICIÓN
b. B. RADIOTERAPIA Y FILARIASIS
c. C. AUMENTO DE LA SECRECIÓN RENINAANGIOTENSINA-ALDOSTERONA
d. D. PERICARDITIS CONSTRICTIVA
78. ES UNA PROTEINA GRANDE DE ADHERENCIA CELULAR QUE LIGA MUCHAS MOLECULAS DE
LA MEC Y FORMA MATRIZ PROVISIONAL DURANTE LA CICATRIZACION:
a. A. CADHERINA
b. B. FIBRONECTINA
c. C. INTEGRINA
d. D. OSTEONECTINA
79. SON MEDIADORES QUIMICOS QUE INTERVIENEN EN LA ESTABILIZACION DE LOS VASOS
RECIEN FORMADOS
a. A. ON Y FCVE
b. B. PROSTAGLANDINAS
c. C. CDK Y CICLINAS
d. D. ANGIOPOYETINAS Y FCDP
80. LA ASCITIS EN UN PACIENTE CON CIRROSIS ALCOHOLO NUTRICIONAL ES DEBIDO A:
a. A) AUMENTO DE LA PRESIÓN HIDROSTÁTICA Y OBSTRUCCIÓN LINFÁTICA
b. B. B) DISMINUCIÓN DE LA PRESIÓN OSMOTICA Y AUMENTO DE LA PRESIÓN
HIDROSTÁTICA
c. C. C) RETENCION DE SODIO Y AUMENTO DE LA PRESIÓN HIDROSTÁTICA
d. D. D) OBSTRUCCIÓN LINFÁTICA Y DISMINUCIÓN DE LA PRESIÓN OSMOTICA
81. ¿CUÁL DE LAS SIGUIENTES SUSTANCIAS INHIBEN LA AGREGACION PLAQUETARIA?
a. A. TROMBOMODULINA
b. B. OXIDO NITRICO Y PROSTACICLINA
c. C. FACTOR ACTIVADOR DEL PLASMINOGENO TISULAR
d. D. ENDOTELINA
82. SUSTANCIA QUE ACTUA COMO FACTOR PROCOAGULANTE:
a. A.ENDOTELINA
b. B. FACTOR TISULAR
c. C.ANTITROMBINA III
d. D.PROSTACICLINA
83. ES UN ESTADO DE HIPERCOAGULABILIDAD DE ALTO RIESGO:
a. A.TABAQUISMO
b. B. USO DE ANTICONCEPTIVOS ORALES
c. C.FIBRILACION AURICULAR
d. D.MIOCARDIOPATIA
84. ES UN ANTICOAGULANTE NATURAL :
a. A.TROMBINA
b. B. ANTITROMBINA
c. C.FIBRINOGENO
d. D.FACTOR DE HAGEMAN
85. LA TROMBOSIS MESENTERICA ES UN EJEMPLO DE:
a. A.INFARTO BLANCO
b. B. INFARTO SEPTICO
c. C.INFARTO ROJO
d. D.INFARTO MURAL
86. EL ORIGEN MAS FRECUENTE DE LAS TROMBOEMBOLIAS PULMONARES ES:
a. A. LESIONES DEL ENDOCARDIO
b. B. VENAS PROFUNDAS Y SUPERFICIALES DE LOS MIEMBROS INFERIORES
c. C. ESTENOSIS MITRAL
d. D. TROMBOS MURALES INTRACARDIACOS
87. LOS INFARTOS BLANCOS SE PRODUCEN CUANDO HAY:
a. A. OBSTRUCCIÓN VENOSA
b. B. EN TEJIDOS CON DOBLE CIRCULACIÓN
c. C. EN TEJIDOS LAXOS
d. D. OBSTRUCCIÓN ARTERIAL EN ÓRGANOS SÓLIDOS CON RIEGO ARTERIAL
TERMINAL
88. ES UN POTENTE VASOCONSTRICTOR QUE SE PRESENTA AL INICIO DE LA LESIÓN
VASCULAR:
a. A.ENDOTELINA
b. B. PROSTACICLINA ENDOTELIAL
c. C.OXIDO NITRICO
d. D.HEPARINA
89. PERTENECEN AL GRUPO DE SHOCK DISTRIBUTIVO EXCEPTO:
a. A.SHOCK ANAFILACTICO
b. B. SHOCKCARDIOGENICO
c. C.SHOCK ENDOTOXICO
d. D.SHOCK NEUROGENICO
90. SON FACTORES SISTEMICOS QUE INFLUYEN EN LA REPARACIÓN DE LAS HERIDAS:
a. A. ESTADO CIRCULATORIO Y HORMONAS
b. B. INFECCIONES Y CUERPOS EXTRAÑOS
c. C. DENERVACION
d. D. ESTRÉS MECANICO
91. SON TIPOS DE TROMBOS, EXCEPTO:
a. A.ARTERIALES
b. B. LIGAMENTOSOS, SUBSEROSOS
c. C.FLEBOTROMBOSIS
d. D.MURALES
92. LAS NEOPLASIAS MALIGNAS PUEDEN SER RESISTENTES A LA APOPTOSIS POR DAÑO A:
a. A.BRCA 1 Y 2
b. B. FAS Y JUN
c. C.RAS
d. D.BCL2
93. LA CAPACIDAD DE INVASION Y METASTASIS DE UNA NEOPLASIA MALIGNA ESTA DADA
POR :
a. A. CAPACIDAD PARA DEGRADAR LA MEC
b. B. SECRECION DE FACTORES DE CRECIMIENTO
c. C. SINTESIS DE COLAGENO
d. D. INSENSIBILIDAD A LAS SEÑALES INHIBITORIAS
94. PROTEINAS DE LA MEC QUE DESESTABILIZAN LAS INTERACCIONES CELULA-MATRIZ PARA
PROMOVER LA ANGIOGENESIS :
a. A. COLAGENO Y ELASTINA
b. B. ACIDO HIALURONICO Y DERMATAN SULFATO
c. C. ESTROMIELISINAS
d. D. SPARK Y TENASCINA
95. NEOPLASIA MALIGNA DE LAS CELULAS GERMINALES DEL TESTICULO
a. A.MELANOMA
b. B. LINFOMA
c. C.SEMINOMA
d. D.MESOTELIOMA
96. CORRELACIONE LOS TÉRMINOS DE LA COLUMNA IZQUIERDA CON LOS DE LA DERECHA
SEGÚN CORRESPONDAN:
46) SHOCK CARDIOGENICO A) QUEMADURAS
47) LINFOMA B) NEOPLASIA MALIGNA
48) INFARTO ROJO C) NEOPLASIA BENIGNA MIXTA
49) ADENOMA PLEOMORFO D) TAPONAMIENTO CARDIACO
50) SHOCK HIPOVOLÉMICO E) TORSION DE QUISTE DE OVARIO
1. SON LAS PRINCIPALES CITOCINAS SECRETADAS EN LA INFLAMACIÓN AGUDA
a. IL 1, IL 8, TNF E IL6
b. TGF-1, NF-KAPPA B
c. VCAM E ICAM
d. INF-Y
2. En la regeneración hepática, la progresión de los hepatocitos del estado G1 a S comienza
con la formación del complejo
a. Ciclina D-CDK4
b. Ciclina A-CDK2
c. Ciclina B-CDK1
d. Ciclina D-CDK6
3. Estos genes se han relacionado con neoplasia endócrina múltiple 1, tumores endócrinos
hipofisiario y pancreáticos
a. BRCA1 y 2
b. TP53
c. MEN-1
d. MSH 2 y 6
4. El factor de Von Willebrand
a. Se encuentra en el tejido endotelial
b. Participa en la hemostasia secundaria
c. Se une a la glucoproteína ib de las plaquetas
d. Es un inhibidor de la coagulación
5. Es un marcador tumoral de tipo hormonal
a. CA125
b. APE
c. Calcitonina
d. Antígeno carcinoembrionario
6. El carcinoma de células renales, el hemangioma cerebeloso y el hepatocarcinoma, se han
relacionado con el siguiente síndrome paraneoplásico
a. Síndrome de secresión inadecuada de hormona antidiurética
b. Policitemia
c. Hipercalcemia
d. Síndrome de Cushing
7. Tipos de generales de señalización
a. Intrínseca-extrinsica
b. Intracelular-extracelular
c. Señal-receptor
d. Autocrina-paracrina-endocrina
8. Tras una hepatectomía parcial
a. Se produce fibrosis
b. El hígado se regenera y los hepatocitos proliferan
c. Se produce cirrosis
d. Se produce una cicatriz
9. Es una antiproteasa que se encuentra en el suero y fluidos
a. GAMA 2 macroglobulina
b. Transpeptidasa
c. Cininas
d. Defensinas
10. Principal citocina activadora de macrófagos por la vía clásica:
a. TNF
b. IL-13 e IL-4
c. TGF-B
d. IFN-Y
11. Son causas de edema de pulmón:
a. Malnutreción e hipertención sistémica
b. Insuficiencia del ventrículo izquierdo, insuficiencia renal e infecciones pulmonares
c. Hipertensión portal y neuroinfección
d. Insuficiencia del ventrículo derecho e infecciones pulmonares
12. Se refiere así, a la pérdida progresiva de la grasa corporal y de la masa corporal magra,
acompañada de una debilidad profunda, anorexia y anemia
a. Efectos locales del cáncer
b. Síndromes paraneoplásicos
c. Caquexia cancerosa
d. Efectos hormonales del cáncer
13. Es u polisacárido de la familia de los glucosa aminoglucanos
a. Fibrilina
b. Fibronectina
c. Acido Hialurónico
d. Cadherina
14. El tiempo de protrombina evalúa la vía intrínseca de la coagulación, que factores
comprende
a. XII, XL y IX
b. VIII, X, y V
c. II, X, y fibrinógeno
d. VII, X, V y fibrinógeno
15. Las metastasis por vía venosa afectan a las siguientes organos:
a. Huesos largos y páncreas
b. Estómago e hígado
c. Hígado y pulmones
d. Hígado y páncreas
16. Señale las funciones celulares que con mayor frecuencia son blanco de los agentes lesivos:
a. Respiración anaerobia y producción de ac. Láctico
b. Producción de enzimas hormonas
c. Los cilios y microvellosidades
d. Fosforilación oxidativa y producción de ATP
1. La falta de diferenciación, es decir, implica una inversión de la diferenciación
hacia un plano más primitivo.
a) Displasia
b) Pleomorfismo
c) Anaplasia
d) Diferenciación
La falta de diferenciación se denomina anaplasia

2. Mutaciones que contribuyen al fenotipo maligno se denomina.


a) Fenotipo mutador
b) Inestabilidad genómica
c) Ninguna de las anteriores
d) Mutación conductora
Genes que regulan la reparación de ADN; la reparación de ADN defectuoso predispone a
mutaciones genómicas (fenotipo mutador).

3. Corresponde al evento principal de la hemostasia primaria *


a) Formación de fibrina
b) Formación del tapón plaquetario
c) Secreción de endotelina
d) Vasodilatación arteriolar
Los productos secretados reclutan otras plaquetas para formar un tapón hemostático
temporal (hemostasia primaria).

4. ¿Cuáles son las características del tejido de granulación?


a) Rico en colágeno y elastina
b) Rico en tejido conjuntivo provisional y nuevos capilares finos
c) Bajo en fibroblastos
d) Abundantes células inflamatorias
El tejido de granulación se forma por migración y proliferación de fibroblastos y por el
depósito de tejido conjuntivo laxo, combinado con los nuevos vasos y leucocitos
intercalados.

5. La estadificación de los cánceres sólidos se basan en qué sistema


a) Sistema de Dukes
b) Ninguna de las anteriores
c) Sistema de bethesda
d) Sistema TNM
El sistema TNM es el sistema de estadificación de cáncer de mayor uso. La mayoría de los
hospitales y centros médicos usan el sistema TNM como método principal en sus informes
de cáncer.

 La T se refiere al tamaño y extensión del tumor principal. El tumor principal se


llama de ordinario el tumor primario.
 La N se refiere a la extensión de cáncer que se ha diseminado a los ganglios (o
nódulos) linfáticos cercanos.
 La M se refiere a si el cáncer se ha metastatizado; es decir, si ha tenido metástasis.
6. Cuál de las siguientes causas es más probable que se acumule el líquido sí existe
variación
a) Permeabilidad vascular
b) Presión linfática
c) Presión oncotica
d) Diapedesis

7. Es la vía más frecuente para la diseminación inicial de los carcinomas


a) Siembra directa de cavidades o superficies corporales
b) Hematógena
c) Linfática
d) Continuidad
8. En el choque séptico se puede producir
a) Enteropatia hemorrágica
b) Daño alveolar difuso
c) Todas las anteriores
d) Encefalopatía isquémica

9. Un embolo gaseoso puede originarse:


a) Por trombosis venosa principal
b) Por fracturas
c) Por cambios bruscos en la presión
d) Por células fetales

10. Es un ejemplo de edema localizado en el que ocurre en casos de


a) Síndrome nefrótico
b) Trichuriasis
c) Cirrosis hepática
d) Insuficiencia cardiaca derecha

11. En la cicatrización por primera intención, en que tiempo la neurovascularización alcanza


su máximo y el tejido de granulación llena la incisión
a) 12 horas
b) 5 días
c) 48 horas
d) 24 horas

12. Los tumores más comunes en los hombres se originan en:


a) Mama, Pulmones, Colon y recto
b) Piel, Leucemia y páncreas
c) Piel, orofaringe y páncreas
d) Próstata, pulmones, colon y recto

13. Son tumores constituidos por células inmaduras que se parecen a las que forman tejido
rudimentario fetal
a) Papiloma
b) Blastoma
c) Hamartoma
d) Osteoma
14. Los infartos palidos se observan en:
a) Casos de obstrucción venosa
b) A y B son ciertas
c) Órganos sólidos
d) Casos de oclusión arterial

Los infartos blancos se producen en órganos sólidos (como corazón, bazo y riñón) con
circulaciones arteriales terminales (es decir, pocas colaterales)
15. ¿Cuál de los siguientes mediadores causan vasoconstricción y broncoconstricción?
a) Tromboxano A2
b) IL-2
c) Leucotrienos B4
d) Ninguna de las anteriores

16. Es un factor de crecimiento con efectos mitogenos sobre los hepatocitos


a) Factor de crecimiento fibroblastico
b) Factor de crecimiento de dispersión (Factor de crecimiento de hepatocitos)
c) Factor de crecimiento epitelial
d) Factor de crecimiento derivado de plaquetas

17. El edema por obstrucción linfática se da por las siguientes condiciones excepto:
a) Neoplasica
b) Postquirurgica
c) Postirradiación
d) Postquimioterapia

18. El virus Epstein Baar produce:


a) Hepatocarcinoma
b) Leucemia de células peludas
c) Linfoma de burkitt
d) Sarcoma de Kaposi

19. En el estado conocido como Shock séptico:


a) Es originado por la pérdida de liquidos
b) Es producido por el paso de bacterias, hongos al torrente circulatorio
c) Carece de respuesta inflamatoria
d) Se considera una respuesta inflamatoria local

20. Su crecimiento se acompaña de infiltración, invasión y destrucción progresiva del tejido


circulante están poco delimitados y son de lenta expansión
a) Neoplasia maligna
b) Neoplasia preinvasiva
c) Cáncer in situ
d) Neoplasia benigna

21. Se media por las interacciones con el factor de Von Willedrand con la glucoproteína IB,
receptor de la superficie de las plaquetas y el colágeno expuesto
a) Activación plaquetaria
b) Adhesión plaquetaria
c) Agregación plaquetaria
d) Fase de iniciación de la coagulación

22. ¿Qué gen supresor de tumores intervienen en la patogenia del carcinoma de mama?
a) Ninguna de las anteriores
b) APC1 APC2
c) BCRA 1 BCRA 2
d) NF1 NF2

23. Especifique las variables que alteran la cicatrización


a) Infección, diabetes, estado nutricional, factores mecánicos, mala perfusión,
tipo, alcance y localización de la lesión
b) Estado nutricional adecuado y esteroides
c) Baja presión local o la torsión de las heridas
d) Infección y diabetes controlada
24. Se caracterizan por degradar la MEC y permite la remodelación y la extensión del tubo
vascular.
a) Tirosina cinasa.
b) MPM metaloproteinasas de matriz
c) Factor transformante beta
d) Factor derivado de plaquetas.
25. La luz UV:
a) Es un carcinógeno químico
b) Hace dímeros en la guanina
c) No es un agente carcinógeno
d) Puede causar carcinoma basocelular

26. Son procesos en la evolución de un trombo excepto:


a) Embolia por recanalización
b) Propagación
c) Trombolisis
d) Recanalización

27. Es una causa de edema por disminución de la presión oncotica


a) Diabetes Mellitus
b) Desnutrición
c) Insuficiencia cardiaca
d) Cáncer

28. Es un término que se aplica a la neoplasia epitelial benigna derivada de las glándulas,
se denomina como:
a) Papiloma
b) Pólipo
c) Angioma
d) Adenoma

29. La mayor parte de las flebotrombosis se producen en:


a) Arterias de mediano calibre
b) Corazón
c) Venas superficiales y profundas de las piernas
d) Arterias cerebrales

30. Una de las razones por las cuales las células neoplásicas son inmortales
a) No se autorrenuevan
b) Respetan las pausas del ciclo celular
c) P53 funciona adecuadamente
d) Expresión telomerasa

31. Una lesión endotelial tendrá como consecuencia:


a) No tiene relación, con la coagulación
b) Desarrollo de un ambiente procoagulante
c) Produce la disolución del coagulo
d) Desarrollo de un ambiente anticoagulante

32. Son acontecimientos plaquetarios excepto


a) Adhesión
b) Migración
c) Cambio de forma y secreción
d) Agregación

33. El efecto warburg consiste en:


a) Incremento del ciclo de Krebs
b) Disminuir la síntesis de biomoléculas
c) Promover la atrofia del tumor
d) La neoplasia maligna fija la glucolisis anaerobia en su metabolismo

34. El shock cardiogénico se produce cuando existe una pérdida de la masa muscular del
ventrículo izquierdo del:
a) 25%
b) 60%
c) 45%
d) 35%
El fracaso grave de bomba (shock cardiógeno) aparece en el 10-15% de los pacientes,
habitualmente con una pérdida ~ 40% de masa ventricular izquierda. El shock cardiogénico
tiene una mortalidad del 70%

35. ¿Cuál es la definición de teratoma?


a) Que se origine de las células de sertoli
b) Que se origine de más de 2 capas germinales
c) Origen en linfocitos B
d) Que se origine de un trofoblasto

36. Los estados de hipercoagulabilidad se dividen en:


a) Fisiológicos y patológicos
b) Genéticos y adquiridos
c) De alto y bajo riesgo
d) Primarios y terciarios

37. Neoplasia maligna de los vasos sanguíneos


a) Linfoma
b) Linfagiosarcoma
c) Leucemia
d) Angiosarcoma

38. Son estados de hipercoagulabilidad de bajo riesgo excepto:


a) Tabaquismo
b) Tabaquismo
c) Infarto agudo al miocardio

39. La inactivación o deleción de P16, adquirida por vía somática se ven en las familias
propensas a:
a) Carcinomas de páncreas, glioblastomas y cánes de esófago
b) Linfomas
c) Adenocarcinoma pancreático y colangiocarcinomas
d) Leucemia mieloide crónica y leucemias linfoblásticas agudas

40. En la cicatriz de primera intención, ¿Cuándo se comienza a observar el tejido de


granulación?
a) Inmediatamente
b) Semanas después
c) A las 24 horas
d) 3 a 7 días después

41. Cual enunciado acerca de las plaquetas es verdad


a) Contiene actina y miosina, microfilamentos y microtubulos
b) Contiene cuerpos densos ricos en fibrinógeno
c) Los precursores son monocitos
d) Contiene gránulos alfa ricos en difosfato de adenosina
La secreción de gránulos plaquetarios (reacción de liberación) ocurre poco después de la
adhesión. Los gránulos a expresan moléculas de selectina de adhesión y contienen factores
de coagulación y de crecimiento; los cuerpos densos o gránulos δ contienen nucleótidos de
adenosina (p. ej., [difosfato de adenosina [ADP] ), calcio y aminas vasoactivas (p. ej.,
histamina). El ADP es un potente mediador de la agregación plaquetaria m (reclutamiento)
y el calcio es importante para la cascada de la coagulación.

42. La estadificación de un tumor maligno se refiere a


a) Grado de diferenciación de las células tumorales
b) Tratamiento quirúrgico
c) Diagnóstico molecular
d) Tamaño de la lesión primaria y si ha realizado metástasis

43. La congestión pasiva crónica del pulmón puede incluir todo lo siguiente, excepto
a) Edema pulmonar
b) Eritema
c) Macrófagos con hemosiderina
d) Induración café
44. La extravasación del líquido al intersticio, cual es el término que se utilizaría
a) Edema
b) Trasudado
c) Exudado
d) Infiltración

45. No siempre responden a las moléculas que inhiben la proliferación de las células
normales, habitualmente por la inactivación de genes supresores de tumores que
modifican componentes de estas vías inhibitorias del crecimiento
a) Insensibilidad a las señales inhibitorias del crecimiento
b) Potencial ilimitado de la replicación
c) Alteración del metabolismo celular
d) Evasión de la apoptosis

46. Una de las maneras en las que un tumor escapa de la respuesta inmune es:
a) Expresando proteínas inhibitorias de linfocitos T
b) Expresando citocinas activadoras de linfocitos T
c) Sobreexpresando moléculas CPH 1
d) Sobreexpresando antígenos

47. ¿A que deben los tumores su potencial replicativo ilimitado?


a) Activación de la telomerasa
b) Responden adecuadamente a las señales apoptosicas
c) Presencia de inhibidores de la telomerasa
d) Carecen capacidad de autorrenovacion

48. La edad influye de manera determinante en el riesgo de cáncer por lo tanto, la mayoría
de los carcinomas se presentan en la edad de:
a) Mayor de 20 años
b) Mayor de 55 años
c) 20 a 30 años
d) 10 a 20 años
49. Un estado inflamatorio crónico puede causar anemia ferropénica en respuesta al:
a) Aumento de albumina
b) Aumento de fibrinógeno
c) Aumento de la hepcidina
d) Aumento de la fibrina

50. En un estado inflamatorio crónico asociado al cáncer


a) Miositis
b) Enfermedad intestinal inflamatoria
c) Condritis
d) Pielonefritis

51. La llamada ¨induración parda¨ es un ejemplo, excepto:


a) Congestión pasiva crónica pulmonar
b) Congestión pasiva crónica esplénica
c) Congestión pasiva del hígado
d) Hemorragia de tubo digestivo

52. El carcinoma epidermoide de pulmón, carcinoma de mama, carcinoma de células


renales y la leucemia/linfoma de linfocitos T en el adulto se han relacionado con el
siguiente síndrome paraneoplasico
a) Síndrome de Cushing
b) Hipercalcemia
c) Hipoglucemia
d) síndrome de secreción inadecuada de hormona antidiurética

53. Afectan al proceso de cicatrización excepto:


a) Desnutrición
b) Tratamiento con glucocorticoides
c) Exceso de ácido ascórbico
d) Diabetes mellitus

54. Es un marcador tumoral de tipo hormonal


a) CA125
b) APE
c) Calcitonina
d) Antígeno carcinoembrionario

55. El restablecimiento de la estructura tisular normal solo se produce:


a) Cuando el tejido residual esta estructuralmente intacto (Resección quirúrgica
parcial)
b) En nefrectomía unilateral
c) Cuando todo el tejido está dañado por inflamación
d) Cuando todo el tejido está dañado por infección

56. Es considerado como el guardián del genoma


a) TP53
b) K-RAS
c) MYC
d) JUN

57. La asbestosis y la silicosis está asociada con qué tipo de neoplasia


a) Carcinoma colonorrectal
b) Carcinoma pancreático
c) Hepatocarcinoma
d) Mesotelioma, carcinoma de pulmón

58. Sustancia generada por sistema de cininas cuando hay daño vascular promueve la
contracción de musculo liso, aparación de dolor y promueve la permeabilidad vascular
a) Bradicinina
b) C5a del complemento
c) Fosfolipasa C
d) Factor de necrosis tumoral

59. La característica histológica predominante de un infarto es:


a) Necrosis grasa
b) Necrosis fibrinoide
c) Necrosis caseosa
d) Necrosis coagulativa

60. Fase del shock donde fracasan mecanismos reflejos de compensación disminuye la
perfusión de órganos vitales
a) Fase latente
b) Fase progresiva
c) Fase no progresiva
d) Fase irreversible

61. Es un carcinógeno quimico de origen natural:


a) Ciclofosfamida
b) Cloruro de vinilo
c) Bencidina
d) Griseofulvina

62. Neoplasia maligna del epitelio placentario


a) Carcinoma embrionario
b) Coriocarcinoma
c) Seminoma
d) Mesotelioma

63. Neoplasia maligna del tejido adiposo


a) Liposarcoma
b) Astrocitoma
c) Neurofibroma
d) Lipoma

64. Contienen una mayor cantidad de sangre oxigenada se caracteriza por el color rojo
brillante y dilatación de las arterias
a) Hiperemia activa
b) Hiperemia pasiva
c) Congestión crónica
d) Congestión pasiva

65. Todo lo siguiente se pueden esperar en la congestión hepática excepto


a) Regiones centrolobulillares distendidas por sangre
b) Infartos en forma de pirámide
c) Patron de nuez moscada
d) Incremento de peso

66. La deficiencia de la glucoproteína GPIB provoca la enfermedad de:


a) Von willebrand
b) Tromboastenia de Glanzmann
c) Nieman Pick
d) Bernard soulier
67. Los principales factores de crecimiento implicados en la síntesis de tejido conjuntivo,
excepto:*
a) PDGF
b) FGF
c) IL-1
d) TGF-BETA

68. Son proteínas compuestas por tres cadenas polipeptidicas distintas trenzadas en triple
hélice:
69. Son neoplasias que tienen peor diferenciación o totalmente indiferenciadas, se refiere
al termino de:
a) Nevo
b) Coristoma
c) Hamartoma
d) Anaplasia
70. Neoplasia maligna de las células de los ductos seminales
a) Coriocarcinoma
b) Carcinoma embrionario
c) Seminoma
d) Tumor de senos endodérmicos

71. Es la fase del shock en la que se activa el eje renina- angiotensina


a) Fase refractaria
b) Fase no progresiva
c) Fase irreversible
d) Fase progresiva

72. Es el colágeno que proporciona fibrillas de anclaje de las células epiteliales a la


membrana
a) Colágeno tipo I
b) Colágeno tipo III
c) Colágeno Tipo IV
d) Colágeno Tipo VII
73. Estos trombos se forman en la circulación venosa, suelen contener mas eritrocitos
incorporados y relativamente pocas plaquetas
a) Trombos arteriales
b) Trombos rojos
c) Trombos murales
d) Vegetaciones
74. ¿Cuáles son las fases de la reparación en las heridas?
a) Infiltración, remodelación, fibrosis y costra
b) Sepsis, formación de tejido de granulación y angiogénesis
c) Angiogenia, tejido de granulación y remodelación
d) Angiogenia, inflamación, contracción, fibrosis e hipoplasia
75. El resultado de múltiples mutaciones que se acumulan de forma independiente en
diferentes células y contribuyen asi a la generación de subclones con características
diferentes es a lo que se le conoce como:
a) Iniciación y promoción
b) Progresión tumoral
c) Acción directa e indirecta
d) Heterogeneidad tumoral

76. Patología donde las personas que sufren mutaciones en el gen TP53 y que tienen una
posibilidad 25 veces mayor de experimentar una neoplasia maligna antes de los años
que la población general
a) Síndrome de desgaste
b) Carcinoma
c) Neoplasia endocrina multiple
d) Sindrome de Li-Fraumeni

77. Si la alteración displasica es intensa y afecta a todo el espesor del epitelio, pero la lesión
no penetra la membrana basal, se habla de:
a) Carcinoma in situ
b) Tumor infiltrante
c) Tumor invasivo
d) Metaplasia

78. Los tipos de VPH de alto riesgo producen proteínas oncogenas que ocasionan lo
siguiente, excepto:
a) Activan ciclinas
b) Inhiben la apoptosis
c) Combaten la senecencia celular
d) Inmunodeficiencia

79. El gen APC inhibidor de las vías de señalización mitogena, se ha relacionado con las
siguientes neoplasias
a) Carcinoma basocelular y meduloblastoma
b) Carcinomas de estómago, colon, páncreas y melanoma
c) Retinoblastomas
d) Schwannoma y meningioma

80. Es una característica del síndrome de Trousseau asociado a carcinomas de páncreas


y estomago se refiere a:
a) Embolos
b) Deficit de glucoproteína 1 B
c) Tromboflebitis migratoria
d) Infarto

81. Que tan resistentes son las células musculares cardiacas a la hipoxia
a) Muy resistentes, mueren después de muchas horas
b) Extremadamente sensibles, mueren a los 5 minutos
c) Bastante sensibles, muerena los 20-30 minutos
d) Poco sensibles, mueren a las pocas horas

82. Son patologías que producen edema por aumento de la presión hidrostática del vaso:
a) Neoplasias y filarias
b) Aumento de la secreción renina angiotensina- aldosterona
c) Glomerulopatias y malnutrición
d) Pericarditis constrictiva y trombosis

83. Se le conoce así cuando el tejido cicatrizal, crece más allá de los límites de la herida
original, además no se contrae
a) Fibromatosis agresivas
b) Cicatriz queloide
c) Granulación exuberante
d) Dehiscencia

84. Factores sistémicos que influyen en la curación de heridas


a) Estrés oxidativo, perdida del tono vascular, anemia
b) Nutrición, situación circulatoria estado metabólico y hormonas
c) Infecciones, nutrición, función renal y cardiaca deficiente
d) Tipo de herida, profundidad de la herida y estado metabólico

85. Marcador tumoral relacionado con el cáncer prostático


a) ALFA FETOPROTEÍNA
b) CA-125
c) CA-19-9
d) PSA

86. Morfológicamente las neoplasias malignas se caracterizan por:


a) Relación núcleo citoplasma normal
b) Arquitectura tisular intacta
c) Pleomorfismo nuclear y celular
d) Figuras mitóticas normales
87. Dentro de las funciones de P53 se encuentran, excepto:
a) Pausar el ciclo celular para reparar el ADN
b) Inducir apoptosis
c) Inducir proliferación celular
d) Inducir senescencia
88. El cáncer es una enfermedad caracterizada por:
a) Daño genético letal
b) Senescencia celular
c) Lesión subletal al ADN
d) Adquisición de mutaciones

89. Se puede producir choque por


a) Todas las anteriores
b) Sepsis
c) Infarto del miocardio
d) Quemaduras

90. Se observa tras una destrucción sustancial de tejido, cuando la lesión inflamatoria afecta
a tejidos que no son capaces de regenerarse o cuando hay abundante exudación de
fibrina en tejidos o cavidades serosas que no es posible eliminar adecuadamente
a) Inflamación crónica
b) Remisión
c) Curación por reposición de tejido conjuntivo
d) Resolución completa

91. Marcador tumoral relacionado con carcinoma de colon, pancreas y pulmón


a) Alfa fetoproteina
b) Antigeno prostático
c) Antigeno carcinoembrionario
d) Enolasa neuronal especifica
92. El gen 9,12 se asocia con
a) Von hippel- lindau  gen VHL (3p25.3)
b) Trisomia 21
c) Cromosoma filadelfia
d) Leucemia mieloide
93. Característica de las señales sinápticas
a) Secreción de neurotransmisores en uniones celulares especiales
b) De difusión mínima y la señal se degrada rápido
c) Se usa para amplificar una respuesta o inhibirla mediante retroalimentación
d) Se libera al torrente circulatorio y actúa distancia sobre células blanco
94. Neoplasia maligna de la glándula tiroides
a) Coriocarcinoma L
b) Adenocarcinoma
c) Condrosarcoma
d) Linfangiosarcoma
95. Se considera como extensión de las neoplasias a sitios anatómicamente separados del
lugar de origen
a) Metastasis
b) Micro invasión
c) Son masas expansivas cohesivas
d) Invasión
96. Son sustancias que inhiben la función plaquetaria
a) Antitrombina
b) ON, prostaciclina y ADPasa
c) Plasminogeno
d) Tromboplastina
97. El gen RB, inhibidor de la progresión celular, se ha relacionado con la siguientes
neoplasias
a) Retinoblastoma, osteosarcoma, carcinoma de mama, colon y pulmón
b) Carcinoma basocelular y meduloblastoma
c) Carcinoma basocelular
d) Carcinoma de células renales y paraganglioma
98. Trombo formado en las cavidades cardiacas
a) Trombo mural
b) Aneurisma
c) Petequia
d) Embolo
99. Tienen mayores probabilidades de sufrir infarto rojo, Excepto:
a) Intestino delgado
b) Pulmon
c) Riñon

100. En la inmortalidad de las células malignas es considerado como factor decisivo


a) Evasión de la senescencia
b) Mutuacion puntual
c) Reordenapmiento de los cromosomas
d) Capacidad de regulación
101. Son algunas causas del retorno venoso
a) Sindrome nefrotico y cirrosis hepática
b) Obstrucción linfática y retención de sodio
c) Insuficiencia cardiaca congestiva y compresión venosa
d) Calor y frio
102. Es el cambio biológico necesario para la progresión de las neoplasias malignas
a) Evasión de la apoptosis
b) Metastasis
c) Potencial replicativo
d) Angiogenia
103. Etapa caracterizada por la hipoperfusión tisular y el inicio del empeoramiento
circulatorio y de los desequilibrios metabólicos incluyendo la acidosis lactica
a) Fase no progresiva
b) Etapa progresiva
c) Etapa reversible
d) Etapa irreversible
104. Un cistoadenoma corresponde con:
a) Neoplasia benigna quística glandular
b) Hiperplasia
c) Neoplasia maligna quística glandular
d) Neoplasia maligna
105. Pertenece a una familia de factores de crecimiento con mas de 20 miembros
a) FCDP
b) FCT BETA
c) FCF
d) FCE
106. Esta via de diseminación es bastante característica de los carcinomas de ovario ya
que frecuentemente se diseminan por la superficie peritoneal
a) Iatrogenica
b) Hematogena
c) Siembra de cavidades y superficies corporales
d) Linfática
107. Se induce por la hipoxia provoca la angiogenia y promueve la migración de las
células endoteliales
a) PDGF
b) TGF BETA
c) FGF
d) VEGF
108. Sufren una conmutación metabólica hacia la glucolisis aerobica efecto warburg que
facilita la síntesis de macromoléculas y orgánulos requeridos para un crecimiento celular
rápido
a) Evasión de la apoptosis
b) Potencial ilimitado de la replicación
c) Alteracion del metabolismo celular
d) Insensibilidad a las señales inhibitorias del crecimiento
109. Cuál es la proteína más abundante del ser humano
a) Vimentina
b) Miosina
c) Caderina
d) Colágeno

110. International normalized ratio valora principalmente en el laboratorio


a) Anticoagulación con heparina
b) Ácido acetilsalicílico
c) Factor VIII y X
d) Anticoagulación oral
111. Responden por lo común al tratamiento con anticuerpos o fármacos que bloquean
la actividad de HER2:
a) Cáncer de mama
b) Adenocarcinoma de pulmón
c) Melanoma
d) Leucemia mielocitica aguda
112. La activación del activador del plasminogeno genera su conversión a plasmina para
a) Activar plaquetas
b) Iniciar la cascada fibrinolitica
c) Secreción y cambio de forma plaquetaria
d) Activación leucocitaria
113. Estos trombos son mas frecuentes en arterias coronarias cerebrales y femorales
conformados por plaquetas, fibrina, eritrocitos y leucocitos degenerados
a) Vegetaciones
b) Trombos murales
c) Trombos arteriales
d) Trombos rojos
114. Permiten determinar el diagnostico, pronostico, la detección de enfermedad residual
mínima y el diagnóstico de la predisposición hereditaria al cáncer
a) Análisis microbiológico
b) Historia clínica
c) Análisis moleculares
d) Análisis morfológicos
115. Vía de la apoptosis que más se desregula en situación de cáncer
a) Necroptosis
b) Vía extrínseca
c) Vía intrínseca
d) Piroptosis
116. Clínicamente el proceso de fibrosis es común en:
a) Mucosas
b) Epitelios
c) Tejidos lábiles
d) Órganos parenquimatosos
117. Que proteína fibrilar suele regenerar mal en el tejido cicatricial
a) Integrina
b) Colágeno
c) Elastina
d) Fibronectina
118. Factor de crecimiento quimiotactico para neutrófilos, macrófagos, células del
musculo liso, estimula la proliferación de fibroblastos y células endoteliales
a) Factor de crecimiento hepatocitario
b) Factor del crecimiento del endotelio vascular
c) Factor de crecimiento derivado de plaquetas
d) Factor de crecimiento epidérmico
119. Es el componente proteico de la MEC encargado de dar resistencia a la tensión
a) Proteoglucanos
b) Laminina
c) Fibronectina
d) Colágeno
120. Es el termino aplicado a un resto heterotopico de células por ejemplo, un nodulo de
tejido pancreático perfectamente desarrollado y con una organización normal en la
submucosa del estomago
a) Teratoma
b) Desmoplasia
c) Hamartoma
d) Coristoma
121. El factor de crecimiento placentario pertenece a la familia de
a) PCE
b) FCH
c) FCVE
d) FCFD

122. Este tipo de células están presentes en la inflamación crónica, son abundantes en
las reacciones inmunitarias mediadas por inmunoglobulina E y en las infecciones
parasitarias.
a) Neutrófilos
b) Linfocitos
c) Eosinófilos
d) Macrófagos.
123. La congestión pasiva crónica del pulmón puede incluir todo lo siguiente excepto.
a) Edema pulmonar
b) Eritema
c) Macrófagos con hemosiderina
d) Induración café
124. Patología en la que se pierde albumina y genera edema:
R= Sindrome Nefrotico
125. Causantes de edema:
R= Enfermedad cardiaca y renal

126. Hematoma mayor a 2cm:


R= Equimosis

127. Se ve implicadoen reacciones inflamatorias ante cristales de urato (causantes de


gota), lípidos (en el síndrome metabolico)
128. Son componentes de la fagocitosis excepto
a) Englobamiento quimiotraccion
b) Reconocimiento y fijacion
c) Opsonizacion
129. Son neoplasias malignas de células germinales del testículo.
Respuesta =SEMINOMAS
130. Una vez que las células tumorales rompen la membrana basal, se habla de R=
TUMOR INFILTRANTE
131. Son neoplasias malignas, su origen es derivado del tejido conectivo o conjuntivo
R= SARCOMA
132. Las células exhiben una variación en su tamaño y forma, es decir, las células de
un mismo tumor no son uniformes
R= PLEOMORFISMO

133. Es un protooncogen que pertenece a los inhibidores de las cinasas de las ciclinas
(INK) **
R= p16

134. Se refiere así, a la pérdida progresiva de la grasa corporal y de la masa corporal


magra, acompañada de una sensibilidad profunda, anorexia y anemia.}
R= CAQUEXIA CANCEROSA

135. Neoplasia maligna de células hematopoyéticas


R= LEUCEMIA

136. De las siguientes opciones, ¿Cuál no es una característica de anaplasia?


R= FALTA DE MITOSIS

137. El marcador tumoral CA 19-9 se relaciona principalmente con


R= CÁNCER DE COLON Y PÁNCREAS

138. Proteína relacionada con el síndrome familiar del retinoblastoma y osteosarcoma


R= RB
139. Tienen una función esencial en muchos aspectos del fenotipo maligno, como la
expresión de los genes del cáncer, el control de la diferenciación y la autorrenovación,
incluso la sensibilidad de la resistencia a los medicamentos
R= CAMBIOS EPIGENÉTICOS
140. Mutaciones que contribuyen al fenotipo maligno se denomina *
R= MUTACIÓN CONDUCTORA
141. Neoplasia benigna de nervio periférico
R= NEUROFIBROMA
142. Células que están presentes en el tejido conectivo contiguos a vasos sanguíneos
por debajo de superficies mucosas de vías respiratorias, la unión de antígeno alérgeno
a anticuerpo IgE, se refiere a:
R= MASTOCITOS
143. El edema que ocurre en la filariasis se debe a:
R= BLOQUEO DE LA CIRCULACIÓN LINFÁTICA.
144. Defecto genético que predispone a sufrir trombosis conocidas como trombofilias y
un 5% a 10% de las personas lo puede presentar
R= DEFICIENCIA DE ANTITROMBINA III, PROTEÍNA C-S Y FACTOR V LEIDEN

145. Corresponde al evento principal de la hemostasia secundaria


R= DEPOSICIÓN DE COAGULO DE FIBRINA
146. . ¿Cómo se clasifican los infartos?
R= ROJOS, BLANCOS; SÉPTICOS
147. La triada de Virchow incluye las siguientes características excepto
R= INCREMENTO DE LA VOLEMIA
148. Es una causa de la disminución de la presión osmótica
R= SÍNDROME NEFRÓTICO

149. ¿Proceso activo en el que la dilatación arteriolar aumenta en el flujo de sangre?


R= HIPEREMIA
150. El edema de origen cardiaco se caracteriza por
R= APARECER POR LAS MAÑANAS

151. La aparición de hematomas puede deberse principalmente


R= ENFERMEDAD DE VON WILLEBRAND
152. La trombosis venosa (Fenómeno de Trousseau) se ha relacionado con síndrome
paraneoplásico
153. R= CARCINOMAS DE PÁNCREAS, BRONCÓGENO Y OTROS CÁNCERES

154. Principal causa que puede generar tromboembolia pulmonar


R= TROMBOSIS RPOFUNDA
155. Son células responsables de la síntesis de nueva matriz extracelular
R= CÉLULAS ENDOTELIALES
156. Es secundario a una insuficiencia de bomba miocárdica debido a las lesiones
intrínsecas del miocardio, comprensión extrínseca u obstrucción de flujo de salida
R= CHOQUE CARDIOGENICO

157. En su mayoría, los casos de shock endotoxico son causados por


R= SEPSIS POR HONGOS

158. 4Es más frecuente la formación de émbolos a partir de


R= TROMBOS
159. El colágeno que proporciona fibrillas de anclaje de las células epiteliales a la
membrana basal
R= COLAGENO TIPO IV
160. La cicatriz de primera intención es dada por heridas con
R= BORDES ALINEADOS
161. Para el anclaje, la migración de células y mantener la polaridad del tejido se requiere
R= CONTROL DE LA PROLIFERACIÓN TISULAR

162. Después de una lesión en un tejido permanente como el músculo cardíaco


R= NO SE REPARA

163. Ejemplos de células estables


R= PARÉNQUIMA DE ORGANOS SÓLIDOS, COMO HIGADO, RIÑON Y PÁNCREAS
164. Son características de la curación de segunda intención
R= LA REACCIÓN INFLAMATORIA ES MÁS INTENSA Y MAYOR CANTIDAD DE
TEJIDO DE GRANULACIÓN
165. La fibrosis es causada por
R= SE MANTIENE LA SÍNTESIS Y SECRECIÓN DE FACTORES DE CRECIMIENTO
Y CITOCINAS FIBROGÉNICAS

166. ¿Cuál es la función de los macrófagos m2 en la inflamación crónica?


R= PRODUCIR Y SECRETAR SUSTANCIAS BIOLOGICAS COMO, FACTORES DE
CRECIMIENTO ACTIVAN FIBROBLASTOS Y FAVORECEN LA ANGIOGENIA
167. Causa fisiológica de hiperemia:
R= Vasodilatación por ejercicio
168. Factor que inhibe la diseminación del trombo:
R= Activador tisular de plasminogeno(PAI)
169. Capacidad de cambiar de un tipo celular a otro:
R=Transdiferenciación
170. Es la capacidad de una célula de transdiferenciarse a distintas estirpes celulares:
R= Plasticidad de desarrollo
171. En esta fase de shock hay liberación de ADH y activación de baroreceptores:
R= Fase inicial progresiva
172. Manifestaciones clínicas presentes en shock séptico:
R= Piel caliente con vasodilatación periférica, hiperemia.
173. Casi siempre oclusiva puente largo de la luz del vaso:
R=Trombosis venosa profunda
174. Tipo de necrosis encéfalo:
R= Licuefactiva
175. Sintomas de la embolia grasa:
R= Taquicardia, bradicardia y disnea
176. Mantiene la integridad del epitelio externo de la cornea:
R=Células madre del limbo
177. Una neoplasia compuesta se divide como:
R= Dos capas celulares y una capa embrionaria
178. Metastasis:
R= Implantes tumorales sin continuidad con el tumor primario
179. Fases de la cascada metastasica:
R=Invasión de la MEC, diseminación vascular, alojamiento y colonización
180. Inhibicion de TP53 mediado por:
R=BAX
181. wich one of the following characteristics not acompany the cancer:
R= Fibrous capsule
182. What are the organs most affected by the hematogenous spread?
R= Liver and lung
183. Wich ARN virus has on oncogene activity?
R=Leucemia and cel T
184. Carcinógeno natural hepático secretado por plantas y hongos:
R= Aflatoxina B1
185. Causas frecuentes de esteatosis:
R=Alcoholismo y desnutrición
186. Mediador químico que produce dilatación y aumento de la permeabilidad y se
libera de tipo físico:
R=Histamina
187. Son células que participan en la reacción inflamatoria excepto:
R= Pericitos
188. Las CX3C son moléculas de adhesión de:
R=Monocitos y Cel T
189. De que vitamina son dependientes las proteínas C y S:
R=Vitamina K
190. Localización más frecuente del carcinoma de células renales:
R=Polos
191. El cáncer renal causa frecuentemente metástasis en:
R=Pulmones y hueso
192. Causa mas frecuente de cáncer renal:
R=Tabaquismo
193. Cual es la proteína viral que induce una rápida degradación de P53:
R=E6
194. Causa frecuente de metástasis en CACU:
R=Pielonefritis y Uremia
195. Lesion primaria de mayor riesgo de producir cáncer de mama:
R= Mastopatia proliferativa con atipia
196. Quimico que dara activación quimiotactica, actividad sobre monocitos y eosinofilos:
R=CXC
197. sustancias que ocacionan fiebre, sueño y perdida del apetito:
R=Citocinas
198. La inflamación fibrinosa se caracteriza por:
R=Fluidos corporales en pericardio y pleura
199. Caracteristicas histológicas de órganos que sufren atrofia, hay presencia de:
R= Formación de vacuolas autofagicas
200. Los leiomiomas se dividen en:
R=Submucosos, subserosos e intramurales
201. Las enfermedades crónicas granulomatosas abarcan varios trastornos que tienen
en común:
R=La incapacidad de destruir agentes etiológicos
202. El infarto de miocardio corresponde a una:
R=cardiopatía isquémica
203. Todas las siguientes sustancias pertenecen al tren de tincion, excepto:
R=Violeta de Genciana
204. La citología exfoliativa puede reportar todo, excepto:
R=El estado nutricional de la células.
205. Carcinoma in situ ductual consiste en:
R=Población maligna de células que no pueden atravesar la Membrana Basal
206. La fosfofructocinasa y la fosforilaza en lesión, activan a:
R=Glucolisis Anaerobia
207. ¿Que moléculas emiten la adhesión de plaquetas al endotelio no lesionado?:
R=Prostacicilinas y Oxido Nitrico
208. Para la formación de granulomas se requiere:
R= reacción inmunitaria mediada por células T
209. Pancreatitis aguda ------------------------ Necrosis Grasa
210. Infeccion Bacteriana----------------------N. Licuefactiva
211. Muerte celular----------------------------- Apoptosis
212. Tuberculosis------------------------------- N. caseosa
213. Infarto al miocardio--------------------- N. Coagulativa

214. SON NEOPLASIAS EPITELIALES GLANDULARES MALIGNAS QUÍSTICAS:


a) CARCINOMAS
b) SARCOMAS
c) ESCLEROMAS
d) CISTOADENOCARCINOMAS
215. NEOPLASIA MALIGNA DE LOS VASOS LINFÁTICOS:
a) HEMANGIOMA
ANGIOSARCOMA
b) LINFOMA
c) LINFANGIOSARCOMA
216. ES EL CAMBIO FUNDAMENTAL PARA OBTENER POTECIAL REPLICATIVO
ILIMITADO A
a) P53,P21,P16
b) ACTIVACION DE LA TELOMERASA
c) HIPERFOSFORILACION DE rb
d) EXPRESION DE ONCOGENES
217. LOS CARCINÓGENOS INDIRECTOS NECESITAN:
a) ACTIVACIÓN METABÓLICA
b) COCARCINÓGENOS
c) AGENTES ALQUILANTES
d) AGENTES ACILANTES
218. LOS SÍNDROMES PARANEOPLÁSICOS PUEDEN PRODUCIR EXCEPTO:
a) TRASTORNOS DERMATOLOGICOS
b) ENDOCRINOPATIAS
c) SEPSIS
d) CAMBIOS VASCULARES Y HEMATOLOGICOS

219. LOS TEJIDOS QUE TIENE UN NIVEL DE REPLICACION BAJO PERO QUE
PUEDEN  SUFRIR DIVISIONES RAPIDAS EN RESPUESTA A ESTIMULOS
ESPECIFICOS SON:
a) LABILES
b) CELULAS MADRE
c) PERMANENTES
d) QUIESCENTES
220. LA CAQUEXIA DEL CANCER SE DEBE A:
a) LA NEOPLASIA COMPITE POR LOS NUTRIENTES CON EL HUESPED
b) ANOREXIA
c) SECRECION DE FACTOR DE NECROSIS TUMORAL
d) DETERIORO GENERAL

221. SON LESIONES SUBLETALES DEL DNA Y DIANAS GENETICAS PARA EL


DESARROLLO DE NEOPLASIAS MALIGNAS EXCEPTO
a) ONCOGENES
b) GENES DE LA APOPTOSIS
c) GENES SUPRESORES DEL CANCER (P53, P21, RB)
d) GENES REPARADORES DEL DNA
e) GENES REPRESORES
222. LA INSENSIBILIDAD A LAS SENALES INHIBITORIAS DEL CRECIMIENTO ESTA
DADO POR LESION A :
a) BRCA-1
b) RB,P53
c) BCL2
d) BAX
223. LOS MARCADORES TUMORALES PUEDEN SER EXCEPTO:
a) ISOENZIMAS
b) ANTIGENOS ONCOFETALES
c) HORMONAS
d) CITOCINAS
224. LAS CELULAS MADRE SE CARACTERIZAN POR:
a) SINTETIZAR HORMONAS
b) CAPACIDAD DE AUTORENOVACION Y REPLICACIÓN ASIMETRICA
c) CAPACIDAD DE SÍNTESIS PROTEICA
d) RESPONDER A ESTIMULOS EXTERNOS

225. LA LESION SUBLETAL DEL DNA POR RADIACIONES ULTRAVIOLETA ES POR:


a) DAÑO AL RB
b) DAÑO A RECEPTORES DE FACTORES DE CRECIMIENTO
c) DAÑO AL RNA MENSAJERO
d) FORMACION DE DIMEROS DE PIRIMIDINA
226. EL VEGF Y LAS ANGIOPOYETINAS INTERVIENEN EN PROCESOS COMO:
a) REMODELACIÓN DE LA MATRIZ EXTRACELULAR
b) DEGRADACIÓN DEL COLAGENO DE LA MATRIZ EXTRACELULAR
c) SÍNTESIS DE COLAGENO POR LOS FIBROBLASTO
d) VASCULOGENESIS Y ANGIOGENESIS
227. LAS PROTEINAS E6 Y E7 DEL VPH DE ALTO RIESGO SE UNEN AL GEN:
a) P27
b) RAS
c) RB Y P53
d) MYC
e) FOS

228. ENZIMAS ENCARGADAS DE LA DEGRADACIÓN DEL COLAGENO Y OTRAS


PROTEÍNAS DE LA MATRIZ EXTRACELULAR:
a) ELASTASA DE LOS NEUTROFILOS
b) CININAS
c) METALOPROTEINASAS
d) CATEPSINA G
229. ABCESOS, ÚLCERAS Y QUEMADURAS SON EJEMPLOS DE CICATRIZACIÓN
POR:
a) TEJIDO CONECTIVO
b) SEGUNDA INTENCIÓN
c) FIBROSIS
d) PRIMERA INTENCIÓN
230. SON CAMBIOS CROMOSOMICOS EN LAS NEOPLASIAS EXCEPTO:
a) AMPLIFICACION
b) TRASLOCACION
c) TRANSCRIPCION
d) DELECION

231. ES UNA CARACTERÍSTICA EXCLUSIVA DE LAS NEOPLASIAS MALIGNAS:


a) NECROSIS
b) ANGIOGENESIS
c) OCASIONAR METASTASIS
d) MITOSIS
232. EL PARECIDO QUE EXISTE ENTRE LA CÉLULA NEOPLÁSICA Y LA CONTRAPARTE NORMAL SE
DENOMINA GRADO DE:
a) PROGRESIÓN
b) DIFERENCIACIÓN
c) INDIFERENCIACIÓN
d) DESDIFERENCIACIÓN
233. SON CELULAS DIFERENCIADAS REPROGRAMADAS:
a) CELULAS MADRE INDUCIDAS
b) CELULAS MADRE TISULARES
c) CELULAS MADRE HEMATOPOYETICAS
d) CELULAS MADRE EMBRIONARIAS
234. ENTRE LAS ONCOPROTEÍNAS (PRODUCTOS PROTÉICOS DE LOS ONCOGENES) TENEMOS:
a) BCL2
b) TELOMERASA
c) FCF
d) P53

235. SON MARCADORES TUMORALES EXCEPTO:


a) GONADOTROPINA CORIONICA HUMANA
b) INSULINA
c) ALFA FETOPROTEINA
d) ANTIGENO PROSTÁTICO ESPECÍFICO
236. CARCINÓGENOS QUÍMICOS EXCEPTO:
a) AGENTES ALQUILANTES
b) AFLATOXINA B
AMINAS AROMATICAS
c) ANTIBIOTICOS
237. UNO DE LOS GENES MUTADO EN LA MAYOR PARTE DE LOS CÁNCERES HUMANOS ES:
a) BAX
b) MYC
c) FIS
d) P53
238. LAS ENDOCRINOPATÍAS DE SÍNDROMES PARANEOPLÁSICOS SE ASOCIAN CON MAYOR
FRECUENCIA A:
a) CA EMBRIONARIO TESTICULAR
b) SEMINOMA
c) ADENOCARCINOMA PROSTÁTICO
d) CÁNCER DE PULMON
239. - ES UNA PROTEINA GRANDE DE ADHERENCIA CELULAR QUE LIGA MUCHAS MOLECULAS DE
LA MEC Y FORMA MATRIZ PROVISIONAL DURANTE LA CICATRIZACION:
a) CADHERINA
b) FIBRONECTINA
c) INTEGRINA
d) OSTEONECTINA
240. SON MEDIADORES QUIMICOS QUE INTERVIENEN EN LA ESTABILIZACION DE LOS VASOS
RECIEN FORMADOS
a) ON Y FCVE
b) PROSTAGLANDINAS
c) CDK Y CICLINAS
d) ANGIOPOYETINAS Y FCDP
241. -LA ASCITIS EN UN PACIENTE CON CIRROSIS ALCOHOLO NUTRICIONAL ES DEBIDO A:
A) AUMENTO DE LA PRESIÓN HIDROSTÁTICA Y OBSTRUCCIÓN LINFÁTICA
B) DISMINUCIÓN DE LA PRESIÓN OSMOTICA Y AUMENTO DE LA PRESIÓN HIDROSTÁTICA
C) RETENCION DE SODIO Y AUMENTO DE LA PRESIÓN HIDROSTÁTICA
D) OBSTRUCCIÓN LINFÁTICA Y DISMINUCIÓN DE LA PRESIÓN OSMOTICA
242. ¿CUÁL DE LAS SIGUIENTES SUSTANCIAS INHIBEN LA AGREGACION PLAQUETARIA?
a) TROMBOMODULINA
b) OXIDO NITRICO Y PROSTACICLINA
c) FACTOR ACTIVADOR DEL PLASMINOGENO TISULAR
d) ENDOTELINA
243. SUSTANCIA QUE ACTUA COMO FACTOR PROCOAGULANTE:
a) ENDOTELINA
b) FACTOR TISULAR
c) ANTITROMBINA III
d) PROSTACICLINA
244. ES UN ESTADO DE HIPERCOAGULABILIDAD DE ALTO RIESGO:
a) TABAQUISMO
b) USO DE ANTICONCEPTIVOS ORALES
c) FIBRILACION AURICULAR
d) MIOCARDIOPATIA
245. ES UN ANTICOAGULANTE NATURAL :
a) TROMBINA
b) ANTITROMBINA
c) FIBRINOGENO
d) FACTOR DE HAGEMAN
246. LA TROMBOSIS MESENTERICA ES UN EJEMPLO DE:
a) INFARTO BLANCO
b) INFARTO SEPTICO
c) INFARTO ROJO
d) INFARTO MURAL
247. EL ORIGEN MAS FRECUENTE DE LAS TROMBOEMBOLIAS PULMONARES ES:
a) LESIONES DEL ENDOCARDIO
b) VENAS PROFUNDAS Y SUPERFICIALES DE LOS MIEMBROS INFERIORES
c) ESTENOSIS MITRAL
d) TROMBOS MURALES INTRACARDIACOS
248. LOS INFARTOS BLANCOS SE PRODUCEN CUANDO HAY:
a) OBSTRUCCIÓN VENOSA
b) EN TEJIDOS CON DOBLE CIRCULACIÓN
c) EN TEJIDOS LAXOS
d) OBSTRUCCIÓN ARTERIAL EN ÓRGANOS SÓLIDOS CON RIEGO ARTERIAL TERMINAL
249. ES UN POTENTE VASOCONSTRICTOR QUE SE PRESENTA AL INICIO DE LA LESIÓN VASCULAR:
a) ENDOTELINA
b) PROSTACICLINA ENDOTELIAL
c) OXIDO NITRICO
d) HEPARINA
250. PERTENECEN AL GRUPO DE SHOCK DISTRIBUTIVO EXCEPTO:
a) SHOCK ANAFILACTICO
b) SHOCKCARDIOGENICO
c) SHOCK ENDOTOXICO
d) SHOCK NEUROGENICO
251. SON FACTORES SISTEMICOS QUE INFLUYEN EN LA REPARACIÓN DE LAS HERIDAS:
a) ESTADO CIRCULATORIO Y HORMONAS
b) INFECCIONES Y CUERPOS EXTRAÑOS
c) DENERVACION
d) ESTRÉS MECANICO
252. -SON TIPOS DE TROMBOS, EXCEPTO:
a) ARTERIALES
b) LIGAMENTOSOS, SUBSEROSOS
c) FLEBOTROMBOSIS
d) MURALES
253. LAS NEOPLASIAS MALIGNAS PUEDEN SER RESISTENTES A LA APOPTOSIS POR DAÑO A:
a) BRCA 1 Y 2
b) FAS Y JUN
c) RAS
d) BCL2
254. LA CAPACIDAD DE INVASION Y METASTASIS DE UNA NEOPLASIA MALIGNA ESTA DADA POR
:
a) CAPACIDAD PARA DEGRADAR LA MEC
b) SECRECION DE FACTORES DE CRECIMIENTO
c) SINTESIS DE COLAGENO
d) INSENSIBILIDAD A LAS SEÑALES INHIBITORIAS
255. PROTEINAS DE LA MEC QUE DESESTABILIZAN LAS INTERACCIONES CELULA-MATRIZ PARA
PROMOVER LA ANGIOGENESIS :
a) COLAGENO Y ELASTINA
b) ACIDO HIALURONICO Y DERMATAN SULFATO
c) ESTROMIELISINAS
d) SPARK Y TENASCINA
256. NEOPLASIA MALIGNA DE LAS CELULAS GERMINALES DEL TESTICULO
a) MELANOMA
b) LINFOMA
c) SEMINOMA
d) MESOTELIOMA
257. CORRELACIONE LOS TÉRMINOS DE LA COLUMNA IZQUIERDA CON LOS DE LA DERECHA
SEGÚN CORRESPONDAN:

46) SHOCK CARDIOGENICO  TAPONAMIENTO CARDIACO

47) LINFOMA  NEOPLASIA MALIGNA

48) INFARTO ROJO  TORSION DE QUISTE DE OVARIO

49) ADENOMA PLEOMORFO NEOPLASIA BENIGNA MIXTA

50) SHOCK HIPOVOLÉMICO  QUEMADURAS

1. Los síntomas del LES suelen aparecer entre los ____________ años.
a. 18 y 30
b. 30 y 50
c. 15 y 40
d. 20 y 60
2. Es una de las manifestaciones más frecuentes en el LES activo
a. Fiebre
b. Mareo
c. Fatiga
d. Ardor
3. Son molestias inespecíficas del LES excepto…
a. Astenia
b. Anorexia
c. Aumento de peso
d. Malestar General
4. En LES, La afección articular suele ser simétrica y afecta con mayor frecuencia a las
pequeñas articulaciones
a. Verdadero
b. Falso
5. Los nódulos subcutáneos, vasculitis cutánea, petequias, ulceras, aftas mucosas y el
fenómeno de Raynaud; son lesiones que pueden aparecer en el curso del LES.
a. Verdadero
b. Falso
6. Los derrames sinoviales son muy frecuentes en los pacientes con LES.
a. Falso
b. Verdadero
7. Es un problema significativo en pacientes con LES; es más frecuente en huesos
trabeculados
a. Artritis
b. Fracturas
c. Osteoporosis
8. Los síntomas de esta manifestación van en detrimento con la calidad de vida del paciente;
se presenta en 61% de los pacientes:
a. Fibromialgia
b. Gastritis
c. Afectación renal
9. La pericarditis es una manifestación cardinal de pacientes con LES:
a. Verdadero
b. Falso
10. Es un medicamento que no se administra en pacientes con LES:
a. AINE
b. Metrotexato
c. Insulina
11. Patología articular crónica degenerativa caracterizada por el deterioro y la pérdida de
cartílago articular junto con la proliferación y remodelación ósea subcondral.

Osteoartitis
Fibromialgia.

Artritis reumatoide.

Lupus Eritematoso.

12. La característica del dolor en la artrosis es:


Inicio Insidioso, mejora con el reposo y empeora con el ejercicio.
Es constante, persisite en reposo y llega a despertar al paciente por las noches.
13. Los datos que se obtienen a la exploración del Px con OA son todos,excepto:
Dolor a la presión

Crujidos articulares percibidos al tacto

Inflamación
Osteofitos en los márgenes de la articulación

14. La principal afectación en las articulaciones IF y CMT de la mano y rodillas la sufren:


Hombres

Mujeres
15. Nódulos presentes en las articulaciones interfalángicas proximales.
Nódulos de Heberden

Nódulos de Bouchard
16. ¿Cuál es la primera línea de tratamiento de la osteoartritis? Acetaminofén, Paracetamol.
Ketorolaco.

Diclofenaco.

17. Son medidas no farmacológicas para la artritis:

A) Evitar sobrecarga
B) Evitar contracturas en extensión
C) Usar calzado adecuado – plantillas

Ayc
A,b y c

18. Son ejemplos de factores sistémicos en la OA,excepto:


Estrógenos

Obesidad
Sexo

Edad
19. La prevalencia de la artrosis aumenta con la edad
Verdadero.
Falso.
20. Terapias Físicas empleadas en el Tx de la OA:
Hidroterapia, Parafina, EMS, masajes

Crioterapia, Masajes, Fluidoterapia, Interferenciales.


Hidroterapia, Crioterapia, T ENS, Masajes Ninguna
de las anteriores.

21. Enfermedad caracterizada por presencia de tender points.


Miopatías inflamatorias.

Fibromialgia.
Artritis reumatoide.

Lupus Eritematoso.

22. ¿Cuántos Tender points debe presentar un paciente para diagnosticarse con fibromialgia?
6-7

3-5

9-11

Once o más.
23. ¿Qué porcentaje de la población mundial se ve afectada por fibromialgia?
8-10%
2-5%
1-10%

15-20%

24. ¿Cuál es la edad más común en que se presenta la fibromialgia?


60-80 años.

20-35 años.

No hay edad preponderante.

20-50 años.
25. ¿A qué sustancia se atribuye la alodinia e hipersensibilidad al dolor?
Triptófano.

Serotonina.

Sustancia P.
Epinefrina.

26. ¿Cuál es la primera línea de tratamiento de la fibromialgia? Acetaminofén, Paracetamol.


Ketorolaco.

Diclofenaco.

27. En la fibromialgia, los niveles de serotonina disminuyen, lo que provoca una


hipersensibilidad al dolor.
Verdadero.
Falso.

28. Los pacientes con fibromialgia no presentan trastornos psiquiátricos.


Verdadero.

Falso.
29. La fibromialgia no es considerada una enfermedad reumatológica.
Verdadero.

Falso.
30. Los estudios de laboratorio de paciente con fibromialgia arrojan:
VSG y FR elevados.
HLA- B7 positivo.
Ana positivos.
Ninguna de las anteriores.

1) Se caracterizan por degradar la MEC y permite la remodelación y la extensión


del tubo vascular
a) Tirocina cinasa
b) MPM Metaloproteinasas de matriz
c) Factor transformante beta
d) Factor derivado de plaquetas

2) La estadificación de los cánceres sólidos se basa en qué sistema


a) Sistema de Dukes
b) Sistema Bethesda
c) Sistema de TNM
d) Ninguna de las anteriores

3) ¿Cuál de las siguientes causas es más probable que se acumule el líquido si


existe variación?
a) Permeabilidad vascular
b) Presión linfática
c) Presión oncótica
d) Diapédesis

4) Dentro de las funciones de P53 se encuentran, excepto:


a) Pausar el ciclo celular para reparar el ADN
b) Inducir apoptosis
c) Inducir proliferación celular
d) Inducir senescencia

5) El cáncer es una enfermedad caracterizada por:


a) Daño genético letal
b) Senescencia celular
c) Lesión subletal del ADN
d) Adquisición de mutaciones letales

6) Es el colágeno que proporciona fibrillas de anclaje de las células epiteliales a la


membrana basal
a) Colágeno tipo I
b) Colágeno tipo III
c) Colágeno tipo IV
d) Colágeno tipo VII

7) El edema que ocurre en la filariasis se debe a


a) Disminución de la presión oncótica
b) Bloqueo de la circulación linfática
c) Aumento de la presión hidrostática venosa
d) Aumento de la permeabilidad vascular

8) La deficiencia de la glucoproteína Gp1b provoca la enfermedad de


a) Von Willebrand
b) Trombastenia de Glanzmann
c) Niemann Pick
d) Bernard-Soulier

9) Los principales factores de crecimiento implicados en la síntesis de tejido


conjuntivo, excepto
a) PDGF
b) FGF
c) IL-1
d) TGF-BETA

10) El gen 9,12 se asocia con


a) Von Hippel Lindau
b) Trisomía 21
c) Cromosoma Filadelfia
d) Leucemia mieloide

11) Fase del shock donde fracasan los mecanismos reflejos de compensación y
disminuye la perfusión de órganos vitales
a) Fase latente
b) Fase progresiva
c) Fase no progresiva
d) Fase irreversible

12) Marcador tumoral relacionado con carcinoma de colon, páncreas y pulmón


a) Alfa fetoproteína
b) Antígeno prostático
c) Antígeno carcinoembrionario
d) Enolasa Neuronal Específica

13) Se puede producir choque por:


a) Sepsis
b) Infarto del miocardio
c) Quemaduras
d) Todas las anteriores

14) Corresponde al evento principal de la hemostasia primaria:


a) Formación de fibrina
b) Formación del tapón plaquetario
c) Secreción de endotelina
d) Vasodilatación arteriolar
15) El gen APC inhibidor de las vías de señalización mitógenas, se ha relacionado
con las siguientes neoplasias:
a) Carcinoma basocelular y meduloblastoma
b) Carcinoma de estómago, colon, páncreas y melanoma
c) Retinoblastomas
d) Schwannoma y meningioma

16) Estos trombos se forman en la circulación venosa, suelen contener más


eritrocitos incorporados y pocas plaquetas
a) Trombo rojo
b) Trombo mural
c) Trombo arterial
d) Vegetaciones

17) Un estado inflamatorio crónico puede causar anemia ferropénica en respuesta


al:
a) Aumento de la albúmina
b) Aumento de la fibrina
c) Aumento de la hepcidina
d) Aumento del fibrinógeno

18) Neoplasia maligna de la glándula tiroides:


a) Coriocarcinoma
b) Adenocarcinoma
c) Condrosarcoma
d) Linfangiosarcoma

19) Se considera como extensión de las neoplasias a sitios anatómicamente


separados del lugar de origen:
a) Metástasis
b) Microinvasión
c) Son masas expansivas cohesivas
d) Invasión

20) Patología donde las personas que sufren mutaciones en el gen TP53 y que
tienen una posibilidad 25 veces mayor de experimentar una neoplasia maligna
antes de los 50 años que la población general:
a) Síndrome de desgaste
b) Carcinoma
c) Neoplasia Endócrina Múltiple
d) Síndrome de Li-Fraumeni
21) Si la alteración displásica es intensa y afecta a todo el espesor del epitelio, pero
la lesión no penetra la membrana basal, se habla de:
a) Carcinoma in situ
b) Tumor infiltrante
c) Tumor invasivo
d) Metaplasia

22) Marcador tumoral relacionado con el cáncer prostático:


a) Alfa fetoproteína
b) Ca-125
c) Ca-19-9
d) PSA

23) Morfológicamente las neoplasias malignas se caracterizan por:


a) Relación núcleo-citoplasma normal
b) Arquitectura tisular intacta
c) Pleomorfismo nuclear y celular
d) Figuras mitóticas normales

24) Neoplasia maligna de las células de los ductos seminales


a) Coriocarcinoma
b) Carcinoma embrionario
c) Seminoma
d) Tumor de senos endodérmicos

25) Son neoplasias malignas de las células germinales del testículo


a) Carcinoma
b) Teratoma
c) Cistoadenoma
d) Seminoma

26) Es una característica del síndrome de Trousseau asociado a carcinomas de


páncreas y estómago, se refiere a:
a) Émbolos
b) Déficit de glicoproteína 1b
c) Tromboflebitis migratoria
d) Infarto

27) Es la fase del shock en la que se activa el eje renina-angiotensina


a) Fase refractaria
b) Fase no progresiva
c) Fase irreversible
d) Fase progresiva
28) ¿Qué tan resistentes son las células musculares cardíacas a la hipoxia?
a) Muy resistentes, mueren después de muchas horas
b) Extremadamente sensibles, mueren a los 5 minutos
c) Bastante sensibles, mueren a los 20-30 minutos
d) Poco sensibles, mueren a las pocas horas

29) Son patologías que producen edema por aumento de la presión hidrostática del
vaso:
a) Neoplasias y filarias
b) Aumento de la secreción renina-angiotensina-aldosterona
c) Glomerulopatías y malnutrición
d) Pericarditis constrictiva y trombosis

30) Se observa tras una destrucción sustancial del tejido cuando la lesión
inflamatoria afecta a tejidos que no son capaces de regenerarse o cuando hay
abundante exudación de fibrina en tejidos o cavidades serosas que no es posible
eliminar adecuadamente
a) Inflamación crónica
b) Remisión
c) Curación por reposición de tejido conjuntivo
d) Resolución completa

31) Características de las señales sinápticas


a) Secreción de neurotransmisores en uniones celulares especiales
b) De difusión mínima y la señal se degrada rápido
c) Se usa para amplificar una respuesta o inhibirla mediante retroalimentación
d) Se libera al torrente circulatorio y actúa a distancia sobre células blanco

32) Se le conoce así cuando el tejido cicatricial crece más allá de los límites de la
herida original, además no se contrae:
a) Fibromatosis agresiva
b) Cicatriz queloide
c) Granulación exhuberante
d) Dehiscencia

33) Factores sistémicos que influyen en la curación de heridas:


a) Estrés oxidativo, pérdida del tono vascular, anemia
b) Nutrición, situación circulatoria, estado metabólico y hormonas
c) Infecciones, nutrición, función renal y cardíaca deficiente
d) Tipo de herida, profundidad de la herida y estado metabólico
34) Son neoplasias que tienen peor diferenciación o totalmente indiferenciadas, se
refiere al término de:
a) Nevo
b) Coristoma
c) Hamartoma
d) Anaplasia

35) Los tipos de VPH de alto riesgo producen proteínas oncógenas que ocasionan
lo siguiente, excepto:
a) Activan ciclinas
b) Inhiben la apoptosis
c) Combaten la senescencia celular
d) Inmunodeficiencia

36) Son sustancias que inhiben la función plaquetaria


a) Antitrombina
b) NO, prostaciclina, ADPasa
c) Plasminógeno
d) Tromboplastina

37) ¿Cuáles son las fases de la reparación en las heridas?


a) Infiltración, remodelación, fibrosis y costra
b) Sepsis, formación de tejido de granulación y angiogénesis
c) Angiogenia, tejido de granulación y remodelación
d) Angiogenia, inflamación, contracción, fibrosis e hipoplasia

38) Es el resultado de múltiples mutaciones que se acumulan de forma


independiente en diferentes células y contribuyen así a la generación de
subclones con características diferentes es a lo que se le conoce como:
a) Iniciación y promoción
b) Progresión tumoral
c) Acción directa e indirecta
d) Heterogeneidad tumoral
Segundo parcial de anatopato

1. La inactivación o deleción de p16, adquirida por vía somática se ven en las familias
propensas a:

Carcinomas de páncreas, glioblastomas y cáncer de esófago

Linfomas

Adenocarcinoma pancreático y colangiocarcinomas

Leucemia mieloide crónica y leucemias linfoblásticas agudas

2. Una de las razones por las cuales las células neoplásicas son inmortales:
No se autorrenuevan
Respetan las pausas del ciclo celular
P53 funciona adecuadamente
Expresan telomerasa
3. Una lesión endotelial tendrá como consecuencia:
No tiene relación con la coagulación
Desarrollo de un ambiente procoagulante
Produce la dilución de un coagulo
Desarrollo de un ambiente anticoagulante
4. El edema por obstrucción linfática se da por las siguientes condiciones excepto:
Neoplasica
Postquirúrgica
Postirradicación
Postquimioterapia
5. En el choque séptico se puede producir:
Enteropatía hemorrágica
Daño alveolar difuso
Todas las anteriores
Encefalopatía isquémica
6. La estadificación de los cánceres sólidos se basa en que sistema:
Sistema de Dukes
Ninguna de las anteriores
Sistema de Bethesda
Sistema de TNM
7. Es un factor de crecimiento con efectos mitógenos sobre los hepatocitos
Factor de crecimiento fibroblástico
Factor de crecimiento de dispersión
Factor de crecimiento epitelial
Factor de crecimiento derivado de las plaquetas
8. Describe el depósito extenso de colágeno que se registra en pulmones, hígado, riñón y
otros órganos, como consecuencia de la inflamación crónica
Fibrosis
9. Tienen mayores probabilidades de sufrir un infarto rojo, excepto. Riñón
10. Neoplasia maligna del nervio periférico Neurofibrosarcoma
11. El gen RB, inhibidor de la progresión celular se ha relacionado con las siguientes
neoplasias Retinoblastoma, osteosarcoma, cáncer de mama y pulmón
12. Se considera como extensión de las neoplasias a sitios anatómicamente separados del
lugar de origen.
Metástasis
Micro invasión
Son masas expansivas cohesivas
invasión
13. Contienen una mayor cantidad de sangre oxigenada se caracteriza por el color rojo
brillante y dilatación de las arteriolas:
Hiperemia activa
Hiperemia pasiva
Congestión crónica
Congestión pasiva
14. Neoplasia maligna del tejido adiposo Liposarcoma
15. Neoplasia maligna del epitelio placentario Coriocarcinoma
16. Es un carcinógeno químico de origen natural PAGINA 323
17. Neoplasia maligna de las células de los ductos seminales
Coriocarcinoma
Carcinoma embrionario
Seminoma
Tumor de senos endodermicos
18. Son características de la congestión pulmonar crónica excepto: Paredes engrosadas,
macrófagos cargadas de hemosiderina.
-Aguda: Capilares alveolares dilatados, edema septal alveolar, hemorragia intraalveolar
19. Es la enfermedad por descomposición con aparición de burbujas de nitrógeno Síndrome
por descompresión
20. Que tan resistentes son las células musculares cardíacas a la hipoxia
Muy resistentes, mueren después de muchas horas
Extremadamente sensibles, mueren a los 5 minutos
Bastante sensibles, mueren a los 20-30 minutos
Poco sensibles, mueren a las pocas horas
21. Es un factor sistémico que afecta la reparación tisular:
Infección
Estado metabólico
Cuerpo extraño
Tamaño y localización de la herida
22. Son neoplasias malignas su origen derivado del tejido conectivo o conjuntivo:
Neuroblastoma
Carcinoma
Sarcoma
Osteoma
23. El cáncer es una enfermedad caracterizada por
Daño genético letal
Senescencia celular
Lesión subletal al DNA
Adquisición de mutaciones letales
24. Son neoplasias que tienen peor diferenciación o totalmente indiferenciadas, se refiere al
término de
Nevo
Coristoma
Hamartoma
Anaplasia
25. Corresponde al evento principal de la hemostasia primaria
Vasodilatación arterial
Formación del tapón plaquetario
Formación de fibrina
Secreción de endotelina
26. Fase del shock donde fracasan mecanismos reflejos de compensación disminuye la
perfusión de órganos vitales Progresiva
27. La característica histológica predominante de un infarto es Necrosis coagulativa
28. Sustancia generada por sistema de cininas cuando hay daño vascular, promueve
contracción de musculo liso, aparición de dolor y promueve la permeabilidad vascular
Endotelina
29. La asbestosis y silicosis está asociada con que tipo de neoplasia Mesotelioma y
carcinoma de pulmón
30. Es considerado como guardián del genoma TP53
31. El restablecimiento de la estructura tisular normal solo se produce: Regeneración
32. Es un marcador tumoral de tipo hormonal Calcitonina
33. Afecta el proceso de cicatrización, excepto: Exceso de acido ascórbico
34. Los estados de hipercoagulabilidad se dividen en: Primarios, secundarios, genéticos y
adquiridos
35. El carcinoma epidermoide de pulmón, carcinoma de mama, carcinoma de células renales
y la leucemia/linfoma de linfocitos T en el adulto se han relacionado con el siguiente
síndrome paraneoplásico. Hipercalcemia
36. Es una causa de edema por disminución de la presión oncótica:
Diabetes mellitus
Desnutrición
Insuficiencia cardíaca
Cáncer
37. Neoplasia benigna del nervio periférico Neurofibroma
38. Células que están presentes en el tejido conectivo contiguos a vasos sanguíneos por
debajo de superficies mucosas de vías respiratoria, la unión de antígeno alergeno a
anticuerpo IgE, se refiere a:
Podocitos
Dendríticas
Macrófagos
Mastocitos
39. Son trastornos adquiridos predisponentes a cáncer Inflamación, mayor replicación e
inflamación crónica, LESIONES PRECURSORAS Y ESTADOS DE INMUNODEFICIENCIA
40. Es un estado inflamatorio crónico asociado al cáncer 279
41. Un estado inflamatorio crónico puede causar anemia ferropénica en respuesta al: al
aumento de hepcidina
42. La edad, influye de manera determinante en el riesgo de cáncer, por lo tanto, la mayoría
de los carcinomas se presentan en la edad de: 55 años
43. Estos trombos se forman en la circulación venosa, suelen contener más eritrocitos
incorporados y relativamente pocas plaquetas:
Trombos arteriales
Trombos rojos
Trombos murales
Vegetaciones
44. Es el colágeno que proporciona fibrillas de anclaje de las células epiteliales a la
membrana basal Colágeno IV
45. Se refiere al reemplazo y recuperación del estado normal, de algunos tejidos
Reparación
Cicatrización
Restitución
Regeneración
46. Son ejemplos del grupo fisiopatológico de edema por disminución de presión oncótica:
Pericarditis constrictiva y trombosis
Filariasis y neoplasias
Desnutrición marasmatica y trombosis
Desnutrición kwashorkior y cirrosis hepática
47. La proliferación exuberante de fibroblastos y otros elementos de tejido conectivo se
denomina:
Queloide
Fibromatosis agresiva
Contractura
Granulación exuberante
48. Son carcinógenos de acción directa excepto
Beta propiolactona
Dimetil sulfato
1 acetil-imidazol
Hidrocarburos aromáticos
49. ¿A que deben los tumores su potencial replicativo ilimitado?
Carecen capacidad de autorrenovación
Presencia de inhibidores de la telomerasa
Responden adecuadamente a las señales apoptósicas
Activación de la telomerasa
50. Se refiere a los cambios reversibles, hereditarios en la expresión genética que se
producen sin mutación:
Cambios epigenéticos
Deleción
Amplificación génica
Cambios cromosómicos
51. Que efecto tiene la disminución de ATP en el pH citosólico:
Disminuye por aumento de ácido láctico
Aumenta por incremento de bicarbonato
Disminuye por aumento de piruvato
Disminuye por incremento de ácido carbónico
52. Es una antiproteasa que inhibe la elastina neutra y es más abundante en tubo digestivo
y pulmón
Colectinas, bradicinina
Bradicinina
Alfa 1 antitripsina
Fibrinopeptidos
53. Mediadores relacionados con el dolor en la inflamación aguda
Leucotrienos y tromboxano A2
Quimiocinas y productos bacterianos
PAF
Prostaglandinas y bradicinina
54. Es una proteína que cuando se une a procaspasa 8 y es utilizada por virus y células
normales para protegerse de apoptosis por medio del receptor de muerte fast:
TNF
FLIP
BH3
FADD
55. El factor de crecimiento de fibroblastos esta implicado en diferentes procesos excepto:
Formación de nuevos vasos sanguíneos
Es mediador de la inflamación
Reparación de heridas
Angiogenia
56. Una de las maneras en las que un tumor escapa de la respuesta inmune es:
Sobreexpresando antígenos
Sobreexpresando moléculas CPH I
Expresando citocinas activadores de linfocitos T
Expresando proteínas inhibidoras de linfocitos T
57. Cual de las siguientes sustancias inhiben la agregación plaquetaria?
ATPASA
FACTOR TISULAR
OXIDO NITROSO
POSTACICLINA
58. Los infartos rojos se producen en caso de: Órganos con doble circulación, oclusiones
venosas, tejidos laxos, previamente congestionados, cuando se reestablece flujo en una
zona de oclusión
59. Masa neoplásica gelatinosa llamada pseudomixoma peritonal es formado a veces por
que tipo de cáncer
Adenocarcinoma intestinal y de vías biliares
Todas las anteriores
Linfoma asociado a mucosa
Carcinoma apendicular y de ovario
60. Morfológicamente las neoplasias malignas se caracterizan por
Relación núcleo citoplasma normal
Arquitectura tisular inactiva
Pleomorfismo nuclear y celular
Figuras mitóticas normales
61. No siempre responden a las moléculas que inhiben la proliferación de las células
normales, habitualmente por la inactivación de genes supresores de tumores que
modifican componentes de estas vías, inhibitorias del crecimiento:
Potencial ilimitado de la replicación
Evasión de la apoptosis
Insensibilidad a las señales inhibitorias del crecimiento
Alteración del metabolismo celular
62. Son células madre que derivan de células adultas reprogramadas Inducibles
63. La fagocito oxidasa más óxido nítrico participan en el estallido respiratorio formando
Peroxinitrito
Anión superóxido
Peróxido de hidrogeno
Hipoclorito
64. Marcador tumoral relacionado con feocromocitomas y tumores relacionados
Catecolaminas y metabolitos
Calcitonina
Hormonas ectópicas
Gonadotropina coriónica humana
65. La cicatriz se define como
Proliferación celular diversa sin interacción
Depósito extensi de colágeno en órganos lesionados (fibrosis)
Estructuras de soporte tisular gravemente dañadas (lesión celular, ulcera)
Reparación por depósito de tejido fibroso
66. Son etapas de la invasión y metástasis de una neoplasia maligna excepto:
Disminución de contacto intracelular por cadherina E
Formación de nueva matriz extracelular
Degradación de la matriz extracelular
Migración de células tumorales
67. En que se podrían emplear en un futuro las CME:
Repoblar órganos dañados
Contra el cáncer
Secreción de citocinas
Inhibir la inflamación crónica
68. El receptor EGFR2 también se conoce como
ERB-B1
EGFR-1
HER2
EGFR
69. Las células atróficas mueren cuando el aporte sanguíneo no es suficiente para mantener
la vida, también se llama
Cambio hialino
Osteoporosis
Cambio graso
Atrofia parda
70. Es el síndrome paraneoplásico más frecuente
Hiperuricemia
Acantosis nigricans
Policitemia
Hipercalcemia
71. Son efectos patológicos de los radicales libres excepto
Daño al ADN
Conversión de superóxido a peróxido por la superóxido dismutasa
Degradación de proteínas
Peroxidación lipídica
72. Los tumores más comunes en los hombres se originan en
Mama, pulmones, colon y recto
Piel, leucemia y páncreas
Piel, orofaringe y páncreas
Próstata, pulmones, colon y recto
73. Todo lo siguiente se pueden esperar en la congestión hepática excepto:
Regiones centrolobulillares distendidas por sangre
Infartos en forma de pirámide
Patrón de nuez moscada
Incremento de peso
74. La bipedestación provoca edema en las partes declives del organismo ¿Cuál de los
siguientes mecanismos explica mejor este fenómeno? Aumento de la presión
hidrostática, inactividad de las piernas
75. Se determina mediante exploración quirúrgica o con técnicas de imagen y depende del
tamaño, la propagación a los ganglios linfáticos locales y regionales, y la presencia de
metástasis remota Estadificación
76. Son ejemplos de shock cardiogénico, excepto:
77. Es la fase del shock en la que se activa el eje renina-angiotensina-aldosterona
Fase refractaria
Fase no progresiva
Fase irreversible
Fase progresiva
78. Se caracterizan por degradar la MEC y permite la remodelación y la extensión del tubo
vascular
Tirocina cinasa
MPM metaloproteinasas de matriz
Factor transformante beta
Factor derivado de plaquetas
79. El edema que ocurre en la filariasis se debe a:
Disminución de la presión oncótica
Bloqueo de la circulación linfática
Aumento de la presión hidrostática venosa
Aumento de la permeabilidad vascular
80. Cual enunciado acerca de las plaquetas es verdad
Contiene cuerpos densos ricos en fibrinógeno
Los precursores son monocitos
Contiene gránulos alfa ricos en difosfato de adenosina
Contiene actina y miosina, microfilamentos y microtúbulos
81. El gen RB, inhibidor de la progresión celular, se ha relacionado con la siguiente
neoplasia:
Retinoblastoma, osteosarcoma, carcinoma de mama, colon y pulmón
Carcinoma basocelular y meduloblastoma
Carcinoma basocelular
Carcinoma de células renales y paraganglioma
82. El gen 9, 22 se asocia con:
Von Hippel-Lindau
Trisomía 21
Cromosoma filadelfia
Leucemia mieloide
83. Características de las señales sinápticas
Secreción de neurotransmisores en uniones celulares especiales
De difusión mínima y la señal se degrada rápido
Se usa para amplificar una respuesta o inhibirla mediante retroalimentación
Se libera al torrente circulatorio y actúa distancia sobre células blanco
84. Este tipo de células están presentes en la inflamación crónica, son abundantes en las
relaciones inmunitarias mediadas por inmunoglobulina E y en infecciones parasitarias
Neutrófilos
Linfocitos
Eosinófilos
Macrófagos
85. En que se diferencia la hiperemia de la congestión Hiperemia=Activa, Congestión=Pasiva
86. En la inmortalidad de las células malignas es considerado como factor decisivo
Evasión de la senescencia
Mutación puntual
Reordenamiento de cromosomas
Capacidad de regulación
87. Dentro de las funciones de p53 se encuentran, excepto:
Pausar el ciclo celular para reparar el ADN
Inducir apoptosis
Inducir proliferación celular
Inducir senescencia
88. Marcador tumoral relacionado con el cáncer prostático
Alfa fetoproteína
CA-125
CA-19-9
PSA
89. Los tipos de VPH de alto riesgo producen proteínas oncogenas que ocasionan lo
siguiente excepto:
Activan ciclinas
Inhiben la apoptosis
Combaten la senescencia celular
Inmunodeficiencia
90. Son sustancias que inhiben la función plaquetaria
Antitrombina
ON, prostaciclina y ADPASA
Plasminógeno
Tromboplastina
91. Neoplasia maligna de la glándula tiroides
Coriocarcinoma L
Adenocarcinoma
Condrosarcoma
Linfangiosarcoma
92. Se puede producir choque por:
Todas las anteriores
Sepsis
Infarto del miocardio
Quemaduras
93. Se observa tras una destrucción de tejido, cuando la lesión inflamatoria afecta a tejidos
que no son capaces de regenerarse o cuando hay abundante exudación de fibrina en
tejidos o cavidades serosas que no es posible eliminar adecuadamente
Inflamación crónica
Remisión
Curación por reposición de tejido conjuntivo
Resolución completa
94. Marcador tumoral relacionado con carcinoma de colon, páncreas y pulmón
Alfa fetoproteína
Antígeno prostático
Antígeno carcinoembrionario
Enolas neuronal específica
95. Son patologías que producen edema por aumento de la presión hidrostática del vaso
Neoplasias y filarias
Aumento de la secreción renina-angiotensina-aldosterona
Glomerulopatías y malnutrición
Pericarditis constrictiva y trombosis
96. Como se denominan las laminaciones producidos por capas pálidas alternantes de
plaquetas mezcladas con fibrina y alternadas con hematíes en el trombo
Embolo
Trombo mural
Flebotrombo
Líneas de zahn
97. Neoplasia benigna del músculo liso
Adenoma pleomorfo
Osteosarcoma
Condroma
Leiomioma
98. Un estado inflamatorio crónico puede causar anemia ferropénica en respuesta al:
Aumento de albúmina
Aumento de fibrina
Aumento de la hepcidina
Aumento del fibrinógeno
99. Son proteínas compuestas por tres cadenas polipeptídicas distintas tranzadas en triple
hélice:
Glucoproteínas adhesivas
Proteoglucanos
Colágenos
Hialuronatos
100. P53 y RB son ejemplos de:
Genes supresores de tumores
Reparación de ADN
Regulador de la apoptosis
Protooncogenes-oncogenes
101. El gen RB, inhibidor de la progresión celular se ha relacionado con la siguiente
neoplasias:
102. Efectos de la trombina Efecto trombolítico
103. Patología donde las personas que sufren mutaciones en el gen p53 y que tienen
una posibilidad 25 veces mayor de experimentar una neoplasia maligna antes de los 50
años que la población general
Síndrome de desgaste
Carcinoma
Neoplasia endocrina múltiple
Síndrome de Li- Fraumeni
104. Mutaciones que contribuyen al fenotipo maligno se denomina
Fenotipo mutador
Inestabilidad genómica
Ninguna de las anteriores
Mutación conductora
105. Analiza la aparición de tumores en poblaciones humanas Epidemiologia
106. Células responsables de la síntesis de nueva matriz extracelular Fibroblastos
107. La vía de propagación más común de los carcinomas es:
Siembra directa de cavidades o superficies corporales
Hematógena
Linfática
Continuidad
108. La extravasación de líquido al intersticio, cual es el término que se utilizaría
Edema
Trasudado
Exudado
Infiltración
109. Las proteínas E6 y E7 del VPH de alto riesgo inhiben los genes: RB y TP53. P21
110. La gradación histopronostica de una neoplasia se refiere a: Grado de
diferenciación
111. La falta de diferenciación, es decir, implica una inversión de la diferenciación
hacia un plano más primitivo
Displasia
Pleomorfismo
Anaplasia
Diferenciación
112. Corresponde al evento principal de la hemostasia primaria
Formación de fibrina
Formación del tapón plaquetario
Secreción endotelial
Vasodilatación arteriolar
113. Cuales son las características del tejido de granulación
Rico en colágeno y elastina
Rico en tejido conjuntivo provisional y nuevos capilares finos
Bajo en fibroblastos
Abundantes células inflamatorias
114. Un émbolo gaseoso puede originarse Diferencia de presión
115. Su crecimiento se acompaña de infiltración, invasión y destrucción progresiva
del tejido circundante, están poco delimitados y son de lenta expansión
Neoplasia maligna
Neoplasia preinvasiva
Cáncer in situ
Neoplasia benigna
116. Es un ejemplo de edema localizado que ocurre en casos de:
Síndrome nefrótico
Trichuriasis
Cirrosis hepática
Insuficiencia cardiaca derecha
117. En el choque séptico se puede producir:
118. Los infartos pálidos se observan en
Casos de obstrucción venosa
A Y B son correctas
Órganos sólidos
Casos de oclusión arterial
119. El virus de Epstein Barr produce:
Hepatocarcinoma
Leucemia de células peludas
Linfoma de Burkitt
Sarcoma de Kaposi
120. En el estado conocido como shock séptico
Es originado por pérdida de líquidos
Es producido por el paso de bacterias, hongos al torrente circulatorio
Carece de respuesta inflamatoria
Se considera una respuesta inflamatoria local
121. Es el término que se aplica a la neoplasia epitelial benigna derivada de las
glándulas, se denomina como
Papiloma
Pólipo
Angioma
Adenoma
122. Son tumores constituidos por células inmaduras que se parecen a las que forman
tejido rudimentario fetal
Papiloma
Blastoma
Hamartoma
Osteoma
123. Cual de los siguientes mediadores causan vasoconstricción y broncoconstricción
Tromboxano A2
IL-2
Leucotrienos B4
Ninguna de las anteriores
124. La congestión pasiva crónica del pulmón puede incluir todo lo siguiente excepto:
lo sacamos en el examen
Edema pulmonar
Eritema
Macrófagos con hemosiderina
Induración café
125. Especifique las variables que alternan la cicatrización
Infección, diabetes, estado nutricional, factores mecánicos, mala perfusión, tipo, alcance y
localización de la lesión
Estado nutricional adecuado y esteroides
Baja presión local o la torsión de las heridas
Infección y diabetes controlada
126. La luz UV:
Es un carcinógeno químico
Hace dímeros en la guanina
No es un agente carcinógeno
Puede causar carcinoma basocelular
127. Son procesos en la evolución de un trombo excepto
Embolia recanalización
Propagación
Trombólisis
Recanalización
128. El shock cardiogénico se produce cuando existe una pérdida de la masa mascular
del ventrículo izquierdo de:
25%
60%
45%
35%
129. La mayor parte de las flebotrombosis se producen en:
Arterias de mediano calibre
Corazón
Venas superficiales y profundas de las piernas
Arterias cerebrales
130. Se media por las interacciones con el factor de von willebrand con la
glucoproteína IB, receptor de la superficie de las plaquetas y el colágeno expuesto
Activación plaquetaria
Adhesión plaquetaria
Agregación plaquetaria
Fase de iniciación de la coagulación
131. Que gen supresor de tumores intervienen en la patogenia del carcinoma de
mama
Ninguna de las anteriores
APC 1 y APC2
BCRA 1 y BCRA 2
NF1 y NF2
132. Son acontecimientos plaquetarios excepto:
Adhesión
Migración
Cambio de forma y secreción
Agregación
133. El efecto Warburg consiste en
134. Neoplasia maligna de los vasos sanguíneos
Linfoma
Linfangiosarcoma
Leucemia
Angiosarcoma
135. Son estados de hipercoagulabilidad de bajo riesgo excepto:
Tabaquismo
Tabaquismo
Infarto agudo al miocardio
136. Cambios celulares que abarcan dos tercios o más del espesor del epitelio
escamoso de la mucosa cervicovaginal. Corresponde a que tipo de lesión
Cáncer in situ NIC III
Infección por VPH
Carcinoma micro invasor
Displasia moderada
137. La deficiencia de la glucoproteína GPIB provoca la enfermedad de:
Von willebrand
Tromboastenia de glanzmann
Nieman pick
Bernard soulier
138. Principales factores de crecimiento implicados en la síntesis de tejido conjuntivo,
excepto:
PDGF
FGF
IL-1
TGF-BETA
139. Si la alteración displásica en intensa y afecta a todo el espesor del epitelio, pero
la lesión no penetra la membrana basal, se habla de:
Carcinoma in situ
Tumor infiltrante
Tumor invasivo
Metaplasia
140. Trombo formado en las cavidades cardiacas:
Trombo mural
Aneurisma
Petequia
Émbolo
141. Tienen mayores probabilidades de sufrir infarto rojo, excepto:
Intestino delgado
Pulmón
Riñón
142. Es la enfermedad por descompresión con aparición de burbujas de nitrógeno
143. En la cicatrización por primera intención, en que tiempo la neovascularización
alcanza su máximo y el tejido de granulación llena la incisión:
12 horas
5 días
144. Cual enunciado acerca de las plaquetas es verdad
Contiene cuerpos densos ricos en fibrinógeno
Los precursores son monocitos
Contiene gránulos alfa ricos en difosfato de adenosina
Contiene actina y miosina, microfilamentos y microtúbulos
145. Todo lo siguiente se pueden esperar en la congestión hepática excepto
Regiones centrolobulillares distendidas por sangre
Infartos en forma de pirámide
Patrón de nuez moscada
Incremento de peso
146. Factores sistémicos que influyen en la curación de heridas
Estrés oxidativo, perdida del tono vascular, anemia
Nutrición, situación circulatoria estado metabólico y hormonas
Infecciones, nutrición, función renal y cardiaca deficiente
Tipo de herida, profundidad de la herida y estado metabólico
147. El gen APC inhibidor de las vías de señalización mitogena, se ha relacionado con
las siguientes neoplasias:
Carcinoma basocelular y meduloblastoma
Carcinomas de estómago, colon, páncreas y melanoma
Retinoblastomas
Schwannoma y meningioma
148. Son neoplasias de células germinales del testículo:
Carcinomas
Teratomas
Cistoadenomas
Seminomas
149. Es una característica del síndrome de Trousseau asociado a carcinomas de
páncreas y estómago se refiere a:
Émbolos
Déficit de glucoproteína 1B
Tromboflebitis migratoria
Infarto
150. ¿Cuáles son las fases de la reparación en las heridas?
Infiltración, remodelación, fibrosis y costra
Sepsis, formación de tejido de granulación y angiogénesis
Angiogenia, tejido de granulación y remodelación
Angiogenia, inflamación, contracción, fibrosis e hipoplasia
151. El resultado de múltiples mutaciones que se acumulan de forma independiente
en diferentes células contribuye así a la generación de subclones con características
diferentes es a lo que se le conoce como
Iniciación y promoción
Progresión tumoral
Acción directa e indirecta
Heterogeneidad tumoral
152. Tipo de colágeno que forma las membranas basales:
Colágeno tipo I
Colágeno tipo V
Colágeno tipo IV
Colágeno Tipo III
153. El estroma reactivo está compuesto por lo siguiente:
Células neoplásicas, tejido conjuntivo y vasos sanguíneos
Células neoplásicas, tejido conjuntivo, vasos sanguíneos y células de sistema inmunitario,
adaptativo e innato
Células neoplásicas y tejido conjuntivo
Tejido conjuntivo, vasos sanguíneos y células de sistema inmunitario, adaptativo e innato.
154. La activación del activador de plasminógeno genera su conversión a plasmina
para:
Activar plaquetas
Iniciar la cascada fibrinolítica
Secreción y cambio de forma plaquetaria
Activación leucocitaria
155. Que enzima secreta la célula neoplásica para destruir la membrana basal del
endotelio y penetrar a la circulación
Oxigenasa, metalasa, gelatinasa
Condroitinsulfatasa ostenectinasa, colagenasa
Colagenasas, gelatinasas, estromelisinas
Sulfatasa, colagenasa, condronectinasa
156. En su mayoría, los casos de shock endotóxico son causados por:
Sepsis por virus
Sepsis por gérmenes gran positivos
Sepsis por hongos
Sepsis por clamydia
157. EL síndrome de poliposis adenomatosa del colon presenta defectos hereditarios
en APC
158. El edema periorbitario es frecuente en:
Cirrosis
Desnutrición
Nefropatía grave
Arritmias
159. Es un estado inflamatorio crónico asociado al cáncer
Miositis
Enfermedad intestinal inflamatoria
Condritis
Pielonefritis
160. Cual es la proteína más abundante en el ser humano
Vimentina
Miosina
Caderina
Colageno
161. Se le denomina así cuando las células parenquimatosas estimulan la síntesis de
un estroma con colágeno abundante:
Desmoplasia
Pétreo
Parénquima
Estroma reactivo
162. La lesión subletal por radiación ultravioleta causa:
Ca de pulmón
Ca de células basales
163. El factor de crecimiento transformante beta tiene actividad:
Mitogenica
Inhibidor de la mitosis
Inhibidor del depósito de colágeno
Fibrogenica
164. Es una característica exclusiva de las neoplasias malignas:
Necrosis
Angiogénesis
Ocasionar metastasis
Mitosis
165. Son marcadores tumorales excepto:
Gonadotropina coriónica humana
Insulina
Alfa fetoproteína
Antígeno prostático específico
166. Entre las oncoproteínas (productos protéicos de los oncogenes) tenemos:
BCL2
Telomerasa
FCF
P53
167. El parecido que existe entre la célula neoplásica y la contraparte normal se
denomina grado de:
Progresión
Diferenciación
Indiferenciación
Desdiferenciación
168. Abscesos, úlceras y quemaduras, son ejemplos de cicatrización por:
Tejido conectivo
Segunda intención
Fibrosis
Primera intención
169. Son células diferenciadas reprogramadas:
Células madre inducidas
Células madre tisulares
Células madre hematopoyéticas
Células madre embrionarias
170. Enzimas encargadas de la degradación del colágeno y otras proteínas de la
matriz extracelular:
Elastasas de los neutrófilos
Cininas
Metaloproteinasas
Catepsina G
171. La lesión subletal del ADN por radiación ultravioleta es por:
Daño al RB
Daño a receptores de factores de crecimiento
Daño al RNA mensajero
Formación de dimeros de pirimidina
172. El VEGF y las angiopoyetinas intervienen en procesos como:
Remodelación de la matriz extracelular
Degradación del colágeno de la matriz extracelular
Síntesis de colágeno por los fibroblastos
Vasculogenesis y angiogeneses
173. Son lesiones subletales del DNA y dianas genéticas para el desarrollo de
neoplasias malignas excepto:
Oncogenes
Genes de la apoptosis
Genes supresores del cáncer (p53, p21, RB)
Genes reparadores del dna
Genes represores
174. Las células madre se caracterizan por:
Sintetizar hormonas
Capacidad de autorrenovación y replicación asimétrica
Capacidad de síntesis proteica
Responder a estímulos externos
175. La insensibilidad a las señales inhibitorias del crecimiento está dado por lesión a:
BRCA-1
RB, P53
BCL2
BAX
176. La caquexia del cáncer se debe a:
La neoplasia compite por los nutrientes con el huésped
Anorexia
Secreción de factor de necrosis tumoral
Deterioro general
177. Es el cambio fundamental para obtener potencial replicativo ilimitado:
P53, p21, p16
Activación de la telomerasa
Hiperfosforilación de rb
Expresión de oncogenes
178. Los carcinógenos indirectos necesitan:
Activación metabólica
Cocarcinógenos
Agentes alquilantes
Agentes acilantes
179. Los tejidos tienen un nivel de replicación bajo pero que pueden sufrir divisiones
rápidas en respuesta a estímulos específicos son:
Lábiles
Células madre
Permanentes
Quiescentes
180. Los síndromes paraneoplásicos pueden producir excepto:
Trastornos dermatológicos
Endocrinopatías
Sepsis
Cambios vasculares y hematológicos
181. Los marcadores tumorales pueden ser excepto:
Isoenzimas
Antígenos oncofetales
Hormonas
Citocinas
182. Son cambios cromosómicos en las neoplasias excepto:
Amplificación
Traslocación
Transcripción
Deleción
183. Carcinógenos químicos excepto:
Agentes alquilantes
Aflotoxina B
Aminas aromáticas
Antibióticos
Funciones del factor de crecimiento transformador beta excepto:

Neoplasia benigna del musculo liso​: LEIOMIONA

Es la magnitud en que las células parenquimatosas neoplasias se asemejan en su


función y forma las células correspondientes del parénquima sano​: DIFERENCIACIÓN

Una lesión endotelial tendrá como consecuencia:

Es un grupo de proteínas grandes multifuncionales cuya principal actividad es inhibir


la angiogénesis:

Masa neoplásica gelatinosa llamada pseudomixoma peritoneal es formado a veces


por que tipo de cánce​r: CANCER DE APENDICE CREO

En el estado conocido como shock séptico:

Son neoplasisas mixtas excepto​:

Las neoplasias mixtas son: adenoma pleomorfo, tumor mixto de las glandulas salivales,
Teratoma, Teratoma quistico del (quiste dermoide) ovario, Hamartomas, Coristoma

¿Cuál es la secuencia en la formación del tapón hemostático primario?

Rotura del endotelio para que exponga al factor de von Willebrand y colágeno , que
promueven la adhesión de plaquetas. Cambio de forma en las plaquetas y la liberación de
gránulos secretores. A los pocos minutos los productos secretados que reclutan a más
plaquetas que pasan a agregación para formar el tapón.

¿Aque gen induce TP53 para dirigir la célula a apoptosis?

Tiempo de protrombina:

Factor de crecimiento placentario pertenece a la familia de: ​VEGF (factor de crecimiento


endotelial vascular)

En un ateroma ulcerado se produce:

EXPONEN A LA MEC SUBENDOTELIAL Y PRODUCEN TURBULENCIAS:

En el carcinoma ovárico, el fibrosarcoma y otros sarcomas mesenquimatosos se han


relacionado con el siguiente síndrome paraneoplasico​ HIPOGLUCEMIA (viene en una
tabla del libro)

Permiten determinar el diagnostico, pronosticos, la dtecccion de enfermedad residual


minima y el diagnostico de la predisposicón hereditaria al cancer​: DIAGNÓSTICO
MOLECULAR Y CITOGENETICO, EJEMPLO PCR.
Es el término que describe la formación de tejido conectivo como respuesta a los
tumores: ​CREO QUE ES LA CÁPSULA PERO NO ESTOY SEGURA, sí, si es. xD Valgo
verga, ni porque lo subraye

Qué entidad se refiere a una forma de coagulopatía de consumo caracterizada por


trombosis microangiopática de círculos vicioso trombosis y hemorragia:
COAGULACIÓN INTRAVASCULAR DISEMINADA

El carcinoma de las células renales, el hemangioma cerebelosos y el


hepatocarcinoma se han relacionado con el siguiente sindrome paraneoplasico
POLICETEMIA

Suelen ocasionar una perdida de función y en la mayoría de los casos para la


transformación se prescisa del daño de dos alelos​: DELECCIONES

¿Cuáles son las células más importantes en la eliminación de agente causales y


tejido muerto y promueven la migración y proliferación de los fibroblastos?
MACROFAGOS TIPO M2

Molecula de adhesión que contiene los granulos alfa de las plaquetas:


P-SELECTINA

La falta de differenciación es decir, implica una inversión de la difrenciación hacia un plano


más primitvo ANAPLASIA
Lo siento ILY

Cómo se clasifican los infartos


infarto rojo infarto blanco o infarto séptico o estéril

son tumores constituidos por células inmaduras que se parecen a las que forman
tejido rudimentario fetal​. Creo que son teratomas

acumulación de cantidades excesivas de colágeno que crece más allá de los bordes
de la herida original y no regresa ​cicatriz queloide

ejemplo de edema por disminución de la presión oncótica ​disminución de la síntesis de


albúmina por lo que entrarían las enfermedades hepáticas graves como la cirrosis terminal y
malnutrición proteínica, la causa más importante de pérdida de albúmina es el síndrome
nefrótico

procesos de la evolución de un trombo


propagación, embolia, disolución, organización y recanalizacion

trombo formado en cavidades cardíacas ​trombos murales

un trombo con lineas de zahn ​significa que el trombo sea formado en sangre que fluía
Se refiere así a la tendencia más perniciosa de los tumores de adquirir una conducta
cada vez más agresiva:​ EVOLUCIÓN Y LA SELCCIÓN GENETICA PUEDEN EXPLICAR
LAS DOS PROPIEDADES MÁS NOCIVAS DE LOS CANCERES: 1) sER MÁS AGRESIVOS
2)RESPONDER PEOR AL TX Es lo unico que encontre :’v Tal cual asi dice el libro

La deficiencia de GPBIIB-IIIA produce la enfermedad de:


TROMBASTENIA DE GLANZMANN

¿En que tiempo las neuronas sufren daños irreversibles a la hipoxia?


3- 4 MINUTOS
LAS CELULAS MIOCARDICAS MUEREN EN 20-30 MIN

Neoplasia maligna de las células mesoteliales:


NEOPLASIAS
1. Neoplasia maligna de la glándula tiroides.
R= ADENOCARCINOMA

2. Se considera como extensión de las neoplasias a sitios anatómicamente


separados del lugar de origen.
R= METASTASIS

3. En la inmortalidad de las células malignas es considerada como factor decisivo.


R= EVASIÓN DE LA SENESCENCIA

4. Son neoplasias que tienen peor diferenciación o totalmente indiferenciadas. Se


refiere al termino de:
R= ANAPLASIA

5. Son neoplasias malignas de células germinales del testículo.


R= SEMINOMAS

6. Es una característica del Sx de Trousseau asociado a carcinoma de páncreas y


estomago se refiere a:
R= TROMBOFLEBITIS MIGRATORIA

7. Patología donde las personas que sufren mutaciones en el gen TP53 y que tienen
posibilidad 25 veces mayor de experimentar una neoplasia maligna antes de los 50
años que la población general.
R= SÍNDROME DE LI-FRAUMENI

8. Son alteraciones diplásicas es intensa y afecta a todo el espesor del epitelio, pero
la lesión no penetra la membrana basal, se habla de:
R= CARCINOMA IN SITU
9. Dentro de las funciones de p53 se encuentran, excepto:
R= INDUCIR PROLIFERACIÓN CELULAR

10. El cáncer es una enfermedad caracterizada por:


R= ADQUISICIONE DE MUTACIONES LETALES.

11. El resultado de múltiples mutaciones que se acumulan de forma independiente


en diferentes células y contribuyen a la generación de subclones con
características diferentes a lo que se le conoce como:
R= PROGRESIÓN TUMORAL.

12. Los tipos de VPH de alto riesgo que producen proteínas oncogenas que
ocasionan lo siguiente excepto:
R= INMUNODEFICIENCIA

13. Neoplasia maligna de las células de los conductos seminales.


R= SEMINOMA

14. Marcador tumoral relacionado con el cáncer prostático.


R= PSA

15. Morfológicamente las neoplasias malignas se caracterizan por:


R= PLEOMORFISMO NUCLEAR Y CELULAR

16. El gen APC, inhibidor de las vías de señalización mitógena, se ha relacionado


con las siguientes neoplasias:
R= CARCINOMAS DE ESTOMAGO, COLÓN, PÁNCREAS Y MELANOMA.

17. El gen 9,12 se asocia con:


R= CROMOSOMA FILADELFIA
18. Marcador tumoral asociado con carcinoma de colon, páncreas y pulmón.
R= ANTÍGENO CARCINOEMBRIONANRIO

19. Neoplasia maligna de tejido adiposo


R= LIPOSARCOMA

20. Neoplasia maligna de epitelio placentario.


R= CORIOCARCINOMA

21. Neoplasia maligna de los vasos sanguíneos


R= ANGIOSARCOMA

22. La inactivación o deleción de p16, adquirida por vía somática se ve en las


familias propensas a:
R= CÁRCINOMA DE PÁNCREAS, GLIOBLASTOMA Y CÁNCERES DE
ESÓFAGO.

23. ¿Cuál es la definición de teratoma?


R= QUE SE ORIGINE DE MÁS DE 2 CAPAS GERMINALES

24. La estadificación de un tumor maligno se refiere a


R= TAMAÑO DE LA LESIÓN PRIMARIA Y SI HA REALIZADO METÁSTASIS

25. ¿Qué gen supresor de tumores intervienen en la patogenia del carcinoma de


mama?
R= BCR1 BCR2

26. El virus de Epstein Barr produce


R= SARCOMA DE KAPOSI
27. El efecto Warburg consiste en
R= LA NEOPLASIA MALIGNA FIJA LA GLUCOLISIS ANAEROBIA EN SU
METABOLISMO

28. Una de las razones por las cuales las células neoplásicas son inmortales
R= EXPRESESAN TELOMERASA

27. La luz UV
R= PUEDE CAUSAR CARCINOMA BASOCELULAR

28. Son tumores constituidos por células inmaduras que se parecen a las que
forman tejido rudimentario fetal.
R= BLASTOMA

29. Los tumores más comunes en los hombres se originan en


R= PROSTATA, PULMONES, COLON Y RECTO.

30. Es el término que se aplica a la neoplasia epitelial benigna derivadas de las


glándulas, se denomina como
R= ADENOMA

31. La falta de diferenciación, es decir, implica una inversión de la diferenciación


hacia un plano más primitivo
R= ANAPLASIA

32. Mutaciones que contribuyen al fenotipo maligno se denomina


R= FENOTIPO MUTADOR

33. La estadificación de los canceres solidos se basa en que sistema


R= SISTEMA DE TNM

33. Su crecimiento se acompaña de infiltración, invasión y destrucción progresiva


del tejido circundante, están poco delimitados y son de lenta expansión
R= NEOPLASIA MALIGNA

34. Es la vía frecuente para la diseminación inicial de los carcinomas


R= LINFÁTICA

35. Una vez que las células tumorales rompen la membrana basal, se habla de
R= TUMOR INFILTRANTE

36. Los tumores tienen capacidad para proliferar sin estímulos externos en
general, como consecuencia de la activación oncogena
R= AUTOSUFICIENCIA DE LAS SEÑALES DE CRECIMIENTO

37. Son neoplasias malignas, su origen es derivado del tejido conectivo o


conjuntivo
R= SARCOMA

38. Las células exhiben una variación en su tamaño y forma, es decir, las células
de un mismo tumor no son uniformes
R= PLEOMORFISMO

39. Es un protooncogen que pertenece a los inhibidores de las cinasas de las


ciclinas (INK) **
R= p16

40. Se refiere así, a la pérdida progresiva de la grasa corporal y de la masa


corporal magra, acompañada de una sensibilidad profunda, anorexia y anemia.}
R= CAQUEXIA CANCEROSA

41. Neoplasia maligna de células hematopoyéticas


R= LEUCEMIA

42. De las siguientes opciones, ¿Cuál no es una característica de anaplasia?


R= FALTA DE MITOSIS

43. El marcador tumoral CA 19-9 se relaciona principalmente con


R= CÁNCER DE COLON Y PÁNCREAS

44. Proteína relacionada con el síndrome familiar del retinoblastoma y


osteosarcoma
R= RB

45. Tienen una función esencial en muchos aspectos del fenotipo maligno, como
la expresión de los genes del cáncer, el control de la diferenciación y la
autorrenovación, incluso la sensibilidad de la resistencia a los medicamentos
R= CAMBIOS EPIGENÉTICOS

46. Mutaciones que contribuyen al fenotipo maligno se denomina *


R= MUTACIÓN CONDUCTORA

47. Neoplasia benigna de nervio periférico


R= NEUROFIBROMA

48. Es considerado como el guardián del genoma, ya que es un gen supresor de


tumores
R= TP53
TRASTORNOS HEMODINÁMICOS.
1. Células que están presentes en el tejido conectivo contiguos a vasos sanguíneos
por debajo de superficies mucosas de vías respiratorias, la unión de antígeno
alérgeno a anticuerpo IgE, se refiere a:
R= MASTOCITOS

2. Cual enunciado acerca de las plaquetas es verdad.


R= PARTICIPAN EN LA HEMOSTASIA SECUNDARIA

3. Corresponde al evento principal de la hemostasia primaria.


R= FORMACIÓN DEL TAPÓN PLAQUETARIO

4. La deficiencia de la glucoproteína GPIB provoca la enfermedad de:


R= SÍNDROME DE BERNARD SOULIER.

5. ¿Qué tan resistentes son las células musculares cardiacas a la hipoxia?


R= BASTANTE SENSIBLES, MUEREN A LOS 20-30 MINUTOS.

6. Son patologías que producen edema por aumento de la presión hidrostática del
vaso.
R= PERICARDITIS CONTRICTIVA Y TROMBOSIS.

7. Un estado inflamatorio crónico puede causar anemia ferropénica en respuesta al:


R= AUMENTO DE LA ALBÚMINA

8. Estos trombos se forman en la circulación venosa, suelen contener más eritrocitos


incorporados y relativamente pocas plaquetas
R= TROMBOS ROJOS

9. Se puede producir choque por:


R= TODAS LAS ANTERIORES (SEPSIS, INFARTO AL MIOCARDIO,
QUEMADURAS)

10. Son sustancias que inhiben la función plaquetaria.


R= ON (OXIDO NITRICO), PROSTACICLINA Y ADPasa.

11. El edema que ocurre en la filariasis se debe a:


R= BLOQUEO DE LA CIRCULACIÓN LINFÁTICA.

12. En la cicatrización de primera intención, en que tiempo la neovascularización


alcanza su máximo y el tejido de granulación llena la incisión.
R= 5 DÍAS

13. Contienen una mayor cantidad de sangre oxigenada se caracteriza por el color
rojo brillante y dilatación de las arteriolas.
R= HIPEREMIA PASIVA

14. Extravasación de líquido al intersticio, cual es el término que se utilizaría:


R= EDEMA

15. Son estados de hipercoagubilidad de bajo riesgo excepto:


R= INFARTO AGUDO AL MIOCARDIO

16. La cicatrización por primera intención ¿Cuándo se comienza a observar el tejido


de granulación?
R= 3 – 7 DÍAS DESPUÉS

17. Los estados de hipercoagubilidad se dividen en


R= GÉNETICOS Y ADQUIRIDOS
18. La congestión pasiva crónica del pulmón puede incluir todo lo siguiente excepto
R= INDURACIÓN CAFÉ

19. La mayor parte de las flebotrombosis se producen en


R= VENAS SUPERCIALES Y PROFUNDAS DE LAS PIERNAS

20. El shock cardiogénico se produce cuando existe una pérdida de la masa del
ventrículo izquierdo del
R= 45%

21. Se media por las interacciones con el factor de Von Willedrand con la
glucoproteína IB, receptor de las superficies de plaquetas y el colágeno expuesto
R= FASE DE INICIACIÓN DE LA COAGULACIÓN

22. Son acontecimientos plaquetarios excepto


R= MIGRACIÓN

23. Son procesos de evolución de un trombo excepto


R= TROMBOLISIS

24. ¿Cuál de los siguientes mediadores causan vasoconstricción y


broncoconstricción?
R= TR COMBOXANO A2

25. Es una causa de edema por disminución de la presión oncotica


R= DESNUTRICIÓN

26. El estado conocido como shock séptico


R= ES PRODUCIDO POR EL PASO DE BACTERIAS, HONGOS AL TORRENTE
CIRCULATORIO
27. Los infartos pálidos se observan en
R= OCLUSIONES ARTERIALES Y TEJIDOS SÓLIDOS

28. El edema por obstrucción linfática se da por las siguientes condiciones excepto
R= POSQUIMIOTERAPIA

29. Es un ejemplo de edema localizado el que ocurre en casos de


R= TRICHIURASIS

30. Fase del shock donde fracasan mecanismos reflejos de compensación


disminuye la perfusión de los órganos vitales
R= FASE IRREVERSIBLE

31. Un embolo gaseosos puede formarse


R= POR CAMBIOS BRUSCOS DE LA PRESIÓN

32. En el choque séptico se puede producir


R= ENCEFALOPATÍA ISQUÉMICA

33. ¿Cuál de las siguientes causas es más probable que se acumule el líquido, si
existe variación?
R= PRESIÓN ONCOTICA

34. Es la fase del shock donde se activa el eje renina-angiotensina


R= FASE NO PROGRESIVA

35. Defecto genético que predispone a sufrir trombosis conocidas como


trombofilias y un 5% a 10% de las personas lo puede presentar
R= DEFICIENCIA DE ANTITROMBINA III, PROTEÍNA C-S Y FACTOR V LEIDEN

36. Corresponde al evento principal de la hemostasia secundaria


R= DEPOSICIÓN DE COAGULO DE FIBRINA

37. ¿Cómo se clasifican los infartos?


R= ROJOS, BLANCOS; SÉPTICOS

38. La triada de Virchow incluye las siguientes características excepto


R= INCREMENTO DE LA VOLEMIA

39. Es una causa de la disminución de la presión osmótica


R= SÍNDROME NEFRÓTICO

40. ¿Proceso activo en el que la dilatación arteriolar aumenta en el flujo de sangre?


R= HIPEREMIA

41. El edema de origen cardiaco se caracteriza por


R= ORTOSTATICO/SUBCUTANEO

42. La aparición de hematomas puede deberse principalmente


R= ENFERMEDAD DE VON WILLEBRAND

43. La trombosis venosa (Fenómeno de Trousseau) se ha relacionado con síndrome


paraneoplásico
R= CARCINOMAS DE PÁNCREAS, BRONCÓGENO Y OTROS CÁNCERES

44. Principal causa que puede generar tromboembolia pulmonar


R= TROMBOSIS RPOFUNDA
45. Son células responsables de la síntesis de nueva matriz extracelular
R= CÉLULAS ENDOTELIALES

46. Es secundario a una insuficiencia de bomba miocárdica debido a las lesiones


intrínsecas del miocardio, comprensión extrínseca u obstrucción de flujo de salida
R= CHOQUE CARDIOGENICO

47. En su mayoría, los casos de shock endotoxico son causados por


R= SEPSIS POR GERMENES GRAM (-)

48. Es más frecuente la formación de émbolos a partir de


R= TROMBOS

REPARACIÓN.
1. Los principales factores de crecimiento implicados en la síntesis de tejido
conjuntivo, excepto: *
R= IL-1

2. Se les conoce así, cuando el tejido cicatrizal crece más allá de los límites de la
herida original, además no se contrae.
R= CICATRIZ QUELOIDE

3. Factores sistémicos que influyen en la curación de las heridas.


R= NUTRICIÓN, SITUACIÓN CIRCULATORIA, ESTADO METABOLICO Y
HORMONAS

4. ¿Cuáles son las fases de la reparación en las heridas?


R= ANGIOGENIA, TEJIDO DE GRANULACIÓN Y REMODELACIÓN.
5. Se observa una destrucción sustancial de tejido, cuando la lesión inflamatoria
afecta a tejidos que nos capaces de regenerarse o cuando hay abundante
exudación de fibrina en tejidos o cavidades serosas, que no es posible eliminar
adecuadamente.
R= CURACIÓN POR REPOSICIÓN DE TEJIDO CONJUNTIVO.

6. Característica de las señales sinápticas


R= SE USA PARA AMPLIFICAR UNA RESPUESTA O INHIBIRLA MEDIANTE
RETROALIMENTACIÓN

7. Todos los siguientes se pueden esperar en la congestión hepática excepto:


R= AUMENTO DE PESO

8. Una lesión endotelial tendrá como consecuencia


R= DESARROLLO DE UN AMBIENTE PROCOAGULANTE

9. Enfoque las variables que alteran la cicatrización.


R= INFECCIÓN, DIABETES, ESTADO NUTRICIONAL, FACTORES MECÁNICOS.
MALA PERFUSIÓN, TIPO, ALCANCE Y LOCALIZACIÓN DE LA LESIÓN.

10. Es un factor de crecimiento con efectos mitógenos sobre los hepatocitos


R= FACTOR DE CRECIMIENTO DE DISPERSIÓN

11. Cuáles son las características del tejido de granulación


R= RICO EN TEJIDO CONJUNTIVO PROVISIONAL Y NUEVOS CAPILARES
FINOS

12. El colágeno que proporciona fibrillas de anclaje de las células epiteliales a la


membrana basal
R= COLAGENO TIPO IV
13. La cicatriz de primera intención es dada por heridas con
R= BORDES ALINEADOS

14. Para el anclaje, la migración de células y mantener la polaridad del tejido se


requiere
R= CONTROL DE LA PROLIFERACIÓN TISULAR

15. Después de una lesión en un tejido permanente como el músculo cardíaco


R= NO SE REPARA

16. Ejemplos de células estables


R= PARÉNQUIMA DE ORGANOS SÓLIDOS, COMO HIGADO, RIÑON Y
PÁNCREAS

16. Son características de la curación de segunda intención


R= LA REACCIÓN INFLAMATORIA ES MÁS INTENSA Y MAYOR CANTIDAD DE
TEJIDO DE GRANULACIÓN

17. El restablecimiento de la estructura tisular normal solo se produce


R= CUANDO EL TEJIDO RESIDUAL ESTÁ ESTRUCTURALEMTE INTACTO
(RESECCIÓN QUIRÚRGICA PARCIAL)

18. La fibrosis es causada por


R= SE MANTIENE LA SÍNTESIS Y SECRECIÓN DE FACTORES DE
CRECIMIENTO Y CITOCINAS FIBROGÉNICAS

19. ¿Cuál es la función de los macrófagos m2 en la inflamación crónica?


R= PRODUCIR Y SECRETAR SUSTANCIAS BIOLOGICAS COMO, FACTORES
DE CRECIMIENTO ACTIVAN FIBROBLASTOS Y FAVORECEN LA ANGIOGENIA
1. La falta de diferenciación, es decir, implica una inversión de la
diferenciación hacia un plano más primitivo.
a) Displasia
b) Pleomorfismo
c) Anaplasia
d) Diferenciación
La falta de diferenciación se denomina anaplasia

2. Mutaciones que contribuyen al fenotipo maligno se denomina.


a) Fenotipo mutador
b) Inestabilidad genómica
c) Ninguna de las anteriores
d) Mutación conductora
Genes que regulan la reparación de ADN; la reparación de ADN defectuoso predispone a mutaciones
genómicas (fenotipo mutador).

3. Corresponde al evento principal de la hemostasia primaria *


a) Formación de fibrina
b) Formación del tapón plaquetario
c) Secreción de endotelina
d) Vasodilatación arteriolar
Los productos secretados reclutan otras plaquetas para formar un tapón hemostático temporal
(hemostasia primaria).

4. ¿Cuáles son las características del tejido de granulación?


a) Rico en colágeno y elastina
b) Rico en tejido conjuntivo provisional y nuevos capilares finos
c) Bajo en fibroblastos
d) Abundantes células inflamatorias
El tejido de granulación se forma por migración y proliferación de fibroblastos y por el depósito de
tejido conjuntivo laxo, combinado con los nuevos vasos y leucocitos intercalados.

5. La estadificación de los cánceres sólidos se basan en qué sistema


a) Sistema de Dukes
b) Ninguna de las anteriores
c) Sistema de bethesda
d) Sistema TNM
El sistema TNM es el sistema de estadificación de cáncer de mayor uso. La mayoría de los hospitales
y centros médicos usan el sistema TNM como método principal en sus informes de cáncer.

 La T se refiere al tamaño y extensión del tumor principal. El tumor principal se llama de


ordinario el tumor primario.
 La N se refiere a la extensión de cáncer que se ha diseminado a los ganglios (o nódulos)
linfáticos cercanos.
 La M se refiere a si el cáncer se ha metastatizado; es decir, si ha tenido metástasis.
6. Cuál de las siguientes causas es más probable que se acumule el líquido sí
existe variación
a) Permeabilidad vascular
b) Presión linfática
c) Presión oncotica
d) Diapedesis

7. Es la vía más frecuente para la diseminación inicial de los carcinomas


a) Siembra directa de cavidades o superficies corporales
b) Hematógena
c) Linfática
d) Continuidad

8. En el choque séptico se puede producir


a) Enteropatia hemorrágica
b) Daño alveolar difuso
c) Todas las anteriores
d) Encefalopatía isquémica

9. Un embolo gaseoso puede originarse:


a) Por trombosis venosa principal
b) Por fracturas
c) Por cambios bruscos en la presión
d) Por células fetales

10. Es un ejemplo de edema localizado en el que ocurre en casos de


a) Síndrome nefrótico
b) Trichuriasis
c) Cirrosis hepática
d) Insuficiencia cardiaca derecha

11. En la cicatrización por primera intención, en que tiempo la neurovascularización


alcanza su máximo y el tejido de granulación llena la incisión
a) 12 horas
b) 5 días
c) 48 horas
d) 24 horas

12. Los tumores más comunes en los hombres se originan en:


a) Mama, Pulmones, Colon y recto
b) Piel, Leucemia y páncreas
c) Piel, orofaringe y páncreas
d) Próstata, pulmones, colon y recto

13. Son tumores constituidos por células inmaduras que se parecen a las que
forman tejido rudimentario fetal
a) Papiloma
b) Blastoma
c) Hamartoma
d) Osteoma
14. Los infartos palidos se observan en:
a) Casos de obstrucción venosa
b) A y B son ciertas
c) Órganos sólidos
d) Casos de oclusión arterial
Los infartos blancos se producen en órganos sólidos (como corazón, bazo y riñón) con circulaciones
arteriales terminales (es decir, pocas colaterales)

15. ¿Cuál de los siguientes mediadores causan vasoconstricción y


broncoconstricción?
a) Tromboxano A2
b) IL-2
c) Leucotrienos B4
d) Ninguna de las anteriores

16. Es un factor de crecimiento con efectos mitogenos sobre los hepatocitos


a) Factor de crecimiento fibroblastico
b) Factor de crecimiento de dispersión (Factor de crecimiento de hepatocitos)
c) Factor de crecimiento epitelial
d) Factor de crecimiento derivado de plaquetas

17. El edema por obstrucción linfática se da por las siguientes condiciones excepto:
a) Neoplasica
b) Postquirurgica
c) Postirradiación
d) Postquimioterapia

18. El virus Epstein Baar produce:


a) Hepatocarcinoma
b) Leucemia de células peludas
c) Linfoma de burkitt
d) Sarcoma de Kaposi

19. En el estado conocido como Shock séptico:


a) Es originado por la pérdida de liquidos
b) Es producido por el paso de bacterias, hongos al torrente circulatorio
c) Carece de respuesta inflamatoria
d) Se considera una respuesta inflamatoria local

20. Su crecimiento se acompaña de infiltración, invasión y destrucción progresiva


del tejido circulante están poco delimitados y son de lenta expansión
a) Neoplasia maligna
b) Neoplasia preinvasiva
c) Cáncer in situ
d) Neoplasia benigna

21. Se media por las interacciones con el factor de Von Willedrand con la
glucoproteína IB, receptor de la superficie de las plaquetas y el colágeno
expuesto
a) Activación plaquetaria
b) Adhesión plaquetaria
c) Agregación plaquetaria
d) Fase de iniciación de la coagulación

22. ¿Qué gen supresor de tumores intervienen en la patogenia del carcinoma de


mama?
a) Ninguna de las anteriores
b) APC1 APC2
c) BCRA 1 BCRA 2
d) NF1 NF2

23. Especifique las variables que alteran la cicatrización


a) Infección, diabetes, estado nutricional, factores mecánicos, mala
perfusión, tipo, alcance y localización de la lesión
b) Estado nutricional adecuado y esteroides
c) Baja presión local o la torsión de las heridas
d) Infección y diabetes controlada
24. Se caracterizan por degradar la MEC y permite la remodelación y la extensión
del tubo vascular.
a) Tirosina cinasa.
b) MPM metaloproteinasas de matriz
c) Factor transformante beta
d) Factor derivado de plaquetas.
25. La luz UV:
a) Es un carcinógeno químico
b) Hace dímeros en la guanina
c) No es un agente carcinógeno
d) Puede causar carcinoma basocelular

26. Son procesos en la evolución de un trombo excepto:


a) Embolia por recanalización
b) Propagación
c) Trombolisis
d) Recanalización

27. Es una causa de edema por disminución de la presión oncotica


a) Diabetes Mellitus
b) Desnutrición
c) Insuficiencia cardiaca
d) Cáncer

28. Es un término que se aplica a la neoplasia epitelial benigna derivada de las


glándulas, se denomina como:
a) Papiloma
b) Pólipo
c) Angioma
d) Adenoma

29. La mayor parte de las flebotrombosis se producen en:


a) Arterias de mediano calibre
b) Corazón
c) Venas superficiales y profundas de las piernas
d) Arterias cerebrales

30. Una de las razones por las cuales las células neoplásicas son inmortales
a) No se autorrenuevan
b) Respetan las pausas del ciclo celular
c) P53 funciona adecuadamente
d) Expresión telomerasa

31. Una lesión endotelial tendrá como consecuencia:


a) No tiene relación, con la coagulación
b) Desarrollo de un ambiente procoagulante
c) Produce la disolución del coagulo
d) Desarrollo de un ambiente anticoagulante

32. Son acontecimientos plaquetarios excepto


a) Adhesión
b) Migración
c) Cambio de forma y secreción
d) Agregación

33. El efecto warburg consiste en:


a) Incremento del ciclo de Krebs
b) Disminuir la síntesis de biomoléculas
c) Promover la atrofia del tumor
d) La neoplasia maligna fija la glucolisis anaerobia en su metabolismo

34. El shock cardiogénico se produce cuando existe una pérdida de la masa


muscular del ventrículo izquierdo del:
a) 25%
b) 60%
c) 45%
d) 35%
El fracaso grave de bomba (shock cardiógeno) aparece en el 10-15% de los pacientes, habitualmente
con una pérdida ~ 40% de masa ventricular izquierda. El shock cardiogénico tiene una mortalidad
del 70%

35. ¿Cuál es la definición de teratoma?


a) Que se origine de las células de sertoli
b) Que se origine de más de 2 capas germinales
c) Origen en linfocitos B
d) Que se origine de un trofoblasto

36. Los estados de hipercoagulabilidad se dividen en:


a) Fisiológicos y patológicos
b) Genéticos y adquiridos
c) De alto y bajo riesgo
d) Primarios y terciarios

37. Neoplasia maligna de los vasos sanguíneos


a) Linfoma
b) Linfagiosarcoma
c) Leucemia
d) Angiosarcoma

38. Son estados de hipercoagulabilidad de bajo riesgo excepto:


a) Tabaquismo
b) Tabaquismo
c) Infarto agudo al miocardio

39. La inactivación o deleción de P16, adquirida por vía somática se ven en las
familias propensas a:
a) Carcinomas de páncreas, glioblastomas y cánes de esófago
b) Linfomas
c) Adenocarcinoma pancreático y colangiocarcinomas
d) Leucemia mieloide crónica y leucemias linfoblásticas agudas
40. En la cicatriz de primera intención, ¿Cuándo se comienza a observar el tejido
de granulación?
a) Inmediatamente
b) Semanas después
c) A las 24 horas
d) 3 a 7 días después

41. Cual enunciado acerca de las plaquetas es verdad


a) Contiene actina y miosina, microfilamentos y microtubulos
b) Contiene cuerpos densos ricos en fibrinógeno
c) Los precursores son monocitos
d) Contiene gránulos alfa ricos en difosfato de adenosina
La secreción de gránulos plaquetarios (reacción de liberación) ocurre poco después de la adhesión.
Los gránulos a expresan moléculas de selectina de adhesión y contienen factores de coagulación y
de crecimiento; los cuerpos densos o gránulos δ contienen nucleótidos de adenosina (p. ej., [difosfato
de adenosina [ADP] ), calcio y aminas vasoactivas (p. ej., histamina). El ADP es un potente mediador
de la agregación plaquetaria m (reclutamiento) y el calcio es importante para la cascada de la
coagulación.

42. La estadificación de un tumor maligno se refiere a


a) Grado de diferenciación de las células tumorales
b) Tratamiento quirúrgico
c) Diagnóstico molecular
d) Tamaño de la lesión primaria y si ha realizado metástasis

43. La congestión pasiva crónica del pulmón puede incluir todo lo siguiente, excepto
a) Edema pulmonar
b) Eritema
c) Macrófagos con hemosiderina
d) Induración café

44. La extravasación del líquido al intersticio, cual es el término que se utilizaría


a) Edema
b) Trasudado
c) Exudado
d) Infiltración

45. No siempre responden a las moléculas que inhiben la proliferación de las células
normales, habitualmente por la inactivación de genes supresores de tumores
que modifican componentes de estas vías inhibitorias del crecimiento
a) Insensibilidad a las señales inhibitorias del crecimiento
b) Potencial ilimitado de la replicación
c) Alteración del metabolismo celular
d) Evasión de la apoptosis

46. Una de las maneras en las que un tumor escapa de la respuesta inmune es:
a) Expresando proteínas inhibitorias de linfocitos T
b) Expresando citocinas activadoras de linfocitos T
c) Sobreexpresando moléculas CPH 1
d) Sobreexpresando antígenos

47. ¿A que deben los tumores su potencial replicativo ilimitado?


a) Activación de la telomerasa
b) Responden adecuadamente a las señales apoptosicas
c) Presencia de inhibidores de la telomerasa
d) Carecen capacidad de autorrenovacion

48. La edad influye de manera determinante en el riesgo de cáncer por lo tanto, la


mayoría de los carcinomas se presentan en la edad de:
a) Mayor de 20 años
b) Mayor de 55 años
c) 20 a 30 años
d) 10 a 20 años
49. Un estado inflamatorio crónico puede causar anemia ferropénica en respuesta
al:
a) Aumento de albumina
b) Aumento de fibrinógeno
c) Aumento de la hepcidina
d) Aumento de la fibrina

50. En un estado inflamatorio crónico asociado al cáncer


a) Miositis
b) Enfermedad intestinal inflamatoria
c) Condritis
d) Pielonefritis

51. La llamada ¨induración parda¨ es un ejemplo, excepto:


a) Congestión pasiva crónica pulmonar
b) Congestión pasiva crónica esplénica
c) Congestión pasiva del hígado
d) Hemorragia de tubo digestivo

52. El carcinoma epidermoide de pulmón, carcinoma de mama, carcinoma de


células renales y la leucemia/linfoma de linfocitos T en el adulto se han
relacionado con el siguiente síndrome paraneoplasico
a) Síndrome de Cushing
b) Hipercalcemia
c) Hipoglucemia
d) síndrome de secreción inadecuada de hormona antidiurética

53. Afectan al proceso de cicatrización excepto:


a) Desnutrición
b) Tratamiento con glucocorticoides
c) Exceso de ácido ascórbico
d) Diabetes mellitus

54. Es un marcador tumoral de tipo hormonal


a) CA125
b) APE
c) Calcitonina
d) Antígeno carcinoembrionario

55. El restablecimiento de la estructura tisular normal solo se produce:


a) Cuando el tejido residual esta estructuralmente intacto (Resección
quirúrgica parcial)
b) En nefrectomía unilateral
c) Cuando todo el tejido está dañado por inflamación
d) Cuando todo el tejido está dañado por infección

56. Es considerado como el guardián del genoma


a) TP53
b) K-RAS
c) MYC
d) JUN

57. La asbestosis y la silicosis está asociada con qué tipo de neoplasia


a) Carcinoma colonorrectal
b) Carcinoma pancreático
c) Hepatocarcinoma
d) Mesotelioma, carcinoma de pulmón

58. Sustancia generada por sistema de cininas cuando hay daño vascular promueve
la contracción de musculo liso, aparación de dolor y promueve la permeabilidad
vascular
a) Bradicinina
b) C5a del complemento
c) Fosfolipasa C
d) Factor de necrosis tumoral
59. La característica histológica predominante de un infarto es:
a) Necrosis grasa
b) Necrosis fibrinoide
c) Necrosis caseosa
d) Necrosis coagulativa

60. Fase del shock donde fracasan mecanismos reflejos de compensación


disminuye la perfusión de órganos vitales
a) Fase latente
b) Fase progresiva
c) Fase no progresiva
d) Fase irreversible

61. Es un carcinógeno quimico de origen natural:


a) Ciclofosfamida
b) Cloruro de vinilo
c) Bencidina
d) Griseofulvina

62. Neoplasia maligna del epitelio placentario


a) Carcinoma embrionario
b) Coriocarcinoma
c) Seminoma
d) Mesotelioma

63. Neoplasia maligna del tejido adiposo


a) Liposarcoma
b) Astrocitoma
c) Neurofibroma
d) Lipoma

64. Contienen una mayor cantidad de sangre oxigenada se caracteriza por el color
rojo brillante y dilatación de las arterias
a) Hiperemia activa
b) Hiperemia pasiva
c) Congestión crónica
d) Congestión pasiva

65. Todo lo siguiente se pueden esperar en la congestión hepática excepto


a) Regiones centrolobulillares distendidas por sangre
b) Infartos en forma de pirámide
c) Patron de nuez moscada
d) Incremento de peso
66. La deficiencia de la glucoproteína GPIB provoca la enfermedad de:
a) Von willebrand
b) Tromboastenia de Glanzmann
c) Nieman Pick
d) Bernard soulier
67. Los principales factores de crecimiento implicados en la síntesis de tejido
conjuntivo, excepto:*
a) PDGF
b) FGF
c) IL-1
d) TGF-BETA

68. Son proteínas compuestas por tres cadenas polipeptidicas distintas trenzadas
en triple hélice:
69. Son neoplasias que tienen peor diferenciación o totalmente indiferenciadas, se
refiere al termino de:
a) Nevo
b) Coristoma
c) Hamartoma
d) Anaplasia
70. Neoplasia maligna de las células de los ductos seminales
a) Coriocarcinoma
b) Carcinoma embrionario
c) Seminoma
d) Tumor de senos endodérmicos

71. Es la fase del shock en la que se activa el eje renina- angiotensina


a) Fase refractaria
b) Fase no progresiva
c) Fase irreversible
d) Fase progresiva

72. Es el colágeno que proporciona fibrillas de anclaje de las células epiteliales a la


membrana
a) Colágeno tipo I
b) Colágeno tipo III
c) Colágeno Tipo IV
d) Colágeno Tipo VII
73. Estos trombos se forman en la circulación venosa, suelen contener mas
eritrocitos incorporados y relativamente pocas plaquetas
a) Trombos arteriales
b) Trombos rojos
c) Trombos murales
d) Vegetaciones
74. ¿Cuáles son las fases de la reparación en las heridas?
a) Infiltración, remodelación, fibrosis y costra
b) Sepsis, formación de tejido de granulación y angiogénesis
c) Angiogenia, tejido de granulación y remodelación
d) Angiogenia, inflamación, contracción, fibrosis e hipoplasia
75. El resultado de múltiples mutaciones que se acumulan de forma independiente
en diferentes células y contribuyen asi a la generación de subclones con
características diferentes es a lo que se le conoce como:
a) Iniciación y promoción
b) Progresión tumoral
c) Acción directa e indirecta
d) Heterogeneidad tumoral

76. Patología donde las personas que sufren mutaciones en el gen TP53 y que
tienen una posibilidad 25 veces mayor de experimentar una neoplasia maligna
antes de los años que la población general
a) Síndrome de desgaste
b) Carcinoma
c) Neoplasia endocrina multiple
d) Sindrome de Li-Fraumeni

77. Si la alteración displasica es intensa y afecta a todo el espesor del epitelio, pero
la lesión no penetra la membrana basal, se habla de:
a) Carcinoma in situ
b) Tumor infiltrante
c) Tumor invasivo
d) Metaplasia

78. Los tipos de VPH de alto riesgo producen proteínas oncogenas que ocasionan
lo siguiente, excepto:
a) Activan ciclinas
b) Inhiben la apoptosis
c) Combaten la senecencia celular
d) Inmunodeficiencia

79. El gen APC inhibidor de las vías de señalización mitogena, se ha relacionado


con las siguientes neoplasias
a) Carcinoma basocelular y meduloblastoma
b) Carcinomas de estómago, colon, páncreas y melanoma
c) Retinoblastomas
d) Schwannoma y meningioma
80. Es una característica del síndrome de Trousseau asociado a carcinomas de
páncreas y estomago se refiere a:
a) Embolos
b) Deficit de glucoproteína 1 B
c) Tromboflebitis migratoria
d) Infarto

81. Que tan resistentes son las células musculares cardiacas a la hipoxia
a) Muy resistentes, mueren después de muchas horas
b) Extremadamente sensibles, mueren a los 5 minutos
c) Bastante sensibles, muerena los 20-30 minutos
d) Poco sensibles, mueren a las pocas horas

82. Son patologías que producen edema por aumento de la presión hidrostática del
vaso:
a) Neoplasias y filarias
b) Aumento de la secreción renina angiotensina- aldosterona
c) Glomerulopatias y malnutrición
d) Pericarditis constrictiva y trombosis

83. Se le conoce así cuando el tejido cicatrizal, crece más allá de los límites de la
herida original, además no se contrae
a) Fibromatosis agresivas
b) Cicatriz queloide
c) Granulación exuberante
d) Dehiscencia

84. Factores sistémicos que influyen en la curación de heridas


a) Estrés oxidativo, perdida del tono vascular, anemia
b) Nutrición, situación circulatoria estado metabólico y hormonas
c) Infecciones, nutrición, función renal y cardiaca deficiente
d) Tipo de herida, profundidad de la herida y estado metabólico

85. Marcador tumoral relacionado con el cáncer prostático


a) ALFA FETOPROTEÍNA
b) CA-125
c) CA-19-9
d) PSA

86. Morfológicamente las neoplasias malignas se caracterizan por:


a) Relación núcleo citoplasma normal
b) Arquitectura tisular intacta
c) Pleomorfismo nuclear y celular
d) Figuras mitóticas normales
87. Dentro de las funciones de P53 se encuentran, excepto:
a) Pausar el ciclo celular para reparar el ADN
b) Inducir apoptosis
c) Inducir proliferación celular
d) Inducir senescencia
88. El cáncer es una enfermedad caracterizada por:
a) Daño genético letal
b) Senescencia celular
c) Lesión subletal al ADN
d) Adquisición de mutaciones

89. Se puede producir choque por


a) Todas las anteriores
b) Sepsis
c) Infarto del miocardio
d) Quemaduras

90. Se observa tras una destrucción sustancial de tejido, cuando la lesión


inflamatoria afecta a tejidos que no son capaces de regenerarse o cuando hay
abundante exudación de fibrina en tejidos o cavidades serosas que no es posible
eliminar adecuadamente
a) Inflamación crónica
b) Remisión
c) Curación por reposición de tejido conjuntivo
d) Resolución completa

91. Marcador tumoral relacionado con carcinoma de colon, pancreas y pulmón


a) Alfa fetoproteina
b) Antigeno prostático
c) Antigeno carcinoembrionario
d) Enolasa neuronal especifica
92. El gen 9,12 se asocia con
a) Von hippel- lindau  gen VHL (3p25.3)
b) Trisomia 21
c) Cromosoma filadelfia
d) Leucemia mieloide
93. Característica de las señales sinápticas
a) Secreción de neurotransmisores en uniones celulares especiales
b) De difusión mínima y la señal se degrada rápido
c) Se usa para amplificar una respuesta o inhibirla mediante
retroalimentación
d) Se libera al torrente circulatorio y actúa distancia sobre células blanco
94. Neoplasia maligna de la glándula tiroides
a) Coriocarcinoma L
b) Adenocarcinoma
c) Condrosarcoma
d) Linfangiosarcoma
95. Se considera como extensión de las neoplasias a sitios anatómicamente
separados del lugar de origen
a) Metastasis
b) Micro invasión
c) Son masas expansivas cohesivas
d) Invasión
96. Son sustancias que inhiben la función plaquetaria
a) Antitrombina
b) ON, prostaciclina y ADPasa
c) Plasminogeno
d) Tromboplastina
97. El gen RB, inhibidor de la progresión celular, se ha relacionado con la siguientes
neoplasias
a) Retinoblastoma, osteosarcoma, carcinoma de mama, colon y pulmón
b) Carcinoma basocelular y meduloblastoma
c) Carcinoma basocelular
d) Carcinoma de células renales y paraganglioma
98. Trombo formado en las cavidades cardiacas
a) Trombo mural
b) Aneurisma
c) Petequia
d) Embolo
99. Tienen mayores probabilidades de sufrir infarto rojo, Excepto:
a) Intestino delgado
b) Pulmon
c) Riñon

100. En la inmortalidad de las células malignas es considerado como factor


decisivo
a) Evasión de la senescencia
b) Mutuacion puntual
c) Reordenapmiento de los cromosomas
d) Capacidad de regulación
101. Son algunas causas del retorno venoso
a) Sindrome nefrotico y cirrosis hepática
b) Obstrucción linfática y retención de sodio
c) Insuficiencia cardiaca congestiva y compresión venosa
d) Calor y frio
102. Es el cambio biológico necesario para la progresión de las neoplasias
malignas
a) Evasión de la apoptosis
b) Metastasis
c) Potencial replicativo
d) Angiogenia
103. Etapa caracterizada por la hipoperfusión tisular y el inicio del empeoramiento
circulatorio y de los desequilibrios metabólicos incluyendo la acidosis lactica
a) Fase no progresiva
b) Etapa progresiva
c) Etapa reversible
d) Etapa irreversible
104. Un cistoadenoma corresponde con:
a) Neoplasia benigna quística glandular
b) Hiperplasia
c) Neoplasia maligna quística glandular
d) Neoplasia maligna
105. Pertenece a una familia de factores de crecimiento con mas de 20 miembros
a) FCDP
b) FCT BETA
c) FCF
d) FCE
106. Esta via de diseminación es bastante característica de los carcinomas de
ovario ya que frecuentemente se diseminan por la superficie peritoneal
a) Iatrogenica
b) Hematogena
c) Siembra de cavidades y superficies corporales
d) Linfática
107. Se induce por la hipoxia provoca la angiogenia y promueve la migración de
las células endoteliales
a) PDGF
b) TGF BETA
c) FGF
d) VEGF
108. Sufren una conmutación metabólica hacia la glucolisis aerobica efecto
warburg que facilita la síntesis de macromoléculas y orgánulos requeridos para
un crecimiento celular rápido
a) Evasión de la apoptosis
b) Potencial ilimitado de la replicación
c) Alteracion del metabolismo celular
d) Insensibilidad a las señales inhibitorias del crecimiento
109. Cuál es la proteína más abundante del ser humano
a) Vimentina
b) Miosina
c) Caderina
d) Colágeno

110. International normalized ratio valora principalmente en el laboratorio


a) Anticoagulación con heparina
b) Ácido acetilsalicílico
c) Factor VIII y X
d) Anticoagulación oral
111. Responden por lo común al tratamiento con anticuerpos o fármacos que
bloquean la actividad de HER2:
a) Cáncer de mama
b) Adenocarcinoma de pulmón
c) Melanoma
d) Leucemia mielocitica aguda
112. La activación del activador del plasminogeno genera su conversión a
plasmina para
a) Activar plaquetas
b) Iniciar la cascada fibrinolitica
c) Secreción y cambio de forma plaquetaria
d) Activación leucocitaria
113. Estos trombos son mas frecuentes en arterias coronarias cerebrales y
femorales conformados por plaquetas, fibrina, eritrocitos y leucocitos
degenerados
a) Vegetaciones
b) Trombos murales
c) Trombos arteriales
d) Trombos rojos
114. Permiten determinar el diagnostico, pronostico, la detección de enfermedad
residual mínima y el diagnóstico de la predisposición hereditaria al cáncer
a) Análisis microbiológico
b) Historia clínica
c) Análisis moleculares
d) Análisis morfológicos
115. Vía de la apoptosis que más se desregula en situación de cáncer
a) Necroptosis
b) Vía extrínseca
c) Vía intrínseca
d) Piroptosis
116. Clínicamente el proceso de fibrosis es común en:
a) Mucosas
b) Epitelios
c) Tejidos lábiles
d) Órganos parenquimatosos
117. Que proteína fibrilar suele regenerar mal en el tejido cicatricial
a) Integrina
b) Colágeno
c) Elastina
d) Fibronectina
118. Factor de crecimiento quimiotactico para neutrófilos, macrófagos, células del
musculo liso, estimula la proliferación de fibroblastos y células endoteliales
a) Factor de crecimiento hepatocitario
b) Factor del crecimiento del endotelio vascular
c) Factor de crecimiento derivado de plaquetas
d) Factor de crecimiento epidérmico
119. Es el componente proteico de la MEC encargado de dar resistencia a la
tensión
a) Proteoglucanos
b) Laminina
c) Fibronectina
d) Colágeno
120. Es el termino aplicado a un resto heterotopico de células por ejemplo, un
nodulo de tejido pancreático perfectamente desarrollado y con una organización
normal en la submucosa del estomago
a) Teratoma
b) Desmoplasia
c) Hamartoma
d) Coristoma
121. El factor de crecimiento placentario pertenece a la familia de
a) PCE
b) FCH
c) FCVE
d) FCFD

122. Este tipo de células están presentes en la inflamación crónica, son


abundantes en las reacciones inmunitarias mediadas por inmunoglobulina E y
en las infecciones parasitarias.
a) Neutrófilos
b) Linfocitos
c) Eosinófilos
d) Macrófagos.
123. La congestión pasiva crónica del pulmón puede incluir todo lo siguiente
excepto.
a) Edema pulmonar
b) Eritema
c) Macrófagos con hemosiderina
d) Induración café
124. Patología en la que se pierde albumina y genera edema:
R= Sindrome Nefrotico
125. Causantes de edema:
R= Enfermedad cardiaca y renal
126. Hematoma mayor a 2cm:
R= Equimosis
127. Se ve implicadoen reacciones inflamatorias ante cristales de urato
(causantes de gota), lípidos (en el síndrome metabolico)
128. Son componentes de la fagocitosis excepto
a) Englobamiento quimiotraccion
b) Reconocimiento y fijacion
c) Opsonizacion
129. Son neoplasias malignas de células germinales del testículo.
Respuesta =SEMINOMAS
130. Una vez que las células tumorales rompen la membrana basal, se habla de
R= TUMOR INFILTRANTE
131. Son neoplasias malignas, su origen es derivado del tejido conectivo o
conjuntivo
R= SARCOMA
132. Las células exhiben una variación en su tamaño y forma, es decir, las
células de un mismo tumor no son uniformes
R= PLEOMORFISMO
133. Es un protooncogen que pertenece a los inhibidores de las cinasas de las
ciclinas (INK) **
R= p16
134. Se refiere así, a la pérdida progresiva de la grasa corporal y de la masa
corporal magra, acompañada de una sensibilidad profunda, anorexia y
anemia.}
R= CAQUEXIA CANCEROSA
135. Neoplasia maligna de células hematopoyéticas
R= LEUCEMIA
136. De las siguientes opciones, ¿Cuál no es una característica de anaplasia?
R= FALTA DE MITOSIS
137. El marcador tumoral CA 19-9 se relaciona principalmente con
R= CÁNCER DE COLON Y PÁNCREAS
138. Proteína relacionada con el síndrome familiar del retinoblastoma y
osteosarcoma
R= RB
139. Tienen una función esencial en muchos aspectos del fenotipo maligno,
como la expresión de los genes del cáncer, el control de la diferenciación y la
autorrenovación, incluso la sensibilidad de la resistencia a los medicamentos
R= CAMBIOS EPIGENÉTICOS
140. Mutaciones que contribuyen al fenotipo maligno se denomina *
R= MUTACIÓN CONDUCTORA
141. Neoplasia benigna de nervio periférico
R= NEUROFIBROMA
142. Células que están presentes en el tejido conectivo contiguos a vasos
sanguíneos por debajo de superficies mucosas de vías respiratorias, la unión de
antígeno alérgeno a anticuerpo IgE, se refiere a:
R= MASTOCITOS
143. El edema que ocurre en la filariasis se debe a:
R= BLOQUEO DE LA CIRCULACIÓN LINFÁTICA.
144. Defecto genético que predispone a sufrir trombosis conocidas como
trombofilias y un 5% a 10% de las personas lo puede presentar
R= DEFICIENCIA DE ANTITROMBINA III, PROTEÍNA C-S Y FACTOR V LEIDEN
145. Corresponde al evento principal de la hemostasia secundaria
R= DEPOSICIÓN DE COAGULO DE FIBRINA
146. . ¿Cómo se clasifican los infartos?
R= ROJOS, BLANCOS; SÉPTICOS
147. La triada de Virchow incluye las siguientes características excepto
R= INCREMENTO DE LA VOLEMIA
148. Es una causa de la disminución de la presión osmótica
R= SÍNDROME NEFRÓTICO

149. ¿Proceso activo en el que la dilatación arteriolar aumenta en el flujo de


sangre?
R= HIPEREMIA
150. El edema de origen cardiaco se caracteriza por
R= APARECER POR LAS MAÑANAS
151. La aparición de hematomas puede deberse principalmente
R= ENFERMEDAD DE VON WILLEBRAND
152. La trombosis venosa (Fenómeno de Trousseau) se ha relacionado con
síndrome paraneoplásico
153. R= CARCINOMAS DE PÁNCREAS, BRONCÓGENO Y OTROS CÁNCERES
154. Principal causa que puede generar tromboembolia pulmonar
R= TROMBOSIS RPOFUNDA
155. Son células responsables de la síntesis de nueva matriz extracelular
R= CÉLULAS ENDOTELIALES

156. Es secundario a una insuficiencia de bomba miocárdica debido a las lesiones


intrínsecas del miocardio, comprensión extrínseca u obstrucción de flujo de
salida
R= CHOQUE CARDIOGENICO

157. En su mayoría, los casos de shock endotoxico son causados por


R= SEPSIS POR HONGOS

158. 4Es más frecuente la formación de émbolos a partir de


R= TROMBOS
159. El colágeno que proporciona fibrillas de anclaje de las células epiteliales a la
membrana basal
R= COLAGENO TIPO IV
160. La cicatriz de primera intención es dada por heridas con
R= BORDES ALINEADOS
161. Para el anclaje, la migración de células y mantener la polaridad del tejido se
requiere
R= CONTROL DE LA PROLIFERACIÓN TISULAR

162. Después de una lesión en un tejido permanente como el músculo cardíaco


R= NO SE REPARA

163. Ejemplos de células estables


R= PARÉNQUIMA DE ORGANOS SÓLIDOS, COMO HIGADO, RIÑON Y
PÁNCREAS
164. Son características de la curación de segunda intención
R= LA REACCIÓN INFLAMATORIA ES MÁS INTENSA Y MAYOR CANTIDAD
DE TEJIDO DE GRANULACIÓN
165. La fibrosis es causada por
R= SE MANTIENE LA SÍNTESIS Y SECRECIÓN DE FACTORES DE
CRECIMIENTO Y CITOCINAS FIBROGÉNICAS

166. ¿Cuál es la función de los macrófagos m2 en la inflamación crónica?


R= PRODUCIR Y SECRETAR SUSTANCIAS BIOLOGICAS COMO,
FACTORES DE CRECIMIENTO ACTIVAN FIBROBLASTOS Y FAVORECEN
LA ANGIOGENIA
167. Causa fisiológica de hiperemia:
R= Vasodilatación por ejercicio
168. Factor que inhibe la diseminación del trombo:
R= Activador tisular de plasminogeno(PAI)
169. Capacidad de cambiar de un tipo celular a otro:
R=Transdiferenciación
170. Es la capacidad de una célula de transdiferenciarse a distintas estirpes
celulares:
R= Plasticidad de desarrollo
171. En esta fase de shock hay liberación de ADH y activación de
baroreceptores:
R= Fase inicial progresiva
172. Manifestaciones clínicas presentes en shock séptico:
R= Piel caliente con vasodilatación periférica, hiperemia.
173. Casi siempre oclusiva puente largo de la luz del vaso:
R=Trombosis venosa profunda
174. Tipo de necrosis encéfalo:
R= Licuefactiva
175. Sintomas de la embolia grasa:
R= Taquicardia, bradicardia y disnea
176. Mantiene la integridad del epitelio externo de la cornea:
R=Células madre del limbo
177. Una neoplasia compuesta se divide como:
R= Dos capas celulares y una capa embrionaria
178. Metastasis:
R= Implantes tumorales sin continuidad con el tumor primario
179. Fases de la cascada metastasica:
R=Invasión de la MEC, diseminación vascular, alojamiento y colonización
180. Inhibicion de TP53 mediado por:
R=BAX
181. wich one of the following characteristics not acompany the cancer:
R= Fibrous capsule
182. What are the organs most affected by the hematogenous spread?
R= Liver and lung
183. Wich ARN virus has on oncogene activity?
R=Leucemia and cel T
184. Carcinógeno natural hepático secretado por plantas y hongos:
R= Aflatoxina B1
185. Causas frecuentes de esteatosis:
R=Alcoholismo y desnutrición
186. Mediador químico que produce dilatación y aumento de la permeabilidad y
se libera de tipo físico:
R=Histamina
187. Son células que participan en la reacción inflamatoria excepto:
R= Pericitos
188. Las CX3C son moléculas de adhesión de:
R=Monocitos y Cel T
189. De que vitamina son dependientes las proteínas C y S:
R=Vitamina K
190. Localización más frecuente del carcinoma de células renales:
R=Polos
191. El cáncer renal causa frecuentemente metástasis en:
R=Pulmones y hueso
192. Causa mas frecuente de cáncer renal:
R=Tabaquismo
193. Cual es la proteína viral que induce una rápida degradación de P53:
R=E6
194. Causa frecuente de metástasis en CACU:
R=Pielonefritis y Uremia
195. Lesion primaria de mayor riesgo de producir cáncer de mama:
R= Mastopatia proliferativa con atipia
196. Quimico que dara activación quimiotactica, actividad sobre monocitos y
eosinofilos:
R=CXC
197. sustancias que ocacionan fiebre, sueño y perdida del apetito:
R=Citocinas
198. La inflamación fibrinosa se caracteriza por:
R=Fluidos corporales en pericardio y pleura
199. Caracteristicas histológicas de órganos que sufren atrofia, hay presencia
de:
R= Formación de vacuolas autofagicas
200. Los leiomiomas se dividen en:
R=Submucosos, subserosos e intramurales
201. Las enfermedades crónicas granulomatosas abarcan varios trastornos que
tienen en común:
R=La incapacidad de destruir agentes etiológicos
202. El infarto de miocardio corresponde a una:
R=cardiopatía isquémica
203. Todas las siguientes sustancias pertenecen al tren de tincion, excepto:
R=Violeta de Genciana
204. La citología exfoliativa puede reportar todo, excepto:
R=El estado nutricional de la células.
205. Carcinoma in situ ductual consiste en:
R=Población maligna de células que no pueden atravesar la Membrana
Basal
206. La fosfofructocinasa y la fosforilaza en lesión, activan a:
R=Glucolisis Anaerobia
207. ¿Que moléculas emiten la adhesión de plaquetas al endotelio no
lesionado?:
R=Prostacicilinas y Oxido Nitrico
208. Para la formación de granulomas se requiere:
R= reacción inmunitaria mediada por células T
209. Pancreatitis aguda ------------------------ Necrosis Grasa
210. Infeccion Bacteriana----------------------N. Licuefactiva
211. Muerte celular----------------------------- Apoptosis
212. Tuberculosis------------------------------- N. caseosa
213. Infarto al miocardio--------------------- N. Coagulativa

214. SON NEOPLASIAS EPITELIALES GLANDULARES MALIGNAS


QUÍSTICAS:
a) CARCINOMAS
b) SARCOMAS
c) ESCLEROMAS
d) CISTOADENOCARCINOMAS
215. NEOPLASIA MALIGNA DE LOS VASOS LINFÁTICOS:
a) HEMANGIOMA
ANGIOSARCOMA
b) LINFOMA
c) LINFANGIOSARCOMA
216. ES EL CAMBIO FUNDAMENTAL PARA OBTENER POTECIAL
REPLICATIVO ILIMITADO A
a) P53,P21,P16
b) ACTIVACION DE LA TELOMERASA
c) HIPERFOSFORILACION DE rb
d) EXPRESION DE ONCOGENES
217. LOS CARCINÓGENOS INDIRECTOS NECESITAN:
a) ACTIVACIÓN METABÓLICA
b) COCARCINÓGENOS
c) AGENTES ALQUILANTES
d) AGENTES ACILANTES
218. LOS SÍNDROMES PARANEOPLÁSICOS PUEDEN PRODUCIR
EXCEPTO:
a) TRASTORNOS DERMATOLOGICOS
b) ENDOCRINOPATIAS
c) SEPSIS
d) CAMBIOS VASCULARES Y HEMATOLOGICOS
219. LOS TEJIDOS QUE TIENE UN NIVEL DE REPLICACION BAJO PERO
QUE PUEDEN  SUFRIR DIVISIONES RAPIDAS EN RESPUESTA A
ESTIMULOS ESPECIFICOS SON:
a) LABILES
b) CELULAS MADRE
c) PERMANENTES
d) QUIESCENTES
220. LA CAQUEXIA DEL CANCER SE DEBE A:
a) LA NEOPLASIA COMPITE POR LOS NUTRIENTES CON EL
HUESPED
b) ANOREXIA
c) SECRECION DE FACTOR DE NECROSIS TUMORAL
d) DETERIORO GENERAL

221. SON LESIONES SUBLETALES DEL DNA Y DIANAS GENETICAS PARA


EL DESARROLLO DE NEOPLASIAS MALIGNAS EXCEPTO
a) ONCOGENES
b) GENES DE LA APOPTOSIS
c) GENES SUPRESORES DEL CANCER (P53, P21, RB)
d) GENES REPARADORES DEL DNA
e) GENES REPRESORES
222. LA INSENSIBILIDAD A LAS SENALES INHIBITORIAS DEL CRECIMIENTO
ESTA DADO POR LESION A :
a) BRCA-1
b) RB,P53
c) BCL2
d) BAX
223. LOS MARCADORES TUMORALES PUEDEN SER EXCEPTO:
a) ISOENZIMAS
b) ANTIGENOS ONCOFETALES
c) HORMONAS
d) CITOCINAS
224. LAS CELULAS MADRE SE CARACTERIZAN POR:
a) SINTETIZAR HORMONAS
b) CAPACIDAD DE AUTORENOVACION Y REPLICACIÓN ASIMETRICA
c) CAPACIDAD DE SÍNTESIS PROTEICA
d) RESPONDER A ESTIMULOS EXTERNOS

225. LA LESION SUBLETAL DEL DNA POR RADIACIONES ULTRAVIOLETA


ES POR:
a) DAÑO AL RB
b) DAÑO A RECEPTORES DE FACTORES DE CRECIMIENTO
c) DAÑO AL RNA MENSAJERO
d) FORMACION DE DIMEROS DE PIRIMIDINA
226. EL VEGF Y LAS ANGIOPOYETINAS INTERVIENEN EN PROCESOS
COMO:
a) REMODELACIÓN DE LA MATRIZ EXTRACELULAR
b) DEGRADACIÓN DEL COLAGENO DE LA MATRIZ EXTRACELULAR
c) SÍNTESIS DE COLAGENO POR LOS FIBROBLASTO
d) VASCULOGENESIS Y ANGIOGENESIS
227. LAS PROTEINAS E6 Y E7 DEL VPH DE ALTO RIESGO SE UNEN AL GEN:
a) P27
b) RAS
c) RB Y P53
d) MYC
e) FOS

228. ENZIMAS ENCARGADAS DE LA DEGRADACIÓN DEL COLAGENO Y


OTRAS PROTEÍNAS DE LA MATRIZ EXTRACELULAR:
a) ELASTASA DE LOS NEUTROFILOS
b) CININAS
c) METALOPROTEINASAS
d) CATEPSINA G
229. ABCESOS, ÚLCERAS Y QUEMADURAS SON EJEMPLOS DE
CICATRIZACIÓN POR:
a) TEJIDO CONECTIVO
b) SEGUNDA INTENCIÓN
c) FIBROSIS
d) PRIMERA INTENCIÓN
230. SON CAMBIOS CROMOSOMICOS EN LAS NEOPLASIAS EXCEPTO:
a) AMPLIFICACION
b) TRASLOCACION
c) TRANSCRIPCION
d) DELECION

231. ES UNA CARACTERÍSTICA EXCLUSIVA DE LAS NEOPLASIAS MALIGNAS:


a) NECROSIS
b) ANGIOGENESIS
c) OCASIONAR METASTASIS
d) MITOSIS

232. EL PARECIDO QUE EXISTE ENTRE LA CÉLULA NEOPLÁSICA Y LA CONTRAPARTE NORMAL SE


DENOMINA GRADO DE:
a) PROGRESIÓN
b) DIFERENCIACIÓN
c) INDIFERENCIACIÓN
d) DESDIFERENCIACIÓN
233. SON CELULAS DIFERENCIADAS REPROGRAMADAS:
a) CELULAS MADRE INDUCIDAS
b) CELULAS MADRE TISULARES
c) CELULAS MADRE HEMATOPOYETICAS
d) CELULAS MADRE EMBRIONARIAS
234. ENTRE LAS ONCOPROTEÍNAS (PRODUCTOS PROTÉICOS DE LOS ONCOGENES) TENEMOS:
a) BCL2
b) TELOMERASA
c) FCF
d) P53

235. SON MARCADORES TUMORALES EXCEPTO:


a) GONADOTROPINA CORIONICA HUMANA
b) INSULINA
c) ALFA FETOPROTEINA
d) ANTIGENO PROSTÁTICO ESPECÍFICO
236. CARCINÓGENOS QUÍMICOS EXCEPTO:
a) AGENTES ALQUILANTES
b) AFLATOXINA B
AMINAS AROMATICAS
c) ANTIBIOTICOS
237. UNO DE LOS GENES MUTADO EN LA MAYOR PARTE DE LOS CÁNCERES HUMANOS ES:
a) BAX
b) MYC
c) FIS
d)P53
238. LAS ENDOCRINOPATÍAS DE SÍNDROMES PARANEOPLÁSICOS SE ASOCIAN CON MAYOR
FRECUENCIA A:
a) CA EMBRIONARIO TESTICULAR
b) SEMINOMA
c) ADENOCARCINOMA PROSTÁTICO
d) CÁNCER DE PULMON
239. - ES UNA PROTEINA GRANDE DE ADHERENCIA CELULAR QUE LIGA MUCHAS MOLECULAS DE
LA MEC Y FORMA MATRIZ PROVISIONAL DURANTE LA CICATRIZACION:
a) CADHERINA
b) FIBRONECTINA
c) INTEGRINA
d) OSTEONECTINA
240. SON MEDIADORES QUIMICOS QUE INTERVIENEN EN LA ESTABILIZACION DE LOS VASOS
RECIEN FORMADOS
a) ON Y FCVE
b) PROSTAGLANDINAS
c) CDK Y CICLINAS
d)ANGIOPOYETINAS Y FCDP
241. -LA ASCITIS EN UN PACIENTE CON CIRROSIS ALCOHOLO NUTRICIONAL ES DEBIDO A:
A) AUMENTO DE LA PRESIÓN HIDROSTÁTICA Y OBSTRUCCIÓN LINFÁTICA
B) DISMINUCIÓN DE LA PRESIÓN OSMOTICA Y AUMENTO DE LA PRESIÓN HIDROSTÁTICA
C) RETENCION DE SODIO Y AUMENTO DE LA PRESIÓN HIDROSTÁTICA
D) OBSTRUCCIÓN LINFÁTICA Y DISMINUCIÓN DE LA PRESIÓN OSMOTICA
242. ¿CUÁL DE LAS SIGUIENTES SUSTANCIAS INHIBEN LA AGREGACION PLAQUETARIA?
a) TROMBOMODULINA
b) OXIDO NITRICO Y PROSTACICLINA
c) FACTOR ACTIVADOR DEL PLASMINOGENO TISULAR
d) ENDOTELINA
243. SUSTANCIA QUE ACTUA COMO FACTOR PROCOAGULANTE:
a) ENDOTELINA
b) FACTOR TISULAR
c) ANTITROMBINA III
d) PROSTACICLINA
244. ES UN ESTADO DE HIPERCOAGULABILIDAD DE ALTO RIESGO:
a) TABAQUISMO
b) USO DE ANTICONCEPTIVOS ORALES
c) FIBRILACION AURICULAR
d) MIOCARDIOPATIA
245. ES UN ANTICOAGULANTE NATURAL :
a) TROMBINA
b) ANTITROMBINA
c) FIBRINOGENO
d) FACTOR DE HAGEMAN
246. LA TROMBOSIS MESENTERICA ES UN EJEMPLO DE:
a) INFARTO BLANCO
b) INFARTO SEPTICO
c) INFARTO ROJO
d) INFARTO MURAL
247. EL ORIGEN MAS FRECUENTE DE LAS TROMBOEMBOLIAS PULMONARES ES:
a) LESIONES DEL ENDOCARDIO
b) VENAS PROFUNDAS Y SUPERFICIALES DE LOS MIEMBROS INFERIORES
c) ESTENOSIS MITRAL
d) TROMBOS MURALES INTRACARDIACOS
248. LOS INFARTOS BLANCOS SE PRODUCEN CUANDO HAY:
a) OBSTRUCCIÓN VENOSA
b) EN TEJIDOS CON DOBLE CIRCULACIÓN
c) EN TEJIDOS LAXOS
d) OBSTRUCCIÓN ARTERIAL EN ÓRGANOS SÓLIDOS CON RIEGO ARTERIAL TERMINAL
249. ES UN POTENTE VASOCONSTRICTOR QUE SE PRESENTA AL INICIO DE LA LESIÓN VASCULAR:
a) ENDOTELINA
b) PROSTACICLINA ENDOTELIAL
c) OXIDO NITRICO
d) HEPARINA
250. PERTENECEN AL GRUPO DE SHOCK DISTRIBUTIVO EXCEPTO:
a) SHOCK ANAFILACTICO
b) SHOCKCARDIOGENICO
c) SHOCK ENDOTOXICO
d) SHOCK NEUROGENICO
251. SON FACTORES SISTEMICOS QUE INFLUYEN EN LA REPARACIÓN DE LAS HERIDAS:
a) ESTADO CIRCULATORIO Y HORMONAS
b) INFECCIONES Y CUERPOS EXTRAÑOS
c) DENERVACION
d) ESTRÉS MECANICO
252. -SON TIPOS DE TROMBOS, EXCEPTO:
a) ARTERIALES
b) LIGAMENTOSOS, SUBSEROSOS
c) FLEBOTROMBOSIS
d) MURALES
253. LAS NEOPLASIAS MALIGNAS PUEDEN SER RESISTENTES A LA APOPTOSIS POR DAÑO A:
a) BRCA 1 Y 2
b) FAS Y JUN
c) RAS
d) BCL2
254. LA CAPACIDAD DE INVASION Y METASTASIS DE UNA NEOPLASIA MALIGNA ESTA DADA POR
:
a) CAPACIDAD PARA DEGRADAR LA MEC
b) SECRECION DE FACTORES DE CRECIMIENTO
c) SINTESIS DE COLAGENO
d)INSENSIBILIDAD A LAS SEÑALES INHIBITORIAS
255. PROTEINAS DE LA MEC QUE DESESTABILIZAN LAS INTERACCIONES CELULA-MATRIZ PARA
PROMOVER LA ANGIOGENESIS :
a) COLAGENO Y ELASTINA
b) ACIDO HIALURONICO Y DERMATAN SULFATO
c) ESTROMIELISINAS
d) SPARK Y TENASCINA
256. NEOPLASIA MALIGNA DE LAS CELULAS GERMINALES DEL TESTICULO
a) MELANOMA
b) LINFOMA
c) SEMINOMA
d) MESOTELIOMA
257. CORRELACIONE LOS TÉRMINOS DE LA COLUMNA IZQUIERDA CON LOS DE LA DERECHA
SEGÚN CORRESPONDAN:

46) SHOCK CARDIOGENICO  TAPONAMIENTO CARDIACO

47) LINFOMA  NEOPLASIA MALIGNA

48) INFARTO ROJO  TORSION DE QUISTE DE OVARIO

49) ADENOMA PLEOMORFO NEOPLASIA BENIGNA MIXTA

50) SHOCK HIPOVOLÉMICO  QUEMADURAS


1.- Patología en la que se pierde albumina y genera edema:
Sindrome Nefrotico

2.- Causantes de edema:


Enfermedad cardiaca y renal
3.- hematoma mayor a 2cm:
Equimosis

4.- Causa fisiológica de hiperemia: Vasodilatacion por ejercicio

5.- Factor que inhibe la diseminación del trombo:


Activador tisular de plasminogeno
6.- Capacidad de cambiar de un tipo celular a otro:
Transdiferenciasion

7.- Es la capacidad de una célula de transdiferenciarse a distintos estirpes celulares:


Plasticidad de desarrollo

8.- En esta fase de shock hay liberación de ADH y activación de baroreceptores:


Fase inicial progresiva

9.- Manifestaciones clínicas presentes en shock séptico:


Piel caliente con vasodilatación periférica, hiperemia.

10.- Casi siempre oclusiva puente largo de la luz del vaso:


Trombosis venosa profunda

11.- Tipo de necrosis encéfalo:


Licuefactiva

12.- Sintomas de la embolia grasa:


Taquicardia, bradicardia y disnea

13.- Mantiene la integridad del epitelio externo de la cornea:


Celulas madre del limbo

14.- Una neoplasia compuesta se divide como:


Dos capas celulares y una capa embrionaria

15.- Metastasis:
Implantes tumorales sin continuidad con el tumor primario

16.- Fases de la cascada metastasica:


Invasion de la MEC, diseminación vascular, alojamiento y colonización

17.- Inhibicion de TP53 mediado por: BAX

18.-wich one of the following characteristics not acompany the cancer:


Fibrous capsule
19.- What are the organs most affected by the hematogenous spread?
Liver and lung

20.-Wich ARN virus has on oncogene activity?


Leucemia and cel T
21.-Carcinogeno natural hepático secretado por plantas y hongos:
Aflatoxina B1

22.- Causas frecuentes de esteatosis:


Alcoholismo y desnutrición

23.- Medidador quimico que produce dilatación y aumento de la permeabilidad y se libera de tipo
físico: Histamina

24.- son células que participan en la reacción inflamatoria excepto:


Pericitos

25.- Las CX3C son moléculas de adhesión de:


Monocitos y Cel T

26.- De que vitamina son dependientes las proteínas C y S:


Vitamina K

27.- Localizacion mas frecuente del carcinoma de células renales:


Polos

28.- El cáncer renal causa frecuentemente metástasis en:


Pulmones y hueso

29.- Causa mas frecuente de cáncer renal:


Tabaquismo

30.- Cual es la proteína viral que induce una rápida degradación de P53:
E6
31.- Causa frecuente de metástasis en CACU:
Pielonefritis y Uremia

32.- Lesion primaria de mayor riesgo de producir cáncer de mama:


Mastopatia proliferativa con atipia

33.- Quimico que dara activación quimiotactica, actividad sobre monocitos y eosinofilos:
CXC

34.- sustancias que ocacionan fiebre, sueño y perdida del apetito:


Citocinas

35.- La inflamación fibrinosa se caracteriza por:


Fluidos corporales en pericardio y pleura
36.- Caracteristicas histológicas de órganos que sufren atrofia, hay presencia de:
Formacion de vacuolas autofagicas

37.- Los leiomiomas se dividen en:


Submucosos, subserosos e intramurales
38.- Las enfermedades crónicas granulomatosas abarca varios trastornos que tienen en común:
La incapacidad de destruir agentes etiológicos

40.- El infarto de miocardio corresponde a una:


cardiopatía isquémica

41.- Todos las siguientes sustancias pertenecen al tren de tincion, excepto:


Violeta de Genciana

42.- La citología exfoliativa puede reportar todo, excepto:


El estado nutricional de la células.

43.- Carcinoma in situ ductual consiste en:


Poblacion maligna de células que no pueden atravesar la Membrana Basal

44.- La fosfofructocinasa y la fosforilaza en lesión, activan a:


Glucolisis Anaerobia

45.- Que moléculas emiten la adhesión de plaquetas al endotelio no lesionado?:


Prostacicilinas y Oxido Nitrico

46.- Para la formación de granulomas se requiere:


reacción inmunitaria mediada por células T

47.- Pancreatitis aguda ------------------------ Necrosis Grasa

48.- Infeccion Bacteriana----------------------N. Licuefactiva

49.- Muerte celular----------------------------- Apoptosis

50.- Tuberculosis------------------------------- N. caseosa

51.- Infarto al miocardio--------------------- N. Coagulativa


1.- Patología en la que se pierde albumina y genera edema:
Sindrome Nefrotico

2.- Causantes de edema:


Enfermedad cardiaca y renal

3.- hematoma mayor a 2cm:


Equimosis

4.- Causa fisiológica de hiperemia: Vasodilatacion por ejercicio

5.- Factor que inhibe la diseminación del trombo:


Activador tisular de plasminogeno

6.- Capacidad de cambiar de un tipo celular a otro:


Transdiferenciasion

7.- Es la capacidad de una célula de transdiferenciarse a distintos estirpes celulares:


Plasticidad de desarrollo

8.- En esta fase de shock hay liberación de ADH y activación de baroreceptores:


Fase inicial progresiva

9.- Manifestaciones clínicas presentes en shock séptico:


Piel caliente con vasodilatación periférica, hiperemia.

10.- Casi siempre oclusiva puente largo de la luz del vaso:


Trombosis venosa profunda

11.- Tipo de necrosis encéfalo:


Licuefactiva

12.- Sintomas de la embolia grasa:


Taquicardia, bradicardia y disnea

13.- Mantiene la integridad del epitelio externo de la cornea:


Celulas madre del limbo

14.- Una neoplasia compuesta se divide como:


Dos capas celulares y una capa embrionaria

15.- Metastasis:
Implantes tumorales sin continuidad con el tumor primario

16.- Fases de la cascada metastasica:


Invasion de la MEC, diseminación vascular, alojamiento y colonización

17.- Inhibicion de TP53 mediado por: BAX

18.-wich one of the following characteristics not acompany the cancer:


Fibrous capsule
19.- What are the organs most affected by the hematogenous spread?
Liver and lung

20.- Wich ARN virus has on oncogene activity?


Leucemia and cel T

21.-Carcinogeno natural hepático secretado por plantas y hongos:


Aflatoxina B1

22.- Causas frecuentes de esteatosis:


Alcoholismo y desnutrición

23.- Medidador quimico que produce dilatación y aumento de la permeabilidad y selibera de tipo
físico: Histamina

24.- son células que participan en la reacción inflamatoria excepto:


Pericitos

25.- Las CX3C son moléculas de adhesión de:


Monocitos y Cel T

26.- De que vitamina son dependientes las proteínas Cy S:


Vitamina K

27.- Localizacion mas freuente del carcinoma de células renales:


Polos

28.- El cáncer renal causa frecuentemente metástasis en:


Pulmones y hueso

29.- Causa mas frecuente de cáncer renal:


Tabaquismo

30.- Cual es la proteína viral que induce una rápida degradación de P53:
E6

31.- Causa frecuente de metástasis en CACU:


Pielonefritis y Uremia

32.- Lesion primaria de mayor riesgo de producir cáncer de mama:


Mastopatia proliferativa con atipia

33.- Quimico que dara activación quimiotactica, actividad sobre monocitos y eosinofilos:
CXC

34.- sustancias que ocacionan fiebre, sueño y perdida del apetito:


Citocinas

35.- La inflamación fibrinosa se caracteriza por:


Fluidos corporales en pericardio y pleura
36.- Caracteristicas histológicas de órganos que sufren atrofia, hay presencia de:
Formacion de vacuolas autofagicas

37.- Los leiomiomas se dividen en:


Submucosos, subserosos e intramurales

38.- Las enfermedades crónicas granulomatosas abarca varios trastornos que tienen en común:
La incapacidad de destruir agentes etiológicos

40.- El infarto de miocardio corresponde a una:


cardiopatía isquémica

41.- Todos las siguientes sustancias pertenecen al tren de tincion, excepto:


Violeta de Genciana

42.- La citología exfoliativa puede reportar todo, excepto:


El estado nutricional de la células.

43.- Carcinoma in situ ductual consiste en:


Poblacion maligna de células que no pueden atravesar la Membrana Basal

44.- La fosfofructocinasa y la fosforilaza en lesión, activan a:


Glucolisis Anaerobia

45.- Que moléculas emiten la adhesión de plaquetas al endotelio no lesionado?:


Prostacicilinas y Oxido Nitrico

46.- Para la formación de granulomas se requiere:


reacción inmunitaria mediada por células T

47.- Pancreatitis aguda ------------------------ Necrosis Grasa

48.- Infeccion Bacteriana----------------------N. Licuefactiva

49.- Muerte celular----------------------------- Apoptosis

50.- Tuberculosis------------------------------- N. caseosa

51.- Infarto al miocardio--------------------- N. Coagulativa


1.-Características microscópicas importantes del Leiomioma
aspecto arremolinado del musculo

2.-Características morfológicas de la apendicitis aguda supurada


absceso de la pared acompañado con ulceración

3.-Ocurre cuando el núcleo de fibroblasto cutáneo de un paciente se introduce en un ovocito


humano enucleado para formar células madre embrionarias que se cultivan se inicia su
diferenciación y se trasplantan al paciente
clonación terapéutica

4.- las células madre progenitoras del hígado se albergan en


conductos de Hering

5.- factor de crecimiento que contribuyye a la reparación de heridas oncogenia, hematopoyesis y


desarrollo de musculo cardiaco y esquelético, la maduración del pulmon y la especificación del
hígado
factor de crecimiento fibroblastico

6.- sun radations are associated with the following type of cáncer
carcinoma basocelular

7.- patron morfologico de necrosis que presentan las celulas muertas por isquemia salvo en el
sistema nerviso central
coagulación o coagulativa

8.- nivel serico de las enzimas necestia de cuanto tiempo para dar el Dx seguro
2-4 hrs

9.- ejemplos del grupo sisiopatologico de edema por disminución de presión osmótica
desnutrición proteica, kwashiorkor

10.- tipo de metaplasia mas frecuente en tejido epitelial


epitelio cilíndrico a plano estratificado
11.- anormalidades encontradas en citología exfoliativa excepto
presencia de anticuerpos
12.- interactua con un receptor TIE2 para reclutar células periendoteliales
angiopoyetina tipo 1

13.- color azul verdoso que adquieren los hematomas en la fase de resolución xq
hemoglobina se transforma en hemosiderina

14.- RNA virus have oncogene activity


leucemia, cel T
15.- coagulo constituido por agregación plaquetaria y deposito de fibrina
tapon hemostático secundario

16.- alteración de escasa frecuencia que se debe a una proliferación exuberante de fibroblastos y
otros elementos de tejido conjuntivo
lesiones desmoides

17.- proceso de remodelación de la reparación de las heridas participan enzimas


metaloproteinasas y gelatinasas

18.- adaptación cel. Crecimiento que puede ser terreno fértil, donde se inicia una proliferación
cancerosa
displasia

*19.- adaptación celular de la diferenciacion que consiste en un cambio reversible de una celula
adulta por una celula adulta
metaplasia

*20.- selectinas integrinas y flicoproteinas tipo mucina atuan como


moléculas de adhesión

*21.- principal quimiotactico endógeno para …..


CXC o alfa

*22.- la lilsosima lactoferrina……


independientes del oxigeno

*23.-principal quimiocina que induce ……..


CX3C

*24.- capacidad de una celula de transdiferenciarse a distintos tipos celulares


plasticidad de desarrollo

*26.- manifestación clínica mas frecuente del canceer renal


hematuria
27.- criterio hitologico para Dx de apendicitis aguda
infiltacion de neutrofilos en la capa muscular

28.- es mitogeno para celuals epiteliales hepetocitos y fibroblastos, es homogeno de TGF alfa, al
neter actividades biológicas similares
TGF

29.- son grandes colecciones de sangre en cavidades excepto


hemostasia
30.- son sustancias con capacidad antitrombotica
plasminogeno tisular y prostaciclina endotelial

31.- tejido de granulación se carateriza por


proliferación de fibroblastos y pequeños vasos sanguíneos

32.- ejemplo de trombosis por estadios de hipercoagulabilidad secundaria


síndrome anticoagulante lupico

33.- infartos rojos aparecen en las siguientes condiciones excepto


en oclusiones arteriales

34.- most cases of septic shock are caused by endotoxin producing bacteria, which are classified
as
gram positivas

35.- en el fribroadenoma mamario se caracteriza sobre todo los siguientes excepto


esta fijo a planos profundos y no se puede delimitar su tamaño

36.- the abnormal triglyceride accumulation linside perenshma cells called


steatosis or change
37.- se pueden encontrar granulomas en las siguientes enfermedades
lepra peromatosa y tuberculosa

38.- son tipos de embolias excepto


embolia linfática

39.- tipo de calcificaicon que se acompaña de hipercalemia y trastornos en el metabolismo del


calcio
metastasica

40.- leiomiomas de acuerdo a su localización en el cuerpo uterino se clasifican en


submucosos, subserosos e intramurales

41.- metabolitos del acido araquidonico por via de ………….


prostaglandinas y tromboxano
42.- pigmento endógeno, no derivado de la hemoglobina
melanina

43.- cuerpos de Russell son acumulos de


inmunoglobulinas en células plasmáticas
44.- los cálculos biliares están formados por
colesterol, calcio y pigmentos biliares
45.- complicaciones mas frecuentes de la apendicitis
peritonitis

46.- la alfa feto proteína se eleva anormalmente en


cáncer primario de hígado, de células germinales del testículo

47.- la citometria de flujo puede medir


los antígenos de membrana, contenido de ADN en células tumorales

48.- la iniciación de la calcificación distrofica extracelular se inicia en


los fosfolipidos de la membrana celular

49.- fenómeno que considera dentro del destino del trombo excepto
propagación

50.- en cual de los siguientes órganos se puede desarrolar un teratoma


ovario
Causa predisponente de la calcificaci6n metastasica
R: Medio alcalino

La biopsia de elecci6n para una lesi6n hiperpigmentada localizada en la piel y que mide un
centimetro de didmetro es:
R: Excisional

Quimiocina, linfotactina para:

Es causa de la calcificacion distréfica.


R: Focos de necrosis

Caracteristicas de necrosis fibrinoide


R: Depdsitos de inmunocomplejos en pared de vasos sanguineos

2:55 histamina, prostaglandina


éCémo la telomerasa inhibe el acortamiendto de los tel6meros?

Son sensores de lesion o estrés celular en la apoptosis. R: Puma y Bid

. La via extrinseca de la apoptosis se activa por: caspasa 8


. De las causas de lesion celular cual es la que afecta de manera directa a la fosforilacién
oxidativa aerdbica. R: Isquemia o hipoxia.
13. La telomerasa es un complejo. R: RNA protejina
14. En la lesi6n celular reversible debida a isquemia/hipoxia la primera activacién bioquimica
se presenta en: disminuci6n de la fosforilacién
15. Fendémenos que caracterizan que la lesion celular es irreversible.
16. Protejnas de transporte y almacenamiento con capacidad antioxidante: Lactoferrina,
transferrina, ferritina.
17. La calcificacién distréfica se presenta en: ateroesclerosis
18. Metaplasia
19. Efectos locales provocados por IL-1, FNT e IL-6 en la reaccion inflamatoria: aumento de la
adhesion y activacion leucocitaria.
20. Moléculas antiapoptoticas: BCL2, BCL-XL
21. Son caracteristicas morfoldgicas de la apoptosis excepto: Fallo de funciones dependientes
de energia
22. Tipo de metaplasia en el esdfago de Barret: de epitelio plano a epitelio cilindrico.
23. La acumulaci6én intracelular de polvo de carbén produce reaccién fibroblastica recibe el
nombre: neumoconiosis
24. Pigmento del envejecimiento: Lipocromo o Lipofuscina
25. Componentes de la inflamaci6n aguda: Modificacidn en el calibre de los vasos y en la
estructura de la microvasculatura.
26. El aumento de la permeabilidad vascular da como resultado:
27. Selectinas, integrinas y glicoproteinas actuan como: moléculas de adhesion.
28. La aspirina inhibe la PGE2 para disminuir la fiebre.
29. Peroxidacion de los lipidos de la membrana, lesiones del DNA son causados por: Radicales
libres derivados de oxigeno.
30. IgG e C3b funcionan como: opsoninas
31. Los principales quimiotacticos enddédgenos: C5a, C3a
32. Tipo de necrosis en encéfalo: necrosis licuefactiva.
33. La fractalquina es una molécula de adhesion para: monocitos y Linfocitos T
34. Los cuerpos de Russell son acimulos de: Inmunoglobulinas en las células plasmaticas.
35. Productos del metabolismo del acido araquidonico por la via de la ciclooxigenasa:
Prostaglandinas y protaciclina, Txa2
36. Grupo de mediadores quimicos que producen inhibicidn de la quimiotaxis y de la
adherencia de neutrofilos: Lipoxinas
37. Calicreina
38. Enfermedad de Crohn
39. Quimiocina que da lugar activacién y quimiotaxis de los neutrofilos, con activacién
limitada sobre monocitos y eosindfilos:
40. Prostaglandina que produce hiperalgesia, que hace que la piel presente hipersensibilidad,
dolor y fiebre: PGE. (Revisala)
41. Proteina que puede unirse a la proteina BCL2, BCL-XL y realiza funciones antiapoptoticas:
BH3
42. Dependiente de calcio
43. En inflamacion son pirégenos enddgenos: IL-1 y FNT
44. Causas mas frecuentes de esteatosis hepatica: alcoholismo y desnutricién.
45. Ejemplos de inflamacién serosa: infecciones virales y quemaduras de segundo grado.
46. 10 veces mas potente que la histamina: FAP
47. En la apoptosis ambas moléculas se unen a la membrana de la mitocondria, hacen poros y
liberan citocromo c: BAX y BAK
48. Factor de crecimiento relacionado con la hipertrofia relacionado con el aumento de la
carga de trabajo: TGF-B
49. La atrofia se produce por:
50. El receptor de muerte mas conocido en la apoptosis se llama: FAS
51. Proteinas plasmaticas de fase aguda de la inflamacidn: Proteina C reactiva, fibrinégeno,
52. Inhibe el apetito: FNT
1.- Patología en la que se pierde albumina y genera edema:
Sindrome Nefrotico

2.- Causantes de edema:


Enfermedad cardiaca y renal

3.- hematoma mayor a 2cm:


Equimosis

4.- Causa fisiológica de hiperemia: Vasodilatacion por ejercicio

5.- Factor que inhibe la diseminación del trombo:


Activador tisular de plasminogeno

6.- Capacidad de cambiar de un tipo celular a otro:


Transdiferenciasion

7.- Es la capacidad de una célula de transdiferenciarse a distintos estirpes celulares:


Plasticidad de desarrollo

8.- En esta fase de shock hay liberación de ADH y activación de baroreceptores:


Fase inicial progresiva

9.- Manifestaciones clínicas presentes en shock séptico:


Piel caliente con vasodilatación periférica, hiperemia.

10.- Casi siempre oclusiva puente largo de la luz del vaso:


Trombosis venosa profunda

11.- Tipo de necrosis encéfalo:


Licuefactiva

12.- Sintomas de la embolia grasa:


Taquicardia, bradicardia y disnea

13.- Mantiene la integridad del epitelio externo de la cornea:


Celulas madre del limbo

14.- Una neoplasia compuesta se divide como:


Dos capas celulares y una capa embrionaria

15.- Metastasis:
Implantes tumorales sin continuidad con el tumor primario

16.- Fases de la cascada metastasica:


Invasion de la MEC, diseminación vascular, alojamiento y colonización

17.- Inhibicion de TP53 mediado por: BAX

18.-wich one of the following characteristics not acompany the cancer:


Fibrous capsule
19.- What are the organs most affected by the hematogenous spread?
Liver and lung

20.-Wich ARN virus has on oncogene activity?


Leucemia and cel T

21.-Carcinogeno natural hepático secretado por plantas y hongos:


Aflatoxina B1

22.- Causas frecuentes de esteatosis:


Alcoholismo y desnutrición

23.- Medidador quimico que produce dilatación y aumento de la permeabilidad y se libera de tipo
físico: Histamina

24.- son células que participan en la reacción inflamatoria excepto:


Pericitos

25.- Las CX3C son moléculas de adhesión de:


Monocitos y Cel T

26.- De que vitamina son dependientes las proteínas C y S:


Vitamina K

27.- Localizacion mas frecuente del carcinoma de células renales:


Polos

28.- El cáncer renal causa frecuentemente metástasis en:


Pulmones y hueso

29.- Causa mas frecuente de cáncer renal:


Tabaquismo

30.- Cual es la proteína viral que induce una rápida degradación de P53:
E6

31.- Causa frecuente de metástasis en CACU:


Pielonefritis y Uremia

32.- Lesion primaria de mayor riesgo de producir cáncer de mama:


Mastopatia proliferativa con atipia

33.- Quimico que dara activación quimiotactica, actividad sobre monocitos y eosinofilos:
CXC

34.- sustancias que ocacionan fiebre, sueño y perdida del apetito:


Citocinas

35.- La inflamación fibrinosa se caracteriza por:


Fluidos corporales en pericardio y pleura
36.- Caracteristicas histológicas de órganos que sufren atrofia, hay presencia de:
Formacion de vacuolas autofagicas

37.- Los leiomiomas se dividen en:


Submucosos, subserosos e intramurales

38.- Las enfermedades crónicas granulomatosas abarca varios trastornos que tienen en común:
La incapacidad de destruir agentes etiológicos

40.- El infarto de miocardio corresponde a una:


cardiopatía isquémica

41.- Todos las siguientes sustancias pertenecen al tren de tincion, excepto:


Violeta de Genciana

42.- La citología exfoliativa puede reportar todo, excepto:


El estado nutricional de la células.

43.- Carcinoma in situ ductual consiste en:


Poblacion maligna de células que no pueden atravesar la Membrana Basal

44.- La fosfofructocinasa y la fosforilaza en lesión, activan a:


Glucolisis Anaerobia

45.- Que moléculas emiten la adhesión de plaquetas al endotelio no lesionado?:


Prostacicilinas y Oxido Nitrico

46.- Para la formación de granulomas se requiere:


reacción inmunitaria mediada por células T

47.- Pancreatitis aguda ------------------------ Necrosis Grasa

48.- Infeccion Bacteriana----------------------N. Licuefactiva

49.- Muerte celular----------------------------- Apoptosis

50.- Tuberculosis------------------------------- N. caseosa

51.- Infarto al miocardio--------------------- N. Coagulativa


PRIMER EXAMEN PARCIAL DE

ANATOMIA PATOLOGICA
OTOÑO 2014
DR. ALEJANDRO J. VAZQUEZ
GUERRA
1.-) CAUSA PREDISPONENTE DE LA
CALCIFICACION METASTASICA:
A) MEDIO ACIDO
B) MEDIO ALCALINO
C) MEDIO NEUTRO
D) TEJIDO LESIONADO
E) TEJIDO NECROSODA
2.-) LA BIOPSIA DE ELECCIÓN PARA
UNA LESIÓN HIPERPIGMENTADA
LOCALIZADA EN LA PIEL Y QUE MIDE
1CM DE DIÁMETRO, ES:
A) INCISIONAL
B) EXCISIONAL
C) BAAF
D) POR RASPADO
E) ENDOSCOPIA
3.-) EL FIJADOR MÁS UTILIZADO PARA
LOS TEJIDOS OBTENIDOS POR
BIOPSIA O NECROPSIA ES:
A) ALCOHOL DE 96º
B) TOLUOL DE 96º
C) BENZOL AL 10%
D) FORMOL AL 10%
E) CITOSPRAY
4.-) ES CAUSA DE LA CALCIFICACIÓN
DISTRÓFICA:
A) INCREMENTO DE LA SECRECIÓN DE
HORMONA PARATIROIDEA
B) DESTRUCCIÓN OSEA
C) INTOXICACIÓN CON VITAMINA D
D) FOCOS DE NECROSIS
E) INSUFICIENCIA RENAL
5.-) EL APOPTOSOMA ESTA FORMADO
POR LAS SIGUIENTES SUSTANCIAS:
A) TNF-1 SE UNE A FAS
B) FLIP SE UNE A LA PROCASPASA 8
C) IAP SE UNE A LA CASPASA 3
D) CITOCROMO C SE UNE APAF-1
E) BH3 BLOQUEA A LAS BCL
6.-) WHAT CHEMICAL MEDIATORS ARE
RESPONSIBLE FOR INCREASING THE BLOOD
FLOW TO AN ARE OF INFLAMATION?
A) HISTAMINE, PROSTAGLANDINS (PGI2, PGE,
AND PGD2), AND NITRIC OXIDE
B) INTEGRINS, SELECTINS, ICAM-1, VCAM1
C) THROMBOXANE A2, LTB4, LTC4, LTD4, LTE4
D) TUMOR NECROSIS FACTOR AND
INTERLEUKIN 1
E) CHEMOKINES C-X-C, C-C, CX3C.
7.-) FORMA POR LA CUAL LA TELOMERASA INHIBE EL
ACORTAMIENTO DE LOS TELOMEROS:
A) AÑADE NUCLEOTIDOS A LOS EXTREMOS DEL
CROMOSOMA
B) REPARA LAS LESIONES DE LAS CADENAS DE
ADN
C) REPARA LAS LESIONES DE LA CADENA DE ARN
D) BLOQUEA A LOS RADICALES LIBRES
DERIVADOS DEL OXIGENO
E) BLOQUEA EL CICLO CELULAR
8.-) ¿CUAL ES LA CARACTERISTICA DEL
NUCLEO CELULAR EN LA APOPTOSIS:
A) CARIORREXIS
B) CARIOLISIS
C) EOSINOFILIA
D) CONDENSACION DE LA CROMATINA
E) HIPERCROMATISMO CELULAR
9.-) LA VIA EXTRÍNSECA DE LA
APOPTOSIS ACTIVA A;
A) CASPASA 1 Y 3
B) CASPASA 9
C) CASPASA 4
D) CASPASAS 8, 10
E) CASPASA 6
10.-) ¿DE LAS CAUSAS DE LESIÓN CELULAR, CUÁL
DE LAS SIGUIENTES ES LA QUE AFECTA DE
MANERA DIRECTA A LA RESPIRACIÓN OXIDATIVA
AÉROBICA EN LA CÉLULA?
A) LOS AGENTES FÍSICOS
B) LA ISQUEMIA / HIPOXIA
C) LOS TRASTORNOS GENÉTICOS
D) LAS REACCIONES INMUNOLÓGICOS
11.-) LA TELOMERASA ES UN COMPLEJO:
A) ADN-PROTEINA
B) ARN-PROTEINA
C) PROTEINA Bcl2
D) PROTEINA IAP
E) TNF ALFA
12.-) EN LESIÓN CELULAR REVERSIBLE DEBIDA
A ISQUEMIA /HIPOXIA LA PRIMERA
ALTERACIÓN BIOQUÍMICA QUE SE PRESENTA
EN:
A) LA SÍNTESIS PROTÉICA
B) ALTERACIONES EN EL NÚCLEO
C) DISMINUCIÓN DE LA FOSFORILACIÓN
OXIDATIVA MITOCONDRIAL
D) LESIÓN DEL CITOESQUELETO
13.-) SON FENÓMENOS QUE CARACTERIZAN E
INDICAN QUE LA LESIÓN CELULAR ES
IRREVERSIBLE:
A) LAS ALTERACIONES DE LOS RIBOSOMAS
B) LA FRAGMENTACIÓN DEL NÚCLEO
CELULAR
C) LA DISFUNCIÓN MITOCONDRIAL ,
PROFUNDOS TRASTORNOS EN LA FUNCIÓN
DE LAS MEMBRANAS CELULARES
D) LA DILATACIÓN DEL RETÍCULO
ENDOPLÁSMICO RUGOSO
14.- )¿CÚALES DE LOS SIGUIENTES SON
FACTORES ANTIOXIDANTES ENZIMÁTICOS?
A) TRANSFERRINA Y LACTOFERRINA
B) VITAMINA A Y VITAMINA E
C) LACTOFERRINA Y CÉRULOPLASMINA
D) CATALASA, GLUTATIÓN PEROXIDASA Y
SUPERÓXIDO DISMUTASAS
15.-) LA CALCIFICACION DISTROFICA SE
PRESENTA EN:
A) HIPERPATIROIDISMO
B) MIELOMA MULTIPLE
C) ATEROMATOSIS
D) INSUFICIENCIA RENAL
E) SARCOIDOSIS
16.- )ADAPTACIÓN CELULAR DEL
CRECIMIENTO QUE PUEDE SER TERRENO
FÉRTIL EN DONDE SE INICIE UNA
PROLIFERACIÓN MALIGNA:
A) ATROFIA
B) HIPERTROFIA
C) HIPERPLASIA PATOLÓGICA
D) HIPERPLASIA FISIOLÓGICA
E) APLASIA
17.-) SON EFECTOS SISTEMICOS PROVOCADAS POR LA
IL-I, EL FNT EN LA REACCION INFLAMATORIA:
A) FIEBRE, PÉRDIDA DEL APETITO , SUEÑO
B) AUMENTO DE LA ADHESIÓN LEUCOCITARIA
C) AUMENTO DE SÍNTESIS DE PROSTAGLANDINAS
D) PROLIFERACIÓN DE FIBROBLASTOS Y AUMENTO
DE LA SÍNTESIS DE COLÁGENA
E) AUMENTO DE LA ACTIVIDAD PROCOAGULANTE Y
DISMINUCIÓN DE LA ANTICOAGULANTE
18.-) LA ATROFIA PUEDE SER:
A) HORMONAL Y COMPENSADORA
B) BENIGNA Y MALIGNA
C) FISIOLÓGICA Y PATOLÓGICA
D) REACTIVA E INDUCIDA
19.-) DE LAS REFERIDAS A CONTINUACIÓN,
SEÑALE EN QUE MUESTRAS PUEDE REALIZARSE
CITOLOGIA EXFOLIATIVA:
A) HUESO, CARTÍLAGO Y MUSCULO ESQUELÉTICO
B) ORINA, EXPECTORACIÓN Y LÍQUIDO DE
ASCITIS
C) DERMIS, EPIDERMIS Y TEJIDO CONECTIVO
LAXO
D) HÍGADO, BAZO Y MESENTERIO
20.-) TIPO DE METAPLASIA MÁS FRECUENTE:
A) DE EPITELIO CILÍNDRÍCO A EPITELIO PLANO
ESTRATIFICADO
B) DE TEJIDO CONECTIVO A CARTÍLAGO
C) DE EPITELIO PLANO A EPITELIO CILÍNDRICO
D) DE TEJIDO MUSCULAR ÓSEO
21.-) LA ACUMULACIÓN INTRACELULAR DE POLVO
DE CARBÓN RECIBE EL NOMBRE DE:
A) GLUCOGENOSIS
B) CUERPOS DE RUSSELL
C) CUERPOS DE COUNCILMAN
D) ANTRACOSIS
E) NEUMOCONIOSIS
22.-) ¿A CÚAL DE LOS PIGMENTOS ENDÓGENOS
QUE SE PUEDEN ACUMULAR
INTRACELULARMENTE SE LE LLAMA “´PIGMENTO
DE ENVEJECIMIENTO”?
A) LIPOFUSCINA
B) BILIRRUBINA
C) HEMOSIDERINA
D) MELANINA
23.-) SON COMPONENTES DE LA INFLAMACIÓN
AGUDA:
A) MODIFICACIÓN EN EL CALIBRE DE LOS VASOS
Y EN LA ESTRUCTURA DE LA
MICROVASCULATURA
B) GRAN DESTRUCCIÓN TISULAR Y REPARACIÓN
POR TEJIDO FIBROSO
C) ATROFIA, HIPERTROFIA E HIPERPLASIA
D) REGENERACIÓN Y CICATRIZACIÓN POR TEJIDO
CONJUNTIVO
24.-)EL AUMENTO DE LA PERMEABILIDAD
VASCULAR DA COMO RESULTADO:
A) AUMENTO DE LAS PROTEÍNAS PLASMÁTICAS
(ALBÚMINA)
B) HEMORRAGIA EN LOS TEJIDOS
C) SALIDA DE UN FLUIDO RICO EN CELULAS
Y PROTEÍNAS AL INTERSTICIO
D) QUÉ SE INICIEN LOS MECANISMOS DE
REPARACIÓN
E) QUÉ SE LIBEREN LOS MEDIADORES DE LA
INFLAMACIÓN
25.-) SELECTINAS, INTEGRINAS Y
GLUCOPROTEÍNAS TIPO MUCINA ACTÚAN COMO:
A) SUSTANCIAS QUIMIOTÁCTICAS ENDÓGENAS
B) OPSONINAS
C) SUSTANCIAS QUE AUMENTAN LA
PERMEABILIDAD VASCULAR
D) SUSTANCIAS VASODILATADORES
E) MOLÉCULAS DE ADHESIÓN
26-) A 60-YEAR-OLD OBESE MAN WAS ADMITTED
TO THE HOSPITAL FOR TREATMENT OF
ALCOHOLISM. HE HAS DIABETES MELLITUS. A
LIVER BIOPSY WAS PERFORMED, AND THE
SPECIMEN SHOWED THAT THE LIVER CELLS
CONTAIN INCREASED AMOUNTS OF
A) HEMOSIDERIN
B) BILE
C) TRIGLYCERIDES
D) BILIRUBIN
E) INSULIN
27.-)PEROXIDACIÓN DE LOS LÍPIDOS DE LA
MEMBRANA, MODIFICACIÓN OXIDATIVA DE LAS
PROTEÍNAS Y LESIONES DEL DNA SON EFECTOS
RELEVANTES PARA LA LESIÓN CELULAR
CAUSADAS POR:
A) HIPOXIA
B) ISQUEMIA
C) ACIDOSIS LÁCTICA
D) RADICALES LIBRES DERIVADOS DEL OXÍGENO
E) FALTA DE PRODUCCIÓN DE ATP
28.-) LOS ANTICUERPOS DE LA IGG, C3B Y LAS
COLECTINAS FUNCIONAN COMO:
A) QUIMIOTÁCTICOS
B) MOLÉCULAS DE ADHESIÓN
C) OPSONINAS
D) VASODILATADORES
E) SUSTANCIAS QUE ESTIMULAN LA REPARACIÓN
TISULAR
29.-) PRINCIPALES QUIMIOTACTICOS ENDOGENOS:
A) C3B Y C3BI
B) C5A Y LTB4
C) FRAGMENTO FC DE IGG Y COLECTINAS
D) E-SELECTINA, P-SELECTINA Y L-SELECTINA
E) COLECTÍNAS Y PROTEÍNAS TIPO MUCINA
30.) A 60-YEAR-OLD MAN WHO HAD GENERALIZED
ATHEROSCLEROSIS DIED 24 HOURS AFTER HAVING
A STROKE. A CEREBRAL INFARCT WAS FOUND AT
AUTOPSY. NECROSIS OF THE BRAIN IS CLASSIFIED
AS:
A) COAGULATIVE NECROSIS
B) LIQUEFACTIVE NECROSIS
C) FAT NECROSIS
D) FIBRINOID NECROSIS
E) CASEOUS NECROSIS
31,.) LAS QUIMIOCINAS CX3C SON MOLÉCULAS DE
ADHESIÓN PARA:
A) MONOCITOS Y LINFOCITOS T
B) MACROFAGOS Y CELULAS B
C) EOSINOFILOS Y BASOFILOS
D) NEUTROFILOS Y MACROFAGOS
E) NEUTROFILOS Y BASOFILOS
32.-) LOS CUERPOS DE RUSSELL SON ACUMULOS
DE:
A) INMUNOGLOBULINAS EN CÉLULAS
PLASMÁTICAS
B) PROTEÍNAS EN LAS CÉLULAS TUBULARES
C) GRASA EN HEPATOCITOS
D) COLESTEROL EN MACRÓFAGOS DE LA LAMINA
PROPIA DE LA VESÍCULA BILIAR
E) GLUCÓGENO EN LAS CÉLULAS EPITELIALES DE
LOS TUBULOS CONTORNEADOS DÍSTALES.
33.-) SON PRODUCTOS DEL METABOLISMO DEL
ÁCIDO ARAQUIDÓNICO POR LA VÍA DE LA
CICLOOXIGENASA:
A) TROMBOXANO Y PROSTAGLANDINAS
B) LEUCOTRIENOS Y LIPOXINAS
C) ION SUPERÓXIDO Y PERÓXIDO DE
HIDROGENO
D) SISTEMA PLASMINOGENO PLASMINA
E.) FIBRINOPÉPTIDOS
34.-) GRUPO DE MEDIADORES QUIMICOS QUE
PRODUCEN INHIBICION DE LA QUIMIOTAXIS DE
NEUTROFILOS Y LA ADHERENCIA DE LOS
MONOCITOS :
A) LEUCOTRIENOS
B) MONOCINAS
C) LIPOXINAS
D) PROSTAGLANDINAS
E) AMINAS VASOACTIVAS
35.-) BRADYKININ IS PRODUCED FROM A HIGH-
MOLECULAR WEIGHT KINOINOGEN
CIRCULATING IN THE BLOOD. THIS REACTION IS
MEDIATED BY
A) COAGULATION FACTOR X
B) KALLIKREIN
C) HAGEMAN FACTOR
D) COMPLEMENT C3
E) PROTEIN
36.-) ESTRUCTURA FORMADA POR UN ACUMULO
DE CÉLULAS EPITELIOIDES, RODEADAS POR UN
COLLAR DE LINFOCITOS Y CÉLULAS
PLASMÁTICAS SE PRESENTA EN:
A) BRUCELOSIS
B) E. COLI
C) CHLAMYDIA TRACHOMATIS
D) NEISSERIA GONORRHEOAE
E) SALMONELLA
37.-) QUIMIOCINA QUE DA LUGAR A ACTIVACIÓN
Y QUIMIOTAXIS DE LOS NEUTROFILOS, CON
ACTIVIDAD LIMITADA SOBRE MONOCITOS Y
EOSINOFILOS:
A) C-X-C O ALFA
B) C-C O BETA
C) C O GAMMA
D) CX 3C
E) CXCR4
38.-) PROSTAGLANDINA QUE PRODUCE
HIPERALGESIA, QUE HACE QUE LA PIEL PRESENTE
HIPERSENSIBILIDAD (DOLOR Y FIEBRE)
A) PGE2
B) PGD2
C) PGF2 ALFA
D) PGI2
E) TxA2
39.-) SISTEMA BACTERICIDA MAS EFICAS DE LOS
NEUTROFILOS
A) H2O2 + MPO + Cl
B) BPI (PROTEINA BACTERICIDA POR
INCREMENTO DE LA PERMEABILIDAD)
C) PBP (PROTEINA BASICA PRINCIPAL
D) DEFENSINAS
E) LACTOFERRINA
40.-) NITRIC OXIDE SYNTHESIS IS AUGMENTED IN
ENDOTHELIAL CELLS BY A CALMODULIN
MEDIATED INFLUX OF WHICH ELEMENT?
A) CALCIUM
B) SODIUM
C) POTASSIUM
D) OXYGEN
E) NITRATE
41.-) PUS O MATERIAL PURULENTO SE
ENCUENTRA FORMADO POR:
A) MACRÓFAGOS Y LINFOCITOS
B) DETRITUS CELULARES, NEUTRÓFILOS Y
LIQUIDO DE EDEMA
C) EXUDADO SEROSO Y HEMORRÁGICO
D) EXUDADO FIBRINOSO Y LEUCOCITOS
E) TEJIDO DE CICATRIZACIÓN
42.-)LA CAUSAS MÁS FRECUENTES DE LA
ESTEATOSIS HEPATICA SON:
A) BACTERIAS, HONGOS
B) TRAUMATISMOS, DESHIDRATACIÓN
C) RADIACIONES IONIZANTES,
QUEMADURAS
D) ALCOHOLISMO, DESNUTRICIÓN
E) CHOQUE ELÉCTRICO, PARÁSITOS
43.-) SON EJEMPLOS DE INFLAMACIÓN SEROSA:
A) APENDICITIS SUPURATIVA AGUDA,
FURUNCULOSIS
B) ULCERA DUODENAL Y GÁSTRICA
C) INFECCIONES VIRALES Y QUEMADURAS DE 2º
GRADO
D) PERICARDITIS FIBRINOSA, PLEURITIS
FIBRINOSA
E) ULCERA DE MUCOSA ORAL, DE MIEMBROS
44.-) MEDIADOR QUIMICO QUE PRODUCE
VASODILATACION Y SU EFECTO ES 10.000 VECES
MAS POTENTE QUE LA HISTAMINA :
A) PROSTAGLANDINA E2
B) PROSTACICLINA
C) FACTOR ACTIVADOR DE LAS PLAQUETAS
D) INTERFERON
E) BRADICININA
45.-)FACTOR DEL COMPLEMENTO C5a SE ACTIVA
POR LA VIA ¿ ? PARA ACTIVAR NEUTROFILOS Y
MONICITOS PARA AUMENTAR LA LIBERACION DE
MAS MEDIADORES INFLAMATORIOS:
A) VIA CLASICA
B) VIA INTRINSECA
C) VIA LIPOOXIOGENASA
D) VIA CICLOOXIGENASA
E) VIA DE LECTINA
46.-) FACTOR DE CRECIMIEMTO RELACIONADO
CON LA HIPERTROFIA POR AUMENTO DE LA
CARGA DE TRABAJO:
A) VEGF
B) FGF
C) HGF
D) TGF BETA
E) TNF
47.-) LA ATROFIA SE PRODUCE POR:
A) MENOR SINTESIS DE FOSFOLIPIDOS
B) AUMENTO EN LA DEGRADACION LIPIDOS
C) MENOR SINTESIS DE CARBOHIDRATOS
D) AUMNETO EN LA SINTESIS DE COLESTEROL
E) AUMENTO EN LA DEGRADACION DE
PROTEINAS
48.-) EL RECEPTOR DE MUERTE MAS CONOCIDO
EN LA APOPTOSIS SE LLAMA:
A) Smac/DIABLO
B) IAP
C) FAS
D) Bcl 2
E) Bax
49.-) SON PROTEINAS PLASMATICAS DE LA FASE
AGUDA EN LA INFLAMACION:
A) CATALASA , DISMUTASA , PEROXIDASA
B) TRANSFERRINA, LACTOFERRINA, FERRITINA
C) ALFA GLICEROFOSFATO, ACETATO
D) PROTEINA C REACTIVA, FIBRINOGENO
PROTEINA AMILOIDE
E) DESMINA,V IMENTINA, QUERATINA
50.-) LOS CUERPOS DE MALLORY SON
ACUMULOS DE:
A) PROTEÍNAS EN LAS CÉLULAS TUBULARES
B) GRASA EN HEPATOCITOS
C) PROTEINAS HIALINO ALCOHOLICAS EN
HIGADO
D) COLESTEROL EN MACRÓFAGOS DE LA LAMINA
PROPIA DE LA VESÍCULA BILIAR
E) GLUCÓGENO EN LAS CÉLULAS EPITELIALES DE
LOS TUBULOS CONTORNEADOS DÍSTALES
1 B 17 A 33 A 49 D
2 B 18 C 34 C 50 C
3 D 19 B 35 B
4 D 20 A 36 A
5 D 21 D 37 A
6 A 22 A 38 A
7 A 23 A 39 A
8 D 24 C 40 A
9 B 25 E 41 B
10 B 26 C 42 D
11 B 27 D 43 C
12 C 28 C 44 C
13 C 29 B 45 C
14 D 30 B 46 D
15 C 31 A 47 E
16 C 32 A 48 C
TIPOS DE BIOPSIAS
● Método diagnóstico y
de tratamiento ● Método diagnóstico.
● Extirpación total de EXCISIONAL INCISIONAL ● Obtiene sólo una parte de la lesión
lesiones pequeñas
● Obtiene la muestra y
elimina la lesión.
PUNCH SACABOCADO

● Uso común en dermatología. ● Se utilizan pinzas de ángulo recto, puede tener dientes en la punta
CONO
● Cilindro hueco con bordes afilados (diámetros: 3, 5, 7, o ser redondas con bordes muy afilados.
9mm.)
● Pinzas de distintos tamaños.
● Asepsia, anestesia,con cuchilla dar vuelta, levantar con
● Displasias graves. Ca in situ. ● Las usamos en: cérvix o en biopsias transendoscópicas.
pinzas y cortar en la parte inferior → se obtiene cilindro.
● Prueba de acetoblanqueo.
Se sutura.
● Se reseca zona de transformación.
● Incisional:dx/ excisional:tx PUNCIÓN
● Generalmente en forma de cono truncado.
● Recomendado para mujeres menores de 30 años que aún quieren
tener hijos → Se mantiene función del cérvix para mantener ● Se usa con agujas gruesas y jeringas.
embarazo. ● El tejido que se obtiene es un cilindro, depende del grosor de la aguja.
● No cura, solo detiene el avance. ● Hay de 2 tipos
● Puede realizarse en pacientes con LIEG si se prevé que no ○ Dirigidas: Con agujas largas con la característica de que sus puntas están marcadas con material
regresará en mucho tiempo. TRANSENDOSCÓPICA
radio opaco para saber la ubicación de aguja. Lo realiza un radiólogo intervencionista. Ejemplos:
pulmón, riñón e hígado.
LEGRADO - CURETAJE ● Se usa para observar el órgano en cuestión en búsqueda de ○ No dirigidas: Se usa una aguja corta para lesiones localizadas, como tiroides, glándula mamaria,
anormalidades y así obtener una muestra de tejido. ganglios. Se inmoviliza la lesión, se pica, se aspira y sale.
● El procedimiento recibe el nombre del órgano o parte del cuerpo que se va ● El top de las biopsias por punción: Biopsia estereotáxica
● Cureta → cucharilla
a visualizar, recibir tratamiento o ambos.
Indicaciones: ○ Permite conocer el 3D el tejido
● Sangrados transvaginales→ Legrados hemostáticos. ○ Se hacen cortes transversales y longitudinales por medio de un programa especial y permite
DE HUSO
● Abortos sólo en huevo muerto retenido, aborto en evolución. obtener las coordenadas exactas de la zona determinada
● NO abortos provocados. ● Se hace en forma elíptica por estética pues sigue los pliegues de flexión. ○ Lo hace un radiólogo intervencionista o neurocirujano.
● Perimenopausicas con sangrados abundantes. ● La cicatriz queda recta ○ Se aplica en cerebro, pulmón o riñón.
● Mola hidatiforme ● El ángulo del huso debe ser menor de 30°
● Fragmentos de placenta retenidos. ○ Son muy caras.
● El huso sirve para nevos, dermatomiositis, lipomas, leiomiomas, fibromas.
● Sinusitis crónica: senos paranasales. ● Es importante hacer el uso 1 o 2 mm de diferencia cuando es benigna,
● Osteomielitis: hueso. cuando es maligna debemos dejar al menos de 3 a 4 mm Biopsias por punción que no necesitan jeringa/ percutáneas:

CUÑA 1. Asistida por vacío.


PIEZA QUIRÚRGICA

● Se realiza en forma de triángulo para coaptación ● Hechas con una pistola al vacío.
● Gastrectomía perfecta. ● Entra y succiona por vacío.
● Orquiectomía ● Se puede usar bisturí/electrobisturí ● Se utiliza para diagnostica Ca de próstata.
● Enucleación ● Utilizada en: higado, pulmón. ● Permite hacer estudio de inmunohistoquímica.
● Amputación
● Nefrectomía
● Colectomía.
TRANSOPERATORIA 2. Tru-cut
● Histerectomía.
● Se utiliza para hígado
● Congelación
● La muestra se mete al criostato (-30° o -40°c) ● Es de aguja gruesa y puede ser dirigida o no dirigida
● Única urgencia del patólogo. ● Es una aguja manual con un disparador con una aguja gruesa, cuya característica es
● 20 min para dar una respuesta. que tiene una hembra y un macho, el macho tiene una hendidura y de estructura
● Objetivo: influir en la decisión del cirujano. sólida.
● Entra el macho dentro de la hembra y se punciona, en ese momento sale el macho
cortando el tejido y se cierra a la hembra y corta el tejido, obteniendo casi un cilindro.

3. Mamotomo

● Se usa una aguja core, que es una aguja cortante gruesa.


● Es un aparato que con guía mastográfica identifica la lesión y ahí
mismo con una aguja central, entra toma la muestra y sale apoyada
con vacío.

SHAVE

● Para lesiones como pólipos, protrusiones de mucosas, nevos y


lesiones en la punta de la nariz.
● Debemos estar seguros de que la lesión es benigna.
ANATOMÍA PATOLÓGICA
Los seres vivos pueden ser estudiados a diferentes niveles. Como toda la materia del universo, están
compuestos de átomos organizados en diferentes niveles de complejidad. (UNAM, 2006)
Átomos y moléculas: Todos los seres vivos están formados por un conjunto de átomos y moléculas
que conforman una estructura material con un alto grado de organización y complejidad, en la que
existen mecanismos moleculares que regulan la comunicación e intercambio de materia y energía con
el ambiente e intervienen en la regulación de las funciones básicas de la vida como son la nutrición, la
reproducción, el crecimiento, la respiración, la excreción, etcétera.
Célula: Que es la unidad funcional, metabólica y reproductiva de los seres vivos.
A nivel celular pueden reconocerse dos niveles de organización claramente diferenciados.
1. Procariontes, que representan a los organismos formados por una célula carente de núcleo
celular.
2. Eucariontes, que constituyen organismos formados por una o más células que presentan núcleo
y diversas estructuras celulares.
Tisular: Los tejidos se caracterizan por ser un conjunto de células que llevan a cabo una tarea en
común y son formados a partir de conjuntos celulares.
Órganos: Los órganos llevan a cabo funciones específicas y tienen una forma reconocible, estas
unidades son fundamentales para la conformación de los sistemas. Los órganos se forman a partir de
grupos de tejidos.
Aparatos y sistemas: Los grupos de órganos forman aparatos o sistemas.
Existen alrededor de 12 aparatos y sistemas que son: cardiovascular, linfático, digestivo, endócrino,
tegumentario, muscular, nervioso, reproductivo, respiratorio, esquelético, urinario e inmune cada uno
con funciones importantes y relevantes para el cuerpo humano.

PATOLOGÍA: Se encarga del estudio de los cambios estructurales, bioquímicos y funcionales que
subyacen a la enfermedad en células, tejidos y órganos. Es la base científica de toda la medicina.
Se divide en
• General: signos y síntomas, p/e: tiene dolor abdominal pero no sé por qué
• Sistémica: ya hay un diagnóstico y el porqué de los síntomas. p/e: es por colitis
OBJETIVO: Explicar con detalle los motivos por los que los signos y síntomas son manifestados por
el paciente, ofreciendo una base racional para la atención clínica y el tratamiento a través de técnicas:
• Moleculares: PCR: identifica el material genético especifico del SARSCOV2
• Microbiológicas: cultivos p/e: bacterias, Escherichia coli, cándida, etc.
• Inmunológicas: inmunofluorescencia, ELISA (VIH, hepatitis, Chagas…)
• Morfológicas: Biopsias al microscopio
Anatomía patológica: Ve tejidos y biopsias a través del microscopio, generalmente su base es la
morfología
Patología clínica: se basa principalmente en técnicas moleculares, microbiológicas e inmunológicas.
“medicina de laboratorio”

1. Etiología: Causa MULTIFACTORIAL (factores genéticos, mutaciones, polimorfismos, factores


adquiridos, infecciones, nutrición, físicos y químicos).
2. Patogenia: Mecanismos de aparición
3. Cambios morfológicos y moleculares: Alteraciones bioquímicas y estructurales que provocan en las
células y órganos del cuerpo.
4. Alteraciones funcionales y manifestaciones clínicas: El resultado final de las alteraciones genéticas,
bioquímicas y estructurales de las células y tejidos. Signos y síntomas. Condicionan la evolución de la
enfermedad.
✓ PATOGENIA: La secuencia de episodios celulares, bioquímicos y moleculares que sucede a la
exposición de las células o tejidos a un agente lesional
✓ CAMBIOS MORFOLÓGICOS: Se correlacionan con las alteraciones estructurales en células o
tejidos características de una enfermedad o diagnósticas de un determinado proceso etiológico.
Hablamos solamente de una característica, podemos usar aditamentos como el microscopio
para ver cómo han cambiado las células en su estructura. P/E: Leucemia antes se detectaba
por la morfología por las características diferentes de las células, a partir de la biología molecular
o proteómica fueron mejorando los diagnósticos para que fueran certeros.
✓ ALTERACIONES FUNCIONALES Y MANIFESTACIONES CLÍNICAS: Las consecuencias
finales de los cambios genéticos, bioquímicos y estructurales en células y tejidos condicionan
su progresión (evolución clínica y pronostico)
Correlación clínico-patológica son fundamentales en el estudio de la enfermedad
< Veo los síntomas y la correlaciono con cierta enfermedad porque ya sé que esos son sus síntomas>
✓ HOMEOSTASIS: La célula puede adaptarse a las necesidades fisiológicas manteniendo un
estado de equilibrio. La célula está en una situación normal o fisiológica. Ante estrés u otra
situación, o se adapta o se lesiona. Las hormonas tiroideas entran en concentración para que
nuestro cuerpo se pueda adaptar a la temperatura de un clima
Médico que tiene una formación especial para identificar las enfermedades mediante el estudio de las
células o los tejidos con un microscopio. (Instituto Nacional del Cáncer de los Institutos Nacionales de
la Salud de EE. UU., s/f)
El patólogo o anatomopatólogo es el médico que se dedica al diagnóstico de enfermedades basándose
en la observación morfológica de lesiones a través de la microscopía y utilizando diversos tipos de
tinciones. El patólogo es el médico que se dedica al diagnóstico basado en la observación morfológica
de lesiones, principalmente a través de la microscopía de luz, utilizando diferentes tinciones
histoquímicas, inmunohistoquímcas, pruebas citogenéticas y genética molecular. (Gregorini, 2017)

1. AUTOPSIA: Es un procedimiento ordenado y minucioso, que incluye la revisión tanto el aspecto


externo como la revisión de los órganos internos.
a. Médico- clínica: autopsias de pacientes que fallecen por “causas naturales” o por una enfermedad.
La autopsia confirma o, en su caso, determina el padecimiento fundamental, las alteraciones
secundarias al mismo y aquellas otras derivadas del tratamiento, describe los hallazgos accesorios
asintomáticos, silentes clínicamente, e investiga la causa de muerte
i. Examen externo: Edad, peso, estatura del cadáver se tienen que conocer de antemano puesto
que son datos orientadores sobre la causa de la muerte. Después se reflejara la constitución
y el estado de nutrición, se deberá anotar cualquier cambio de coloración, existencia de
patología cutánea, cicatrices (recientes o antiguas) o cualquier otro signo que pudiera dirigir
la investigación hacia una enfermedad determinada.
ii. Examen interno: Se sigue un proceso similar a la autopsia médico-legal. Únicamente señalar
que si se detecta durante el proceso autopsico algún signo que pudiera tener un significado
no natural debe suspenderse inmediatamente la autopsia y poner tal hecho en conocimiento
de la autoridad judicial. Debe actuarse del mismo modo si el hallazgo tiene lugar durante la
práctica de pruebas complementarias.
b. Médico- legal: son las exploraciones autópsicas que, en colaboración con los médicos forenses, se
efectúen a demanda de éstos. Son, en general, autopsias parciales, en las que la autopsia clínica, el
número y volumen de los órganos a estudiar ha sido seleccionado por el médico forense, para que lo
efectuemos con nuestra propia tecnología, como examen auxiliar.
2. BIOPSIA: Procedimiento por medio del cual se obtiene un fragmento de tejido de un ser vivo,
con el objetivo de someterlo a un estudio microscópico y establecer un diagnóstico. Sirve para
hacer diagnóstico, para indicar tratamiento, para definir pronóstico, evolución y extensión de
una lesión y existen varios tipos:
a. Incisional: En esta técnica se toma sólo un fragmento de la lesión.
b. Excisional: Para esta técnica se toma toda la lesión.
c. Aspiración/ Punción: Esta se realiza mediante una aguja o trocar y se aspira tejido de la
lesión. Aquí entra la Biopsia por Aspiración por Aguja Fina (BAAF).
d. Congelación o transoperatoria: Para esta técnica se toma un fragmento de la lesión
durante la operación y permite un diagnóstico en menos de media hora con resultado
positivo o negativo o con la espera de cortes de parafina si se tiene duda del diagnóstico.
Se utiliza el criostato mediante fijación física de la muestra por congelación.
e. Postoperatoria: Es mediante estudio de lesión por parafina.
f. Sacabocados: Con pinzas especiales.
g. Punch: Mediante lápiz especial.

3. CITOLOGÍA EXFOLIATIVA:
Procedimiento que se encarga de exfoliar las células de manera mecánica.
Consiste en recolectar las células que se obtienen a partir de un raspado manual (exfoliación
mecánica) o de un desprendimiento espontáneo del cuerpo (Exfoliación espontánea)
Indicaciones:
Mujeres de 25-64 años que nunca se han realizado el estudio o que tengan factores de riesgo
como:
• Inicio de vida sexual antes de los 18
• Múltiples parejas sexuales
• Infección por VPH
• Enfermedades de transmisión sexual
• Tabaquismo (factor predisponente para cáncer de células escamosas)
• Desnutrición
• Deficiencia de antioxidantes
• Inmunodeficiencias
• Uso de anticonceptivos orales combinados a largo plazo
Recomendaciones
1. Realizar la toma 2 semanas después del primer día del ultimo menstrual (entre los días 10-
18)
2. Realizar la toma antes de cualquier otra exploración cervical o vaginal
3. Evitar hacer el examen durante la menstruación o antes de 3 días de finalizado el ultimo
periodo menstrual. Si hay sangrado, anormal por menstruación, debe diferirse la toma de la
citología, ya que la sangre dificulta el diagnóstico
4. Evitar 48 horas previas a la toma: ducha vaginal, uso de tampones, espumas, gelatinas u
otras cremas o medicamento vaginales, no tener sexo durante 48 horas previas a la muestra
5. Evitar el uso de lubricante para no contaminar la muestra
6. Si existe flujo vaginal que cubra el cérvix, removerlo cuidadosamente con una torunda
húmeda de solución fisiológica. Si la cervicitis o vaginitis son intensas debe darse tratamiento y
diferir la toma.
Material
• Guantes y googlees
• Espátula de Airé. Cepillo endocervical, citobrush.
• Laminilla rotulada
• Fijador, citospray (deja las células intactas)
• Espejo vaginal o pato
• Para lubricar: solución salina
Técnica
• Colocar espejo vaginal de lado, verificar que el tornillo no este atorado, giro de 45 ° y abrirlo,
visualizar el cérvix, con el tornillo fijar el espejo vaginal.
• Con la espátula de airé tiene 2 partes: la parte que parece hueso exocérvix (3 vueltas hacia la
derecha) y la larga que parece manita endocérvix. (metes la punta con cuidado y hacia el lado
contrario, hacia el lado izquierdo 3 vueltas) Con el cepillo endocervical se introduce ¾ partes y
gira 15-20° sacas y extiendes. Con el citobrush la parte larga endo y la corta exo, con una sola
toma se toma la muestra, se introduce y dan 3 vueltas y el extendido se puede dividir la laminilla
a la mitad lado dcho. endo e izquierdo exo, Con el citobrush un solo barrido largo.
• Extender en laminilla
• Fijación:
a) Alcohol etílico 96%
b) Citospray (alcohol base y una sustancia serosa)
c) Alcoholes: Metanol 100%, Propanolol 80%, Isopropano 80%
d) Otros aerosoles sin laca
• Desatornillas el espejo, volteas el espejo y lo vas cerrando poco a poco y sacas como se metió
Si no hay cérvix, tomar de paredes vaginales cercanas al cérvix.
Hay que ver si dentro de los factores de riesgo hay inmunocompromiso: VIH, artritis, lupus etc.
Hay que hacerle su prueba ANUAL
Si no, hay que preguntarse si tiene +69 años, ya que tiene menos riesgo de Cáncer
Cervicouterino, y preguntar si en los últimos 10 años se ha hecho la prueba, y hay que juntar
por lo menos 3 pruebas negativas
Si no, hay que preguntar si esta embarazada o vida sexual activa e igual juntar 3 pruebas
consecutivas negativas y de ahí cada 2-3 años
Aunque sea histerectomía PARCIAL aun así se le hace el estudio y los mismos 3 seguidos
Si una px tuvo previamente CaCu o una metaplasia o cáncer insitu.
Dentro del conducto cervical vamos a encontrar tejido columnar o cilíndrico y en el exocérvix es
escamoso, con la edad el tejido interno se empieza a exponer hacia afuera hacia la cavidad vaginal
Con el pH de la vagina hay un cambio adaptativo: METAPLASIA, ese tejido va a ser metaplasico
porque no puede aguantar el cambio de PH.
Zona de transformación: Importantísimo porque ahí van a pasar casi todas las neoplasias, si la
identificamos podemos tomar una muestra adecuada. Región o tejido metaplásica entre la unión
escamo columnar (UEC) nueva y antigua. Células cilíndricas, todas juntitas acomodadas. nucleos
parabasales.
Conforme están mas cercanas a la membrana basal se van haciendo más redondas
Todas las células de la membrana basal: células basales ( son redondas, conforme se van haciendo
mas superficiales cambian su conformación morfológica que cuando llegan hacia la superficie las
células son)
Estrato superficial. Células aplanadas con núcleo alargado
Basales: muy redondas núcleo 1 a 1
Parabasales: cambian la conformación y 1 a 2,3,4 citoplasma
Intemediarias: forma poligonal (De aquí en adelante)
Superficiales: poligonal
Escamas: poligonal
Células escamosas superficiales: Son las que mas deberíamos encontrar, eosinofílico (rosa,
morada, naranja) núcleo picnotico casi siempre centrado
Células intermedias: Caracterisitcas similares, solo cambia su citoplasma cianofilico (azulado-
grisáceo)
ENDOCERVIX: células columnares o cilíndricas, núcleo excéntrico, células alargaditas, si las vemos
de frente vemos esa característica parecido a panal de abejas que es normal, cianofilicas
4. NECROPSIA: Es el estudio de un cadáver mediante la observación cuidadosa, incluyendo la
apertura de sus cavidades y el examen de todos sus órganos, el protocolo de autopsia con
todos los datos de la misma y la descripción de los hallazgos efectuados en todos os órganos
y cavidades. Tiene como objetivo determinar la causa de muerte, rectificar los certificados de
defunción, efectuar estudios estadísticos, elaborar políticas de salud, docencia o fines jurídicos.
Existen dos tipos:
a. Clínica que es por toda muerte natural como infarto, neoplasias, neumonía,
etc.
b. Médico legal o forense como muertes violentas, accidentes, caídas,
quemados, ahogados, etc.

Principales serotipos importantes


**16,18 son los mas importantes relacionados con neoplasia intraepitelial
Los otros también pueden causar cancer pero si haces una citología exfoliativa y ves VPH y lo mandas
a serotificar y hay 16 y 18 PELIGRO muy cancerígenos
TRANSMISIÓN
1. Contacto sexual
2. Fomites
3. Transmisión vertical
4. Autoinoculación
COLPOSCOPÍA
• Visualización de paredes vaginales y fondo de saco
• Aumento de 60 a 40 veces
• Utilización de acido acético del 5% al 10%
• No es específica para VPH
INMUNOHISTOQUÍMICA
Técnica que permite localizar células en los tejidos mediante IgG.

El cáncer cervicouterino (CaCu) es uno de los principales problemas de salud pública en el mundo,
acontece en 10% de todos los cánceres, es la tercera neoplasia más diagnosticada.
El cérvix normal se compone de diferentes tipos de células epiteliales, el canal cervical medio y el
cérvix superior están compuestos por epitelio columnar secretor, originado embriológicamente de la
invaginación de los conductos müllerianos. Existe un pequeño potencial neoplásico para este tipo de
células. La vagina y el ectocérvix distal están compuestos de epitelio escamoso, estas células
escamosas reemplazan las células columnares mullerianas originales cuando se forma el canal
uterovaginal. Existe un pequeño potencial neoplásico para este tipo celular.
La unión escamo-columnar (UEC) es el punto donde las células escamosas y columnares se
encuentran. Esta unión se encuentra típicamente entre el ectocérvix central y el canal cervical inferior,
pero la localización varía a lo largo de la vida de la mujer, por el desarrollo fetal y la menopausia. La
unión escamocolumnar original es una zona de transformación. La transformación normal de un tipo
celular maduro en otro es llamada metaplasia. Cuando la metaplasia ocurre, existe siempre un
potencial neoplásico.
En las mujeres en edad reproductiva, la UEC se mueve hacia afuera por influencia hormonal. El pH
ácido vaginal provoca irritación mecánica que induce el proceso de metaplasia escamosa, resultando
una nueva UEC. Esta área entre la original y la nueva UEC es referida como la zona de transición. Las
células escamosas inmaduras metaplásicas en esta zona de transformación son teóricamente las más
vulnerables a neoplasia. La mayoría de los carcinomas de células escamosas se originan en la UEC.
En mujeres jóvenes la UEC es localizada fuera del orificio cervical externo y el tumor tiende a crecer
hacia afuera (crecimiento exofítico), en contraste, en pacientes de mayor edad, la UEC es localizada
en el canal cervical, por lo que el cáncer cervical tiende a crecer hacia adentro, a lo largo del canal
cervical (crecimiento endofítico). Las células de reserva en la unión escamocolumnar han sido
vigiladas con interés como origen del adecocarcinoma cervical. Sin embargo, la mayoría de los
adenocarcinomas cervicales surgen en la unión escamocolumnar.
La coilocitosis ha sido descrita en muestras de Papanicolaou por décadas y es reconocida como
muestra de displasia leve. En 1970 Meisels y Fortin descubrieron que el VPH era el origen de atipia
coilocítica. El VPH es asociado a un alto grado de lesiones cervicales y es considerado el agente
causal en el desarrollo de cáncer cervicouterino. Esta relación ha sido descrita por Bosch y cols. y es
ahora bien aceptado el modelo para entender la oncogénesis mediada por virus.
FACTOR DE RIESGO: VPH, antecedentes.
ADAPTACIONES DE CÉLULA: Son las respuestas funcionales y estructurales reversibles ante
situaciones de estrés fisiológico y a ciertos estímulos patológicos, frente a los cuales se desarrollan
estados nuevos, aunque de equilibrio alterado, en los que la célula consigue sobrevivir y mantener su
función, puede consistir en aumento de tamaño de las células (hipertrofia), y de la actividad funcional,
incremento de su número (hiperplasia), disminución de su tamaño y actividad metabólica (atrofia), o
cambio en el fenotipo de dichas células (metaplasia). La lesión celular es reversible hasta cierto punto,
cuando se vuelve irreversible produce la muerte celular.
La adaptación es REVERSIBLE. Adaptación = reversibilidad
TIPOS:
• Hipertrofia: Aumento del tamaño de las células y de su actividad funcional. (por el estés fisiológico
se adaptan y se hacen más grandes) p/e: ponerse mamado.
La hipertrofia puede ser fisiológica o patológica, y se debe a un aumento de las exigencias funcionales
o la estimulación por hormonas o factores de crecimiento. El estímulo más habitual para la hipertrofia
muscular es el aumento del esfuerzo.
El crecimiento masivo del útero de forma fisiológica durante el embarazo es un buen ejemplo de
aumento del tamaño de un órgano inducido por las hormonas y que se debe fundamentalmente a la
hipertrofia de las fibras musculares.
Las dos vías bioquímicas más importantes implicadas en la hipertrofia muscular son la fosfoinositol 3-
cinasa/Akt (que se considera la más importante en condiciones fisiológicas, como la hipertrofia por
ejercicio) y las señales distales de los receptores acoplados a la proteína G (que se inducen por
muchos factores de crecimiento y agentes vasoactivos, que se consideran más importantes en la
hipertrofia patológica). La hipertrofia se puede asociar también a un cambio de las proteínas
contráctiles de una forma adulta a otras fetales o neonatales

• Hiperplasia: incremento del número de células (con el propósito de tratar de compensar algo, y
también aumenta su función, aunque no necesariamente) p/e: hiperplasia prostática, la próstata crece
y empieza a obstruir la luz de la uretra y disminuye el chorro de la orina.
Puede ser fisiológica o patológica.
• Atrofia: Disminución del tamaño celular y de su actividad metabólica. p/e: hemiplejia, alguna
discapacidad con uso de sillas de ruedas, uso de yeso, férula etc.
La atrofia fisiológica es frecuente durante el desarrollo normal. Algunas estructuras embrionarias, como
el notocordio o el conducto tirogloso, experimentan atrofia durante el desarrollo fetal. El tamaño del
útero disminuye al poco tiempo del parto y esto es una forma de atrofia fisiológica.
La atrofia patológica depende de la causa de base y puede ser local o generalizada. Las causas
frecuentes de atrofia son las siguientes:
• Reducción de la carga de trabajo (atrofia por desuso).
• Pérdida de la inervación (atrofia por denervación).
• Reducción de la irrigación
• Nutrición inadecuada.
• Pérdida de la estimulación endocrina
• Presión.
La atrofia se produce por una menor síntesis de proteínas con aumento de su degradación en las
células. La síntesis de proteína se reduce por la menor actividad metabólica. La degradación de las
proteínas celulares tiene lugar principalmente a través de la vía de la ubicuitinaproteasoma. La
deficiencia de nutrientes y el desuso activan a las ubicuitina ligasas, que unen el pequeño péptido
ubicuitina con las proteínas celulares, de forma que estas quedan marcadas para su degradación por
los proteasoma. En muchas situaciones, la atrofia se asocia a un aumento de la autofagia, con el
consiguiente aumento del número de vacuolas autofágicas.

• Metaplasia: Cambio en el fenotipo celular. (características externas o físicas, estructurales que va a


tener una célula) Hablamos de un riesgo inminente de que DESARROLLEN cáncer, pero NO
SIGNIFICA CANCER. La metaplasia es REVERSIBLE, cuando hay cambio en el material genético ya
es irreversible (cáncer o neoplasia) TODO CANCER Y NEOPLASIA viene de una METAPLASIA.
Es un cambio reversible en el que una célula diferenciada (epitelial o mesenquimal) se sustituye por
otro tipo celular. Puede ser una sustitución adaptativa de las células que son sensibles al estrés por
otros tipos celulares que resisten mejor este entorno adverso.
La metaplasia epitelial más frecuente es la cilíndrica a escamosa que se describe en la vía respiratoria
en respuesta a la irritación crónica. En los fumadores habituales, el epitelio cilíndrico ciliado normal de
la tráquea y los bronquios se sustituye por un epitelio escamoso estratificado. Una deficiencia de
vitamina A (ácido retinoico) induce la metaplasia escamosa del epitelio respiratorio.
La metaplasia de tejido conjuntivo es la formación de cartílago, hueso o tejido adiposo (tejidos
mesenquimales) en zonas que normalmente no contienen estos elementos. Por ejemplo, la formación
de hueso dentro del músculo se llama miositis osificante y se describe en algunos casos tras una
hemorragia intramuscular. Este tipo de metaplasia se puede explicar peor como una respuesta
adaptativa y puede ser secundaria a una lesión celular o tisular.
La metaplasia es resultado de una reprogramación de las células madre que existen en los tejidos
normales o de células mesenquimales indiferenciadas presentes en el tejido conjuntivo. En el cambio
metaplásico, estas células precursoras se diferencian por una nueva vía. La diferenciación de las
células madre hacia una estirpe determinada se debe a señales generadas por las citocinas, los
factores de crecimiento y los componentes de la matriz extracelular en el entorno celular. Estos
estímulos externos inducen la expresión de los genes que dirigen a la célula hacia una vía de
diferenciación específica. En situaciones de deficiencia o exceso de vitamina A, se sabe que el ácido
retinoico regula la transcripción génica de forma directa a través de los receptores de retinoides
nucleares que pueden influir en la diferenciación de los progenitores derivados de las células madre
de los tejidos. Se ignora cómo otros estímulos externos inducen la metaplasia, pero está claro que
también deben modificar de alguna forma la actividad de los factores de transcripción que regulan la
diferenciación.
La lesión celular se produce cuando las células se someten a un estrés tan importante que no pueden
adaptarse ya, cuando se exponen a agentes con capacidad lesiva inherente o cuando presentan
alteraciones intrínsecas. Las lesiones pueden progresar a través de un estadio reversible y culminar
en la muerte celular.
Lesión celular reversible. En las fases precoces o formas leves de lesión, los cambios funcionales y
morfológicos son reversibles si se elimina el estímulo lesivo. Las características de las lesiones
reversibles son una reducción de la fosforilación oxidativa, con la consiguiente depleción de las
reservas energéticas celulares en forma de adenosina trifosfato (ATP) y el edema celular secundario
a cambios en las concentraciones de iones y la entrada de agua. Además, se pueden encontrar
alteraciones en diversos orgánulos intracelulares, como las mitocondrias o el citoesqueleto.

Es la consecuencia final de una lesión celular progresiva, es uno de los acontecimientos más
importantes en la evolución de la enfermedad en cualquier tejido u órgano. Cuando persiste la
agresión, la lesión llega a ser irreversible y la célula no se puede recuperar y muere, se divide en:
• Necrosis: se relaciona con un proceso patológico, consecuencia de la desnaturalización de las
proteínas intracelulares y la digestión enzimática de la célula con daños mortales. Relacionado con
inflamación, y por consecuencia también con edema.
• Apoptosis: se relaciona con un proceso fisiológico, una célula muere por la activación de un
programa de “suicidio” interno, lo que implica un des ensamblaje orquestado de componentes
celulares: se produce una afectación mínima del tejido adyacente y una inflamación mínima, si es que
la hay. Se observan condensación y fragmentación de la cromatina. Afectación mínima del tejido. No
va a romper su membrana celular porque no hay desnaturalización, se va a mantener intacta, no tiene
un proceso inflamatorio por lo tanto no va a haber edema, Si se desnaturalizan proteínas la membrana
plasmática se rompe.
Es un mecanismo de muerte celular inducida por un proceso de suicidio programado y regulado en el
que las células que van a morir activan enzimas intrínsecas que degradan al ADN nuclear y las
proteínas del núcleo y citoplasma. Células apoptósicas se disgregan en cuerpos apoptósicos que
contienen porciones de citoplasma y núcleo.
La célula muerta y sus fragmentos son ingeridos antes de que su contenido sea extravasado por lo
que NO HAY inflamación.
✓ Apoptosis fisiológica: en embriogenia, involución de tejidos dependientes de hormonas tras
suspensión hormonal (ciclo menstrual), homeostasia de proliferación celular, eliminación de
linfocitos nocivos y muerte de células cuyo propósito ya se cumplió.
✓ Apoptosis patológica: afectación del ADN, acumulación de proteínas mal plegadas, muerte
celular por infecciones.
Mecanismo de apoptosis: apoptosis consecuencia de activación de caspasas.
a. Fase inicial: caspasas se convierten en catalíticamente activas.
b. Fase de ejecución: Caspasas estimulan la degradación de componentes celulares
especiales.
Vías de activación: convergen en la activación de las caspasas que media la fase final de la apoptosis.
i. Vía intrínseca (mitocondrial) de la apoptosis: principal mecanismo, consecuencia
del aumento de la permeabilidad de la membrana externa mitocondrial con la
liberación de moléculas inductoras de muerte (citocromo C) y controladas por la
familia de proteínas BCL2, BCLX y MCL1 (antiapoptósicas), BAX y BAK
(proapoptósicas), BAD, BIM, BID, Puma y Noxa (sensoras del estrés y daño
celular)
Una vez liberado al citosol el citocromo c se une a APAF1 que forma el
apoptosoma y desencadena la activación de las caspasas. Las IAP bloquean la
actividad de las caspasas y mantienen a las células con vida.
ii. Vía extrínseca (iniciada por receptores de muerte): es directa, FAS (CD96) y TNF
1 rcpt activan a proteínas adaptadoras y caspasas iniciadoras.
Comienza con la implicación de receptores de muerte de la membrana plasmática
en varias células que activan la cascada inflamatoria.
Esta vía puede ser inhibida por FLIP que se une a procaspasa 8.
Ejemplos de apoptosis:
1. Carencia de factores de crecimiento: apoptosis activada por vía mitocondrial/intrínseca por
disminución de BCL2 y BCL-XL
2. Daño del ADN: Con la radiación p53 induce apoptosis.
3. Mal plegamiento de proteínas: que se acumulan en el RE.
4. Apoptosis inducida por la familia de rcpts de TNF: implicada en la eliminación de linfocitos.
5. Apoptosis mediada por linfocitos T citotóxicos.
Trastornos asociados a la desregulación de la apoptosis
1. Trastornos asociados a apoptosis defectuosa e incremento de la supervivencia celular.
2. Trastornos asociados al aumento de la apoptosis y exceso de muerte celular: caracterizados
por pérdida de células.
a. Enfermedades neurodegenerativas.
b. Lesión isquémica.
c. Infecciones víricas.
• Necroptosis: Fusión de apoptosis y necrosis.
Hay pérdida del ATP, edema, generación de ERO, liberación de
enzimas lisosómicas y rotura de la membrana plasmática.
Se inicia por la unión a su receptor de un ligando, interviene las cinasas
asociadas a receptores RIP1 y RIP3. Puede ir acompañada de
liberación de IL-1, la caspasa 1 y 11 son responsables de la muerte
celular que indica pérdida de integridad de las membranas celulares y
liberación de mediadores inflamatorios.

• Piroptosis: En células infectadas por microbios y conlleva a la


activación de la caspasa 1 a fin de generar IL1 biológicamente activa.
Se asocia en esteatohepatitis, pancreatitis aguda, lesión por
reperfusión y enfermedades neurodegenerativas.

• AUTOFAGIA: Es una respuesta adaptativa de las células a la


privación de nutrientes; es, en esencia, un autocanibalismo destinado
a mantener la viabilidad. Puede culminar también en la muerte celular
y es fisiológica y patológica. Puede ser de tres tipos:

1. Mediada por chaperonas: traslocación directa a través de la membrana lisosómica.


2. Microautofagia: invaginación de la membrana lisosómica para la liberación del material.
3. Macroautofagia: forma principal que implica secuestro y transporte de porciones de citosol en
una vacuola de doble membrana.
Tiene varias etapas:
1. Formación de la membrana de aislamiento (fagóforo) y nucleación dentro de ella.
2. Elongación de la vesícula. Requiere sistemas de ubicutina.
3. Maduración del autofagosoma.
4. Fusión del autofagosoma con lisosomas.
5. Degradación del contenido.
Genes: Atg necesarios para la formación del autofagosoma.
Marcador de autofagia: LC3
Autofagia en enfermedades:
1. Cáncer.
2. Trastornos neurodegenerativos.
3. Enfermedades infecciosas.
4. Enfermedades inflamatorias intestinales.
➢ Hipoxia (primer parámetro de lesión celular) por:
• Isquemia (pérdida de irrigación sanguínea) p/e: diabéticos descontrolados: tienen disminución
de irrigación en extremidades y pueden complicarse, como en el pie diabético. Compresión de
un vaso sanguíneo que disminuye su luz y provocará hipoxia.
• Oxigenación inadecuada (Insuficiencia cardiorrespiratoria) p/e: Los órganos principales que se
encargan de la oxigencion, primero pulmones (enfermedades relacionadas con afectación del
intercambio gaseoso podrán provocar esta lesion celular, como trombosis, EPOC, asma,
COVID19, tuberculosis…) y después corazón (no llega sangre a los pulmones porque la bomba
está fallando, como valvulopatía, hipertrofia ventricular izquierda, insuficiencia cardiaca,
bloqueos…)
• Pérdida de la capacidad de transporte de oxígeno (anemia, intoxicacipon por monóxido de
carbono) p/e: están bien pulmones y corazón, pero mal las células encargadas del transporte
del oxígeno (cuando disminuyen estas células, disminuye la Hb y produce anemia, intoxicación
por algunos gases como personas que ocupan fogeros, quemadores… encerrados en espacios
cerrados).

➢ Elementos físicos
1. Traumatismo
2. Calor: los extremos siempre van a ser dañinos.
3. Frío: los extremos siempre van a ser dañinos.
4. Radiación: no sólo se da por bombas atómicas, también por estudios de gabinete
5. Descarga eléctrica:

➢ Sustancias químicas y fármacos (sustancias para fumigar, solventes que usan para drogarse,
medicamentos recetados que pueden ser polifarmacia, alcohol)

➢ Reacciones inmunitarias: Debidas por inmunidad como artritis, esclerodermia,

➢ Trastornos genéticos: Deben ser identificados desde el nacimiento (tamiz para identificar las más
frecuentes, o manifestaciones clínicas)

➢ Desequilibrios nutricionales: Dos extremos, desnutrición, caquexia y obesidad.


Las lesiones celulares pueden ser
1. Reversibles
a) Edema celular: perdida de la homeostasis ionica e hídrica (no función de las bombas
de iones de la membrana plasmática que dependen de la energía)
b) Cambio graso: vacuolas lipídicas citoplasmáticas
2. Irreversibles (Necrosis tisular)
a) Necrosis coagulativa: muy característica de la hipoxia
• Es el patrón más frecuente
• Desnaturalización protéica
• Conservación de la célula y de la estructura tisular
• Característico de la muerte por hipoxia en todos los tejidos, excepto el encéfalo
- Heterolisis (leucocitos)
- Autolisis
b) Licuefactiva
• Predomina la autolisis y heterolisis sobre la desnaturalización proteica
• Zona necrosada es blanda y llena de liquido (no siempre edematoso, puede ser
por infección y acumulación de pus).
❖ Bacteriana
• Se puede presentar en SNC
c) Necrosis gangrenosa:
• Necrosis coagulativa apicada a una extremidad isquémica
• Estructura tisular (el tejido necrosado mantiene la estructura tisular) se va a
mantener (común a la coagulativa)
• Cuando se llega a asociar con una infección bacteriana es: Gangrena húmeda
• Ejemplo: pie del diabético: inicia por necrosis por hipoxia (por isquemia), y si
después de eso se infecta (pierden sensibilidad y caminan y no se dan cuenta si
se pincharon o algo) y es una GANGRENA HUMEDA
El tejido rojo es tejido de granulación, significa que la herida se está reparando. Lo
que parece nata es tejido necrótico.La necrosis no necesariamente es negra.
d) Necrosis caseosa: Característica de las lesiones tuberculosas
• Material blanco, friable <<aspecto de queso>>
• Friable = muy delicado, si yo toco muy probablemente el tejido se desprenderá
muy fácilmente
e) Necrosis grasa
• Se liberan acidos frasos a partir de los triglicéridos
• Forman complejos con calcio (se ven blancecinos puntos)
• Crean jabones
• Areas blancas calcáreas (saponificación de la grasa)
• p/e: pancreatitis
f) Necrosis fibrinoide
• Patrón patológico debido a un deposito de antígenos y anticuerpos en los vasos
sanguíneos
• Se tienen que identificar antigenos y anticuerpos

TIPOS DE LESIÓN (NATURALEZA)


• DURACION
• GRAVEDAD
Mecanismos intracelulares de lesión celular:
1. DISMINUCIÓN DE ATP: Lesiones pueden producirse por una causa muy importante: hipoxia,
que puede ser secundaria a la isquemia, las mitocondrias se afectan, disminuye fosfo oxidativa,
disminuye atp disminuye bomba se sodio. Si hay hipoxia disminuye el ATP y se activan 3
procesos importantes
1. Glucólisis anaeróbica
2. Se alteran las bombas de la membrana celular
3. Producción de proteínas se ve alterada

2. DAÑO MITOCONDRIAL: Crea un canal de alta conductancia en la membrana de la mitocondria


(poro de transición de la permeabilidad mitocondrial) que induce la insuficiencia de la
fosforilación oxidativa y agotamiento progresivo de ATP que culmina en la necrosis celular.
❖ CICLOSPORINA: medicamento para algunos padecimientos como
inmunosupresor, se puede utilizar en artritis reumatoide en donde hay
autoinmunidad a Ac que van a destruir a las mismas células del paciente al
reconocerlas como extrañas. La ciclosporina bloquea que se forme el poro de
transición de la permeabilidad mitocondrial. Entonces evita que la célula entre
en necrosis. Se utiliza para cualquier proceso que se quiera bloquear la
necrosis y la donde haya procesos de destrucción celular propia.

3. FLUJO DE ENTRADA DE CA Y PÉRDIDA DE LA HEMOSTASIA DE CA: La elevación de ca


induce lesión por varios mecanismos (agotamiento de ATP)
Se pierde la homeostasis del calcio, si se altera la membrana. Entra mucho sodio y agua t
también mucho calcio. El calcio siempre es bueno mientras haya homeostasis.
El calcio fuera de homeostasis puede ser lesivo, aumenta en el citoplasma, activa enzimas (CA
= ACtivación de muchas enzimas celulares:
-Fosfolipasas: degradan a fosfolípidos (membrana celular)
- Proteasas: rompe a proteínas ( el citoesqueleto de la membrana celular tiene proteínas
y se van a derrumbar o eliminar de la membrana celular )
- Endonucleasas: alteran el material genpetico, el núcleo
- ATPasas: dañan aún más el ATP (si de por si ya estaba bajo, el poquito que sobra lo
empieza a eliminar más las enzimas )
A mayor cantidad de lesión celular, se forman los poros y por lo tanto la célula se daña

4. ESTRÉS OXIDATIVO: Los radicales libres son especies químicas que tienen un electrón
desapareado en sus orbitales externos, son altamente reactivos y atacan a moléculas
adyacentes.
a. ERO (Especies Reactivas de Oxígeno): radicales libres derivados de O2, producidas en
las células durante la respiración mitocondrial (neutrófilos y macrófagos) y degradadas
por sistemas de defensa. El exceso de estas causa Estrés oxidativo.
b. Generación de radicales libres:
i. Reacciones de oxido-reducción.
ii. Absorción de energía radiante.
iii. Leucocitos los generan en la inflamación.
iv. Metabolismo enzimático de sustancias químicas exógenas o fármacos.
v. Metales de transición
vi. NO
c. Eliminación de radicales libres:
i. Antioxidantes bloquean formación de radicales (vitaminas E, A, C)
ii. Enzimas:
1. Catalasa que descompone el H2O2
2. Superoxido dismutasa que convierte el O2 en H2O2
3. Glutatión peroxidasa que cataliza la descomposición de los radicales libres.
d. Efectos patológicos de radicales
i. Peroxidación lipídica en membranas.
ii. Modificación oxidativa de proteínas.
iii. Lesiones de ADN.
5. DEFECTOS EN LA PERMEABILIDAD DE MEMBRANAS
o Mecanismos de daño de las membranas:
▪ Especies reactivas de O2
▪ Síntesis de fosfolípidos reducida
▪ Incremento de la degradación de fosfolípidos
▪ Anomalías citoesqueléticas
o Consecuencias del daño
▪ Daño en membranas mitocondriales: se abren poros y disminuye ATP y liberación de
proteínas que inducen apoptosis.
▪ Daño en membrana plasmática: Pérdida del equilibrio osmótico.
▪ Lesión de membranas lisosómicas: digestión de ADN y ARN

6. DAÑO DE ADN Y PROTEÍNAS: Casi siempre induce apoptosis.


Isquemia: lesión celular más común, consecuencia de la hipoxia inducida por la reducción del flujo
sanguíneo (obstrucción arterial mecánica) o por la reducción del drenaje venoso. Tiende a provocar
una lesión celular y tisular rápida y grave.
Disminución de ATP hace que falle la bomba Na, que conduce a flujo de salida de K, entrada de Na y
H2O y por consiguiente un edema celular.
Si la hipoxia continua el citoesqueleto se dispersa y forma vesículas en la superficie celular y generar
muerte celular.
P/e: en el miocardio se produce entrada masiva de Ca en la zona isquémica y se produce la muerte
celular por necrosis.

El restablecimiento del flujo sanguíneo en los tejidos isquémicos puede favorecer la recuperación de
las células si están lesionadas de forma reversible, pero también puede exacerbar la lesión e inducir
muerte celular.
La lesión por reperfusión se produce por los siguientes mecanismos:
1. Estrés oxidativo: durante reoxigenación se forman ERO y ERN.
2. Sobrecarga de Ca intracelular: favorece el poro de transición y con esto la disminución del ATP.
3. Inflamación.
4. Activación del sistema de complemento.
Por toxicidad farmacológica, tiene dos mecanismos generales:
1. Toxicidad directa
2. Conversión de metabolitos tóxicos: modificación por oxidasas de funci´´on mixta del citocromo
P450 en el RE.

Acumulación de sustancias endógenas “normales” y de sustancias exógenas y que no tiene la


capacidad la célula de poderlas metabolizar o eliminar.
En el citoplasma, orgánulos, núcleo y pueden ser sintetizadas por células afectadas o en otro lugar.
Las principales formas de acumulaciones intracelulares anómalas son:
1. Eliminación inadecuada de una sustancia normal por defectos en el mecanismo de
empaquetamiento y transporte como el cambio graso hepático (esteatosis).
2. Metabolismo anómalo: célula normal tiene lípidos en su interior pero si su metabolismo esta
alterado no puede metabolizar o procesar esas moléculas de grasa y se acumula.
3. Defecto de plegamiento y transporte de proteínas: no se pliegan bien las proteínas. No se
empaquetan, no se transportan y no se secretan. En el interior de la célula se acumulan muchas
proteínas anomalas, esta relacionada con patologías: Jacob, alzheimer, deficiencia de alfa 1
antitrepsina, parkinson, hipercolesterolemia familiar, fibrosis quística.
4. Cuando hay falta de enzima, se acumulan los sustratos. Están relacionadas con los procesos
hereditarios
Acumulación de lípidos
1. Esteatosis (cambio graso): acumulación de triglicéridos en células parenquimatosas del hígado,
corazón y riñón, las causas incluyen toxinas, desnutrición proteínica DM, obesidad y anoxia, las
causas más frecuentes de hígado graso son el consumo excesivo de alcohol y el hígado graso
no alcohólico por diabetes y obesidad.
2. Colesterol y ésteres de colesterol: La mayoría de células utilizan colesterol para la síntesis de
las membranas celulares y las acumulaciones pueden manifestarse en varías patologías:
a. Aterosclerosis: células presentan aspecto espumoso y los agregados de ellas en la
íntima producen ateromas de colesterol amarillos que cuando se rompen liberan su
contenido al espacio extracelular. Los ésteres de colesterol extracelulares cristalizan
formando largas agujas que aparecen como características hendiduras en los cortes
tisulares.
b. Xantomas: La acumulación intracelular de colesterol en macrófagos es propia de los
estados hiperlipidémicos hereditarios y adquiridos que forman células espumosas que
generan masas tumorales conocidas como xantomas.
c. Colesterolosis: La acumulación focal de macrófagos cargados de colesterol en la lámina
propia de la vesícula biliar.
d. Enfermedad de Niemann-Pick tipo C causada por mutaciones que afectan a una enzima
implicada en el tráfico de colesterol, que induce la acumulación de este en múltiples
órganos.
Acumulación de proteínas: Las acumulaciones de proteínas parecen como gotículas redondeadas
eosinófilas, vacuolas o agregados en el citoplasma de aspecto AMORFO, FIBRILAR O CRISTALINO
y obedecen a ciertas causas:
1. Reabsorción de gotículas en túbulos renales proximales:
a. En enfermedades renales asociadas a la pérdida de proteínas en la orina (proteinuria).
b. Pequeñas cantidades de proteínas filtradas por el glomérulo y que son reabsorbidas por
pinocitosis y provoca un aumento de reabsorción de las proteínas en las vesículas
haciendo que aparezcan como gotículas hialinas rosadas en el citoplasma de la célula
tubular.
c. Es un proceso reversible.
2. Las proteínas pueden ser secretadas.
a. El RE se distiende sensiblemente produciendo grandes inclusiones eosinófilas
homogéneas llamados cuerpos de Russel.
3. Transporte intracelular defectuosos y secreción de proteínas esenciales.
a. Las mutaciones en las proteínas ralentizan el plegamiento y generan intermedios
parcialmente plegados, agregados en el RE de los hepatocitos que no son secretados y
que puede provocar efisema.
b. Acumulación de proteínas citoesqueléticas.
i. Microtúbulos
ii. Filamentos de actina delgados
iii. Filamentos de miosina gruesos.
iv. Filamentos intermedios que ofrecen un entramado celular flexible que sirve para
organizar el citoplasma y resistir a las fuerzas que inciden sobre la célula y se
diferencian en 5 clases:
1. Filamentos de queratina. P/e: Hialina alcohólica.
2. Neurofilamentos. P/e: ovillos neurofribrilafres.
3. Filamentos de desmina.
4. Filamentos de vimetina (tejido conjuntivo).
5. Filamentos gliales.
4. Agregación de proteínas anómalas/proteinopatías/enfermedades por agregación de proteínas:
causa directa o indirecta de cambios patológicos.

Cambio hialino: Alteración en las células o en el espacio extracelular que da un aspecto


HOMOGÉNEO, VITRIO Y ROSADO a los cortes histológicos de rutina teñidos con HyE. Este cambio
es originado por diversas alteraciones y no corresponde a un patrón específico de acumulación.
o Acumulación Hialina Intracelular: Cuerpos de Russel (alcohólica).
o Acumulación Hialina Extracelular: Tejido fibroso colágeno de las cicatrices antiguas puede
aparecer hialinizado. P/e: En HTA las paredes de las arteriolas se hialinizan como
consecuencia de las proteínas plasmáticas extravasadas y del deposito del material de
membrana basal.

Acumulación de Glucógeno: El glucógeno es una fuente de energía fácilmente accesible que se


almacena en el citoplasma de las células sanas. En pacientes con alteraciones del metabolismo de la
glucosa o glucógeno se observa un depósito intracelular excesivo. (DM, glucogenosis)
Las masas de glucógeno aparecen como vacuolas transparentes en el citoplasma y se identifica mejor
cuando está fijado en alcohol absoluto.
La tinción con carmín de Best o la reacción de PAS da lugar a una coloración rosa-violácea del
glucógeno.

Acumulación de pigmentos: Pueden ser pigmentos exógenos y pigmentos endógenos.


a. Exógenos
i. Carbón: contaminante en áreas urbanas. Cuando es captado por los macrófagos
en los alvéolos y transportado a través de los vasos linfáticos a los ganglios
linfáticos de la región traqueoabdominal produciendo antracosis en los tejidos
pulmonares o neumoconiosis de los mineros de carbón.
ii. Tatuajes: Los pigmentos inoculados son fagocitados por los macrófagos dérmicos
en los que residen durante toda la vida de la persona tatuada.
b. Endógenos
i. Lipofuscina/ lipocromo/pigmento de desgaste: insoluble, compuesto por polímeros
de lípidos y fosfolípidos formando complejos proteícos y es un signo indicador de
lesión por radicales libres y peroxidación lipídica. Se observa en células que
experimentan cambios regresivos lentos y es particularmente visible en el hígado
y corazón de pacientes de edad avanzada o que padecen desnutrición grave o
caquexia cancerosa.
ii. Melanina: Pigmento endógeno pardo-negro que se forma cuando la enzima
tirosinasa cataliza la oxidación de la tirosina para formar dihidroxifenilanina en los
melanocitos, está presente en los pacientes con alcaptonuria (enfermedad
metabólica).
iii. Hemosiderina: Derivada de la hemoglobina, es un pigmento de color amarillo
dorado a pardo, granular o cristalino que constituye una de las principales formas
de almacenamiento de hierro junto con apoferritina constituyente de a mayoría de
los tipos celulares. Esta acumulación es característica de las equimosis.
1. Causas de hemosiderosis:
a. Aumento de la absorción de hierro de la dieta por un error congénito
del metabolismo (hemocromatosis).
b. Anemias hemolíticas en las que la lisis prematura de eritrocitos
induce liberación de cantidades anómalas de hierro.
c. Transfusiones sanguíneas repetidas.

Depósito anómalo en los tejidos de sales de Ca con cantidades menores de sales de hierro, magnesio
y otros minerales, hay dos formas:
1. Distrófica: El depósito es local en tejidos que se están muriendo.
En áreas de necrosis coagulativa, caseosa o licuefactiva y en focos de necrosis enzimática de
grasas, presente en ateromas de aterosclerosis avanzada y válvulas cardíacas dañadas o de
ancianos. Las sales de Ca aparecen como gránulos o agregados blanquecinos finos percibidos
como depósitos arenosos.
Provoca disfunción orgánica.

2. Metastásica: Consecuencia de hipercalcemia secundaria a algún trastorno del metabolismo del


calcio, puede afectar a tejidos normales siempre que haya hipercalcemia, son 4 causas
principales que causan esta calcificación:
a. Aumento de la secreción de la hormona paratiroidea (PTH) que causa resorción ósea.
b. Resorción del tejido óseo consecuencia de tumores primarios de médula ósea o
metástasis difusa.
c. Trastornos relacionados con la vitamina D: intoxicación, sarcoidosis, hipercalcemia
idiopática de la lactancia.
d. Insuficiencia renal: causante de retención de fosfato que induce hiperparotiroidismo
secundario.
Las sales se presentan como depósitos amorfos no cristalinos o como cristales de
hidroxiapatita.
Mitología: Tot (egipcio) y Hermes (griego) hallaron el elixir de la juventud.
Literatura: Shakespeare describió las 7 edades del hombre.
El envejecimiento celular es debido al progresivo deterioro de la función y la viabilidad celulares
causado por anomalías genéticas y por la acumulación de efectos nocivos celulares y moleculares
asociados a exposición a estímulos externos. Se ve influido por un número limitado de genes
(anomalías genéticas aceleran el proceso p/e: Sx Werner, Sx Bloom y ataxia telangiectasa)
El envejecimiento es condicionado por numerosos mecanismos:
1. Daño del ADN: por factores exógenos (físicos, químicos, biológicos) y endógenos (ERO que
amenazan la integridad del ADN).
Las afectaciones persisten y se acumulan a medida que envejece la célula.
Las células madre hematopoyéticas sufren 14 mutaciones promedio al año y es probable que
este daño acumulado explique neoplasias comunes de la edad avanzada.
2. Senescencia celular: todas las células normales tienen una capacidad de replicación limitada y
tras una cantidad de divisiones quedan sumidas en un estado terminal de ausencia de división
y consta de dos mecanismos.
a. Desgaste de telómeros:
Longitud de los telómeros, disminuyen conforme vamos creciendo, y las células tienen
más probabilidad de lesión celular y de que haya esa replicación.
A mayor replicación, mayor probabilidad de un fallo en esa replicación.
Si las proteínas no detectan eso (como P53) se sigue haciendo esa replicación mal hecha
que puede llegar a cáncer.
La telomerasa mantiene la longitud del telómero y se cree que las células tumorales la
ocupan.
Si hay un daño en ese ADN, necesita a fuerza el ciclo celular para que se repare o se
vaya a apotosis, si no pasa empieza a mutar y se siguen reproduciendo y causan cancer
Senescencia celular: acortamiento de telómeros (relacionado con aumento de divisiones
celulares)
En algún momento se acaba la replicación celular, dicha disminuye y hay menos celulas
Ante cualquier lesion celular puede haber un deficit de formacion de proteínas, por lo
tanto las células no van a tenr proteínas y van a cambiar su funcionamiento
Cuando hay alguna alteración o deficit de algunos nutrientes, y puede que haya
disminución de transmisión de señales por insulina o que disminuya la señalización tipo
TOR (buscar) lo que provoca sirturinas alteradas y la transcripción tambien, aquí podría
aumentar la reparación del ADN junto con la homeostasis de las proteínas. Si los
nutrientes y dieta están bien aprovechadas, se puede contrarrestar el envejecimiento
celular (retardarlo )
SI NOSOTROS ESTAMOS RELACIONANDO TODO CON INFLAMACIÓN NUESTRAS
CELULAS VAN A EMPEZAR A FALLAR, SE ACTIVARÁ LA SENESCENCIA CELULAR
Y POR LO TANTO ENVEJECEN CELULAS. SI NOSOTROS NOS NUTRIMOS
ADECUADAMENTE TODAS LAS SEÑALIZACIONES DE LESIÓN CELULAR SERÁN
PRO- REPARACIÓN O HOMEOSTASIS Y SE CONTRARRESTARÁ EL
ENVEJECIMIENTO.

b. Activación de genes supresores tumorales: CDKN2A implicada en la senescencia y


codifica dos proteínas supresoras tumorales p16 y INK4a que controlan la progresión de
la fase G1 a la fase S en el ciclo celular.
3. Homeostasia defectuosa de proteínas: Las proteínas mantienen conformaciones de
plegamiento correctas y otro sistema degrada las proteínas mal plegadas por autofagia-
lisosoma y ubicutina-proteosoma.
4. Desregulación de la sensibilidad a los nutrientes: comer menos aumenta la longevidad, circuitos
neurohormonales controlan el metabolismo
a. Vía de transmisión de señales de la insulina y el factor de crecimiento insulínico que
favorecen el estado anabólico además del crecimiento y replicación celular.
b. Sirtuinas: Existen 7 tipos (la 6 es la mejor) y estas favorecen la expresión de varios genes
que aumentan la longevidad. (En el vino)
i. Contribuyen a las adaptaciones metabólicas de restricción calórica.
ii. Favorecen la integridad del genoma activando las enzimas reparadoras del ADN.
ANATOMÍA PATOLÓGICA
La inflamación es una respuesta de los tejidos vascularizados a las infecciones y al daño tisular, es
una respuesta protectora esencial para la supervivencia y está regulada por mediadores como
leucocitos, linfocitos, anticuerpos y proteínas del complemento.
Si hay una lesión: la inflamación va a contenerla y la va a aislar, el proceso de inflamación hace que
las células y proteínas lleguen a los tejidos dañados o necróticos y a los organismos invasores.
La reacción inflamatoria se desarrolla a través de pasos secuenciales:
1. Agente responsable es reconocido.
2. Leucocitos y proteínas plasmáticas son reclutados pasando le la circulación al sitio de la lesión.
3. Leucos y proteínas son activados y actúan juntos.
4. Reacción es controlada y concluida.
5. Tejido dañado es reparado.
Sus propiedades fundamentales son:
1. Componentes de la respuesta inflamatoria: principales vasos sanguíneos (vasodilatación para
aumentar permeabilidad) y leucocitos (ingieren y destruyen agentes nocivos)
2. Consecuencias nocivas de la inflamación: Reacción inflamatoria protectora puede convertirse
en crónica y patológica.
3. Inflamación local y sistémica: La reacción tisular es una respuesta local a una lesión localizada,
la sistémica origina patologías.
4. Mediadores de la inflamación: inician y amplifican la respuesta inflamatoria y determinan sus
pautas, gravedad y manifestaciones clínicas y patológicas.
5. Inflamación aguda y crónica:
a. Aguda/innata: minutos-horas caracterizada por exudado de proteínas y fluidos
plasmáticos (edema) y la migración de neutrófilos.
b. Crónica/adaptativa: Duración prolongada, asociada a la destrucción de los tejidos,
presencia de linfocitos y macrófagos, proliferación de vasos sanguíneos y depósito del
tejido conjuntivo.
6. Conclusión de la inflamación e inicio de la reparación tisular: La inflamación concluye cuando el
agente responsable es eliminado y provoca la reparación tisular mediante la regeneración de
las células supervivientes.
Celso S. I d. C: signos cardinales de la inflamación Eritema (rubor), tumor (hinchazón), calor y dolor.
Virchow S. XIX: Pérdida de la función.
1. Infecciones víricas, fúngicas, bacterianas,
parasitarias y toxinas.
2. Necrosis tisular: induce la inflamación con
independencia de la causa de muerte celular, es decir,
por isquemia, traumatismo o agresión física/química.
3. Cuerpos extraños: Causan inflamación por sí
mismo o por lesión tisular traumática, transporte de
microorganismos o sustancias endógenas nocivas
(cristales de urato, colesterol y lípidos).
4. Reacciones inmunitarias/hipersensibilidad:
Microbios, sustancias ambientales, etc., producen
enfermedades autoinmunitarias y alergias persistentes
y difíciles de curar.

El reconocimiento de agentes agresores es el primer


paso de todas las reacciones inflamatorias, existen
diversos receptores:
1. Receptores microbianos de las células: endosomas y citosol, el mejor pertenece a la familia de
los receptores de tipo señuelo (TLR) que se expresan en múltiples tipos celulares como céulas
epiteliales, dendríticas, macrófagos y otros leucos.
2. Sensores de lesión celular: Receptores activan un complejo citosólico multiproteínico llamado
inflamasoma que induce la producción de la IL-1 que recluta leucocitos y da lugar a la
inflamación, también se ve implicado en las reacciones inflamatorias de cristales de urato,
lípidos, colesterol y depósitos amiloides en el cerebro.
3. Otros receptores celulares implicados en la inflamación: expresan receptores para fragmentos
Fc de los anticuerpos y las proteínas de complemento y fomentan ingestión y destrucción de
los microorganismos.
4. Proteínas circulantes: sistema del complemento que produce inflamación.
Tres componentes principales:
1. Dilatación de pequeños vasos y aumento de flujo sanguíneo.
2. Aumento de la permeabilidad de la microvasculatura que permite que las proteínas plasmáticas
y los leucocitos abandonen la circulación.
3. Migración de los leucocitos desde la microcirculación, acumulación en el foco de la lesión, y
activación de los mismos para eliminar el agente causal.
Algunos mediadores actúan sobre pequeños vasos sanguíneos próximos y facilitan el flujo sanguíneo
del plasma y el reclutamiento de los leucocitos circulantes que se dirigen al sitio en el que se localiza
el agente agresor.
Reacciones de los vasos sanguíneos: consisten en el flujo de sangre y en la permeabilidad de los
vasos diseñados para optimizar el movimiento de las proteínas plasmáticas y de los leucocitos. Y se
inician inmediatamente después de la lesión, pueden ser los siguientes:
1. Cambios en el flujo y calibre de los vasos: vasodilatación inducida por acción de mediadores
como la histamina sobre músculo liso vascular, que afecta primero a las arteriolasy aumenta el
flujo sanguíneo que es lo que provoca el calor y eritema y que va seguida de un aumento en la
permeabilidad de la microvasculatura con liberación de proteínas en tejidos extravasculares.
La pérdida de líquido y el mayor diámetro de los vasos conducen a un flujo sanguíneo más lento
y aumento de la viscosidad sanguínea y que provoca estasis.
2. Aumento de la permeabilidad vascular/extravasación vascular: la contracción de las células
endoteliales determina el incremento de los espacios interendoteliales y es inducido por
bradicilina, histamina, leucotrienos y otros mediadores.
La extravasación comienza con un retraso entre 2-12 horas después de la lesión y dura varias
horas.
La lesión endotelial induce necrosis celular y desprendimiento, los neutrófilos se adhieren al
endotelio y pueden lesionar células endoteliales, aquí la extravasación comienza
inmediatamente después de la lesión y se mantiene varias horas.
Los vasos linfáticos participan también en la inflamación aguda ya que limpian los líquidos
extravasculares. A veces experimentan linfagitis y linfadenitis.

- Exudación: La extravasación de líquido, proteínas y células sanguíneas del sistema vascular al


tejido intersticial o cavidades corporales.
- Exudado: Líquido extravascular que presenta una concentración elevada de proteínas y restos
celulares.
- Trasudado: Líquido con bajo contenido de proteínas y material celular escaso o nulo de baja
densidad. Ultrafiltrado de plasma producido como consecuencia de un desequilibrio osmótico o
hidrostático a través de las paredes vasculares sin aumento de la permeabilidad de los vasos.
- Edema: exceso de líquido en el líquido intersticial o en cavidades serosas, puede ser exudado
o trasudado.
- Pus: exudado inflamatorio purulento rico en leucocitos y células muertas.
- Transcitosis: Incremento del transporte de líquidos y proteínas a través de la célula endotelial.
Reclutamiento de leucocitos para los sitios de inflamación: los cambios en el flujo sanguíneo y la
permeabilidad vascular van seguidos por la entrada de los leucocitos a los tejidos. Los neutrófilos y
macrófagos ingieren y destruyen bacterias y otros agentes dañinos y producen factores de crecimiento
que generan la lesión tisular y prolongan la inflamación, el trayecto de los leucocitos de la luz del vaso
al tejido va a ser un proceso que transcurren en varias fases:
1. En la luz es marginamiento, reclutamiento y adhesión al endotelio después se desprenden y se
vuelven a unir haciendo el rodamiento vascular (selectinas las más importantes) y es mediada
por moléculas de adhesión complementarias presentes en dos tipos celulares (selectinas e
integrinas) e incrementadas por citocinas.
TNF e IL-1 actúan sobre células endoteliales e inducen la expresión coordinada de numerosas
moléculas de adhesión (mediada por integrinas)
2. Migración a través del endotelio y la pared vascular/ diapédesis: en vénulas poscapilares, donde
las quimiocinas actúan sobre los leucocitos adherentes y estimulan a las células para que
migren a través de los espacios intersticiales a favor del gradiente de concentración químico.
3. Migración en los tejidos hacia el estímulo quimiotáctico: los leucocitos se desplazan por los
tejidos hacia el sitio de la lesión y sus mediadores son quimicionas, componentes del sistema
del complemento y metabolitos del ácido araquidónico.
Fagocitosis y eliminación del agente causal: el reconocimiento de agentes nocivos induce diversas
respuestas en los leucocitos designados como activación leucocítica mediante vías de señalización
que dan lugar al aumento de Ca citosólico y activación de enzimas (cinasa C, fosfolipasa A2).
Como tal la fagocitosis evoluciona a través de una secuencia de tres pasos
1. Reconocimiento y fijación de la partícula que va a ser ingerida: Los receptores fagocíticos
pueden ser de manosa, barredores/depuradores y de opsoninas.
2. Atrapamiento con formación posterior de una vacuola fagocítica/ fagosoma: después de que
una partícula se ha unido a los receptores extensores del citoplasma la envuelven y se forma el
fagosoma que se funde con el gránulo lisosómico y descarga el contenido del gránulo en el
fagolisosoma.
3. Destrucción y degradación del material ingerido: La muerte de los microbios es causada por
especies reactivas de oxígeno (ERO)/ intermediarios reactivos de oxígeno y por especies
reactivas de nitrógeno que destruyen los residuos fagocitados.
a. ERO se producen por el ensamblaje y la activación rápida de una oxidasa de múltiples
componentes, que oxida la NADPH y reduce el oxígeno a anión superóxido. Estas ERO
se producen en el lisosoma y fagolisosoma donde actúan contra las partículas ingeridas
sin dañar a la célula anfitrión e intervienen en la lesión celular que acompaña a la
inflamación.
El suero, los líquidos tisulares y las células anfitrión poseen mecanismos antioxidantes
que protegen ante estos radicales y son la enzima superóxido dismutasa, catalasa (que
inhibe la toxicidad de H2O2), glutatión peroxidasa, ceruloplastina y la fracción libre de
hierro de la transferrina sérica.
b. Óxido nítrico: es un gas soluble que participa en la muerte microbiana y hay tres tipos, la
endotelial eNOS que provoca la vasodilatación, neuronal nNOS que aumenta el Ca y
calmodulina dependiente y la inducible iNOS que se asocia a sepsis por el aumento de
Ca y afecta principalmente a corazón y pulmones. Todos estos regulados por GMPc.
c. Enzimas lisosómicas y otras proteínas de los lisosomas: neutrófilos y monocitos
contienen gránulos lisosómicos que contribuyen a la muerte de microbios y que cuando
liberan su contenido participan en la lesión tisular. Hay dos tipos principales de gránulos:
i. Menores/ específicos/ secundarios: contienen lisozima, colagenasa, gelatinasa,
lactoferrina, activador de plasminógeno, histaminasa y fosfatasa alcalina.
ii. Gránulos mayores/azurófilos/primarios: que contienen mieloperoxidasa, factores
bactericidas, hidrofasas y proteasas neutras que degradan colágeno, membrana
basal, fibrina, elastina y cartílago dando lugar a la inflamación.
d. Trampas extracelulares de neutrófilos: son redes fibrilares que aportan una elevada
concentración de sustancias antimicrobianas en sitios de infección y que evitan la
diseminación de los microorganismos atrapándolos en las fibrillas, son producidas por
neutrófilos en respuesta a patógenos infecciosos y mediadores inflamatorios.
e. Lesión tisular mediada por leucocitos: Que son causantes destacados de lesión celular
y tejidos normales en distintas circunstancias.
i. Defensa normal contra infecciones.
ii. Enfermedades autoinmunitarias.
iii. Reacción excesiva contra sustancias ambientales.
f. Terminación de la respuesta inflamatoria: Cuando el agresor es expulsado del
organismo.

Son sustancias que inician y regulan las acciones inflamatorias, los más importantes son las aminas
vasoactivas, productos lipídicos, citocinas y productos de activación del complemento, todos estos son
secretados o producidos por proteínas plasmáticas, los principales tipos celulares que producen
mediadores de la inflamación son los macrófagos, células dendríticas y mastocitos que detectan a los
invasores y dañan los tejidos.
Los mediadores derivados del plasma se producen sobre todo en el hígado y están presentes en la
circulación como precursores inactivos que para adquirir sus propiedades deben ser activados por
proteólisis.
Los mediadores activos solo son producidos en respuesta a diversos estímulos y la mayoría son de
vida breve, se degradan con rapidez y son inactivados por enzimas, son depurados o inhibidos,
además un mediador puede estimular la liberación de otros.

1. Aminas vasoactivas: histamina y serotonina


Almacenadas como moléculas preformadas en las células, entre las fuentes más abundantes de
histamina se encuentran los mastocitos de tejido conjuntivo adyacente a vasos, basófilos y plaquetas
sanguíneas, esta se almacena en gránulos de los mastocitos y es liberada en respuesta a estímulos
como lesión física, traumatismo, frío, calor o mecanismos desconocidos, reacciones alérgicas,
anafilotoxinas, etc.
Los leucocitos secretan proteínas liberadoras de histaminas.
Los neuropéptidos y citocinas estimulan la liberación de histaminas también.
La histamina causa la dilatación de las arteriolas y aumenta la permeabilidad de las vénulas.
2. Metabolitos del AA:
Los mediadores lipídicos PG y Lx productos del AA presente en los fosfolípidos de membrana
estimulan las reacciones vasculares y celulares en la inflamación aguda.
El AA es un AG de 20 carbonos que se obtiene a partir de la dieta o por conversión del ácido graso
linoleico.
Los estímulos liberan AA a partir de fosfolípidos de membrana durante la acción de fosfolipasas
celulares sobre todo fosfolipasa A que comprenden el aumento del Ca citoplasmático y la activación
de diversas cinasas en respuesta a estímulos externos.
Los mediadores derivados de AA son los eicosanoides que son sintetizados a partir de dos tipos de
enzimas: COX y Lipoxigenasas y están presentes en toda la inflamación.
3. PG
Son producidas por mastocitos, macrófagos, células endoteliales y participan en reacciones vasculares
y sistémicas de la inflamación. Generadas por acción de las COX.
COX1 es producida en respuesta a estímulos inflamatorios y COX2 es inducida por procesos
inflamatorios, las PG se dividen en series en función de sus características estructurales y se codifican
con letras y números. PGE2, PGD2, PGE2a, PGI y TxA.
El TxA es un potente agregante plaquetario y vasoconstrictor inestable y que se inactiva rápidamente.
El epitelio vascular expresa PGI2 (prostaciclina) que vasodilata e inhibe la agregación plaquetaria,
favoreciendo la formación de edema y es producida por mastocitos, también está implicada en la fiebre
y es hiperalgésica.
4. Leucotrienos
Son producidos por leucocitos, mastocitos por acción de la lipooxigenasa y se ven implicados en
reacciones del músculo vascular y liso en reclutamiento leucocítico, hay tres tipos de lipooxigenasas
siendo la 5lipooxigenasa la predominante en los neutrófilos.
5. Lipoxinas
También se originan a partir del AA por la vía de la lipooxigenasa, reducen la inflamación inhibiendo el
reclutamiento de los leucocitos, inhiben la quimiotaxis y la adhesión al endotelio de los neutrófilos.
6. Inhibidores farmacológicos de PG y Lx
a. AINES
i. COX2: Coxibs
ii. COX1 y 2: ibuprofeno, ASA
iii. Inhibidores de lipooxigenasa: Zileuton
iv. Corticoesteroides
v. Antagonistas de receptores de leucotrienos: Montelukast
7. Citocinas y quimiocinas
Las citocinas son proteínas producidas por linfocitos, macrófagos y células dendríticas que median y
regulan las reacciones inmunitarias e inflamatorias.

A. TNF e IL-1: desempeñan funciones esenciales en el reclutamiento de los leucocitos


favoreciendo su adhesión al endotelio y su migración a través de los vasos y son producidas
por macrófagos y células dendríticas, el TNF se genera también a partir de linfocitos T y
mastocitos y la IL1 por células epiteliales.
La producción de TNF es inducida por señales transmitidas a través de TLR u otros sensores
microbianos en tanto que la síntesis de IL1 es estimulada por las mismas señales y ambas
influyen en reacciones inflamatorias locales y sistémicas.
Sus funciones principales son:
- Activación endotelial.
- Activación de leucocitos, síntesis de colágeno y proliferación de células sinoviales.
- Respuesta de fase aguda sistémica como la fiebre, sepsis, apetito, etc.
B. Quimiocinas: son una familia de proteínas que actúan como quimiotácticas para tipos
específicos de leucocitos, existen alrededor de 40 distintas y 20 receptores diferentes.
a. Las quimiocinas C-X-C tienen un residuo de aa que separa los dos primeros de los 4
residuos de cisteína conservados y actúan principalmente sobre los neutrófilos, la IL8 es
característica de este grupo y causa vasodilatación y quimiotaxia de los neutrófilos,
b. Las quimiocinas C-C tienen los dos primeros residuos de cisteína conservados, estos
cuentan con la proteína quimiotáctica de monocitos MPC-1, eotaxina y la quimicina
RANTES, estas quimiocinas atraen a monocitos, eosinófilos, basófilos y linfocitos.
c. Quimiocinas C carecen del primero y el tercero de los residuos de cisteína conservados.
d. Quimiocinas CX-C: fractalquina que presenta dos formas y es inducida por citocinas
inflamatorias y tiene una potente actividad quimiotáctica.
Las quimiocinas median su actividad uniéndose a 7 receptores transmembrana unidos a proteína
G llamados CXCR/CCR y actúan como correceptores de una glucoproteína del VIH.
Las quimiocinas desempeñan dos funciones importantes:
- En la inflamación aguda: favorecen la fijación de leucocitos al endotelio a fin de
aumentar la afinidad de las integrinas y estimulan la quimiotaxia de los mismos.
- En el mantenimiento de la arquitectura tisular.

C. Sistema del complemento: Es un conjunto de proteínas solubles y receptores de membrana que


funcionan fundamentalmente en defensa del anfitrión contra los microbios y en reacciones
inflamatorias patológicas. Las proteínas del sistema del complemento están presentes de forma
inactiva en el plasma.
La escisión de la C3 se puede producir por una de las 3 vías
1. Clásica: desencadenada por la fijación de C1 a un Ac IgM o IgG que se ha combinado
con un Ag.
2. Alternativa: Por infección microbiana.
3. Lectina: Manosa se une a muchos CHO sobre los microbios y activa el componente
C1.
Las vías determinan la formación de enzima C3 convertasa que se divide en C3a y C3b. En la
inflamación C3a, C4a y C5a son productos de la escisión de los componentes del complemento
qu estimulan la liberación de histamina y aumentan la permeabilidad vascular y causan
vasodilatación. C3b y C3bi actúan como opsoninas y favorecen la fagocitosis.
El inhibidor C1 (INH C1) bloquea la activación de C1, que es la primera proteína de la vía clásica
del complemento.
El factor acelerador de la degradación (DAF) evita la formación de C3 convertasas y el CD59
inhibe la del complejo de ataque a la membrana.
8. Factor activador de plaquetas (PAF)
Es un mediador derivado de fosfolípidos que fue descubierto como factor causante de la agregación
plaquetaria y ejerce múltiples efectos inflamatorios.
Plaquetas, basófilos, mastocitos, neutrófilos, macrófagos y células endoteliales elaboran PAF que
induce la vasoconstricción y broncoconstricción y aumento de la permeabilidad venular.
9. Productos de coagulación
La cascada de coagulación está dada por reacciones enzimáticas que producen trombina que a su
vez, forman coágulos de fibrina mediante enzimas, sustrato y cofactores.
Existen 13 factores de coagulación, de los cuales el más importante es el 3 ya que es considerado
el factor tisular.
Todas las formas de lesión tisular que dan lugar a coagulación inducen inflamación e incrementan
la probabilidad de la coagulación anómala.
10. Cininas
Son péptidos vasoactivos derivados de proteínas plasmáticas (cimógenos) por acción de proteasas
(calicreínas). La calicreína descompone un precursor de la glucoproteína plasmática para inducir
bradicinina que aumenta la permeabilidad vascular e induce la contracción del músculo liso,
dilatación vascular y dolor cuando se inyecta en la piel, es de corta duración y es rápidamente
inactivada por la cinasa que interviene como mediador en cortas formas de reacción alérgicacomo
la anafilaxia.
11. Neuropéptidos
Los neuropéptidos son secretados por nervios sensitivos y diversos leucocitos y pueden intrvenir
en el inicio y regulación de respuestas inflamatorias, los más reconocidos de esta familia son la
sustancia P y la neurocinina A. Estos van a aumentar la permeabilidad vascular y secreci´n
hormonal.

Los rasgos morfológicos de la inflamación aguda son la dilatación de los pequeños vasos y la
acumulación de leucocitos en el tejido extravascular.
1. Inflamación serosa
Se caracteriza por la exudación de líquido con bajo contenido de células en los espacios creados
por la lesión celular o en las cavidades corporales revestidas por el peritoneo, pleura o pericardio.
El líquido no está infectado y no contiene grandes cantidades de leucocitos, la acumulación de este
en las cavidades se le llama derrame.
2. Fibrinosa
Al ser mayor el aumento de la permeabilidad vascular las moléculas grandes (fibrinógeno) salen
del torrente sanguíneo formándose fibrina que se deposita en el espacio extracelular, cuando la
extravasación es importante se desarrolla un exudado fibrinoso.
La fibrina aparece como una red de fibras o coágulo amorfo, si no se elimina estimula el desarrollo
hacia el interior de fibroblastos y vasos que conlleva a la formación de la cicatriz.
3. Purulenta/supurativa/absceso
Se caracteriza por la producción de pus, un exudado formado por neutrófilos, residuos licuados de
células necróticas y líquido de edema, la causa más frecuente es infección bacteriana
(estafilococos). P/e: apendicitis aguda
Los abscesos tienen una región central que aparece como una masa de leucocitos necróticos y
células tisulares, se observa dilatación vascular y proliferación parenquimatosa y fibroblástica,
indicativas de inflamación crónica y reparación.
4. Úlceras
Defecto o excavación local en la superficie de un órgano o tejido inducido por esfacelación
(desprendimiento) de tejido necrótico inflamado, sólo se produce en la necrosis tisular y está
presente cerca de la superficie donde se dio la inflamación. (frecuente en boca, estomago, intestino
y vías genitourinarias, piel, tejido subcutáneo de EI y en personas de edad avanzada con trastornos
circulatorios).

Todos los procesos inflamatorios agudos evolucionan de una de las tres formas siguientes:
1. Resolución completa/Resolución/Remisión: Es la evolución habitual cuando la lesión es limitada
y de duración corta. Una vez que se ha conseguido eliminar el agente causal, todas las
reacciones inflamatorias deberían concluir con el restablecimiento del estado normal.
2. Curación por reposición de tejido conjuntivo (cicatrización/fibrosis): Se observa tras una
destrucción sustancial de tejido, es decir, cuando la inflamación afecta a tejidos que no son
capaces de regenerarse o cuando hay abundante exudación de fibrina en tejidos o cavidades
serosas que no es posible de eliminar adecuadamente.
3. Progresión de la inflamación aguda a crónica.
Es una respuesta de duración prolongada en la que la inflamación, la lesión de los tejidos y los intentos
de reparación coexisten con combinaciones variables.

1. Infecciones persistentes: por microorganismos difíciles de erradicar como micobacterias,


hongos, virus y parásitos que a veces provocan hipersensibilidad retardada y en ocasiones
adoptan un patrón específico denominado reacción granulomatosa.
2. Enfermedades por hipersensibilidad: La inflamación crónica desempeña un importante papel en
un grupo de enfermedades que son causadas por activación excesiva e inapropiada del sistema
inmunitario y en fases tardías predomina la fibrosis. P/e: artritis reumatoide y esclerosis múltiple,
enfermedades alérgicas (asma bronquial)
3. Exposición prolongada a agentes potencialmente tóxicos, exógenos (partículas de silice que al
ser inhaladas causa silicosis) o endógenos.

1. Infiltración de células mononucleares (macrófagos, linfocitos y células plasmáticas).


2. Destrucción de los tejidos: inducida por el agente causal persistente o por células inflamatorias.
3. Intentos de curación: mediante la reposición del tejido conjuntivo dañado mediante angioedema
y fibrosis.

1. Macrófagos: células predominantes en inflamación crónica, secretan citocinas y factores d


crecimiento que actúan sobre diversas células destruyendo a los agentes invasores y tejidos
extraños, activando otras células (LnT). Estas células actúan como filtros de partículas,
microbios y células senescentes, funcionan como células efectoras que eliminan
microorganismos en las respuestas inmunitarias celular y tumoral.
Los macrófagos son células tisulares derivadas de CMH de médula ósea y de células
progenitoras del saco vitelino y el hígado fetal durante el desarrollo inicial, estos están
distribuidos de forma difusa en la mayoría de los tejidos conjuntivos, se hallan en localizaciones
específicas como hígado, bazo, ganglios linfáticos, SNC y pulmones y que en conjunto forman
el sistema mononuclear fagocítico.
Los precursores comprometidos de la médula ósea dan lugar a monocitos (semivida 1día) que
pasan a la sangre, migran a varios tejidos y se diferencian en macrófagos.
Hay dos vías principales de la activación de los macrófagos: la alternativa y la clásica
a. Clásica: inducida por productos microbianos como endotoxinas que se vinculan a los
TLR y otros sensores por señales derivadas de linfocitos T (INFy) en respuestas
inmunitarias o por sustancias extrañas.
Los macrófagos activados por la vía clásica también se llaman M1 y producen
NO y ERO y aumentan las enzimas lisosómicas.
b. Alternativa: Es inducida por citocinas distintas del INFy como la IL-4,3 producidas por
linfocitos T y otras células. La función esencial de las M2 es la reparación de los tejidos,
estas células secretan factores de crecimiento que favorecen la angiogenia, activan los
fibroblastos y estimulan la síntesis de colágeno.
Los productos de los macrófagos activados eliminan agentes lesivos e inician la reparación y
las funciones de los macrófagos son esencialmente las siguientes:
A) Ingieren y eliminan microbios y tejidos muertos.
B) Inician el proceso de reparación de tejidos y participan en la formación de cicatriz y
fibrosis.
C) Secretan mediadores de la inflamación como citocinas (TNF, IL-1, quimiocinas y
otros) y eicosanoides por lo que son esenciales para el comienzo y propagación de
las reacciones inflamatorias.
D) Los macrófagos exponen antígenos a los linfocitos T y responden a señales
procedentes de los mismos, estableciendo un asa de retroalimentación para la
defensa contra numerosos microbios, mediante respuestas inmunitarias mediadas
por células.

2. Linfocitos
Los agentes extraños activan los linfocitos T y B que amplifican y propagan la inflamación
crónica, estos Ln van a actuar como mediadores de la actividad adaptativa que aporta defensa
contra patógenos infecciosos y que cuando se activan tiende a ser persistente y grave.
Los Ln T y B estimulados por antígenos utilizan distintos pares de moléculas de adhesión para
migrar a los sitios de inflamación, TNF, IL-1 y quimiocinas fomentan el reclutamiento de los
leucocitos estableciendo condiciones necesarias para la persistencia de la respuesta
inflamatoria.
Los LnTCD4 favorecen la inflamación e influyen en la naturaleza de la reacción inflamatoria y
existen 3 subgrupos de citocinas y que generan diferentes tipos de inflamación:
A) TH1 producen citocina IFNy que activa macrófagos por la vía clásica.
B) TH2 que secretan Il-4, IL-5 e Il-3 que reclutan y activan eosinófilos y son
responsables de la vía alternativa de la activación de macrófagos.
C) TH17 que secretan IL-17 y otras citocinas que inducen la secreción de
quimiocinas responsables del reclutamiento de neutrófilos y monocitos en la
reacción.
TH1 y TH17 intervienen en la defensa contra bacterias, virus y enfermedades
autoinmunes, mientras que TH2 actúa contra parásitos, helmintos y alergias.
Los LnB activados y las células plasmáticas productoras de anticuerpos están a menudo
presentes en los focos de inflamación crónica, pueden ser específicos frente a Ags extraños en
el lugar de la inflamación o desarrollarse contra componentes tisulares alterados.

3. Eosinófilos
Son abundantes en las reacciones inmunitarias mediadas por IgE y en las infecciones
parasitarias, su reclutamiento depende de las moléculas de adhesión similares a las utilizadas
por los neutrófilos y de quimiocinas específicas.
Estás células presentan gránulos que contienen la proteína básica principal altamente catiónica
que es tóxica para los parásitos y que causa lisis de células epiteliales en mamíferos.
4. Mastocitos
Se distribuyen ampliamente en los tejidos conjuntivos y participan en reacciones inflamatorias
agudas y crónicas, en su superficie expresan el receptor FcdeltaRI que se une a la porción Fc
del Ac IgE. En las reacciones de hipersensibilidad inmediata los Acs IgE ligandos a los
receptores para Fc provocando la desgranulación y liberación de mediadores como histamina
y PG.
5. Neutrófilos
Inducidos por microbios persistentes o por mediadores elaborados por los macrófagos
activados y linfocitos T. P/e: osteomielitis.
Es una forma de inflamación crónica caracterizada por cúmulos de macrófagos activados a menudo
por linfocitos T y a veces asociada a necrosis central. p/e: tuberculosis.
La formación de granulomas es un intento por parte de la célula de contener a un agente causal que
sea difícil de erradicar, a menudo se registra una intensa activación de LnT que conduce a la activación
de macrófagos, aquellos macrófagos pueden desarrollar un abundante citoplasma y comenzar a
asemejarse a células epiteliales por lo que se denominan epitelioides.
Algunos de esos macrófagos se fusionan formando células gigantes multinucleadas.
Existen dos tipos de granulomas de diferente patogenia:
A. Granulomas de cuerpo extraño: ocasionados por cuerpos extraños inertes, se suelen formar en
torno a materiales como talco, suturas u otras fibras, las células epitelioides y gigantes se
yuxtaponen a la superficie del cuerpo extraño y el material extraño es identificado en el centro
del granuloma.
B. Granulomas inmunitarios: son causados por diversos agentes, capaces de inducir una
respuesta inmunitaria persistente mediada por LnT, es difícil de erradicar y los macrófagos
activan los LnT para producir citocinas como la Il-2 que activa otros LnT perpetuando la
respuesta inflamatoria.
Células epitelioides: Macrófagos activados en los granulomas que presentan un citoplasma granular
rosado con bordes celulares no diferenciados.
Células gigantes multinucleadas: Constan de una gran masa de citoplasma con numerosos núcleos y
derivan de la fusión de múltiples macrófagos activados.

La inflamación se asocia a reacciones sistémicas inducidas por citocinas que conforman la respuesta
de fase aguda, TNF, IL-6 y 1 son las más importantes y consiste en diversas alteraciones:
1. Fiebre: elevación de la temperatura corporal de 1-4GC por acción de pirógenos IL-1 y TNF y
PG que a su vez incrementan la producción de COX.
2. Proteínas de fase aguda: son proteínas plasmáticas sintetizadas en el hígado, las más
conocidas son la proteína C reactiva, el fibrinógeno y la proteína amiloide A sérica.
3. Leucocitosis: Recuento suele ascender hasta 15,000-20,000 c/ml o en casos extremos de
40,000-100,000 c/ml, la leucocitosis se produce por aceleración de la liberación de células a
partir de la reserva postmitótica de la médula ósea y se asocia a la elevación del número de
neutrófilos inmaduros en la sangre (desviación a la izquierda).
4. Aumento del pulso y presión arterial por redireccionamiento del flujo del lecho vascular cutáneo.
5. Sepsis en infecciones bacterianas graves que termina en shock hipertensivo y trastornos
metabólicos.
1. Perspectiva general de la reparación de tejidos: Reparación hace referencia al restablecimiento
de la arquitectura y la función tisular tras una lesión, mientras que la reparación se suele asociar
a tejidos parenquimatosos y conjuntivos, en tanto que la cicatrización se relaciona con epitelios
de superficie. La reparación de los tejidos dañados tiene lugar a través de dos tipos de
reacciones: la regeneración por proliferación de células residuales, de células madre adultas y
el depósito de tejido conjuntivo para formar una cicatriz.
a. Regeneración: algunos tejidos son capaces de reemplazar los componentes dañados y
de recuperar el estado normal mediante la regeneración que se produce por la
proliferación de las células que han sobrevivido a la lesión y que mantienen la capacidad
de proliferar.
La regeneración de células y tejidos implica proliferación celular promovida por factores
de crecimiento y que divide a los tejidos del cuerpo en tres grupos:
o Lábiles (en continua división): Entre las células lábiles se cuentan las
hematopoyéticas y epitelios superficiales de piel, cavidad oral, vagina y cuello
uterino, además de epitelios cúbicos y cilíndricos. Aquí las células lesionadas
son rápidamente reemplazadas por la proliferación de células residuales y
diferenciación de células madre adultas y es un proceso dirigido por factores
de crecimiento denominados factores estimuladores de colonias CSF.
El hígado tiene una gran capacidad de regeneración tras hepatectomía parcial
mediante dos mecanismos: la proliferación de hepatocitos remanentes y la
repoblación a partir de células progenitoras, donde intervienen Il-6, HGF y
TGFa que estimulan el metabolismo celular la entrada de células en en ciclo
celular.
o Estables: Las células de estos tejidos están en reposo y presentan una mínima
actividad proliferativa en estado normal y son capaces de dividirse en
respuesta a la lesión o pérdida de masa tisular, su proliferación es
particularmente importante en la cicatrización de las heridas.
o Tejidos permanentes: Las células de estos tejidos están diferenciadas
terminalmente y que no son proliferativas en la vida posnatal y originan una
cicatriz.
b. Depósito de tejido conjuntivo (formación de cicatriz): Si los tejidos lesionados no son
capaces de restablecerse por completo la reparación se lleva a cabo por aposición de
tejido conjuntivo (fibroso) que puede dar la formación de una cicatriz.
El término fibrosis se emplea habitualmente para describir el depósito extenso de
colágeno que se registra en pulmones, hígado, riñón y otros órganos como consecuencia
de la inflamación crónica. Cuando la fibrosis se desarrolla en un espacio tisular ocupado
por exudado inflamatorio el proceso se denomina organización.
Cuando la reparación no puede realizarse solo mediante regeneración es posible que se
proceda a reponer las células lesionadas con tejido conjuntivo dando lugar a la formación
de una cicatriz que puede tener lugar si la lesión tisular es grave o crónica y causa daño
en las células parenquimatosas y los epitelios, así como en la red de tejido conjuntivo o
si las células que no se dividen resultan lesionadas.
La aposición de tejido conjuntivo se produce en dos fases: migración de fibroblastos al
sitio de lesión y su proliferación y el depósito de proteínas de MEC regidos por citocinas
y factores de crecimiento (TGFb importante para la síntesis y depósito de proteínas de
tejido conjuntivo).
El resultado del proceso de reparación se ve influido por el equilibrio entre síntesis y
degradación de las proteínas de la MEC.

1. Angiogenia: formación de nuevos vasos sanguíneos que aportan los nutrientes y el oxígeno
necesarios para el proceso de reparación y que son permeables debido a las uniones
intraendoteliales incompletas y a que el VEGV incrementa la permeabilidad vascular.
Es el proceso de desarrollo de nuevos vasos sanguíneos a partir de los existentes, esencial en
los sitios de lesión, desarrollo de circulaciones colaterales en localizaciones de isquemia y en
el aumento de tamaño de los tumores y sigue los siguientes pasos:
a. Vasodilatación en respuesta al NO y aumento de la permeabilidad inducido por VEGF.
b. Separación de los pericitos de la superficie abluminal y degradación de la membrana
basal para permitir la formación de nuevos brotes.
c. Migración de las células endoteliales hacia el área de la lesión tisular.
d. Proliferación de las células endoteliales inmediatamente por detrás del borde anterior
de las células que migran.
e. Remodelación en tubos capilares.
f. Reclutamiento de células periendoteliales a fin de formar un vaso maduro.
g. Supresión de la proliferación y migración endotelial y depósito de la membrana basal.
En esta formación intervienen PGF, Ang 1 y 2, VEGV, NO, TGDb entre otros.
2. Formación de tejido de granulación: La migración y proliferación de fibroblastos y el depósito de
tejido conjuntivo laxo junto con los vasos y leucocitos entremezclados forman en tejido de
granulación. Su aspecto histológico se caracteriza por la proliferación de fibroblastos y nuevos
capilares finos y de pared delgada en una matriz celular laxa a menudo con células inflamatorias
mezcladas.
3. Remodelación del tejido conjuntivo: La maduración y reorganización del tejido conjuntivo genera
una cicatriz fibrosa estable dando lugar a la formación de una cicatriz que puede removerse con
el tiempo.

La reparación de tejidos se ve alterada por distintas influencias que reducen la calidad o la idoneidad
del proceso reparador, las variables que modifican pueden ser extrínsecas o intrínsecas y sistémicas
o locales.
- La infección es una de las causas clínicamente más importantes de retraso en la
cicatrización, prolonga la inflamación y puede incrementar el alcance de la lesión tisular a
nivel local.
- Diabetes: que es un trastorno metabólico que afecta la repartición de tejidos por muchas
razones y una de las principales causas sistémicas de cicatrización anómala.
- Estado nutricional: vitamina C inhibe la síntesis de colágeno y retrasa la cicatrización.
- Glucocorticoides: Tienen efectos anafilácticos y su administración puede debilitar la cicatriz.
- Factores mecánicos: Como el aumento de la presión a nivel local o la torsión.
- Mala perfusión, p/e: en aterosclerosis
- Cuerpos extraños
- El tipo de alcance y lesión
- Localización de la lesión

1. Cicatrización de heridas cutáneas


A) Cicatrización por primera intención/unión primaria: cuando la lesión afecta solo a la
capa epitelial, el principal mecanismo de reparación es la regeneración epitelial y consta
de tres procesos: inflamación, proliferación celular y maduración de la cicatriz de tejido
conjuntivo.
Las heridas inducen una rápida activación de las vías de la coagulación que determina
la formación de un coágulo sanguíneo en la superficie de la lesión que contiene fibrina,
fibronectina y proteínas del complemento que sirve para detener la hemorragia.
Primeras 24 horas de la lesión: neutrófilos.
24-48 hrs: células epiteliales.
Día 3: Neutrófilos comienzan a ser reemplazados por macrófagos.
Día 5: Neurovascularización alcanza su máximo.
Semana 2: Acumulación de colágeno y proliferación de fibroblastos, se comienza el proceso de
blanqueamiento.
B) Cicatrización por segunda intención/unión secundaria: cuando la pérdida de células o
tejidos es más extensa como en heridas grandes, abscesos, úlcera o necrosis
isquémica en órganos parenquimatosos donde combina la regeneración y
cicatrización.
En las heridas que producen pérdida extensa de tejido el coágulo es mayor y en la
zona lesionada hay más exudado y residuos necróticos, además de que la inflamación
es más intensa.
Las heridas grandes requieren más tejido de granulación a fin de rellenar los espacios
vacíos y aportar una infraestructura subyacente que sirva de base a la regeneración
del epitelio tisular.
Contracción que se produce en heridas superficiales para ayudar a cerrar la herida.

2. Resistencia de la herida: la recuperación de la resistencia a la tracción o tensión de rotura es


debida al incremento de la síntesis de colágeno superior a su degradación durante los dos
primeros meses de cicatrización y se relaciona a modificaciones estructurales de las fibras de
colágeno.
3. Fibrosis de órganos parenquimatosos: El depósito de colágeno es parte de la cicatrización
normal de la herida y la fibrosis hace referencia al exceso de depósito de colágeno y otros
componentes de la MEC en un tejido y está inducido por estímulos lesivos persistentes como
infecciones o reacciones inflamatorias crónicas y se suele asociar a la pérdida de tejido y que
a menudo causa disfunción orgánica sustancial e insuficiencia de los tejidos afectados, la
principal citocina implicada en la fibrosis es TGFb.

Las complicaciones de la reparación tisular tienen su origen en anomalías en cualquiera de los


componentes básicos del proceso como la formación de la cicatriz defectuosa, exceso de la formación
de componentes reparadores o desarrollo de contracturas.
La formación inadecuada de tejido de granulación o de cicatriz da lugar a dos tipos de complicaciones:
dehiscencia (rotura) de la herida (por aumento de presión) y úlceras (consecuencia de la
vascularización inapropiada durante la cicatrización).
La formación excesiva de componentes que participan en el proceso de reparación da lugar a cicatrices
hipertróficas (acumulación de colágeno) y queloides (crece más allá de la herida original y no se
contrae).
La granulación exuberante protruye por encima del nivel de la piel circulante y bloquea la
reepitelización y debe ser eliminado por cauterización o escisión quirúrgica.
La contracción del tamaño de una herida es una parte importante del proceso normal de la cicatrización
pero su exceso deforma la herida y tejidos circundantes.
1. ¿EN DÓNDE SE PRODUCEN LAS C. VASODILATADORES
ESPECIES REACTIVAS DE OXÍGENO EN LA
D. MOLÉCULAS DE ADHESIÓN
FAGOCITOSIS?
6. LA CELULA CARACTERÍSTICA DE LA
A. NUCLEO
INFLAMACION GRANULOMATOSA:
B. LISOSOMA Y FAGOLISOSOMA
A. CELULAS DE KUPFFER
C. REL Y MITOCONDRIA
B. HISTIOCITOS SINUSALES
D. RER Y LISOSOMA
C. MACROFAGOS ALVEOLARES
2. LA LIPOFUSCINA SE CARACTERIZA POR:
D. CELULAS EPITELIOIDES.
A. SER UN PIGMENTO EXÓGENO
7. ¿CUAL ES EL EFECTO DE LA HISTAMINA
B. SER UNA ACUMULACIÓN DE SOBRE LOS VASOS SANGUÍNEOS?
TRIGLICÉRIDOS
A. VASODILATACIÓN
C. SER UN PIGMENTO DE DESGASTE
B. VASOCONSTRICCIÓN
D. SER UN PIGMENTO FISIOLÓGICO
C. QUIMIOTAXIS
3. LA ACUMULACIÓN INTRACELULAR DE
D. LESIÓN ENDOTELIAL
POLVO DE CARBÓN RECIBE EL NOMBRE
DE: 8. LA BIOPSIA DE ELECCIÓN PARA
SINUSITIS CRONICA ES:
A. GLUCOGENOSIS
A. LEGRADO.
B. CUERPOS DE RUSSELL
B. EN CUÑA.
C. CUERPOS DE COUNCILMAN
C. EXCISIONAL EN HUSO.
D. ANTRACOSIS
D. EN SACABOCADOS.
4. PEROXIDACIÓN DE LOS LÍPIDOS DE LA
MEMBRANA, MODIFICACIÓN OXIDATIVA DE 9. LOS TIPOS DE VIRUS DEL PAPILOMA
LAS PROTEÍNAS Y LESIONES DEL DNA SON HUMANO QUE PRODUCEN EN FORMA MÁS
EFECTOS RELEVANTES PARA LA LESIÓN FRECUENTE CA EPIDERMOIDE SON:
CELULAR CAUSADAS POR:
A. TIPOS 2, 4, 7
A. HIPOXIA
B. TIPOS 33, 35, 51
B. ISQUEMIA
C. TIPOS 11, 42, 44
C. ACIDOSIS LÁCTICA
D. TIPOS 16, 18
D. RADICALES LIBRES DERIVADOS DEL
OXÍGENO 10. TIPO DE CALCIFICACION QUE APARECE
EN ZONAS DE NECROSIS COAGULATIVA
5. EL FRAGMENTO DE FC DE IgG, C3B Y LAS CASEOSA O LICUEFACTIVA
COLECTINAS FUNCIONAN COMO:
A. DISTROFICA
A. QUIMIOTÁCTICOS
B. METASTASICA
B. OPSONINAS
C. CAMBIO HIALINO
D. METAPLASIA D. METAPLASIA
11. PROTEINA DE LA FASE AGUDA QUE EN 16. ES UN CAMBIO MORFOLÓGICO
INFLAMACION CRONICA PUEDE CAUSAR CaRACTERIZADO POR CONDENSACIÓN Y
AMILOIDOSIS SECUNDARIA FRAGMENTACIÓN DE LA CROMATINA QUE
SUCEDE EN:
A. PROTEINA C RECATIVA
A. HIPERTROFIA
B. PROTEINA AMILOIDE SERICA
B. NECROSIS
C. FIBRINOGENO
C. APOPTOSIS
D. PROTEINA S
D. ATROFIA
12. EN LA MUERTE CELULAR DE TIPO
NECROPTOSIS, LA CARACTERÍSTICA 17. LA REGENERACIÓN HEPÁTICA QUE SE
PRINCIPAL ES: PRODUCE TRAS LA HEPATECTOMÍA
PARCIAL ES UN EJEMPLO DE:
A. NO ACTIVA A LAS CASPASAS
A. METAPLASIA
B. ES IDÉNTICA A LA NECROSIS
B. ATROFIA
C. ES IDÉNTICA A LA APOPTOSIS
C. HIPERPLASIA
D. ACTIVA LAS CASPASAS 6 Y 3
D. HIPERTROFIA
13. EN UN INFARTO CEREBRAL ES COMÚN
ENCONTRAR EL PATRÓN DE NECROSIS: 18. ES UNA BIOPSIA CUYO OBJETIVO ES
UNICAMENTE DIAGNOSTICO:
A. CASEOSA
A. BIOPSIA INCISIONAL
B. GRASA
B. BIOPSIA EXCISIONAL
C. LICUEFACTIVA
C. ESPÉCIMEN QUIRÚRGICO
D. COAGULATIVA
D. BIOPSIA POR CURETAJE
14. TIPO DE NECROSIS EN EL PIE
DIABÉTICO 19. ACUMULACIÓN INTRACELULAR PROPIA
DEL ENVEJECIMIENTO:
A. NECROSIS CASEOSA
A. LIPOFUSCINA
B. NECROSIS GRASA
B. HEMOSIDERINA
C. LICUEFACCIÓN
C. FOSFOLÍPIDOS
D. NECROSIS COAGULATIVA
D. LÍPIDOS
15. EN LOS PACIENTES CON
HIPERTENSIÓN ARTERIAL EL MIOCARDIO 20. SON CAUSAS PATOLÓGICAS DE
SUFRE: APOPTOSIS:
A. HIPERPLASIA A. NUMERO CONSTANTE DE CÉLULAS
B. HIPERTROFIA B. DURANTE LA EMBRIOGENESIS
C. ATROFIA C. PROTEÍNAS MAL PLEGADAS
D. CAMBIOS HORMONALES A. REMISIÓN COMPLETA, CICATRIZACIÓN,
FIBROSIS O PROGRESIÓN A RESPUESTA
21. LA MIGRACIÓN DE LOS LEUCOCITOS A
CRÓNICA
TRAVÉS DEL ENDOTELIO SE LLAMA:
B. CICATRIZACIÓN, PÉRDIDA DE LA
ADHESIÓN.
FUNCIÓN Y PROGRESIÓN A RESPUESTA
QUIMIOTAXIS. CRÓNICA

DIAPÉDESIS. C. RESOLUCIÓN COMPLETA, FORMACIÓN


DE PUS Y PROGRESIÓN A RESPUESTA
RODAMIENTO. CRÓNICA
22. EN PATOLOGÍAS COMO ENFERMEDAD D. RESOLUCIÓN COMPLETA, CURACIÓN
DE ALZHEIMER Y COREA DE HUNTINGTON, POR REPOSICIÓN DE TEJIDO CONJUNTIVO
LA APOPTOSIS ES INDUCIDA POR: Y PROGRESIÓN A RESPUESTA CRÓNICA
A. DAÑO AL DNA 26. LA INTOXICACIÓN POR MONÓXIDO DE
B. LIPOPEROXIDACIÓN CARBONO SE RELACIONA CON LO
SIGUIENTE:
C. CARENCIA DE FACTORES DE
CRECIMIENTO SUSTANCIAS QUÍMICAS

D. MAL PLEGAMIENTO DE PROTEÍNAS REACCIONES INMUNITARIAS

23. SON LLAMADAS TAMBIÉN HIPOXIA


HIPERSENSIBILIDAD. PUEDEN DIRIGIRSE RADIACIÓN
CONTRA AUTOANTÍGENOS O SER
SUSTANCIAS ANÓMALAS CONTRA 27. ES UN FENOMENO PATOLOGICO QUE
SUSTANCIAS AMBIENTALES O CONTRA SUCEDE DURANTE LA FAGOCITOSIS
MICROBIOS.
A. ENGLOBAMIENTO
INFECCIONES.
B. UNION
CUERPOS EXTRAÑOS.
C. REGURGITACION
REACCIONES INMUNITARIAS.
D. ATRAPAMIENTO
NECROSIS TISULAR.
28. TODAS LAS CÉLULAS TIENEN
24. EL MECANISMO DE LA ATROFIA RECEPTORES CITOSÓLICOS QUE
COMIENZA CON: RECONOCEN UNA AMPLIA DIVERSIDAD DE
MOLÉCULAS LIBERADAS O ALTERADAS
A. REDUCCIÓN DE NECESIDADES COMO CONSECUENCIA DEL DAÑO
METABÓLICAS CELULAR. ESTOS RECEPTORES ACTIVAN
B. AUTOFAGIA UN COMPLEJO CITOSÓLICO
MULTIPROTEÍNICO LLAMADO:
C. APOPTOSIS
REACCIÓN ANTIGENO-ANTICUERPO.
D. AUMENTO DE SÍNTESIS DE PROTEÍNAS
INFLAMASOMA
25. CUÁLES SON LAS FORMAS DE
EVOLUCIÓN DE LA INFLAMACIÓN AGUDA LECTINA DE UNIÓN A MANOSA
SISTEMA DEL COMPLEMENTO
29. PROCEDIMIENTO DONDE, SE EXTIRPA DEL ÁCIDO ARAQUIDÓNICO EN LA
PARTE DE LA LESIÓN, EXCLUSIVAMENTE INFLAMACIÓN?
CON UN
A. LEUCOCITOS, MASTOCITOS, CÉLULAS
PROPÓSITO DIAGNÓSTICO. ENDOTELIALES Y PLAQUETAS
A. BIOPSIA TREPANACIÓN B. MASTOCITOS, NEUTRÓFILOS,
EOSINÓFILOS Y BASÓFILOS
B. BIOPSIA ASPIRACIÓN
C. LINFOCITOS, LEUCOCITOS,
C. BIOPSIA INCISIONAL
NEUTRÓFILOS Y PLAQUETAS
D. BIOPSIA EXCISIONAL
D. PLAQUETAS, EOSINOFILOS, BASOFILOS
30. SON MOLÉCULAS SENSORAS SOLO
34. SON CAUSADOS POR CUERPOS
BH3 DE LA APOPTOSIS :
RELATIVAMENTE INERTES, EN AUSENCIA
A. BCLX DE REACCIONES INMUNITARIAS MEDIADAS
POR LINFOCITOS T. SUELEN FORMARSE
B. BAD, BIM, BID EN TORNO A MATERIALES COMO TALCO,
C. BAX BAK SUTURAS O UTRAS FIBRAS.

D. BCL2 GRANULOMAS DE CUERPO EXTRAÑO

31. ¿CUAL ES LA PROTEINA QUE TIENE HIPERSENSIBILIDAD


FUNCION ANTIAPOPTOSICO?: FAGOCITOSIS.
APAF-1 GRANULOMAS INMUNITARIOS.
P53 35. ES UNA ALTERACIÓN EN EL
BLC2 DESARROLLO, LA MORFOLOGÍA Y
MADURACIÓN DE LA CÉLULA
CASPASA 3
A. DISPLASIA
32. SE OBSERVA TRAS UNA DESTRUCCIÓN
SUSTANCIAL DE TEJIDO, CUANDO LA B. HIPERPLASIA
LESIÓN INFLAMATORIA AFECTA A TEJIDOS C. NEOPLASIA
QUE NO SON CAPACES DE REGENERARSE
O CUANDO HAY ABUNDANTE EXUDACIÓN D. METAPLASIA
DE FIBRINA EN TEJIDOS O CAVIDADES
36. SON PROTEÍNAS DE FASE AGUDA QUE
SEROSAS, QUE NO ES POSIBLE ELIMINAR
AUMENTAN EN RESPUESTA ESTÍMULOS
ADECUADAMENTE.
INFLAMATORIOS.
REMISIÓN.
PROTEÍNA C REACTIVA Y FIBRINA
INFLAMACIÓN CRÓNICA.
PROTEÍNA C REACTIVA, FIBRINÓGENO Y
RESOLUCIÓN COMPLETA. HEPCIDINA.

FIBROSIS ALBUMINA Y TRANSFERRINA


33. CUÁLES SON LAS PRINCIPALES AUMENTO DE VSG (VELOCIDAD DE
CELULAS QUE GENERAN METABOLITOS SEDIMENTACIÓN GLOBULAR).
37. ES LA DEFICIENCIA DE OXÍGENO, D. GRANULOMA POR BACILOS
INTERFIERE EN LA RESPIRACIÓN GRAMNEGATIVOS
OXIDATIVA AEROBIA Y ES UNA CAUSA
42. SON CAUSAS DE INFLAMACION
IMPORTANTE Y COMÚN DE LESIÓN Y
EXCEPTO
MUERTE CELULARES
A. ANTIBIOTICOS
A. ISQUEMIA
B. REACCIONES INMUNITARIAS
B. APOPTOSIS
C. CUERPO EXTRAÑO
C. NECROSIS
D. NFECCIONES CUERPO EXTRAÑO
D. HIPOXIA
43. LA FAGOCITO OXIDASA MAS OXIDO
38. LA METAPLASIA ES UNA ADAPTACIÓN
NITRICO PARTICIPA EN EL ESTALLIDO
CELULAR CORRESPONDIENTE A:
RESPIRATORIO
A. CAMBIO DE GENOTIPO
FORMANDO:
B. CAMBIO DE UNA CÉLULA MADURA POR
A. ANIÓN SUPEROXIDO
OTRA CÉLULA MADURA
B. PEROXIDO DE HIDROGENO
C. CAMBIO EN NÚMERO
C. PEROXINITRITO
D. CAMBIO EN TAMAÑO
D. HIPOCLORITO
39. EN UNA VÁLVULA CARDIACA DAÑADA
ES COMÚN ENCONTRAR: 44. TIPO DE NECROSIS QUE SE PRODUCE
EN LA PANCREATITIS AGUDA:
A. PIGMENTO INTRACELULAR
A. NECROSIS CASEOSA
B. CALCIFICACIÓN DISTRÓFICA
B. NECROSIS LICUEFACTIVA
C. CALCIFICACIÓN FISIOLÓGICA
C. NECROSIS COAGULATIVA
D. CALCIFICACIÓN METASTÁSICA
D. NECROSIS GRASA
40. LA CALCIFICACIÓN DISTRÓFICA SE
ENCUENTRA EN: 45. PROCEDIMIENTO DONDE, SE EXTIRPA
LA LESIÓN COMPLETA EN UN SOLO
A. ÁREAS DE NECROSIS.
TIEMPO. SE UTILIZA CUANDO SE DESEA
B. LA REABSORCION ÓSEA. RESECAR LA TOTALIDAD DE LA LESIÓN YA
SEA BENIGNA O MALIGNA, DEJANDO
C. LA INTOXICACIÓN CON VITAMINA D.
MÁRGENES DE SEGURIDAD PARA EVITAR
D. EL INCREMENTO DE LA SECRECIÓN DE LA RECIDIVA.
HORMONA PARATIROIDEA.
A. BIOPSIA ASPIRACIÓN
41. ESTE TIPO DE LESIÓN PRESENTA BAJA
B. BIOPSIA INCISIONAL
RESPUESTA DE LINFOCITOS T:
C. BIOPSIA TREPANACIÓN
A. GRANULOMA POR M. TUBERCULOSIS
D. BIOPSIA EXCISIONAL
B. GRANULOMA POR CUERPO EXTRAÑO
C. GRANULOMA INMUNITARIO
46. PUEDE DEBERSE A MUTACIONES, D. MUERTE CELULAR PROGRAMADA POR
ENVEJECIMIENTO O FACTORES IL-1
AMBIENTALES DESCONOCIDOS, SE
50. ¿QUÉ EFECTO TIENE LA DISMINUCIÓN
RECONOCE EN LA ACTUALIDAD COMO
DE ATP EN EL PH CITOSÓLICO?
RASGO PROPIO DE DIVERSAS
ENFERMEDADES NEURODEGENERATIVAS, A. DISMINUYE POR AUMENTO DE
COMO ALZHEIMER, COREA DE PIRUVATO
HUNTINGTON O EL PARKINSONISMO.
B. DIMINUYE POR AUMENTO DE ÁCIDO
APOPTOSIS POR DAÑO AL ADN. LACTICO
APOPTOSIS MEDIADA POR LINFOCITOS T C. AUMENTA POR INCREMENTO DE
CITOTÓXICOS. BICARBONATO
APOPTOSIS POR MAL PLEGAMIENTO DE D. DISMINUYE POR INCREMENTO DE
PROTEÍNAS. ÁCIDO CARBÓNICO
APOPTOSIS POR CARENCIA DE FACTORES 1.-) PIGMENTO ENDÓGENO, NO DERIVADO
DE CRECIMIENTO. DE LA HEMOGLOBINA:
47. LA DIGESTION EXZIMATICA ES EL A) HEMOSIDERINA
EVENTO PREDOMINANTE EN EL SIGUIENTE
TIPO DE NECROSIS: B) BILIRRUBINA

A. NECROSIS GRASA C) ANTRACOSIS

B. N. DE COAGULACION D) MELANINA

C. N. DE LICUEFACCION E) TATUAJE

D. NECROSIS FIBRINOSA 2.-) CALCIFICACIÓN PATOLÓGICA QUE SE


ACOMPAÑA DE NIVELES ELEVADOS DE
48. LOS RECEPTORES DE MANOSA, LOS CALCIO SÉRICO ES:
RECEPTORES, BARREDORES
(DEPURADORES) Y LOS RECEPTORES DE A) DISTROFICA
DIVERSAS OPSONINAS PERTENECEN A B) METASTÁSICA
LOS RECEPTORES DE: RODAMIENTO.
C) DISPLASICA
ÚNICAMENTE RECEPTORES DE LA
ACTIVACIÓN DEL COMPLEMENTO. D) HORMONAL

FAGOCITOSIS 3.-) LA INICIACIÓN DE LA CALCIFICACIÓN


DISTROFICA INTRACELULAR SE INICIA EN:
DE LAS ESPECIES REACTIVAS DEL
OXÍGENO A) RETÍCULO ENDOPLASMICO RUGOSO

49. LA PIROPTOSIS SE DEFINE COMO: B) RETÍCULO ENDOPLASMICO LISO

A. LESIÓN CELULAR C) MITOCONDRIAS

B. MUERTE CELULAR PROGRAMADA POR D) MEMBRANA CELULAR


CASPASA 9 E) NÚCLEO
C. ADAPTACIÓN CELULAR
4.-) EL ASPECTO VÍTREO HOMOGÉNEO Y D) REPARACIÓN TISULAR
ROSADO A LOS CORTES HISTOLÓGICOS
E) REACCIONES ALÉRGICAS
HABITUALES TEÑIDO CON HEMATOXILINA
Y EOSINA SE LLAMA: 8.-) ES EJEMPLO DE INFLAMACIÓN
PURULENTA:
A) CAMBIO GRASO
A) DERRAME PLEURAL
B) ACUMULACIÓN INTRACELULAR DE
GLUCÓGENO B) AMPOLLAS DE LAS QUEMADURAS
C) CAMBIO HIALINO C) VESÍCULAS CAUSADAS POR INFECCIÓN
VIRAL (HERPES)
D) COLESTEROLOSIS
D) LIQUIDO DE ASCITIS
E) ATEROESCLEROSIS
E) APENDICITIS AGUDA SUPURATIVA
5.-) LOS CUERPOS DE RUSSELL SON
ACUMULOS DE: 9.-) SE DENOMINA BIOPSIA AL ESTUDIO DE:
A) INMUNOGLOBULINAS EN CÉLULAS A) LAS CAVIDADES DE UN CADÁVER
PLASMÁTICAS
B) UN FROTIS O EXTENDIDO CELULAR
B) PROTEÍNAS EN LAS CÉLULAS
TUBULARES C) UN FRAGMENTO DE TEJIDO

C) GRASA EN HEPATOCITOS D) CÉLULAS EN DESCAMACIÓN EN


CUALQUIER LIQUIDO CORPORAL
D) COLESTEROL EN MACRÓFAGOS DE LA
LAMINA PROPIA DE LA VESÍCULA BILIAR E) LAVADO BRONQUIAL

E) GLUCÓGENO EN LAS CÉLULAS 10.-) SON SUSTANCIAS QUE ACTÚAN


EPITELIALES DE LOS TUBULOS COMO ANTIOXIDANTES NATURALES:
CONTORNEADOS DÍSTALES. A) SUPERÓXIDO DISMUTASA, GLUTATION
6.-) EL LLAMADO PIGMENTO DE DESGASTE PEROXIDASA
SE FORMA CON MUCHA FRECUENCIA EN: B) TETRACLORURO DE CARBONO Y
A) METAPLASIA CLORURO DE MERCÚRIO

B) HIPERPLASIA C) ÓXIDO NÍTRICO Y XANTINA OXIDASA

C) HIPERTROFIA D) METALES DE TRANSICIÓN COMO


HIERRO Y COBRE
D) ATROFIA
E) PERÓXIDO DE HIDRÓGENO Y ION
E) HIPOPLASIA HIDROXILO
7.-) LOS MACRÓFAGOS MODIFICADOS O 11.-) LAS PRINCIPALES CITOCINAS QUE
CÉLULAS EPITELIOIDES SON INTERVIENEN EN LA RESPUESTA
CARACTERÍSTICAS DE: INFLAMATORIA SON:
A) INFLAMACIÓN AGUDA A) QUIMIOCINAS
B) INFLAMACIÓN CRÓNICA B) INTERFERON ALFA, INTERFERON BETA
C) INFLAMACIÓN GRANULOMATOSA C) IL1 Y FNT
D) PROSTAGLANDINAS Y TROMBOXANO D) PROLIFERACIÓN DE FIBROBLASTOS Y
AUMENTO DE LA SISTESIS DE COLÁGENA
E) LEUCOTRIENOS
E)AUMENTO DE LA ACTIVIDAD
12.-) LOS SIGUIENTES SON EFECTOS DE LA
PROCOAGULANTE Y DISMINUCIÓN DE LA
HIPOXIA EN LA CÉLULA:
ANTICOAGULANTE.
A) AUMENTO DE LA PRODUCCIÓN DE ATP
16.-)TIPO DE NECROSIS QUE SE SUCEDE
B) AUMENTO DEL PH INTRACELULAR SECUNDARIA A LA LESIÓN HIPOXICA LETAL
EN LA MAYORÍA DE LOS TEJIDOS:
C) DISMINUCIÓN DEL CALCIO
INTRACELULAR A) COLICUATIVA

D) DISMINUCIÓN DE LA CONCENTRACIÓN B) GRASA


DE SODIO Y AGUA
C) CASEOSA
E) DISMINUCIÓN DE LA FOSFORILACIÓN
D) COAGULATIVA
OXIDATIVA
E) GANGRENA HUMEDA
13.-)EL FIJADOR DE TEJIDOS UTILIZADO DE
FORMA RUTINARIA: 17.-) LA LENTIFICACIÓN DE LA
CIRCULACIÓN EN LA INFLAMACIÓN AGUDA
A) XILOL AL 40%
SE DEBE A:
B) ALCOHOL DEL 96
A) LA VASOCONSTRICCIÓN ARTERIOLAR
C) TOLUENO
B) VASODILATACIÓN VENULAR
D) FORMOL AL 10%
C) EL AUMENTO DE LA PERMEABILIDAD DE
E) ALCOHOL ABSOLUTO LA MICROVASCULATURA

14.-)MEDIADORES RELACIONADOS CON EL D) LA APERTURA DE NUEVOS LECHOS


DOLOR EN LA INFLAMACIÓN AGUDA: VASCULARES

A)PAF Y SUSTANCIA P E) LA MARGINACIÓN LEUCOCITARIA


B) QUIMIOCINAS Y PRODUCTOS 18.-) SON SUSTANCIAS QUE ACTÚAN
BACTERIANOS C) PROSTAGLANDINAS Y COMO OPSONINAS:
BRADICININA
A) INTEGRINAS, SELECTINAS E
D)LEUCOTRIENOS Y TROMBOXANO A2 INMUNOGLOBULINAS

E) C5a, C5bi Y MAC B) C5A, LTB4, IL-8

15.-) SON REACCIONES DE LA FASE AGUDA C) HISTAMINA, SEROTONINA


PROVOCADAS POR LA IL-1, EL TNF
D) BRADICININA, CALICREÍNA
A) FIEBRE, PERDIDA DEL APETITO
E) C3B, IGG, Y COLECTINAS
B) AUMENTO DE LA ADHESIÓN
19.-) LAS PROLONGACIONES O
LEUCOCITARIA
PSEUDOPODOS DE LOS LEUCOCITOS
C) AUMENTO DE SÍNTESIS DE CONTIENEN EN SU INTERIOR FILAMENTOS
PROSTAGLANDINAS DE:
A) COLÁGENO Y ELASTINA A) ACCIÓN DIRECTA O INDIRECTA
B) COLÁGENO TIPO III B) ESTÍMULOS ENDÓGENOS
C) PROTEOGLUCANOS C) ESTÍMULOS EXÓGENOS
D) ACTINA Y MIOSINA D) FRAGMENTACIÓN DEL ADN
E) MICROTÚBULOS E) CONDENSACIÓN DE LA CROMATINA
20.-) EN LA HEMOSIDEROSIS SISTÉMICA, 24.-) EL DESPRENDIMIENTO DE LOS
EL PGIMENTO NO LESIONA A LAS RIBOSOMAS DEL RER EN LA LESIÓN
CÉLULAS, PERO SI SU ACUMULACIÓN ES CELULAR PRODUCE:
EXCESIVA, PRODUCE UNA ENFERMEDAD
A) DISMINUCIÓN DE LA ACTIVIDAD DE LA
CONOCIDA COMO:
BOMBA DE NA-K
A) OCRONOSIS
B) DISMINUCIÓN DE LA FOSFORILACIÓN
B) HEMOCROMATOSIS OXIDATIVA
C) ALCAPTONURIA C) AUMENTA LA GLUCOLISIS ANAEROBIA
D) ANTRACOSIS D)DISMINUCIÓN DE LA SÍNTESIS DE
PROTEÍNAS
E)ASBESTOSIS
E) AUMENTO DE LA CONCENTRACIÓN DEL
21.-) ES CARACTERÍSTICA DE LA
NA INTRACELULAR
INFLAMACIÓN CRÓNICA:
25.-)LOS CUERPOS DE WEIBEL PALADE DE
A) INFILTRACIÓN DE NEUTRÓFILOS
LAS CELULAS ENDOTELIALES CONTIENEN:
B) DURACIÓN MUY CORTA (MINUTOS,
A)SELECTINA P
HORAS O ALGUNOS DÍAS)
B)INTEGRINAS
C) INFILTRACIÓN DE CÉLULAS
MONONUCLEARES (MACRÓFAGOS) C)SELECTINA L
D) POCA DESTRUCCIÓN TISULAR D)ICAM
E) ES UN TIPO DE INFLAMACIÓN DE MUY 26.-)SON EJEMPLO DE SUBSTANCIAS
RARA PRESENTACIÓN QUÍMICAS QUE CAUSAN LESIÓN CELULAR
DE MANERA INDIRECTA AL
22.-) ES CARACTERISTICA DE LA
TRANSFORMARSE, POR VÍA ENZIMÁTICA,
CALCIFICACION METASTASICA:
EN RADICALES LIBRES:
A) NIVELES NORMALES DE CALCIO
A) CLORURO MERCÚRICO
B) SE PRESENTA EN NECROSIS CASEOSA
B) PARACETAMOL Y TETRACLORURO DE
C) PUEDE SER INTRA O EXTRACELULAR CARBONO

D) SE PRESENTA EN TEJIDOS NORMALES C) CIANURO, CICUTA


E) PROVOCA DISFUNCION ORGANICA D) QUIMIOTERÁPICOS,

23.-) LAS SUSTANCIAS QUÍMICAS PUEDEN


PRODUCIR LESIÓN CELULAR POR:
27.-) EL PATRÓN MORFOLÓGICO DE 31.-) ENTRE LOS FACTORES DE RIESGO
NECROSIS EN EL QUE PREDOMINA LA PARA EL CA CU SE ENCUENTRA, EXCEPTO:
DEGRADACIÓN ENZIMÁTICA ES LA:
A) VAGINOSIS BACTERIANA DE
A) N. LICUEFACTIVA REPETICIÓN
B) N. GANGRENOSA B) CANDIDOSIS VULVOVAGINAL
REPETITIVA
C) N. COAGULATIVA
C) ENFERMEDADES DE TRANSMISION
D) N. CASEOSA
SEXUAL, PROMISCUIDAD
28.) FACTORES DE ADHESION QUE
D) INICIO DE ACTIVIDAD SEXUAL DESPUÉS
PERTENECEN A LAS INMUNOGLOBULINAS
DE LOS 30 AÑOS
A)LIPOXINAS
32.-) WHICH OF THE FOLLOWING
B) SELECTINAS MEDIATORS OF INFLAMMATION HAS
CHEMOTACTIC PROPERTIES AND IS
C)ICAM, VECAM, PCAM INCREASED IN PERSONS TAKING ASPIRIN?
D)FACTOR DE LEWIS X A) THROMBOXANE A2
29.-) ES UN MEDIADOR QUIMICO QUE TIENE B) PROSTAGLANDIN E2
ACTIVIDAD BACTERICIDAY PRODUCE
VASODILATACION POR RELAJACION C) PLATELET-ACTIVATING FACTOR
DIRECTA DEL MUSCULO LISO
D) LEUKOTRIENE B4
ARTERIOLAR:
E) INTERLEUKIN-1
A)PROSTAGLANDINAS
33.) NITRIC OXIDE SYNTHESIS IS
B)C3a Y c5a
AUGMENTED IN ENDOTHELIAL CELLS BY A
C)OXIDO NITRICO CALMODULIN-MEDIATED INFLUX OF WHICH
ELEMENT?
D)FACTORDE NECROSIS TUMORAL
A) CALCIUM
E) LIPOXINAS
B) SODIUM
30.-)SON FENÓMENOS QUE
CARACTERIZAN E INDICAN QUE LA LESIÓN C) POTASSIUM
CELULAR ES IRREVERSIBLE:
D) OXYGEN
A) LAS ALTERACIONES DE LOS
E) NITRATE
RIBOSOMAS
34.-) ORGANO O TEJIDO QUE AUMENTA DE
B) LA FRAGMENTACIÓN DEL NÚCLEO
TAMAÑO POR AUMENTO DE SU NUMERO
CELULAR
DE CELULAS:
C) LA DISFUNCIÓN MITOCONDRIAL Y LA
A) ATROFIA
PERDIDA DE LA INTEGRIDAD DE LAS
MEMBRANAS CELULARES B) METAPLASIA
D) LA DILATACIÓN DEL RETÍCULO C) HIPERTROFIA
ENDOPLÁSMICO RUGOSO
D) HIPERPLASIA
E) DISPLASIA B) FORMACIÓN DE VACUOLAS
AUTOFAGICAS
35.-)WHICH OF THE FOLLOWING
MEDIATORS OF INFLAMMATION CAUSES C) PRESENCIA DE MITOCONDRIAS
PAIN : GIGANTES
A) NITRIC OXIDE D) CARIORREXIS HINCHAZON CELULAR
B) COMPLEMENT C3A E) TUMEFACCION MITOCONDRIAL
C) BRADYKININ 39.-) PARA LA FORMACIÓN DE
GRANULOMAS INMUNITARIOS SE
D) LEUKOTRIENE B4
REQUIERE:
E) INTERLEUKIN-1
A) PRESENCIA DE NEUTROFILOS
36.-)LA INFLAMACIÓN FIBRINOSA SE
B) REACCION INMUNITARIA MEDIADA POR
CARACTERIZA POR:
LINFOCITOS T
A) SE PRODUCE POR BACTERIAS
C) PARTICIPACIÓN DEL SISTEMA DE
PIOGENAS COMO EL ESTAFILOCOCO
COAGULACIÓN
B) SE PRESENTA EN ABSCESOS,
D) LIBERACIÓN DE ANTICUERPOS
APENDICITIS AGUDA
40.-) UNA CARACTERÍSTICA HISTOLÓGICA
C) SE PRESENTA EN QUEMADURAS O
DE LOS ORGANOS QUE SUFREN ATROFIA
INFECCIONES VIRALES
ES:
D) EL LIQUIDO LIGERO SE LOCALIZA EN EL
A) HIPERPLASIA DEL RETÍCULO
INTERIOR DE LA EPIDERMIS
ENDOPLASMICO RUGOSO
E) SE PRESENTA EN INFLAMACIÓN DE
B) FORMACIÓN DE VACUOLAS
CAVIDADES CORPORALES COMO
AUTOFAGICAS
PERICARDIO Y PLEURA
C) PRESENCIA DE MITOCONDRIAS
37.-) CORAZON ANORMALMENTE GRANDE
GIGANTES
POR OBSTRUCCIÓN VALVULAR MITRAL
OAORTICA PRODUCE: D) CARIORREXIS HINCHAZON CELULAR
A) HIPERPLASIA 41.-)SON CAUSAS DE HIPOXIA EXCEPTO:
B) METAPLASIA A) INSUFICIENCIA CARDIORESPIRATORIA
C) DISPLASIA B) ANEMIA
D) HIPERTROFIA C) INTOXICACIÓN POR MONÓXIDO DE
CARBONO
E) ATROFIA
D) ATEROESCLEROSIS
38.-)UNA CARACTERÍSTICA HISTOLÓGICA
DE LOS ORGANOS QUE SUFREN ATROFIA E) ANEURISMAS
ES:
42.-) SON COMPONENTES DE LA
A) HIPERPLASIA DEL RETÍCULO INFLAMACIÓN AGUDA:
ENDOPLASMICO RUGOSO
A) MODIFICACIONES EN EL CALIBRE DE A) LIMITADO AL GROSOR DE LA CAPA
LOS VASOS Y EN LA ESTRUCTURA MUSCULAR
MICROVASCULATURA
B) LIMITADO AL GROSOR DE LA CAPA
B) GRAN DESTRUCCIÓN TISULAR Y SEROSA
REPARACIÓN POR TEJIDO FIBROSO
C) LIMITADO A LA SUBMUCOSA
C) ATROFIA, HIPERTROFIA E HIPERPLASIA
D) LIMITADO AL GROSOR DEL EPITELIO
D) REGENERACIÓN Y CICATRIZACIÓN POR
E) LIMITADO AL TEJIDO GRASO
TEJIDO CONJUNTIVO
PARA CADA TIPO DE MUERTE CELULAR
E) NECROSIS Y APOPTOSIS
ELIJA LA PATOLOGÍA QUE SEA MÁS
43.-)SON FENÓMENOS LEUCOCITARIOS EN CARACTERÍSTICA.
LA LUZ VASCULAR:
46) APOPTOSIS
A) DIAPÉDESIS, SUEÑO DE ONDAS LENTAS
47) NECROSIS LICUEFACTIVA
B) QUIMIOTAXIS, ENGLOVAMIENTO
48) NECROSIS COAGULATIVA
C) MARGINACIÓN, RODAMIENTO Y
49) NECROSIS GRASA
ADHESIÓN
50) NECROSIS CASEOSA
D) OPSONIZACIÓN, FIEBRE
A) TUBERCULOSIS
C) FAGOCITOSIS Y DEGRADACIÓN
B) PANCREATITIS AGUDA
44.-) DENTRO DE LOS EJEMPLOS
ESPECÍFICOS DE APOPTOSIS, ESTÁ LA C) INFARTO DE MIOCARDIO
INDUCIDA POR LESIÓN DEL ADN CELULAR,
SEÑALE CÚAL ES EL GEN INVOLUCRADO D) INFECCIONES BACTERIANAS FOCALES
EN ESTE MECANISMO: E) MUERTE CELULAR POR RECHAZO
A) BEL 2 INMUNITARIO

B) CED 3
C) P53
D) CED 9
45.) EL CANCER IN SITU SE LOCALIZA :
ÚTERO GRÁVIDO
En el embarazo no involuciona el cuerpo lúteo y así persiste la secreción de progesterona. Las
glándulas endometriales muestran mayor secreción y núcleos hipercromáticos, las células
predeciduales se transforman en deciduales (con más organelos), de citoplasma abundante, límites
netos y disposición epiteloidea, lo que constituye la decidua (del verbo decidere : caer). Las células
deciduales controlan el grado de invasión del trofoblasto. El endometrio puede mostrar el fenómeno
de Arias-Stella: glándulas con hipersecreción y núcleos atípicos (grandes, irregulares,
hipercromáticos).
ÚTERO ATRÓFICO
El cérvix de la mujer menopáusica no sufre estímulos hormonales, y por tanto sus células dejan de
madurar, presentando un aspecto similar todas las capas de su epitelio; así pues las células
descamadas que veremos en la extensión corresponderán a células de las capas profundas o basales
y dicho patrón citológico recibirá la calificación de ‘atrófico’.
Por tanto será normal ver patrones diferentes en la extensión de una mujer de 20 ó 60 años, en relación
con su estado hormonal, sin que por ello exista patología ninguna.

HIPERPLASIA PROSTÁTICA BENINGA


La hiperplasia benigna de próstata es un diagnóstico histológico, que incluye proliferación de músculo
liso y células epiteliales.
A partir de los 30-40 años empiezan a aparecer focos de hiperplasia en el tejido glandular y
fibromuscular. Por encima de la quinta o sexta década de vida se da una segunda fase de crecimiento.
La hiperplasia de la próstata se da cuando la proliferación celular excede la muerte celular programada
(apoptosis), como resultado de la estimulación del crecimiento celular, inhibición de la apoptosis o
ambas.
Es posible relacionar los síntomas de hiperplasia prostática benigna, ya sea con el componente
obstructivo de la próstata (obstrucción mecánica o dinámica), o con la respuesta secundaria de la
vejiga a la resistencia a la salida.
A medida que el agrandamiento prostático tiene lugar, la obstrucción mecánica puede ser
consecuencia del crecimiento hacia la luz uretral o cuello vesical, conduciendo así a una mayor
resistencia de salida de la vejiga.
La hiperplasia prostática benigna inicia predominantemente en la zona de transición periuretal, a
diferencia del cáncer de próstata, que tiende a ocurrir en zonas más periféricas, esto explica el por qué
el tamaño de la próstata percibida en tacto rectal muchas veces no se correlaciona con la
sintomatología (la zona media no es fácilmente accesible a la palpación).
El componente dinámico de la obstrucción prostática explica la naturaleza variable de los síntomas
experimentados por los pacientes. Mientras que las quejas por síntomas irritativos miccionales, son a
consecuencia de la respuesta secundaria de la vejiga al aumento de la resistencia de salida.

ATEROSCLEROSIS DE MONCKENBERG
La esclerosis de Monckeberg es una enfermedad de baja prevalencia y causa desconocida, en la cual
arterias de mediano y pequeño calibre sufren calcificación de la capa media generándose una
reducción de la luz.
EXAMEN EXTRAORDINARIO DE ANATOMIA
PATOLOGICA

PRIMAVERA 2012

DR. ALEJANDRO J. VAZQUEZ GUERRA


1.-) DEFECTOS EN ESTOS GENES PUEDEN
CONVERTIRLOS EN ONCOGENES Y
CONTRIBUIR AL CRECIMIENTO CÁNCER:
A) PROTOONCOGENES
B) GENES INDUCTORES DE APOPTOSI
C) GEN P21
D) GEN P53
E) GENES REPARADORES
2.-) ¿QUÉ VITAMINA ES NECESARIA PARA
LA HIDROXILACIÓN DEL PROCOLAGENO?
A) A
B) B
C) C
D) E
E) D
3.-) PRECURSORES CELULARES
ENDOTELIALES QUE PARTICIPAN EN LA
VASCULOGÉNESIS:
A) AGENTES PROANGIOGÉNICOS
B) ANGIOBLASTOS
C) AGENTES ANTIANGIOGÉNICOS
D) ANGIOPOYETINAS
4.-) LA CURACIÓN DE UNA HERIDA CON
DAÑO CÉLULAR EXTENSO ES UN EJEMPLO
DE…
A) CURACIÓN POR PRIMERA INTENCIÓN
B) CURACIÓN POR SEGUNDA INTENCIÓN
C) CURACIÓN POR TERCERA INTENCIÓN
D) NO EXISTE CURACIÓN
5.-) ¿ALREDEDOR DE QUE DÍA SE FORMAN
LOS PUENTES DE COLÁGENO EN LA
CURACIÓN DE HERIDAS POR PRIMERA
INTENCIÓN?
A) INMEDIATAMENTE
B) A LAS 24 HORAS
C) ALREDEDOR DEL DÍA 5
D) NO SE FORMAN PUENTES DE
COLÁGENO EN LA CURACIÓN POR
PRIMERA INTENCIÓN
6.-) CAMBIO EN LA DIFERENCIACIÓN DE
UNA CÉLULA DE UN TIPO CELULAR A
OTRO:
A) PLASTICIDAD DEL DESARROLLO.
B) TRANSDIFERENCIACIÓN.
C) MUTACIÓN.
D) CLONACIÓN.
E) DIFERENCIACIÓN ESTOCÁSTICA
7.-) LA APOPTOSIS SE PRESENTA EN:
A) PANCREATITIS AGUDA
B) INFECCIONES BACTERIANAS
C) INFARTO DE MIOCARDIO
D)MAMA LACTANTE DURANTE EL DESTETE
E) TUBERCULOSIS
8.-) CARACTERÍSTICA DE LA
CALCIFICACIÓN DISTROFICA:
A) NIVELES ELEVADOS DE CALCIO
B) SE PRESENTA EN TEJIDOS NORMALES
C) AFECTA RIÑONES, PULMONES
D) NO PRODUCE DISFUNCIÓN CLÍNICA
E) SE PRESENTA EN ÁREAS DE NECROSIS
9.-) SU FORMACIÓN EN EL MARGEN DE LA
HERIDA AYUDA A LA CONTRACCIÓN DE LA
HERIDA.
A) FIBROBLASTOS
B) MACRÓFAGOS
C) MIOFIBROBLASTOS
D) PROTEÍNAS PLASMÁTICAS
10.-) SON SUSTANCIAS QUE ACTÚAN
COMO OPSONINAS:
A)INTEGRINAS, SELECTINAS E
INMUNOGLOBULINAS
B) C5A, LTB4, IL-8
C) HISTAMINA, SEROTONINA
D) BRADICININA, CALICREÍNA
E) C3B, IGG, Y COLECTINAS
11.-) SU EPITELIZACIÓN COMPLETA ES
MÁS LENTA:
A) CICATRICES DE PRIMERA INTENCIÓN
B) CICATRICES DE SEGUNDA INTENCIÓN
C) CICATRICES CON BORDES LÍNEALES
CON POCA PÉRDIDA DE TEJIDO
CONJUNTIVO
D) HERIDAS CON LOS BORDES
CONFRONTADOS
12.-) SON EJEMPLOS DE INFLAMACIÓN
SEROSA. EXCEPTO:
A) DERRAME PLEURAL
B) AMPOLLAS DE LAS QUEMADURAS
C) VESÍCULAS CAUSADAS POR INFECCIÓN
VIRAL (HERPES)
D) LIQUIDO DE ASCITIS
E) APENDICITIS AGUDA SUPURATIVA
13.-) ES UNA ALTERACIÓN LOCAL DE LA
CICATRIZACIÓN.
A) ESTADO NUTRICIONAL
B) ENFERMEDADES CRONICO
DEGENERATIVAS
C) FORMACIÓN EXCESIVA DEL TEJIDO DE
GRANULACIÓN
D) BORDES CON TEJIDOS NECROZADOS
14.-) ES UN PROCESO ACTIVO EN EL CUAL
LA DILATACIÓN ARTERIAL AUMENTA EL
FLUJO DE SANGRE:
A) CONGESTIÓN
B) HIPOXIA
C) HIPEREMIA
D) INFILTRACIÓN
15.-) ES UNA EXTRAVASACIÓN DE SANGRE
HACIA EL ESPACIO EXTRAVASCULAR
PUEDE SER EXTERNA O QUEDAR
CONTENIDA EN UN TEJIDO :
A) EQUIMOSIS
B) HEMORRAGIA
C) CONGESTIÓN
D) LINFEDEMA
E) NEUMOTORAX
16.-) ES UNA ACUMULACIÓN DE SANGRE,
CAUSADO POR LA ROTURA DE VASOS
CAPILARES, QUE APARECE
GENERALMENTE COMO RESPUESTA
CORPORAL RESULTANTE DE UN GOLPE:
A) PURPURAS
B) HEMATOMAS
C) HIPEREMIA
D) HEMOSIDERINA
17.-) EL PROCESO INICIAL DE
VASOCONSTRICCIÓN TRAS UNA LESIÓN
ESTA MEDIADO POR:
A) FACTOR DE VON WILRBRAN
B) ENDOTELINA.
C) CITOSINAS
D) TROMBINA Y ENDOTELINA
E) TROMBOPLASTINA
18.-) PROCESO PASIVO, DEBIDO A LA
REDUCCIÓN DEL FLUJO DE SALIDA DE 1
TEJIDO ACUMULA HEMOGLOBINA
DESOXIGENADA :
A) HIPEREMIA
B) ERITEMA
C) OSMOSIS
D) CONGESTIÓN
19.-) ¿CUÁLES SON LOS COMPONENTES
DE LA HEMOSTASIA ?
A) ENDOTELIO, PLAQUETAS Y
ANTITROMBINA
B) ENDOTELIO, PLAQUETAS Y
MONOCITOS
C) ENDOTELIO, PLAQUETAS Y CASCADA
DE COAGULACIÓN
D) LA PARED VASCULAR, MONOCITOS Y
ERITROCITOS
20.-) ¿QUE FACILITA LA ADHERENCIA Y
ACTIVACIÓN PLAQUETARIA?
A) LA AGREGACIÓN DE
MICROORGANISMOS
B) LA EXPOSICIÓN DE LA MATRIZ
EXTRACELULAR MUY TROMBOGENICA
C) LA FORMACIÓN DE TROMBINA
D) LA AUSENCIA DE PLAQUETAS
E) FORMACION DE PLASMINA
21.-) EL CUADRO SE CARACTERIZA POR LA
SÚBITA APARICIÓN DE DISNEA, CIANOSIS Y
SHOCK, SEGUIDO DE ALTERACIONES
NEUROLÓGICAS QUE VAN DESDE LA
CEFALEA A CONVULSIONES Y COMA
A) EMBOLIA AÉREA
B) EMBOLIA DE GRASA Y MEDULA ÓSEA
C) EMBOLIA PULMONAR
D) EMBOLIA DE LÍQUIDO AMNIÓTICO
E) TROMBOEMBOLIA SISTÉMICA
22.-) EL SÍNDROME POR DESCOMPRESIÓN
SE DA PRINCIPALMENTE EN:
A) BOXEADORES
B) MAESTROS
C) INGENIEROS
D) BUCEADORES
E) NADADORES
23.-) SE CONSIDERAN FACTORES DE
RIESGO PARA PRODUCIR COLELITIASIS,
EXCEPTO:
A) EDAD
B) ANTICONCEPTIVOS ORALES
C) OBESIDAD
D) DIETA RICA EN FIBRA
24.-) ES LA PRINCIPAL COMPLICACIÓN DE
LOS CÁLCULOS BILIARES POR
OBSTRUCCIÓN DEL CUELLO DE LA
VESÍCULA O DEL CONDUCTO CÍSTICO:
A) COLECISTITIS LITIASICA AGUDA
B) COLECISTITIS CRÓNICA
C) COLANGITIS
D) COLEDOCOLITIASIS
E) NO EXISTE NINGUNA COMPLICACIÓN
25.-) LOS CASOS MÁS GRAVES DONDE LA
VESÍCULA BILIAR ESTÁ NECRÓTICA, VERDE
NEGRUZCO Y PUEDE PRESENTAR
PERFORACIONES SE DENOMINA:
A) COLECISTITIS GANGRENOSA
B) COLECISTITIS AGUDA CALCULOSA
C) COLECISTITS ENFISEMATOSA AGUDA
D) NO EXISTE SITUACIÓN
26.-) ENTRE LOS FACTORES DE RIESGO MÁS
IMPORTANTES PARA LA PRESENTACIÓN DE IAM
DESTACAN:
A) HIPERLIPIDEMIAS FAMILIARES, ENFERMEDADES
PULMONARES OBSTRUCTIVAS, SÍNDROMES
PARANEOPLÁSICOS
B) INSUFICIENCIA RENAL Y PULMONAR, DISLIPIDEMIAS,
DIABETES, SEDENTARISMO, DEPRESIÓN,
C) ESTRÉS, OBESIDAD, NÍVELES ELEVADOS DE
COLESTEROL Y ENZIMAS PANCREÁTICAS, HIPERHIDROSIS
D) TABAQUISMO OBESIDAD, HIPERTENSIÓN ARTERIAL,
DIABETES MELLITUS, HIPERCOLESTEROLEMIA Y
ATEROESCLEROSIS
E) ESTRÉS, INTOXICACIÓN POR ALCOHOL Y TRASTORNOS
DEL ESTADO DE ÁNIMO
27.-) ¿CUÁL ES EL VIRUS ARN
ONCÓGENO?
A) VIRUS DE LA HEPATITIS B
B) VIRUS DEL PAPILOMA HUMANO
C) VIRUS DE LA LEUCEMIA DE CÉLULAS T
D) VIRUS DE EPSTEIN-BARR
28.-) .- ES EJEMPLO DE TROMBOSIS POR
ESTADO DE HIPERCOAGULABILIDAD
PRIMARIA:
A) ANTICOAGULANTE Y LUPICO Y CÁNCER
B) TABAQUISMO Y ANTICONCEPTIVOS
ORALES
C) DEF. DE ANTITROMBINA III Y DE PROTEÍNA
CYS
D) PRÓTESIS VALVULARES E
HIPERESTROGENISMO
E) SÍNDROME NEFRÓTICO Y
MIOCARDIOPATÍAS
29.-) LA LESIÓN CELULAR POR
TETRACLORURO DE CARBONO Y TYLENOL
SE CARACTERIZAN POR:
A) DAÑO RENAL GRAVE
B) SU CONVERSIÓN EN METABOLITOS
ACTIVOS MUY TÓXICOS
C) LESIÓN CELULAR REVERSIBLE
D) ACTIVAR LOS MECANISMOS DE
APOPTOSIS
30.-) ADAPTACIÓN CELULAR QUE
CONSISTE EN EL AUMENTO EN EL
NÚMERO DE CÉLULAS DE UN ÓRGANO O
TEJIDO CON EL CONSECUENTE AUMENTO
DE VOLÚMEN DEL MISMO:
A) HIPERPLASIA
B) METAPLASIA
C) HIPERTROFIA
D) ATROFIA
E) HIPOPLASIA
31.-) A LA ACUMULACIÓN ANORMAL DE
TRIGLICÉRIDOS DENTRO DE LAS CÉLULAS
PARENQUIMATOSAS SE LE LLAMA:
A) GLUCOGENOSIS
B) COLAGENOSIS
C) ESTEATOSIS
D) ANTRACOSIS
E) HEMOSIDEROSIS
32.-) A CÚAL DE LOS PIGMENTOS
ENDÓGENOS QUE SE PUEDEN ACUMULAR
INTRACELULARMENTE SE LE LLAMA
“PIGMENTO DEL ENVEJECIMIENTO”?
A) LIPOFUSCINA
B) BILIRRUBINA
C) HEMOSIDERINA
D) MELANINA
33.-) ¿QUÉ TIPO DE BIOPSIA SE UTILIZA
PARA PADECIMIENTOS EN PROSTATA?
A) BIOPSIA ENDOSCOPICA
B) BIOPSIA EN SACABOCADO
C) BIOPSIA EN CUÑA
D) BIOPSIA POR PUNCIÓN
E) ENDOSCOPICA
34.-) PROBLEMAS CLÍNICOS EN RIÑONES
CAUSADOS POR UN SHOCK
A) OLIGURIA, ANURIA .
B) POLIURIA Y ACIDOSIS
C) HIPERPLASIA RENAL
D) ISQUEMIA
E) DISURIA
35.-) SI SE INMOVILIZA UNA EXTREMIDAD
CON UN APARATO DE YESO POR TIEMPO
PROLONGADO, LOS MUSCULOS
INMOVILIZADOS PRESENTARAN:
A) HIPERPLASIA
B) HIPERTROFIA
C) AGENESIA
D) ATROFIA
E) APLASIA
36.-) SE LLAMA PERDIDA DE LA POLARIDAD
CELULAR NORMAL CUANDO:
A) HAY PERDIDA DE LA RELACIÓN NÚCLEO
CITOPLASMA CELULAR
B) HAY PERDIDA DE LA ORIENTACIÓN DE LAS
CÉLULAS ANAPLÁSICAS
C) HAY PERDIDA DE LAS FUNCIONES
CELULARES
D) HAY PERDIDA DEL TAMAÑO DE LAS
CÉLULAS ANAPLÁSICAS
E) HAY PERDIDA DE LA FORMA DE LAS
CÉLULAS ANAPLÁSICAS
37.-) EL RANGO PROMEDIO PARA LA
FIJACIÓN ADECUADA DE UN ESPÉCIMEN
QUIRÚRGICO EN FORMOL ES:
A) DE 1 A 3 HORAS
B) DE 8 A 12 HORAS
C) DE 72 A 78 HORAS
D) DE 4 A 6 DIAS
E) DE 48 A 72 HORAS
38.-) LA TROMBOSIS ARTERIAL
GENERALMENTE ES SECUNDARIA
A:
A) ESTASIS CIRCULATORIA
B) ATEROESCLEROSIS
C) CÁNCER
D) DEFICIENCIAS ENZIMÁTICAS
39.-)UN TUMOR ES CLÍNICAMENTE
DETECTABLE CUANDO TIENE UN PESO DE
1 GRAMO, EL CUAL CONTIENE:
A) 10 9 CELULAS
B) B) 10 6 CELULAS
C) C) 10 5 CELULAS
D) D) 10 7 CELULAS
E) E) 10 4 CELULAS
40.-) EL GRADO EN QUE LAS CÉLULAS
PARENQUIMATOSAS DE UNA
NEOPLASIA SE PARECEN A LA
CONTRAPARTE NORMAL SE
DENOMINA:
A) DIFERENCIACIÓN
B) INDIFERENCIACIÓN
C) DESDIFERENCIACIÓN
D) REGRESIÓN
E) PROGRESIÓN
41.-) CÓMO SE DENOMINAN A LOS
TROMBOS QUE PUEDEN TENER
LAMINACIONES PRODUCIDOS POR
CAPAS PÁLIDAS ALTERNANTES DE
PLAQUETAS MEZCLADAS CON
FIBRINA Y HEMATÍES?
A) EMBOLO
B) TROMBO MURAL
C) FLEBOTROMBO
D) LÍNEAS DE ZAHN
42.-) ¿CÓMO SE CLASIFICAN LOS
INFARTOS?
A) ROJOS, BLANCOS, SÉPTICOS
B) CARDIOGÉNICO, HIPOVOLÉMICO,
SÉPTICO.
C) DE PRIMERA INTENCIÓN, DE SEGUNDA
INTENCIÓN
D) AGUDOS, CRÓNICOS
E) CARDIACO, RENAL
43.-) EL PRIMER PUNTO DE ATAQUE DE LA
HIPOXIA EN LA CELULA ES:
A) RESPIRACIÓN ANAEROBICA
B) FOSFORILACION OXIDATIVA
C) GLUCÓLISIS ANAEROBIA
D) SÍNTESIS DE CREATINCINASA
44.-) ¿CUÁL DE LOS SIGUIENTES
ES ANTIOXIDANTES NO
ENZIMÁTICO?
A) TRANSFERRINA
B) CATALASAS
C) GLUTATIÓN PEROXIDASA
D) SUPERÓXIDO DISMUTASAS
45.-) LOS SIGUIENTES SON
PIGMENTOS DERIVADOS DE LA
HEMOGLOBINA:
A) LIPOFUCSINA Y MELANINA
B) HEMOSIDERINA Y BILIRRUBINA
C) CARBON Y TATUAJE
D) PIGMENTO DE DESGASTE O
ENVEJECIMIENTO
CORRELACIONAR LAS SIGUIENTES COLUMNAS
CONTESTANDO EN SU HOJA DE RESPUESTA

46.) CONDROSARCOMAE A) EPITELIAL BENIGNA


47.)CORIOCARCINOMAD B) MESENQUIMATOSA
BENIGNA
48.) HEMANGIOMAB C) NO ES NEOPLASIA
49.) PAPILOMA A D) EPITELIAL MALIGNA
50.) GRANULOMA C E) MESENQUIMATOSA
MALIGNA
1EXA. EXAMEN PARCIAL DE
ANATOMIA PATOLOGICA
OTOÑO 2014
DR. ALEJANDRO J. VAZQUEZ GUERRA
1.-)PIGMENTO ENDÓGENO, NO DERIVADO DE LA
HEMOGLOBINA:
A) HEMOSIDERINA
B) BILIRRUBINA
C) ANTRACOSIS
D) MELANINA
E)TATUAJE
2.-) CALCIFICACIÓN PATOLÓGICA QUE SE
ACOMPAÑA DE NIVELES ELEVADOS DE CALCIO
SÉRICO ES:
A) DISTROFICA
B) METASTÁSICA
C) DISPLASICA
D) HORMONAL
3.-) LA INICIACIÓN DE LA CALCIFICACIÓN
DISTROFICA INTRACELULAR SE INICIA EN:
A) RETÍCULO ENDOPLASMICO RUGOSO
B) RETÍCULO ENDOPLASMICO LISO
C) MITOCONDRIAS
D) MEMBRANA CELULAR
E) NÚCLEO
4.-) EL ASPECTO VÍTREO HOMOGÉNEO Y ROSADO A
LOS CORTES HISTOLÓGICOS HABITUALES TEÑIDO
CON HEMATOXILINA Y EOSINA SE LLAMA:
A) CAMBIO GRASO
B) ACUMULACIÓN INTRACELULAR DE GLUCÓGENO
C) CAMBIO HIALINO
D) COLESTEROLOSIS
E) ATEROESCLEROSIS
5.-) LOS CUERPOS DE RUSSELL SON ACUMULOS
DE:
A) INMUNOGLOBULINAS EN CÉLULAS
PLASMÁTICAS
B) PROTEÍNAS EN LAS CÉLULAS TUBULARES
C) GRASA EN HEPATOCITOS
D) COLESTEROL EN MACRÓFAGOS DE LA LAMINA
PROPIA DE LA VESÍCULA BILIAR
E) GLUCÓGENO EN LAS CÉLULAS EPITELIALES DE
LOS TUBULOS CONTORNEADOS DÍSTALES.
6.-) EL LLAMADO PIGMENTO DE DESGASTE SE
FORMA CON MUCHA FRECUENCIA EN:
A) METAPLASIA
B) HIPERPLASIA
C) HIPERTROFIA
D) ATROFIA
E) HIPOPLASIA
7.-)LOS MACRÓFAGOS MODIFICADOS O CÉLULAS
EPITELIOIDES SON CARACTERÍSTICAS DE:
A) INFLAMACIÓN AGUDA
B) INFLAMACIÓN CRÓNICA
C) INFLAMACIÓN GRANULOMATOSA
D) REPARACIÓN TISULAR
E) REACCIONES ALÉRGICAS
8.-) ES EJEMPLO DE INFLAMACIÓN PURULENTA:
A) DERRAME PLEURAL
B) AMPOLLAS DE LAS QUEMADURAS
C) VESÍCULAS CAUSADAS POR INFECCIÓN VIRAL
(HERPES)
D) LIQUIDO DE ASCITIS
E) APENDICITIS AGUDA SUPURATIVA
9.-) SE DENOMINA BIOPSIA AL ESTUDIO DE:
A) LAS CAVIDADES DE UN CADÁVER
B) UN FROTIS O EXTENDIDO CELULAR
C) UN FRAGMENTO DE TEJIDO
D) CÉLULAS EN DESCAMACIÓN EN CUALQUIER
LIQUIDO CORPORAL
E) LAVADO BRONQUIAL
10.-) SON SUSTANCIAS QUE ACTÚAN COMO
ANTIOXIDANTES NATURALES:
A) SUPERÓXIDO DISMUTASA, GLUTATION
PEROXIDASA
B) TETRACLORURO DE CARBONO Y CLORURO DE
MERCÚRIO
C) ÓXIDO NÍTRICO Y XANTINA OXIDASA
D) METALES DE TRANSICIÓN COMO HIERRO Y COBRE
E) PERÓXIDO DE HIDRÓGENO Y ION HIDROXILO
11.-) LAS PRINCIPALES CITOCINAS QUE
INTERVIENEN EN LA RESPUESTA INFLAMATORIA
SON:
A) QUIMIOCINAS
B) INTERFERON ALFA, INTERFERON BETA
C) IL1 Y FNT
D) PROSTAGLANDINAS Y TROMBOXANO
E) LEUCOTRIENOS
12.-) LOS SIGUIENTES SON EFECTOS DE LA
HIPOXIA EN LA CÉLULA:
A) AUMENTO DE LA PRODUCCIÓN DE ATP
B) AUMENTO DEL PH INTRACELULAR
C) DISMINUCIÓN DEL CALCIO INTRACELULAR
D) DISMINUCIÓN DE LA CONCENTRACIÓN DE
SODIO Y AGUA
E) DISMINUCIÓN DE LA FOSFORILACIÓN
OXIDATIVA
13.-)EL FIJADOR DE TEJIDOS UTILIZADO DE
FORMA RUTINARIA:
A) XILOL AL 40%
B) ALCOHOL DEL 96
C) TOLUENO
D) FORMOL AL 10%
E) ALCOHOL ABSOLUTO
14.-)MEDIADORES RELACIONADOS CON EL DOLOR EN
LA INFLAMACIÓN AGUDA:
A)PAF Y SUSTANCIA P
B) QUIMIOCINAS Y PRODUCTOS BACTERIANOS
C) PROSTAGLANDINAS Y BRADICININA
D)LEUCOTRIENOS Y TROMBOXANO A2
E) C5a, C5bi Y MAC
15.-) SON REACCIONES DE LA FASE AGUDA
PROVOCADAS POR LA IL-1, EL TNF
A) FIEBRE, PERDIDA DEL APETITO
B) AUMENTO DE LA ADHESIÓN LEUCOCITARIA
C) AUMENTO DE SÍNTESIS DE PROSTAGLANDINAS
D) PROLIFERACIÓN DE FIBROBLASTOS Y
AUMENTO DE LA SISTESIS DE COLÁGENA
E)AUMENTO DE LA ACTIVIDAD PROCOAGULANTE
Y DISMINUCIÓN DE LA ANTICOAGULANTE.
16.-)TIPO DE NECROSIS QUE SE SUCEDE
SECUNDARIA A LA LESIÓN HIPOXICA LETAL EN
LA MAYORÍA DE LOS TEJIDOS:
A) COLICUATIVA
B) GRASA
c) CASEOSA
D) COAGULATIVA
E) GANGRENA HUMEDA
17.-) LA LENTIFICACIÓN DE LA CIRCULACIÓN EN
LA INFLAMACIÓN AGUDA SE DEBE A:
A) LA VASOCONSTRICCIÓN ARTERIOLAR
B) VASODILATACIÓN VENULAR
C) EL AUMENTO DE LA PERMEABILIDAD DE LA
MICROVASCULATURA
D) LA APERTURA DE NUEVOS LECHOS
VASCULARES
E) LA MARGINACIÓN LEUCOCITARIA
18.-) SON SUSTANCIAS QUE ACTÚAN COMO
OPSONINAS:
A) INTEGRINAS, SELECTINAS E
INMUNOGLOBULINAS
B) C5A, LTB4, IL-8
C) HISTAMINA, SEROTONINA
D) BRADICININA, CALICREÍNA
E) C3B, IGG, Y COLECTINAS
19.-) LAS PROLONGACIONES O PSEUDOPODOS DE
LOS LEUCOCITOS CONTIENEN EN SU INTERIOR
FILAMENTOS DE:
A) COLÁGENO Y ELASTINA
B) COLÁGENO TIPO III
C) PROTEOGLUCANOS
D) ACTINA Y MIOSINA
E) MICROTÚBULOS
20.-) EN LA HEMOSIDEROSIS SISTÉMICA, EL
PGIMENTO NO LESIONA A LAS CÉLULAS, PERO SI
SU ACUMULACIÓN ES EXCESIVA, PRODUCE UNA
ENFERMEDAD CONOCIDA COMO:
A) OCRONOSIS
B) HEMOCROMATOSIS
C) ALCAPTONURIA
D) ANTRACOSIS
E)ASBESTOSIS
21.-) ES CARACTERÍSTICA DE LA INFLAMACIÓN
CRÓNICA:
A) INFILTRACIÓN DE NEUTRÓFILOS
B) DURACIÓN MUY CORTA (MINUTOS, HORAS O
ALGUNOS DÍAS)
C) INFILTRACIÓN DE CÉLULAS MONONUCLEARES
(MACRÓFAGOS)
D) POCA DESTRUCCIÓN TISULAR
E) ES UN TIPO DE INFLAMACIÓN DE MUY RARA
PRESENTACIÓN
22.-) ES CARACTERISTICA DE LA CALCIFICACION
METASTASICA:
A) NIVELES NORMALES DE CALCIO
B) SE PRESENTA EN NECROSIS CASEOSA
C) PUEDE SER INTRA O EXTRACELULAR
D) SE PRESENTA EN TEJIDOS NORMALES
E) PROVOCA DISFUNCION ORGANICA
23.-) LAS SUSTANCIAS QUÍMICAS PUEDEN
PRODUCIR LESIÓN CELULAR POR:
A) ACCIÓN DIRECTA O INDIRECTA
B) ESTÍMULOS ENDÓGENOS
C) ESTÍMULOS EXÓGENOS
D) FRAGMENTACIÓN DEL ADN
E) CONDENSACIÓN DE LA CROMATINA
24.-) EL DESPRENDIMIENTO DE LOS RIBOSOMAS
DEL RER EN LA LESIÓN CELULAR PRODUCE:
A) DISMINUCIÓN DE LA ACTIVIDAD DE LA
BOMBA DE NA-K
B) DISMINUCIÓN DE LA FOSFORILACIÓN
OXIDATIVA
C) AUMENTA LA GLUCOLISIS ANAEROBIA
D)DISMINUCIÓN DE LA SÍNTESIS DE PROTEÍNAS
E) AUMENTO DE LA CONCENTRACIÓN DEL NA
INTRACELULAR
25.-)LOS CUERPOS DE WEIBEL PALADE DE LAS
CELULAS ENDOTELIALES CONTIENEN:
A) SELECTINA P
B) INTEGRINAS
C) SELECTINA L
D) ICAM
26.-) SON EJEMPLO DE SUBSTANCIAS QUÍMICAS
QUE CAUSAN LESIÓN CELULAR DE MANERA
INDIRECTA AL TRANSFORMARSE, POR VÍA
ENZIMÁTICA, EN RADICALES LIBRES:
A) CLORURO MERCÚRICO
B) PARACETAMOL Y TETRACLORURO DE
CARBONO
C) CIANURO, CICUTA
D) QUIMIOTERÁPICOS,
27.-) EL PATRÓN MORFOLÓGICO DE NECROSIS EN
EL QUE PREDOMINA LA DEGRADACIÓN
ENZIMÁTICA ES LA:
A) N. LICUEFACTIVA
B) N. GANGRENOSA
C) N. COAGULATIVA
D) N. CASEOSA
28.) FACTORES DE ADHESION QUE PERTENECEN
A LAS INMUNOGLOBULINAS
A) LIPOXINAS
B) SELECTINAS
C) ICAM, VECAM, PCAM
D) FACTOR DE LEWIS X
29.-) ES UN MEDIADOR QUIMICO QUE TIENE
ACTIVIDAD BACTERICIDAY PRODUCE
VASODILATACION POR RELAJACION DIRECTA
DEL MUSCULO LISO ARTERIOLAR:
A) PROSTAGLANDINAS
B) C3a Y c5a
C) OXIDO NITRICO
D) FACTOR DE NECROSIS TUMORAL
E) LIPOXINAS
30.-)SON FENÓMENOS QUE CARACTERIZAN E
INDICAN QUE LA LESIÓN CELULAR ES
IRREVERSIBLE:
A) LAS ALTERACIONES DE LOS RIBOSOMAS
B) LA FRAGMENTACIÓN DEL NÚCLEO CELULAR
C) LA DISFUNCIÓN MITOCONDRIAL Y LA
PERDIDA DE LA INTEGRIDAD DE LAS
MEMBRANAS CELULARES
D) LA DILATACIÓN DEL RETÍCULO
ENDOPLÁSMICO RUGOSO
31.-) ENTRE LOS FACTORES DE RIESGO PARA EL CA
CU SE ENCUENTRA, EXCEPTO:
A) VAGINOSIS BACTERIANA DE REPETICIÓN
B) CANDIDOSIS VULVOVAGINAL REPETITIVA
C) ENFERMEDADES DE TRANSMISION SEXUAL,
PROMISCUIDAD
D) INICIO DE ACTIVIDAD SEXUAL DESPUÉS DE
LOS 30 AÑOS
32.-) WHICH OF THE FOLLOWING MEDIATORS OF
INFLAMMATION HAS CHEMOTACTIC PROPERTIES
AND IS INCREASED IN PERSONS TAKING ASPIRIN?
A) THROMBOXANE A2
B) PROSTAGLANDIN E2
C) PLATELET-ACTIVATING FACTOR
D) LEUKOTRIENE B4
E) INTERLEUKIN-1
33.) NITRIC OXIDE SYNTHESIS IS AUGMENTED IN
ENDOTHELIAL CELLS BY A CALMODULIN-
MEDIATED INFLUX OF WHICH ELEMENT?
A) CALCIUM
B) SODIUM
C) POTASSIUM
D) OXYGEN
E) NITRATE
34.-) ORGANO O TEJIDO QUE AUMENTA DE
TAMAÑO POR AUMENTO DE SU NUMERO DE
CELULAS:
A) ATROFIA
B) METAPLASIA
C) HIPERTROFIA
D) HIPERPLASIA
E) DISPLASIA
35.-) WHICH OF THE FOLLOWING MEDIATORS
OF INFLAMMATION CAUSES PAIN :
A) NITRIC OXIDE
B) COMPLEMENT C3A
C) BRADYKININ
D) LEUKOTRIENE B4
E) INTERLEUKIN-1
36.-) LA INFLAMACIÓN FIBRINOSA SE
CARACTERIZA POR:
A) SE PRODUCE POR BACTERIAS PIOGENAS
COMO EL ESTAFILOCOCO
B) SE PRESENTA EN ABSCESOS, APENDICITIS
AGUDA
C) SE PRESENTA EN QUEMADURAS O
INFECCIONES VIRALES
D) EL LIQUIDO LIGERO SE LOCALIZA EN EL
INTERIOR DE LA EPIDERMIS
E) SE PRESENTA EN INFLAMACIÓN DE
CAVIDADES CORPORALES COMO PERICARDIO Y
PLEURA
37.-) CORAZON ANORMALMENTE GRANDE POR
OBSTRUCCIÓN VALVULAR MITRAL O AORTICA
PRODUCE:
A) HIPERPLASIA
B) METAPLASIA
C) DISPLASIA
D) HIPERTROFIA
E) ATROFIA
38.-)UNA CARACTERÍSTICA HISTOLÓGICA DE LOS
ORGANOS QUE SUFREN ATROFIA ES:
A) HIPERPLASIA DEL RETÍCULO ENDOPLASMICO
RUGOSO
B) FORMACIÓN DE VACUOLAS AUTOFAGICAS
C) PRESENCIA DE MITOCONDRIAS GIGANTES
D) CARIORREXIS HINCHAZON CELULAR
E) TUMEFACCION MITOCONDRIAL
39.-) PARA LA FORMACIÓN DE GRANULOMAS
INMUNITARIOS SE REQUIERE:
A) PRESENCIA DE NEUTROFILOS
B) REACCION INMUNITARIA MEDIADA POR
LINFOCITOS T
C) PARTICIPACIÓN DEL SISTEMA DE COAGULACIÓN
D) LIBERACIÓN DE ANTICUERPOS
40.-) UNA CARACTERÍSTICA HISTOLÓGICA DE LOS
ORGANOS QUE SUFREN ATROFIA ES:
A) HIPERPLASIA DEL RETÍCULO ENDOPLASMICO
RUGOSO
B) FORMACIÓN DE VACUOLAS AUTOFAGICAS
C) PRESENCIA DE MITOCONDRIAS GIGANTES
D) CARIORREXIS HINCHAZON CELULAR
41.-) SON CAUSAS DE HIPOXIA EXCEPTO:
A) INSUFICIENCIA CARDIORESPIRATORIA
B) ANEMIA
C) INTOXICACIÓN POR MONÓXIDO DE CARBONO
D) ATEROESCLEROSIS
E) ANEURISMAS
42.-) SON COMPONENTES DE LA INFLAMACIÓN
AGUDA:
A) MODIFICACIONES EN EL CALIBRE DE LOS VASOS
Y EN LA ESTRUCTURA MICROVASCULATURA
B) GRAN DESTRUCCIÓN TISULAR Y REPARACIÓN
POR TEJIDO FIBROSO
C) ATROFIA, HIPERTROFIA E HIPERPLASIA
D) REGENERACIÓN Y CICATRIZACIÓN POR TEJIDO
CONJUNTIVO
E) NECROSIS Y APOPTOSIS
43.-) SON FENÓMENOS LEUCOCITARIOS EN LA
LUZ VASCULAR:
A) DIAPÉDESIS, SUEÑO DE ONDAS LENTAS
B) QUIMIOTAXIS, ENGLOVAMIENTO
C) MARGINACIÓN, RODAMIENTO Y ADHESIÓN
D) OPSONIZACIÓN, FIEBRE
C) FAGOCITOSIS Y DEGRADACIÓN
44.-) DENTRO DE LOS EJEMPLOS ESPECÍFICOS DE
APOPTOSIS, ESTÁ LA INDUCIDA POR LESIÓN DEL
ADN CELULAR, SEÑALE CÚAL ES EL GEN
INVOLUCRADO EN ESTE MECANISMO:
A) BEL 2
B) CED 3
C) P53
D) CED 9
45.) .) EL CANCER IN SITU SE LOCALIZA :
A) LIMITADO AL GROSOR DE LA CAPA
MUSCULAR
B) LIMITADO AL GROSOR DE LA CAPA SEROSA
C) LIMITADO A LA SUBMUCOSA
D) LIMITADO AL GROSOR DEL EPITELIO
E) LIMITADO AL TEJIDO GRASO
PARA CADA TIPO DE MUERTE CELULAR ELIJA LA
PATOLOGÍA QUE SEA MÁS CARACTERÍSTICA.

46) APOPTOSIS (E)


47) NECROSIS LICUEFACTIVA (D)
48) NECROSIS COAGULATIVA (C)
49) NECROSIS GRASA (B)
50) NECROSIS CASEOSA (A)

A) TUBERCULOSIS
B) PANCREATITIS AGUDA
C) INFARTO DE MIOCARDIO
D) INFECCIONES BACTERIANAS FOCALES
E) MUERTE CELULAR POR RECHAZO INMUNITARIO
1) d 12 e 23 d 35 C 46 E
2) b 13 c 24 e 36 E 47 D
3) c 14 a 25 a 37 D 48 C
4 c 15 d 26 38 B 49 B
5 a 16 c 27 c 39 B 50 A
6 d 17 e 28 40 B 33 A
7 e 18 29 41 E
8 c 19 c 30 42 A
9 a 20 c 31 43 C
10 a 21 c 32 44 C
11 c 22 d 34 D 45 D
El tratamiento para H. Pylori ……. respuesta b) Il-6 e il-1
correcta b c) Il-12 e il-2
d) INF e Il-1
1.Es un tipo especializado de tejido, característico
e) TNF e Il-6
de la curación, de aspecto rosado, blanco y de
apariencia granular en la superficie de las heridas: 7.En la biología celular de las carcinogenesis
química se observan los siguientes pasos
a) Tejjido conectivo
b) Tejido epitelial a) Mutaciones y angiogénesis
c) Tejido mesenquimatoso b) Mutaciones y metástasis
d) Tejido conjuntivo c) Iniciacion y promoción
e) Tejido de degranulacion d) Apoptosis y emperipolesis
e) Expansión clonal y mitosis
2.Son carcinógenos de acción directa:
8.En más del 95% de los casos el origen de una
f) Hidrocarburos aromáticos
tromboembolia pulmonar es:
g) Aflatoxina B
h) Cloruro de vinilo a) Necrosis isquémica
i) Griseofulvina b) Embolia aérea
j) Agentes alquilantes y acilantes c) Embolia de líquido amniótico
d) Embolia grasa y medula ósea
3.A 20 year old woman suffered extensive burns
e) Trombosis venosa profunda
and developed large irregular scars over her
hands. These scars limite the moviment of her 9.¿Es un producto de las celulas endoteliales
fingers, and she unable to fully extend them. normales y un cofactor esencial para la unión de
What are these lesión called? las plaquetas a los elementos de la MEC?
a) Desmoids a) Proteina C
b) Aggressive fibromatoses b) Acetil Coenzima A
c) Contractures c) ATP
d) Keloids d) Factor de Von Willebrand
e) Wound dehiscences e) Fibrina
4.Tambien conocido como factor de dispersión: 10.La formación insuficiente del tejido de
granulación puede causar:
a) FGF
b) VEGF a) Tejido necrótico y cuerpos extraños
c) HGF b) Diabetes y cáncer
d) TGF-a c) Contracturas y cicatrización queloide
e) PDGF d) Movilización precoz de las heridas
e) Deshiscencia de la herida y ulceración.
5.Which the following is the most common site
for arteriolar thromboembolization? 11.Which of the following subtances is an
anticoagulant secreted by endotelial cells?
a) Brain
b) Heart a) Von willebrand factor
c) Kidneys b) Prothrombin
d) Lower extremities c) Inhibitor of plasminogen activator
e) Eyes d) Thrombomodulin
e) Tissue factor
6.Citocinas que participan en la curación de las
heridas:

a) Il-2 e il-4
12.Es una de las consecuencias importantes 18.Wich growth factor is the most potent
cuando el embolo produce una oclusión vascular promotor of angiogénesis in the granulation
parcial o completa: tissue?

a) Necrosis isquémica a) Angiotensin


b) Embolo aérea b) Platelet derived growth factor
c) Embolia pulmonar c) Tumor growth factor b
d) Embolia grasa y medula osea d) Vascular endotelial growth factor
e) Embolo de líquido amniótico e) Epidermal Growth factor

13.Which of the following tumors occurs most 19. 60 year old man had congestive heart failure
often in the ovary? and sodium retention. Pitting edema of the lower
extremities in this patientis most likely associated
a) Condroma
with n increased blood level of:
b) Hemangioma
c) Leiomyoma a) Albumin
d) Transitional cell papiloma b) Globulin
e) Cystadenoma c) Aldosterone
d) Troponin
14.Son carcinógenos de acción indirecta:
e) Plasminogen
a) Beta propiolactona
20. Nombre que recibe la formación de vasos
b) Dimetil sulfato, Ciclofosfamida
durante el desarrollo embrionario
c) 1 Acetil-imidazol
d) Hidrocarburos aromáticos a) Hematopoyesis
e) Diepoxibutano b) Vasculogenia
c) Angiogenia
15.Es una alteración de escasa frecuencia que se
d) Neovascularizacion
debe a una proliferación exuberante de
fibroblastos y otros elementos de tejido 21. Son ejemplos de factores sistémicos que
conjuntivo afectan la cicatrización

a) Lesiones desmoides a) Hematoma y tejido necrótico


b) Hiperplasia beninga b) Infeccion local y cuerpos extraños
c) Neoplasias c) Alergia a suturas y denervación
d) Carne orgullosa d) Diabetes o cáncer
e) Contracturas y cicatrización queloide
16.Mediadores que son potentes vasodilatadores
e inhibidores de la agregación plaquetaria 22. Se produce cuando un embolo venoso
atraviesa una comunicación interauricular o
a) Especies reactivas de oxígeno y NO
interventricular para llegar a la circulación
b) prostaciclina y el oxido nítrico
sistémica:
c) Factores de coagulación
d) Glucoproteínas aunadas al endotelio a) Embolia aérea
b) Embolia de grasa
17.En las endocrinopatías en los síndromes
c) Embolia paradójica
paraneoplasicos se caracterizan por:
d) Embolia de medula osea
a) Producción hormona ectópica
23. Célula con capacidad de autorrenovarse y de
b) Secreción de mucopolisacáridos
generar estirpes de células diferenciadas
c) Producción de lípidos
d) Secreción de proteínas a) Células estables
e) Producción de polisacáridos b) Celulas madre
c) Células pluripotenciales b) Liberacion de catecolaminas y
d) Células madre embrionarias antidiuréticos
e) Células quiescientes c) Sintesis de oxido nítrico y anuria
d) Glucolisis anaeróbica y acidosis láctica
24. ¿Qué nombre recibe la mutacion especifica
metabólica
del factor V?
e) Necrosis tubular aguda con insuficiencia
a) Mutacion de Leiden renal
b) Defecto de a fibrinólisis
30.The most common cause of mitral stenosis in
c) Deficiencia de proteína COS
adults is:
d) Déficit de antitrombina III
e) Homocistenemia a) Floppy mital valve
b) Rheumatic endocarditis
25Calcificación de válvula mitral área funcional de
c) Bacterial endocarditis
menos de 1cm3, ocacionan en primera instancia:
d) Mitral anular calcification
a) Hiperemia e) Nonbacterial trombotic endocarditis
b) Hemorragia
31.Tanto células como núcleos desarrollan
c) Congestión Pulmonar
variaciones en tamaño y forma:
d) Hemotorax
e) Neumotorax a) Diferenciación
b) Anaplasia
26.¿Cuál es la diferencia entre hematoma,
c) Pleomorfismo
petequia, purputa y equimosis?
d) Perdida de polaridad
a) La localización anatómica de cada una de e) Hipercromatismo
ellas
32.La función de esta proteasa degrada al
b) La causa de la hemorragia
colágeno tipo IV y …….. a la metástasis de celulas
c) El tamaño de las hemorragias
tumorales?
d) La capa en donde ocurre , mucosa,
submucosa y serosa a) Caspazas
e) Por los órganos donde se presentan b) Metaloproteinasas
c) Telomerasa
27.¿El edema de origen renal se caracteriza por?
d) Anhidrasa carbonica
a) Postural
33,Genes incluidos en el genoma humano que
b) Presentarse en la región sacra
regula el crecimiento y diferenciación celular
c) Localizaciones en tejido laxo
a) Genes implicados en la reparación del
28.Son patologías que producen edema por
ADN
aumento de la presión hidrostática del vaso:
b) Protooncogenes
a) Neoplasis y filariasis c) P53
b) Aumeto de la secreción renina- d) Oncogenes (PP:84)
Angiotensina-Aldosterona e) Genes inductores de apoptosis
c) Glomerulopatias y malnutrición
34.Son aquellos que se caracterizan por aplicar
d) Pericarditis constictiva y trombosis
frenos a la proliferación celular, formando una
e) Gastroenteropatias con perdida de
red de puntos de control que impiden el
proteínas
crecimiento incontrolado
29.La fase progresiva del shock se caracteriza por:
a) Genes inhibidores apoptosis
a) Activación del sistema renina- b) Genes de la apotosis
angiotensina c) Genes supresores tumorales
d) Genes pre apoptosicos a) A la ingestión del metabolito fúngico
e) Genes reparadores aflatoxina B2
b) Mutacion del P450
35.Correlacionar las siguientes columnas
c) Mutacion de G6PDH
contestando en su hoja de respuesta:
d) Al efecto iniciador de fármacos con daño
a) Angiosarcoma / mesenquimatosa al hígado
maligna
41. La ciclina E/CDK2 controla la transición entre:
b) Coriocarcinoma o Seminoma /Epitelio
maligna (D) a) G1-S
c) Hemangioma o Linfangioma b) G0-G1
/Mesenquima benigna
EXAMENES VASQUEZ GUERA
d) Cistoadenoma o papiloma escamoso
/epitelial benigna 42.- La patología o la anytomia patológica es una
e) Glaucoma/ No es neoplasia (C) rama de la medicina que estudia:
36.La caquexia en el cáncer se da por el efecto de
Los cambios estructurales y funcionales de la
la siguiente citosina.
enfermedad.
a) PDFG
b) TGB 43.- Causa predisponente de la calcificación
c) TNF metastasica.
d) Adenalina
e) IL-8 a) medio acido
b) medio alcalino
37.Celulas linfoides que son cpaces de destruir c) medio neutro
celulas tumorales sin una sensibilización previa: d) tejido lesionado
a) Netrofilos e) tejido necrosado
b) Natural killer
c) Eosinofilos 44.- La biopsia de elección para una lesión
d) Celulas CD hiperpigmentada localizada en la piel y que mide
e) Macrófagos 1cm de diámetro, es:

38.Marcador tumoral relacionado con carcino de a) Incisional


colon y páncreas: b) Excisional
c) BAAF
a) Alfa fetoproteina
d) Por raspado
b) Antígeno prostático
e) Endoscopia
c) Antigeno carcinoembrionario
d) Enolasa neuronal especifica
45.- El fijador más utilizado para los tejidos
e) Inmunoglobulinas
obtenidos por biopsia o necropsia es:
39.Es la perdida de grasa corporal , masa magra,
debilidad profunda anorexia y anemia: a) Alcohol de 96%
b) Toluol de 96%
a) Inanición c) Benzol al 10%
b) Caquexia d) Formol al 10%
c) Marasmatica e) Citospray
d) Kwashorkor

40.¿A qué se asocia el carcinoma hepatocelular? 46.- Es causa de calcificación distrofiaca:

a) Incremento de la secreción de PTH


b) Destrucción ósea c) Caspasa 4
c) Intoxicación con vitamina D d) Caspasa 8 y 10
d) Focos de necrosis e) Caspasa 6
e) Insuficiencia renal
52- ¿De las causas de lesión celular, cuál de las
47.- El apoptosoma está formado por las siguientes es la que afecta de manera directa a la
siguientes sustancias: respiración oxidativa aeróbica en la célula?

a) TNF-1 se une a FAS a) Los agentes físicos


b) FLIP se une a la procaspasa 8 b) La isquemia y la hipoxia
c) IAP se une a la caspasa 3 c) Los trastornos genéticos
d) Citocromo C se une APAF-1 d) Las reacciones inmunológicas.
e) BH3 bloquea a las BCL
48.- What chemical mediators are responsable 53.- La telomerasa es un complejo:
for increasing the blood flow to an are of
inflamation? a) ADN- proteína
b) ARN- Proteina
a) Histamine, Prostaglandins and nitric c) Proteina Bcl2
oxide d) Proteina IAP
b) Integrins, selextins, icam-1 vcam1 e) TNF alfa
c) Thromboxane A2, LTB4, LTC4, LTE4
d) Tumor necrosis factor and interluukin 1 54.- En lesión celular reversible debida a isquemia
e) Chemokines CXC, CC, CX3C / hipoxia la primera alteración bioquímica que se
presenta en:
49.- Forma por la cual la telomerasa inhibe el
acortamiento de los telomeros: a) La síntesis proteíca
b) Alteraciones en el nucleo
a) Añade nucleótidos a los extremos del c) Disminución de la fosforilacion oxidativa
cromosoma mitocondrial
b) Repara las lesiones de las cadenas de d) Lesión del citoesqueleto
ADN
c) Repara las lesiones de la cadena de ARN 55.- Son fenómenos que caracterizan e indican
d) Bloque a los radicales libres derivados que la lesión celular es irreversible:
del oxigeno
e) Bloquea el ciclo celular. a) Las alteraciones de los ribosomas
b) La fragmentación del núcleo celular
50.- ¿Cuál es la característica del nucleo celular c) La disfunción mitocondrial, profundos
en la apoptosis? trastornos en la función de las
membranas celulares
a) Cariorerexis d) La dilatación del retículo endoplásmico
b) Cariolisis rugoso
c) Eosinofilia
d) Condensacion de la cromatina 56.- ¿Cuáles de los siguientes son factores
e) Hipercromatismo celular. antioxidantes enzimáticos?

51.- La via extrínseca de la apoptosis activa a: a) Transferrina y Lactoferrina


b) Vitamina A y vitamina E
a) Caspasa 1 y 3 c) Lactoferrina y ceruloplasmina
b) Caspasa 9
d) Catalasa, Glutation peroxidasa y
superoxido dismutasa. a) De epitelio cilíndrico a epitelio plano
estratificado
57.- La calcificación distrofica se presenta en: b) De tejido conectivo a cartílago
c) De epitelio plano a epitelio cilíndrico
a) Hiperparatiroidismo d) De tejido muscular a óseo.
b) Mieloma multiple
c) Ateromatosis 63.- La acumulación intracelular de polvo de
d) Insuficiencia renal carbón recibe el nombre de:
e) Sarcoidosis.
a) Glucogenosis
58.- Adaptación celular del crecimiento que b) Cuerpos de Rusesell
puede ser terreno fértil en donde se inicie una c) Cuerpos de Councilman
proliferación maligna: d) Antracosis
e) Neumoconiosis
a) Atrofia
b) Hipertrofia 64.- ¿A cual de los pigmentos endógenos que
c) Hiperplasia patológica (revisar) se puede acumular intracelularmente se le llama
d) Hiperplasia fisiológica “Pigmento del envejecimiento”?
e) Aplasia
a) Lipofuscina
59.- son efectos sistémicos provocadas por la IL-1, b) Bilirrubina
el TNF en la reacción inflamatoria: c) Hemosiderina
d) Melanina
a) Fiebre, pérdida del apetito, sueño
b) Aumento de la adhesión leucocitaria 65.- Son componentes de la inflamación aguda:
c) Aumento de la síntesis de
prostaglandinas a) Modificación en el calibre de los vasos y
d) Proliferación de fibroblastos y aumento en la estructura de la microvasculatura.
de la síntesis de colágena b) Gran destrucción tisular y reparación por
e) Aumento de la actividad procoagulante y tejido fibroso
disminución de la anticoagulante. c) Atrofia, hipertrofia e hiperplasia
d) Regeneración y cicatrización por tejido
60.- La atrofia puede ser: conjuntivo.

a) Hormonal y compensadora 66.- El aumento de la permeabilidad vascular da


b) Benigna y maligna como resultado:
c) Fisiológica y patológica
d) Reactiva e inducida a) Aumento de las proteínas plasmáticas
(albumina)
61.- De las referidas a continuación, señale en b) Hemorragia en los tejidos
que muestras puede realizarse citología c) Salida de un fluido rico en células y
exfoliativa: proteínas al intersticio.
a) hueso, cartílago y musculo esquelético d) Que se inicien los mecanismos de
b) orina, expectoración y liquido de ascitis reparación
c) dermis, epidermis y tejido conectivo laxo e) Que se liberen los mediadores de la
d) hígado, bazo y mesenterio. inflamación.

62.- Tipo de metaplasia más frecuente:


67.- Selectinas, integrinas y glucoproteínas tipo d) E-selectina, P-Selectina y L-selectina
mucina actúan como: e) Colectinas y proteínas tipo mucina.

a) Sustancias quimiotacticas edogenas 72.- A 60 – year-old man who had generalized


b) Opsoninas aterosclerosis died 24 hours after having a stroke.
c) Sustancias que aumentan la Acerebral infarct was found at autopsy. Necrosis
permeabilidad vascular of the brain is classified as:
d) Sustancias vasodilatadoras
e) Moléculas de adhesión a) Coagulative necrosis
b) Liquefactive necrosis
68.- A 60-year-old obese man was admitted to c) Fat necrosis
the hospital for treatment of alcoholism. He has d) Fibrinoid necrosis
diabetes mellitus. A liver biopsy ws performed, e) Caseous necrosis
and the specimen showed that liver cells contain
increased amounts of: 73.- Las quimiocinas CX3C son moléculas de
adhesión para:
a) Hemosiderin
b) Bile a) Monocitos y Linfocitos T
c) Triglycerides b) Macrófagos y celulas B
d) Bilirubin c) Eosinofilos y basófilos
e) Insulin d) Neutrófilos y macrófagos
e) Neutrófilos y basófilos
69.- Peroxidacion de los lípidos de la membrana,
modificación oxidativa de las proteínas y lesiones 74.- Los cuerpos de Russell son acumulos de:
del DNA son efectos relevantes para la lesión a) inmunoglobulinas en células plasmáticas.
celular causados por: b) Proteínas en las células tubulares
c) Grasa en hepatocitos
a) Hipoxia d) Colesterol en macrófagos de la lámina propia
b) Isquemia de las vesículas biliar.
c) Acidosis láctica e) Glucogeno en las celulas epiteliales de los
d) Radicales libres derivados del oxigeno tubulos contorneados distales.
(ERO)
e) Falta de producción de ATP 75.- Son productos del metabolismo del acido
araquidónico por la via de las ciclooxigenasas:
70.- los anticuerpos de la IgG, C3B y las colectinas
funcionan como: a) Tromboxano y prostaglandinas
b) Leucotrienos y lipoxinas
a) Quimiotacticas c) Ion superoxido y peróxido de hidrogeno
b) Moléculas de adhesión d) Sistema plasminogeno plasmina
c) Opsoninas e) Fibrinopeptidos.
d) Vasodilatadores
e) Sustancias que estimulan la reparación 76.- Grupo de mediadores químicos que
tisular. producen inhibición de la quimiotaxis de
neutrófilos y la adherencia de los monocitos.
71.- Principales quimiotacticos endógenos:
a) Leucotrienos
a) C3B y C3BI b) Monocinas
b) C5A y LTB4 c) Lipoxinas
c) Fragmento Fc de IgG y colectinas d) Prostaglandinas
e) Aminas vasoactivas d) Defensinas
e) Lactoferrina
77.- Bradykinin is produced from a high-moecular
weight kinoinogen circulating in the blood. This 82.- Nitric oxide sinthesis is augmented in
reaction is mediated by: endotelial cells by a calmodulin medianted influex
of which element?
a) Coagulation factor X
b) Kallikrein a) Calcium
c) Hageman factor b) Sodium
d) Complement C3 c) Potassium
e) Protein d) Oxygen
e) Nitrate
78.- Estructura formada por un acumulo de
celulas epiteloides, rodeadas por un collar de 83.- Pus o material purulento se encuentra
linfocitos y celulas plasmáticas se presenta en: formado por:

a) Brucelosis a) Macrófagos y linfocitos


b) E. Coli b) Deritus celulares, neutrófilos y liquido de
c) Clamydia Trachomatis edema
d) Neisseria Gonorrheoae c) Exudado seroso y hemorrágico
e) Salmonella d) Exudado fibrinoso y leucotitos
e) Tejido de cicatrización
79.- Quimioxina que da lugar a la activación dy
quimiotaxis de los neutrófilos , con actividad 84.- la causa mas frecuente de la esteatosis
limitada sobre monocitos y eosinofilos: hepática son:

a) CXC o alfa a) Bacterias, hongos


b) CC o beta b) Traumatismos, deshidratación
c) C o gamma c) Radiaciones ionizantes, quemaduras
d) CXC3C d) Alcoholismo, desnutrricion
e) CXCR4 e) Choque eléctrico, parasitos

80.- Prostaglandina que produce hiperalgesia, 85.- Son ejemplos de inflamación serosa:
que hace que la piel presente hipersensibilidad
(dolor y fiebre) a) Apendicitis supurativa aguda,
furunculosis
a) PGE2 b) Ulceración duodenal y gástrica
b) PGD2 c) Infecciones virales y quemaduras de 2
c) PGF2 grado
d) PGI2 d) Pericarditis fibrinosa, pleuritis fibrinosa
e) TxA2 e) Ulcera de mucosa oral, de miembros.

81.- sistema bactericida más eficaz de los 86.- Mediador químicos que produce
neutrófilos vasodilatación y su efecto es 10 000 veces mas
potente que la histamina:
a) H2O2 + MPO + Cl
b) BPI Proteina bactericida por incremento a) Prostaglandina E2
de la permeabilidad b) Prostaciclina
c) PBP Proteina básica principal c) Factor Activado de las Plaquetas
d) Interferon 92.- LOS CUERPOS DE MALLORY SON ACUMULOS
e) Bradicinina DE:

A) PROTEÍNAS EN LAS CÉLULAS TUBULARES


87.- Factor del complemento C5a se activa por la
B) GRASA EN HEPATOCITOS
via ¿? Para activar netrofilos y monocitos para
C) PROTEINAS HIALINO ALCOHOLICAS EN HIGADO
aumentar la leberacion de mas mediadores
D) COLESTEROL EN MACRÓFAGOS DE LA LAMINA
inflamatorios:
PROPIA DE LA VESÍCULA BILIAR
E) GLUCÓGENO EN LAS CÉLULAS EPITELIALES DE
a) Via clásica
LOS TUBULOS CONTORNEADOS DÍSTALES
b) Via intrínseca
c) Via lipooxigenasa
EXAMEN II
d) Via ciclooxigenasa
e) Via de las lectinas
93.- PIGMENTO ENDÓGENO, NO DERIVADO DE LA
HEMOGLOBINA:
88.- Factor de crecimiento relacionado con la
hipertrofia por aumento de la carga de trabajo: A) HEMOSIDERINA
B) BILIRRUBINA
a) VEGF C) ANTRACOSIS
b) FGF D) MELANINA
c) HGF E) TATUAJE
d) TGF beta
e) TNF 94.- CALCIFICACIÓN PATOLÓGICA QUE SE
ACOMPAÑA DE NIVELES ELEVADOS DE CALCIO
89.- LA ATROFIA SE PRODUCE POR: SÉRICO ES:
A) MENOR SINTESIS DE FOSFOLIPIDOS
B) AUMENTO EN LA DEGRADACION LIPIDOS A) DISTROFICA
C) MENOR SINTESIS DE CARBOHIDRATOS B) METASTÁSICA
D) AUMNETO EN LA SINTESIS DE COLESTEROL C) DISPLASICA
E) AUMENTO EN LA DEGRADACION DE D) HORMONAL
PROTEINAS
95.- LA INICIACIÓN DE LA CALCIFICACIÓN
90.- EL RECEPTOR DE MUERTE MAS CONOCIDO DISTROFICA INTRACELULAR SE INICIA EN:
EN LA APOPTOSIS SE LLAMA:
A) RETÍCULO ENDOPLASMICO RUGOSO
A) Smac/DIABLO
B) RETÍCULO ENDOPLASMICO LISO
B) IAP
C) MITOCONDRIAS
C) FAS
D) MEMBRANA CELULAR
D) Bcl 2
E) NÚCLEO
E) Bax
96.- EL ASPECTO VÍTREO HOMOGÉNEO Y ROSADO
91.- SON PROTEINAS PLASMATICAS DE LA FASE
A LOS CORTES HISTOLÓGICOS HABITUALES
AGUDA EN LA INFLAMACION:
TEÑIDO CON HEMATOXILINA Y EOSINA SE
A) CATALASA , DISMUTASA , PEROXIDASA LLAMA:
B) TRANSFERRINA, LACTOFERRINA, FERRITINA
A) CAMBIO GRASO
C) ALFA GLICEROFOSFATO, ACETATO
B) ACUMULACIÓN INTRACELULAR DE
D) PROTEINA C REACTIVA, FIBRINOGENO
GLUCÓGENO
PROTEINA AMILOIDE
C) CAMBIO HIALINO
E) DESMINA,V IMENTINA, QUERATINA
D) COLESTEROLOSIS
E) ATEROESCLEROSIS
102.- LAS PRINCIPALES CITOCINAS QUE
97.- EL LLAMADO PIGMENTO DE DESGASTE SE INTERVIENEN EN LA RESPUESTA INFLAMATORIA
FORMA CON MUCHA FRECUENCIA EN: SON:

A) METAPLASIA A) QUIMIOCINAS
B) HIPERPLASIA B) INTERFERON ALFA, INTERFERON BETA
C) HIPERTROFIA C) IL1 Y TNF
D) ATROFIA D) PROSTAGLANDINAS Y TROMBOXANO
E) HIPOPLASIA E) LEUCOTRIENOS

98.- LOS MACRÓFAGOS MODIFICADOS O 103.- LOS SIGUIENTES SON EFECTOS DE LA


CÉLULAS EPITELIOIDES SON CARACTERÍSTICAS DE: HIPOXIA EN LA CÉLULA:

A) INFLAMACIÓN AGUDA A) AUMENTO DE LA PRODUCCIÓN DE ATP


B) INFLAMACIÓN CRÓNICA B) AUMENTO DEL PH INTRACELULAR
C) INFLAMACIÓN GRANULOMATOSA C) DISMINUCIÓN DEL CALCIO INTRACELULAR
D) REPARACIÓN TISULAR D) DISMINUCIÓN DE LA CONCENTRACIÓN DE
E) REACCIONES ALÉRGICAS SODIO Y AGUA
E) DISMINUCIÓN DE LA FOSFORILACIÓN
99.- ES EJEMPLO DE INFLAMACIÓN PURULENTA: OXIDATIVA

A) DERRAME PLEURAL
104.- EL FIJADOR DE TEJIDOS UTILIZADO DE
B) AMPOLLAS DE LAS QUEMADURAS
FORMA RUTINARIA:
C) VESÍCULAS CAUSADAS POR INFECCIÓN VIRAL
(HERPES) A) XILOL AL 40%
D) LIQUIDO DE ASCITIS B) ALCOHOL DEL 96
E) APENDICITIS AGUDA SUPURATIVA C) TOLUENO
D) FORMOL AL 10%
100.- SE DENOMINA BIOPSIA AL ESTUDIO DE: E) ALCOHOL ABSOLUTO

A) LAS CAVIDADES DE UN CADÁVER


105.- MEDIADORES RELACIONADOS CON EL
B) UN FROTIS O EXTENDIDO CELULAR
DOLOR EN LA INFLAMACIÓN AGUDA:
C) UN FRAGMENTO DE TEJIDO
D) CÉLULAS EN DESCAMACIÓN EN CUALQUIER A) PAF Y SUSTANCIA P
LIQUIDO CORPORAL B) QUIMIOCINAS Y PRODUCTOS BACTERIANOS
E) LAVADO BRONQUIAL C) PROSTAGLANDINAS Y BRADICININA
D) LEUCOTRIENOS Y TROMBOXANO A2
101.- SON SUSTANCIAS QUE ACTÚAN COMO E) C5a, C5bi Y MAC
ANTIOXIDANTES NATURALES:
106.- SON REACCIONES DE LA FASE AGUDA
A) SUPERÓXIDO DISMUTASA, GLUTATION
PROVOCADAS POR LA IL-1, EL TNF
PEROXIDASA
B) TETRACLORURO DE CARBONO Y CLORURO DE A) FIEBRE, PERDIDA DEL APETITO
MERCÚRIO B) AUMENTO DE LA ADHESIÓN LEUCOCITARIA
C) ÓXIDO NÍTRICO Y XANTINA OXIDASA C) AUMENTO DE SÍNTESIS DE PROSTAGLANDINAS
D) METALES DE TRANSICIÓN COMO HIERRO Y D) PROLIFERACIÓN DE FIBROBLASTOS Y
COBRE AUMENTO DE LA SISTESIS DE COLÁGENA
E) PERÓXIDO DE HIDRÓGENO Y ION HIDROXILO E) AUMENTO DE LA ACTIVIDAD PROCOAGULANTE
Y DISMINUCIÓN DE LA ANTICOAGULANTE
107.- TIPO DE NECROSIS QUE SE SUCEDE
SECUNDARIA A LA LESIÓN HIPOXICA LETAL EN LA 112.- ES CARACTERÍSTICA DE LA INFLAMACIÓN
MAYORÍA DE LOS TEJIDOS: CRÓNICA:

A) COLICUATIVA A) INFILTRACIÓN DE NEUTRÓFILOS


B) GRASA B) DURACIÓN MUY CORTA (MINUTOS, HORAS O
C) CASEOSA ALGUNOS DÍAS)
D) COAGULATIVA C) INFILTRACIÓN DE CÉLULAS MONONUCLEARES
E) GANGRENA HUMEDA (MACRÓFAGOS)
D) POCA DESTRUCCIÓN TISULAR
18.- LA LENTIFICACIÓN DE LA CIRCULACIÓN EN LA E) ES UN TIPO DE INFLAMACIÓN DE MUY RARA
INFLAMACIÓN AGUDA SE DEBE A: PRESENTACIÓN

A) LA VASOCONSTRICCIÓN ARTERIOLAR
113.- ES CARACTERISTICA DE LA CALCIFICACION
B) VASODILATACIÓN VENULAR
METASTASICA:
C) EL AUMENTO DE LA PERMEABILIDAD DE LA
MICROVASCULATURA A) NIVELES NORMALES DE CALCIO
D) LA APERTURA DE NUEVOS LECHOS B) SE PRESENTA EN NECROSIS CASEOSA
VASCULARES C) PUEDE SER INTRA O EXTRACELULAR
E) LA MARGINACIÓN LEUCOCITARIA D) SE PRESENTA EN TEJIDOS NORMALES
E) PROVOCA DISFUNCION ORGANICA
109.- SON SUSTANCIAS QUE ACTÚAN COMO
OPSONINAS: 114.- LAS SUSTANCIAS QUÍMICAS PUEDEN
PRODUCIR LESIÓN CELULAR POR:
A) INTEGRINAS, SELECTINAS E
INMUNOGLOBULINAS A) ACCIÓN DIRECTA O INDIRECTA
B) C5A, LTB4, IL-8 B) ESTÍMULOS ENDÓGENOS
C) HISTAMINA, SEROTONINA C) ESTÍMULOS EXÓGENOS
D) BRADICININA, CALICREÍNA D) FRAGMENTACIÓN DEL ADN
E) C3B, IGG, Y COLECTINAS E) CONDENSACIÓN DE LA CROMATINA

110.- LAS PROLONGACIONES O PSEUDOPODOS 115.- EL DESPRENDIMIENTO DE LOS RIBOSOMAS


DE LOS LEUCOCITOS CONTIENEN EN SU INTERIOR DEL RER EN LA LESIÓN CELULAR PRODUCE:
FILAMENTOS DE:
A) DISMINUCIÓN DE LA ACTIVIDAD DE LA BOMBA
A) COLÁGENO Y ELASTINA DE NA-K
B) COLÁGENO TIPO III B) DISMINUCIÓN DE LA FOSFORILACIÓN
C) PROTEOGLUCANOS OXIDATIVA
D) ACTINA Y MIOSINA C) AUMENTA LA GLUCOLISIS ANAEROBIA
E) MICROTÚBULOS D) DISMINUCIÓN DE LA SÍNTESIS DE PROTEÍNAS
E) AUMENTO DE LA CONCENTRACIÓN DEL NA
111.- EN LA HEMOSIDEROSIS SISTÉMICA, EL INTRACELULAR
PGIMENTO NO LESIONA A LAS CÉLULAS, PERO SI
SU ACUMULACIÓN ES EXCESIVA, PRODUCE UNA 116.- LOS CUERPOS DE WEIBEL PALADE DE LAS
ENFERMEDAD CONOCIDA COMO: CELULAS ENDOTELIALES CONTIENEN:

A) OCRONOSIS A) SELECTINA P
B) HEMOCROMATOSIS B) INTEGRINAS
C) ALCAPTONURIA C) SELECTINA L
D) ANTRACOSIS D) ICAM
E) ASBESTOSIS
117.- SON EJEMPLO DE SUBSTANCIAS QUÍMICAS 122.- ENTRE LOS FACTORES DE RIESGO PARA EL
QUE CAUSAN LESIÓN CELULAR DE MANERA CA CU SE ENCUENTRA, EXCEPTO:
INDIRECTA AL TRANSFORMARSE, POR VÍA
A) VAGINOSIS BACTERIANA DE REPETICIÓN
ENZIMÁTICA, EN RADICALES LIBRES:
B) CANDIDOSIS VULVOVAGINAL REPETITIVA
A) CLORURO MERCÚRICO C) ENFERMEDADES DE TRANSMISION SEXUAL,
B) PARACETAMOL Y TETRACLORURO DE PROMISCUIDAD
CARBONO D) INICIO DE ACTIVIDAD SEXUAL DESPUÉS DE LOS
C) CIANURO, CICUTA 30 AÑOS
D) QUIMIOTERÁPICOS
123.- WHICH OF THE FOLLOWING MEDIATORS OF
118.- EL PATRÓN MORFOLÓGICO DE NECROSIS INFLAMMATION HAS CHEMOTACTIC PROPERTIES
EN EL QUE PREDOMINA LA DEGRADACIÓN AND IS INCREASED IN PERSONS TAKING ASPIRIN?
ENZIMÁTICA ES LA
A) THROMBOXANE A2
A) N. LICUEFACTIVA B) PROSTAGLANDIN E2
B) N. GANGRENOSA C) PLATELET-ACTIVATING FACTOR
C) N. COAGULATIVA D) LEUKOTRIENE B4
D) N. CASEOSA E) INTERLEUKIN-1

119.- FACTORES DE ADHESION QUE PERTENECEN 124.- NITRIC OXIDE SYNTHESIS IS AUGMENTED IN
A LAS INMUNOGLOBULINAS ENDOTHELIAL CELLS BY A CALMODULIN-
MEDIATED INFLUX OF WHICH ELEMENT?
A) LIPOXINAS
B) SELECTINAS A) CALCIUM
C) ICAM, VECAM, PCAM B) SODIUM
D) FACTOR DE LEWIS X C) POTASSIUM
D) OXYGEN
120.- ES UN MEDIADOR QUIMICO QUE TIENE E) NITRATE
ACTIVIDAD BACTERICIDAY PRODUCE
VASODILATACION POR RELAJACION DIRECTA DEL 125.- ORGANO O TEJIDO QUE AUMENTA DE
MUSCULO LISO ARTERIOLAR: TAMAÑO POR AUMENTO DE SU NUMERO DE
CELULAS:
A) PROSTAGLANDINAS
B) C3a Y c5a A) ATROFIA
C) OXIDO NITRICO B) METAPLASIA
D) FACTOR DE NECROSIS TUMORAL C) HIPERTROFIA
E) LIPOXINAS D) HIPERPLASIA
E) DISPLASIA
121.- SON FENÓMENOS QUE CARACTERIZAN E
INDICAN QUE LA LESIÓN CELULAR ES 126.- WHICH OF THE FOLLOWING MEDIATORS OF
IRREVERSIBLE: INFLAMMATION CAUSES PAIN :

A) LAS ALTERACIONES DE LOS RIBOSOMAS A) NITRIC OXIDE


B) LA FRAGMENTACIÓN DEL NÚCLEO CELULAR B) COMPLEMENT C3A
C) LA DISFUNCIÓN MITOCONDRIAL Y LA PERDIDA C) BRADYKININ
DE LA INTEGRIDAD DE LAS MEMBRANAS D) LEUKOTRIENE B4
CELULARES E) INTERLEUKIN-1
D) LA DILATACIÓN DEL RETÍCULO ENDOPLÁSMICO
RUGOSO 127.- LA INFLAMACIÓN FIBRINOSA SE
CARACTERIZA POR:
A) SE PRODUCE POR BACTERIAS PIOGENAS COMO 132.- SON COMPONENTES DE LA INFLAMACIÓN
EL ESTAFILOCOCO AGUDA:
B) SE PRESENTA EN ABSCESOS, APENDICITIS
A) MODIFICACIONES EN EL CALIBRE DE LOS
AGUDA
VASOS Y EN LA ESTRUCTURA
C) SE PRESENTA EN QUEMADURAS O
MICROVASCULATURA
INFECCIONES VIRALES
B) GRAN DESTRUCCIÓN TISULAR Y REPARACIÓN
D) EL LIQUIDO LIGERO SE LOCALIZA EN EL
POR TEJIDO FIBROSO
INTERIOR DE LA EPIDERMIS
C) ATROFIA, HIPERTROFIA E HIPERPLASIA
E) SE PRESENTA EN INFLAMACIÓN DE
D) REGENERACIÓN Y CICATRIZACIÓN POR TEJIDO
CAVIDADES CORPORALES COMO PERICARDIO Y
CONJUNTIVO
PLEURA
E) NECROSIS Y APOPTOSIS
128.- CORAZON ANORMALMENTE GRANDE POR
133.- SON FENÓMENOS LEUCOCITARIOS EN LA
OBSTRUCCIÓN VALVULAR MITRAL O AORTICA
LUZ VASCULAR:
PRODUCE:
A) DIAPÉDESIS, SUEÑO DE ONDAS LENTAS
A) HIPERPLASIA
B) QUIMIOTAXIS, ENGLOVAMIENTO
B) METAPLASIA
C) MARGINACIÓN, RODAMIENTO Y ADHESIÓN
C) DISPLASIA
D) OPSONIZACIÓN, FIEBRE
D) HIPERTROFIA
C) FAGOCITOSIS Y DEGRADACIÓN
E) ATROFIA

134.- DENTRO DE LOS EJEMPLOS ESPECÍFICOS DE


129.- UNA CARACTERÍSTICA HISTOLÓGICA DE LOS
APOPTOSIS, ESTÁ LA INDUCIDA POR LESIÓN DEL
ORGANOS QUE SUFREN ATROFIA ES:
ADN CELULAR, SEÑALE CÚAL ES EL GEN
A) HIPERPLASIA DEL RETÍCULO ENDOPLASMICO INVOLUCRADO EN ESTE MECANISMO:
RUGOSO
A) BEL 2
B) FORMACIÓN DE VACUOLAS AUTOFAGICAS
B) CED 3
C) PRESENCIA DE MITOCONDRIAS GIGANTES
C) P53
D) CARIORREXIS HINCHAZON CELULAR
D) CED 9
E) TUMEFACCION MITOCONDRIAL

135.- EL CANCER IN SITU SE LOCALIZA :


130.- PARA LA FORMACIÓN DE GRANULOMAS
INMUNITARIOS SE REQUIERE: A) LIMITADO AL GROSOR DE LA CAPA
MUSCULAR
A) PRESENCIA DE NEUTROFILOS
B) LIMITADO AL GROSOR DE LA CAPA SEROSA
B) REACCION INMUNITARIA MEDIADA POR
C) LIMITADO A LA SUBMUCOSA
LINFOCITOS T
D) LIMITADO AL GROSOR DEL EPITELIO
C) PARTICIPACIÓN DEL SISTEMA DE
E) LIMITADO AL TEJIDO GRASO
COAGULACIÓN
D) LIBERACIÓN DE ANTICUERPOS
136. PARA CADA TIPO DE MUERTE CELULAR ELIJA
LA PATOLOGÍA QUE SEA MÁS CARACTERÍSTICA.
131.- SON CAUSAS DE HIPOXIA EXCEPTO (duda):
46) APOPTOSIS (E)
A) INSUFICIENCIA CARDIORESPIRATORIA 47) NECROSIS LICUEFACTIVA (D)
B) ANEMIA 48) NECROSIS COAGULATIVA (C)
C) INTOXICACIÓN POR MONÓXIDO DE CARBONO 49) NECROSIS GRASA (B)
D) ATEROESCLEROSIS 50) NECROSIS CASEOSA (A)
E) ANEURISMAS
A) TUBERCULOSIS
B) PANCREATITIS AGUDA
C) INFARTO DE MIOCARDIO variable entre 10 y 12 cm la biopsia de elección a
D) INFECCIONES BACTERIANAS FOCALES utilizar es: en cuña
E) MUERTE CELULAR POR RECHAZO
INMUNITARIO / hepatitis viral 142.- Es la causa de calcificación distrofica: focos
de necrosis

Fotos 143.- Que molécula se unen a PAF-1 para evitar


que el aumento de la permeabilidad mitocondrial
137.- ¿A qué gen induce TP53 para que dirija a la y salida del citocromo c ocacione la apoptosis por
célula hacia la apoptosis? la activación de la procaspasa 9: BCL2 BCL- X1
MCL-1
a) RB
b) BAX 144.- De las causas de lesión celular cual de las
c) BCL-2 siguientes es la que afeca de manera direct a la
d) BCL-X1 respiración oxidativa aerobica en la celula:
e) FLIP isquemia e hipoxia

138.- ¿Son proteínas que facilitan la entrada y 145.- Dentro de los ejemplos específicos de
progresión de las células en el ciclo celular? apoptosis esta la inducida por lesión de DNA
celular, señale cual es el gen involucrado en este
a) Metaloproteinas meanismo: P53
b) Integrinas
c) Selectinas 146.- Son enzimas que producen apoptosis
d) Fibrinopeptidos celular: Caspasas
e) Ciclinas
147.- El aumento de la permeabilidad vascular da
como resultado: Exudado
139.- Factor de crecimiento que promueve la
cicatrización de heridas, la angiogenia, la 148.- En el aumento de la permeabilidad vascular
hematopoyesis y desarrollo de musculos. las citocinas, IL-1, TNF, INF gamma ocacionan :
retracción endotelial
a) HGF
b) TGF beta 149.- Potentes quimiotacticos y activadores
c) FGF leucocitarios: C5a y LTB4
d) VEGF
e) TGF alfa 150.- Mediadores que participan en el dolor:
PGE2 y bradicinina
140.- La ciclina E/CDK2 controla la transición
entre: 151.- Son ejemplos de inflamación serosa:
infecciones virales y quemaduras de segunda
a) G1 – S grado.
b) G0 – G1
c) G2 – M 152.- Prostaglandinas, leucotrienos, tromboxanos
d) M – G0 se producen por el metabolismo del: acido
e) M – G1 araquidónico

141.- si durante una colecistectomía se observan 153.- Estructura formada por acumulo de celulas
en el hígado del paciente 3 nodulos de diámetro epiteloides redeadas por un collar de linfocitos y
cel plasmáticas: granuloma
154.- son ejemplos de sustancias químicas que 167.- La biopsia de elección para una lesión para
causan lesión de manera indirecta al lesión en estomago de 1 cm de diámetro es:
transformrse en radicales libres: paracetamol y Laparoscopia
tetracloruro de carbono.
168.- El fijador más utilizado para biopsia por
aspiración es: Alcohol del 96%
155.- los cuerpos de weibel palade de las celulas
endoteliales contiene: Selectina P 169.- Es causa de la calcificación distrofica es:
Ganglio linfático tuberculoso (necrosis de tejido)
156.- las quimiocinas CX3C son moléculas de
adhesión y agentes quiotacticos para: Monocitos 170.- El apoptosoma esta formado por las
y Linfocito T siguientes sustancias: Citocromo C y APAF-1

157.- El calibre de las agujas utilizadas en la 171.- Es el factor que incrementa el flujo
biopsia de aspiración es de : 21 – 22 sanguíneo en la inflamación:

158.- La lisozima, lactoferrina, proteína básica 172.- ¿Cuál es la característica del nucleo celular
mayor destruyen microorganismos por en la apoptosis? : condensación de cromatina
mecanismos: independientes a oxigeno.
173.- La via intrínseca de la apoptosis activa a:
159.- Es la secuencia de la fagocitosis: Caspasa 9
Reconocimiento, ingestión, muerte y
degradación. 174.- Las causas de lesión celular cuales son las
que afectan directamente a la fosforilación
160.- Grupo de mediadores químicos que oxidativa: isquemia e hipoxia
produce la inhibición de las quimiotaxis y de la
adhesión de los monocitos: lipoxinas 175.- En la necroptosis el necrosoma esta
formado por: RIP 1 RIP 3 y caspasa 8
161.- Estudio de gabinete de elección para el Dx
de colecistitis crónica litiasica: Ultrasonido 176.- La Piroptosis se caracteriza por su
respectiva : Caspasa 1 y 11
162.- Mediadores que participan en la adherencia
plaquetaria (no sabemos que): ecosanoides. 177.- Son características que indican que la lesión
celular es irreversible: Disfunción mitocondrial y
163.- El TNF y la IL-1 produce vasodilatación por: daño a la membrana
liberación de histamina.
178.- Cual de estos es un mecanismo antioxidante
164.- Es una antiproteasa que inhibe a la elastasa enzimático: Catalasa , glutatión peroxidasa.
y es abundante en el tubo digestivo y pulmón:
Alfa 1 –antitripsina 179.- La sobre carga sistémica …. Se le conoce
como: Respuesta: Hemosiderosis
165.- Biopsia de elección en pacientes jóvenes
con lesión escamosa: cono cervical 180.-Terreno fértil para inductor de cáncer:
Metaplasia
AUDIO
181.- Son efectos sistémicos provocados por la IL-
166.- Sustancia que participa en la hipertrofia 1, TNF: Fiebre, sueño, disminución del apetito.
fisiológica: IGF - b
182.- La atrofia puede ser: fisiológica y patológica 198.- son mediadores químicos que los producen
…. Lipoxinas
183.- Son características morfológicas de la
apoptosis exepto: e) 199.- Ingles ……La bradicinina es producida por
….. Respuesta Calicreina
184.- ¿Cuáles son las celulas …….? Neutrofilo (d)
200.- Granuloma formado por un cumulo de
185.- El acumulo intracelular de carbono se le celulas epiteloides con un cordon linfocitario y
denomina: Antracosis multicelulas Respuesta: lepra

186.- autofagocitosis (a) 201.- Quimicina que funciona como quimiotactica


y activación de los neutrófilos CXC o Alfa
187.- son comoponenetes de la inflamación
aguda: cambios en la microvasculatura de los 201.- Prostaglandina que produce hiperalgesia:
vasos. PGE2

188.- el aumeto de la permeabilidad da como 202.- Respuesta deficiente: inmunidad innata


resultado: salida de un liquido rico en proteínas Respuesta excesiva: inmunidad adaptativa

189.- selectinas, integrinas y actúan como: 203.- Ingles …. Calcio


moléculas de adhesión.
204.- La pus esta formado por: detritos celulares,
190.- (ingles) Tratamiento de alcoholismo liquido de edema, celulas necróticas
8biopsia de hígado): triglicéridos 205.- la causa mas frecuente de esteatosis
hepática son: alcoholismo y desnutrición.
191.- peroxidacion de los lípidos de membrana,
lesiones de DNA y desplegamiento de proteínas 206.- son efectos de inflamación serosa:
son efectos de: EROS infecciones virales y quemaduras de 2do grado

192.- Los anticuerpos de la igG funcionan como: 207.- mediador que produce vasodilatación 10
opsoninas 000 veces mas potentes que la histamina: PAF
193.- Los principales quimiotacticos endógenos
son: C5a y LTB4 208.- factor del complemento que activa la via
lipooxigenasa en neutrófilo: C5a
194.- Ingles …. Un hombre de 60 años que tenia
una hemosiderosis generalizada con fallo cerebral 209.- factor de crecimiento en el que participa la
y hipertrofia por aumento de la carga de trabajo:
(B) Necrosis licuefactiva TGF beta

195.- Quimioxinas CX3C…..son moléculas de 210.- la atrofia se produce por: aumento por la
adhesión para: Monocitos y linfocitos T degradación de proteínas

196.- Los cuerpos de russell son acumulos de: 211.- El receptor de muerte mas conocido en la
inmunoglobulinas en Celulas plasmáticas apoptosis se llama: Fas (CD95)

197.- son productos del metabolismo del acido 212.- son proteínas plasmáticas de la fase aguda
araquidónico mediado por las ciclooxigenasas: en la inflamación: Proteina C reactiva, SSA,
Prostaglandinas y tromboxano fibrinógeno
213.- los cuerpos de mallory son acumulos de: E) INSUFICIENCIA RENAL
proteínas alcoholicas en hígado. ++

218.-) EL APOPTOSOMA ESTA FORMADO POR LAS


SIGUIENTES SUSTANCIAS:
214.-) CAUSA PREDISPONENTE DE LA
A) TNF-1 SE UNE A FAS
CALCIFICACION METASTASICA:
B) FLIP SE UNE A LA PROCASPASA 8
A) MEDIO ACIDO
C) IAP SE UNE A LA CASPASA 3
B) MEDIO ALCALINO
D) CITOCROMO C SE UNE APAF-1
C) MEDIO NEUTRO
E) BH3 BLOQUEA A LAS BCL
D) TEJIDO LESIONADO

E) TEJIDO NECROSODA
219.-) WHAT CHEMICAL MEDIATORS ARE
215.-) LA BIOPSIA DE ELECCIÓN PARA UNA
RESPONSIBLE FOR INCREASING THE BLOOD
LESIÓN HIPERPIGMENTADA LOCALIZADA EN LA
FLOW TO AN ARE OF INFLAMATION?
PIEL Y QUE MIDE 1CM DE DIÁMETRO, ES:
A) HISTAMINE, PROSTAGLANDINS (PGI2, PGE,
A) INCISIONAL
AND PGD2), AND NITRIC OXIDE
B) EXCISIONAL
B) INTEGRINS, SELECTINS, ICAM-1, VCAM1
C) BAAF
C) THROMBOXANE A2, LTB4, LTC4, LTD4, LTE4
D) POR RASPADO
D) TUMOR NECROSIS FACTOR AND INTERLEUKIN
E) ENDOSCOPIA 1

216.-) EL FIJADOR MÁS UTILIZADO PARA LOS E) CHEMOKINES C-X-C, C-C, CX3C.
TEJIDOS OBTENIDOS POR BIOPSIA O NECROPSIA
ES:
220.-) FORMA POR LA CUAL LA TELOMERASA
A) ALCOHOL DE 96º
INHIBE EL ACORTAMIENTO DE LOS TELOMEROS:
B) TOLUOL DE 96º
A) AÑADE NUCLEOTIDOS A LOS EXTREMOS DEL
C) BENZOL AL 10%
CROMOSOMA
D) FORMOL AL 10%
B) REPARA LAS LESIONES DE LAS CADENAS DE
E) CITOSPRAY
ADN
217.-) ES CAUSA DE LA CALCIFICACIÓN
C) REPARA LAS LESIONES DE LA CADENA DE ARN
DISTRÓFICA:
D) BLOQUEA A LOS RADICALES LIBRES
A) INCREMENTO DE LA SECRECIÓN DE
HORMONA PARATIROIDEA DERIVADOS DEL OXIGENO
B) DESTRUCCIÓN OSEA E) BLOQUEA EL CICLO CELULAR
C) INTOXICACIÓN CON VITAMINA D

D) FOCOS DE NECROSIS 221.-) ¿CUAL ES LA CARACTERISTICA DEL


NUCLEO CELULAR EN LA APOPTOSIS: 225.-) EN LESIÓN CELULAR REVERSIBLE DEBIDA A
ISQUEMIA /HIPOXIA LA PRIMERA ALTERACIÓN
A) CARIORREXIS
BIOQUÍMICA QUE SE PRESENTA EN:
B) CARIOLISIS
A) LA SÍNTESIS PROTÉICA
C) EOSINOFILIA
B) ALTERACIONES EN EL NÚCLEO
D) CONDENSACION DE LA CROMATINA
C) DISMINUCIÓN DE LA FOSFORILACIÓN
E) HIPERCROMATISMO CELULAR
OXIDATIVA MITOCONDRIAL

D) LESIÓN DEL CITOESQUELETO


222.-) LA VIA EXTRÍNSECA DE LA APOPTOSIS
226.-) SON FENÓMENOS QUE CARACTERIZAN E
ACTIVA A;
INDICAN QUE LA LESIÓN CELULAR ES
A) CASPASA 1 Y 3 IRREVERSIBLE:

B) CASPASA 9 A) LAS ALTERACIONES DE LOS RIBOSOMAS

C) CASPASA 4 B) LA FRAGMENTACIÓN DEL NÚCLEO CELULAR

D) CASPASAS 8, 10 C) LA DISFUNCIÓN MITOCONDRIAL ,


PROFUNDOS TRASTORNOS EN LA FUNCIÓN DE
E) CASPASA 6 LAS MEMBRANAS CELULARES

D) LA DILATACIÓN DEL RETÍCULO


ENDOPLÁSMICO RUGOSO
223.-) ¿DE LAS CAUSAS DE LESIÓN CELULAR, CUÁL
DE LAS SIGUIENTES ES LA QUE AFECTA DE 227.- )¿CÚALES DE LOS SIGUIENTES SON
MANERA DIRECTA A LA RESPIRACIÓN OXIDATIVA FACTORES ANTIOXIDANTES ENZIMÁTICOS?
AÉROBICA EN LA CÉLULA?
A) TRANSFERRINA Y LACTOFERRINA
A) LOS AGENTES FÍSICOS
B) VITAMINA A Y VITAMINA E
B) LA ISQUEMIA / HIPOXIA
C) LACTOFERRINA Y CÉRULOPLASMINA
C) LOS TRASTORNOS GENÉTICOS
D) CATALASA, GLUTATIÓN PEROXIDASA Y
D) LAS REACCIONES INMUNOLÓGICOS SUPERÓXIDO DISMUTASAS

228.-) LA CALCIFICACION DISTROFICA SE


PRESENTA EN:
224.-) LA TELOMERASA ES UN COMPLEJO:
A) HIPERPATIROIDISMO
A) ADN-PROTEINA
B) MIELOMA MULTIPLE
B) ARN-PROTEINA
C) ATEROMATOSIS
C) PROTEINA Bcl2
D) INSUFICIENCIA RENAL
D) PROTEINA IAP
E) SARCOIDOSIS
E) TNF ALFA
229.- )ADAPTACIÓN CELULAR DEL CRECIMIENTO
QUE PUEDE SER TERRENO FÉRTIL EN DONDE SE
INICIE UNA PROLIFERACIÓN MALIGNA:
A) ATROFIA B) DE TEJIDO CONECTIVO A CARTÍLAGO

B) HIPERTROFIA C) DE EPITELIO PLANO A EPITELIO CILÍNDRICO

C) HIPERPLASIA PATOLÓGICA D) DE TEJIDO MUSCULAR ÓSEO

D) HIPERPLASIA FISIOLÓGICA 234.-) LA ACUMULACIÓN INTRACELULAR DE


POLVO DE CARBÓN RECIBE EL NOMBRE DE:
E) APLASIA
A) GLUCOGENOSIS
230.-) SON EFECTOS SISTEMICOS PROVOCADAS
POR LA IL-I, EL TNF EN LA REACCION B) CUERPOS DE RUSSELL
INFLAMATORIA:
C) CUERPOS DE COUNCILMAN
A) FIEBRE, PÉRDIDA DEL APETITO , SUEÑO
D) ANTRACOSIS
B) AUMENTO DE LA ADHESIÓN LEUCOCITARIA
E) NEUMOCONIOSIS
C) AUMENTO DE SÍNTESIS DE
235.-) ¿A CÚAL DE LOS PIGMENTOS ENDÓGENOS
PROSTAGLANDINAS
QUE SE PUEDEN ACUMULAR
D) PROLIFERACIÓN DE FIBROBLASTOS Y INTRACELULARMENTE SE LE LLAMA “´PIGMENTO
AUMENTO DE ENVEJECIMIENTO”?

DE LA SÍNTESIS DE COLÁGENA A) LIPOFUSCINA

E) AUMENTO DE LA ACTIVIDAD PROCOAGULANTE B) BILIRRUBINA


Y
C) HEMOSIDERINA
DISMINUCIÓN DE LA ANTICOAGULANTE
D) MELANINA
231.-) LA ATROFIA PUEDE SER:
236.-) SON COMPONENTES DE LA INFLAMACIÓN
A) HORMONAL Y COMPENSADORA AGUDA:

B) BENIGNA Y MALIGNA A) MODIFICACIÓN EN EL CALIBRE DE LOS VASOS Y


EN LA ESTRUCTURA DE LA MICROVASCULATURA
C) FISIOLÓGICA Y PATOLÓGICA
B) GRAN DESTRUCCIÓN TISULAR Y REPARACIÓN
D) REACTIVA E INDUCIDA
POR TEJIDO FIBROSO
232.-) DE LAS REFERIDAS A CONTINUACIÓN,
C) ATROFIA, HIPERTROFIA E HIPERPLASIA
SEÑALE EN QUE MUESTRAS PUEDE REALIZARSE
CITOLOGIA EXFOLIATIVA: D) REGENERACIÓN Y CICATRIZACIÓN POR TEJIDO
CONJUNTIVO
A) HUESO, CARTÍLAGO Y MUSCULO ESQUELÉTICO
237.-)EL AUMENTO DE LA PERMEABILIDAD
B) ORINA, EXPECTORACIÓN Y LÍQUIDO DE ASCITIS
VASCULAR DA COMO RESULTADO:
C) DERMIS, EPIDERMIS Y TEJIDO CONECTIVO
LAXO A) AUMENTO DE LAS PROTEÍNAS PLASMÁTICAS

D) HÍGADO, BAZO Y MESENTERIO (ALBÚMINA)

233.-) TIPO DE METAPLASIA MÁS FRECUENTE: B) HEMORRAGIA EN LOS TEJIDOS

A) DE EPITELIO CILÍNDRÍCO A EPITELIO PLANO C) SALIDA DE UN FLUIDO RICO EN CELULAS


ESTRATIFICADO
Y PROTEÍNAS AL INTERSTICIO
D) QUÉ SE INICIEN LOS MECANISMOS DE A) QUIMIOTÁCTICOS

REPARACIÓN B) MOLÉCULAS DE ADHESIÓN

E) QUÉ SE LIBEREN LOS MEDIADORES DE LA C) OPSONINAS


INFLAMACIÓN
D) VASODILATADORES
238.-) SELECTINAS, INTEGRINAS Y
E) SUSTANCIAS QUE ESTIMULAN LA REPARACIÓN
GLUCOPROTEÍNAS TIPO MUCINA ACTÚAN COMO:
TISULAR
A) SUSTANCIAS QUIMIOTÁCTICAS ENDÓGENAS
242.-) PRINCIPALES QUIMIOTACTICOS
B) OPSONINAS ENDOGENOS:

C) SUSTANCIAS QUE AUMENTAN LA A) C3B Y C3BI


PERMEABILIDAD VASCULAR
B) C5A Y LTB4
D) SUSTANCIAS VASODILATADORES
C) FRAGMENTO FC DE IGG Y COLECTINAS
E) MOLÉCULAS DE ADHESIÓN
D) E-SELECTINA, P-SELECTINA Y L-SELECTINA
239-) A 60-YEAR-OLD OBESE MAN WAS
E) COLECTÍNAS Y PROTEÍNAS TIPO MUCINA
ADMITTED TO THE HOSPITAL FOR TREATMENT
OF ALCOHOLISM. HE HAS DIABETES MELLITUS. A 243.) A 60-YEAR-OLD MAN WHO HAD
LIVER BIOPSY WAS PERFORMED, AND THE GENERALIZED ATHEROSCLEROSIS DIED 24 HOURS
SPECIMEN SHOWED THAT THE LIVER CELLS AFTER HAVING A STROKE. A CEREBRAL INFARCT
CONTAIN INCREASED AMOUNTS OF WAS FOUND AT AUTOPSY. NECROSIS OF THE
BRAIN IS CLASSIFIED AS:
A) HEMOSIDERIN
A) COAGULATIVE NECROSIS
B) BILE
B) LIQUEFACTIVE NECROSIS
C) TRIGLYCERIDES
C) FAT NECROSIS
D) BILIRUBIN
D) FIBRINOID NECROSIS
E) INSULIN
E) CASEOUS NECROSIS

244,.) LAS QUIMIOCINAS CX3C SON MOLÉCULAS


240.-)PEROXIDACIÓN DE LOS LÍPIDOS DE LA
DE ADHESIÓN PARA:
MEMBRANA, MODIFICACIÓN OXIDATIVA DE LAS
PROTEÍNAS Y LESIONES DEL DNA SON EFECTOS A) MONOCITOS Y LINFOCITOS T
RELEVANTES PARA LA LESIÓN CELULAR
CAUSADAS POR: B) MACROFAGOS Y CELULAS B

A) HIPOXIA C) EOSINOFILOS Y BASOFILOS

B) ISQUEMIA D) NEUTROFILOS Y MACROFAGOS

C) ACIDOSIS LÁCTICA E) NEUTROFILOS Y BASOFILOS

D) RADICALES LIBRES DERIVADOS DEL OXÍGENO 245.-) LOS CUERPOS DE RUSSELL SON ACUMULOS
DE:
E) FALTA DE PRODUCCIÓN DE ATP
A) INMUNOGLOBULINAS EN CÉLULAS
241.-) LOS ANTICUERPOS DE LA IGG, C3B Y LAS PLASMÁTICAS
COLECTINAS FUNCIONAN COMO:
B) PROTEÍNAS EN LAS CÉLULAS TUBULARES D) NEISSERIA GONORRHEOAE

C) GRASA EN HEPATOCITOS E) SALMONELLA

D) COLESTEROL EN MACRÓFAGOS DE LA LAMINA 250.-) QUIMIOCINA QUE DA LUGAR A


PROPIA DE LA VESÍCULA BILIAR ACTIVACIÓN Y QUIMIOTAXIS DE LOS
NEUTROFILOS, CON ACTIVIDAD LIMITADA SOBRE
E) GLUCÓGENO EN LAS CÉLULAS EPITELIALES DE
MONOCITOS Y EOSINOFILOS:
LOS TUBULOS CONTORNEADOS DÍSTALES
A) C-X-C O ALFA
246.-) SON PRODUCTOS DEL METABOLISMO DEL
ÁCIDO ARAQUIDÓNICO POR LA VÍA DE LA B) C-C O BETA
CICLOOXIGENASA:
C) C O GAMMA
A) TROMBOXANO Y PROSTAGLANDINAS
D) CX 3C
B) LEUCOTRIENOS Y LIPOXINAS
E) CXCR4
C) ION SUPERÓXIDO Y PERÓXIDO DE HIDROGENO
251.-) PROSTAGLANDINA QUE PRODUCE
D) SISTEMA PLASMINOGENO PLASMINA HIPERALGESIA, QUE HACE QUE LA PIEL PRESENTE
HIPERSENSIBILIDAD (DOLOR Y FIEBRE)
E.) FIBRINOPÉPTIDOS
A) PGE2
247.-) GRUPO DE MEDIADORES QUIMICOS QUE
PRODUCEN INHIBICION DE LA QUIMIOTAXIS DE B) PGD2
NEUTROFILOS Y LA ADHERENCIA DE LOS
C) PGF2 ALFA
MONOCITOS :
D) PGI2
A) LEUCOTRIENOS B) MONOCINAS
C) LIPOXINAS D) PROSTAGLANDINAS E) TxA2
E) AMINAS VASOACTIVAS
252.-) SISTEMA BACTERICIDA MAS EFICAS DE LOS
248.-) BRADYKININ IS PRODUCED FROM A HIGH- NEUTROFILOS
MOLECULAR WEIGHT KINOINOGEN CIRCULATING
IN THE BLOOD. THIS REACTION IS MEDIATED BY A) H2O2 + MPO + Cl

A) COAGULATION FACTOR X B) BPI (PROTEINA BACTERICIDA POR


INCREMENTO DE LA PERMEABILIDAD)
B) KALLIKREIN
C) PBP (PROTEINA BASICA PRINCIPAL
C) HAGEMAN FACTOR
D) DEFENSINAS
D) COMPLEMENT C3
E) LACTOFERRINA
E) PROTEIN
253.-) NITRIC OXIDE SYNTHESIS IS AUGMENTED
249.-) ESTRUCTURA FORMADA POR UN IN ENDOTHELIAL CELLS BY A CALMODULIN
ACUMULO DE CÉLULAS EPITELIOIDES, RODEADAS MEDIATED INFLUX OF WHICH ELEMENT?
POR UN COLLAR DE LINFOCITOS Y CÉLULAS
PLASMÁTICAS SE PRESENTA EN: A) CALCIUM

A) BRUCELOSIS B) SODIUM

B) E. COLI C) POTASSIUM

C) CHLAMYDIA TRACHOMATIS D) OXYGEN


E) NITRATE 258.-)FACTOR DEL COMPLEMENTO C5a SE
ACTIVA POR LA VIA ¿ ? PARA ACTIVAR
254.-) PUS O MATERIAL PURULENTO SE
NEUTROFILOS Y MONICITOS PARA AUMENTAR LA
ENCUENTRA FORMADO POR:
LIBERACION DE MAS MEDIADORES
A) MACRÓFAGOS Y LINFOCITOS INFLAMATORIOS:

B) DETRITUS CELULARES, NEUTRÓFILOS Y A) VIA CLASICA


LIQUIDO DE EDEMA
B) VIA INTRINSECA
C) EXUDADO SEROSO Y HEMORRÁGICO
C) VIA LIPOOXIOGENASA
D) EXUDADO FIBRINOSO Y LEUCOCITOS
D) VIA CICLOOXIGENASA
E) TEJIDO DE CICATRIZACIÓN
E) VIA DE LECTINA
255.-)LA CAUSAS MÁS FRECUENTES DE LA
259.-) FACTOR DE CRECIMIEMTO RELACIONADO
ESTEATOSIS HEPATICA SON:
CON LA HIPERTROFIA POR AUMENTO DE LA
A) BACTERIAS, HONGOS CARGA DE TRABAJO:

B) TRAUMATISMOS, DESHIDRATACIÓN F) A) VEGF

C) RADIACIONES IONIZANTES, QUEMADURAS G) B) FGF

D) ALCOHOLISMO, DESNUTRICIÓN H) C) HGF

E) CHOQUE ELÉCTRICO, PARÁSITOS I) D) TGF BETA

256.-) SON EJEMPLOS DE INFLAMACIÓN SEROSA: J) E) TNF

A) APENDICITIS SUPURATIVA AGUDA, K)


FURUNCULOSIS

B) ULCERA DUODENAL Y GÁSTRICA


260.-) LA ATROFIA SE PRODUCE POR:
C) INFECCIONES VIRALES Y QUEMADURAS DE 2º
L) A) MENOR SINTESIS DE FOSFOLIPIDOS
GRADO
M) B) AUMENTO EN LA DEGRADACION
D) PERICARDITIS FIBRINOSA, PLEURITIS
LIPIDOS
FIBRINOSA
N) C) MENOR SINTESIS DE
E) ULCERA DE MUCOSA ORAL, DE MIEMBROS
CARBOHIDRATOS
257.-) MEDIADOR QUIMICO QUE PRODUCE
O) D) AUMNETO EN LA SINTESIS DE
VASODILATACION Y SU EFECTO ES 10.000 VECES
COLESTEROL
MAS POTENTE QUE LA HISTAMINA :
P) E) AUMENTO EN LA DEGRADACION DE
A) PROSTAGLANDINA E2
PROTEINAS
B) PROSTACICLINA

C) FACTOR ACTIVADOR DE LAS PLAQUETAS


261.-) EL RECEPTOR DE MUERTE MAS CONOCIDO
D) INTERFERON EN LA APOPTOSIS SE LLAMA:

E) BRADICININA A) Smac/DIABLO

B) IAP
C) FAS 265.-) LA DIGESTION EXZIMATICA ES EL EVENTO
PREDOMINANTEEN EL SIGUIENTE TIPO DE
D) Bcl 2
NECROSIS:
E) Bax
A) N. DE COAGULACION

B) N. DE LICUEFACCION *
262.-) SON PROTEINAS PLASMATICAS DE LA FASE
C) NECROSIS CASEOSA
AGUDA EN LA INFLAMACION:
D) NECROSIS GRASA
A) CATALASA , DISMUTASA , PEROXIDASA
E) GANGRENA SECA
B) TRANSFERRINA, LACTOFERRINA, FERRITINA

C) ALFA GLICEROFOSFATO, ACETATO


266.) EN EL HIGADO GRASO POR ALCOHOLISMO
D) PROTEINA C REACTIVA, FIBRINOGENO
CRONICO, PARTICIPAN LOS SIGUIENTES
PROTEINA AMILOIDE
MECANISMOS EXCEPTO:
E) DESMINA,V IMENTINA, QUERATINA
A) AUMENTO DE LA SINTESIS DE ACIDOS GRASOS
263.-) LOS CUERPOS DE MALLORY SON LIBRES
ACUMULOS DE:
B) MENOR UTILIZACION DE TRIGLICERIDOS
A) PROTEÍNAS EN LAS CÉLULAS TUBULARES
C) AUMENTO DE ALFA GLICEROFOSFATO *
B) GRASA EN HEPATOCITOS
D) BLOQUEO DE LA EXCRECION DE
C) PROTEINAS HIALINO ALCOHOLICAS EN HIGADO LIPOPROTEINAS

D) COLESTEROL EN MACRÓFAGOS DE LA LAMINA EXCRESION NORMAL DEL ACETATO


PROPIA DE LA VESÍCULA BILIAR

E) GLUCÓGENO EN LAS CÉLULAS EPITELIALES DE


267-) EN LA LESION CELULAR POR ISQUEMIA-
LOS TUBULOS CONTORNEADOS DÍSTALES
REPERFUCION HAY……

A) AUMENTO DE IONES DE Ca EN EL
PRIMER EXAMEN PARCIAL DE ANATOMIA LIQUIDO EXTRACELULAR
PATOLOGICA
B) AUMENTO DE IONES DE Ca EN EL
CITOSOL *

264.-) EN LA METAPLASIA ES VALIDO LO C) AUMENTO DE IONES DE Na EN EL


SIGUIENTE: ESPACIO EXTRACELULAR

A.) ES UN TRASTORNO DEL CRECIMIENTO D) AUMENTO DE Na EN EL CITOSOL

B.) ES UN CAMBIO REVERSIBLE * E) HAY EQUILIBRIO DE Ca EN EL ESPACIO


EXTRACELULAR Y EN EL CITOSOL
C.) AFECTA SOLO A TEJIDOS EPITELIALES
268.-) CUAL DE LOS SIGUIENTES
D.) ES UN CAMBIO IRREVERSIBLE Y PROGRESIVO
FACTORES ES ANTIAPOPTOSICO:
E.) ES UNA LESION CELULAR REVERSIBLE
A) P53

B) BCL-2
C) Bax C) CURACIÓN POR TERCERA INTENCIÓN

D) NO EXISTE CURACIÓN

273.-) ¿ALREDEDOR DE QUE DÍA SE FORMAN


EXAMEN EXTRAORDINARIO DE LOS PUENTES DE COLÁGENO EN LA
ANATOMIA PATOLOGICA CURACIÓN DE HERIDAS POR PRIMERA
INTENCIÓN?

A) INMEDIATAMENTE
269.-) DEFECTOS EN ESTOS GENES PUEDEN
B) A LAS 24 HORAS
CONVERTIRLOS EN ONCOGENES Y
CONTRIBUIR AL CRECIMIENTO CÁNCER: C) ALREDEDOR DEL DÍA 5
A) PROTOONCOGENES D) NO SE FORMAN PUENTES DE
B) GENES INDUCTORES DE APOPTOSI COLÁGENO EN LA CURACIÓN POR
C) GEN P21 PRIMERA INTENCIÓN
D) GEN P53 274.-) CAMBIO EN LA DIFERENCIACIÓN DE
UNA CÉLULA DE UN TIPO CELULAR A OTRO:
E) GENES REPARADORES
A) PLASTICIDAD DEL DESARROLLO.

B) TRANSDIFERENCIACIÓN.
270.-) ¿QUÉ VITAMINA ES NECESARIA PARA
LA HIDROXILACIÓN DEL PROCOLAGENO? C) MUTACIÓN.
A) A D) CLONACIÓN.
B) B E) DIFERENCIACIÓN ESTOCÁSTICA
C) C 275.-) LA APOPTOSIS FISIOLOGICA SE
PRESENTA EN:
D) E
A) PANCREATITIS AGUDA
E) D
B) INFECCIONES BACTERIANAS
271.-) PRECURSORES CELULARES
ENDOTELIALES QUE PARTICIPAN EN LA C) INFARTO DE MIOCARDIO
VASCULOGÉNESIS:
D )MAMA LACTANTE DURANTE EL DESTETE
A) AGENTES PROANGIOGÉNICOS
E) TUBERCULOSIS
B) ANGIOBLASTOS
276.-) CARACTERÍSTICA DE LA
C) AGENTES ANTIANGIOGÉNICOS CALCIFICACIÓN DISTROFICA:
D) ANGIOPOYETINAS A) NIVELES ELEVADOS DE CALCIO
272.-) LA CURACIÓN DE UNA HERIDA CON B) SE PRESENTA EN TEJIDOS NORMALES
DAÑO CÉLULAR EXTENSO ES UN EJEMPLO
DE… C) AFECTA RIÑONES, PULMONES

A) CURACIÓN POR PRIMERA INTENCIÓN D) NO PRODUCE DISFUNCIÓN CLÍNICA

B) CURACIÓN POR SEGUNDA INTENCIÓN E) SE PRESENTA EN ÁREAS DE NECROSIS


277.-) SU FORMACIÓN EN EL MARGEN DE LA B) ENFERMEDADES CRONICO
HERIDA AYUDA A LA CONTRACCIÓN DE LA DEGENERATIVAS
HERIDA.
C) FORMACIÓN EXCESIVA DEL TEJIDO DE
A) FIBROBLASTOS GRANULACIÓN

B) MACRÓFAGOS D) BORDES CON TEJIDOS NECROZADOS

C) MIOFIBROBLASTOS

D) PROTEÍNAS PLASMÁTICAS 282.-) ES UN PROCESO ACTIVO EN EL CUAL


LA DILATACIÓN ARTERIAL AUMENTA EL
278.-) SON SUSTANCIAS QUE ACTÚAN COMO
FLUJO DE SANGRE:
OPSONINAS:
A) CONGESTIÓN
A)INTEGRINAS, SELECTINAS E
INMUNOGLOBULINAS B) HIPOXIA

B) C5A, LTB4, IL-8 C) HIPEREMIA

C) HISTAMINA, SEROTONINA D) INFILTRACIÓN

D) BRADICININA, CALICREÍNA 283.-) ES UNA EXTRAVASACIÓN DE SANGRE


HACIA EL ESPACIO EXTRAVASCULAR PUEDE
E) C3B, IGG, Y COLECTINAS
SER EXTERNA O QUEDAR CONTENIDA EN UN
279.-) SU EPITELIZACIÓN COMPLETA ES MÁS TEJIDO :
LENTA:
A) HIPEREMIA
A) CICATRICES DE PRIMERA INTENCIÓN
B) HEMORRAGIA
B) CICATRICES DE SEGUNDA INTENCIÓN
C) CONGESTIÓN
C) CICATRICES CON BORDES LÍNEALES CON
D) LINFEDEMA
POCA PÉRDIDA DE TEJIDO CONJUNTIVO
E) NEUMOTORAX
D) HERIDAS CON LOS BORDES
CONFRONTADOS 284) These gallstones surgically removed
along with the gallbladder of a 50-year-old
280.-) SON EJEMPLOS DE INFLAMACIÓN
obese woman are composed predominantly
SEROSA. EXCEPTO:
of:
A) DERRAME PLEURAL

B) AMPOLLAS DE LAS QUEMADURAS

C) VESÍCULAS CAUSADAS POR INFECCIÓN


VIRAL (HERPES)

D) LIQUIDO DE ASCITIS

E) APENDICITIS AGUDA SUPURATIVA A) CALCIUM CARBONATE


281.-) ES UNA ALTERACIÓN LOCAL DE LA B) CHOLESTEROL
CICATRIZACIÓN.
C) BILIRUBIN
A) ESTADO NUTRICIONAL
D) BILIVERDIN
E) CALCIUM HYDROXYAPATITE 289.-) EL CUADRO SE CARACTERIZA POR LA
SÚBITA APARICIÓN DE DISNEA, CIANOSIS Y
285.-) EL PROCESO INICIAL DE
SHOCK, SEGUIDO DE ALTERACIONES
VASOCONSTRICCIÓN TRAS UNA LESIÓN ESTA
NEUROLÓGICAS QUE VAN DESDE LA CEFALEA
MEDIADO POR:
A CONVULSIONES Y COMA
A) FACTOR DE VON WILRBRAN
A) EMBOLIA AÉREA
B) ENDOTELINA.
B) EMBOLIA DE GRASA Y MEDULA ÓSEA
C) CITOSINAS
C) EMBOLIA PULMONAR
D) TROMBINA Y ENDOTELINA
D) EMBOLIA DE LÍQUIDO ANMIÓTICO
E) TROMBOPLASTINA
E) TROMBOEMBOLIA SISTÉMICA
286-) PROCESO PASIVO, DEBIDO A LA
290.-) EL SÍNDROME POR DESCOMPRESIÓN
REDUCCIÓN DEL FLUJO DE SALIDA DE UN
SE DA PRINCIPALMENTE EN:
TEJIDO, ACUMULA HEMOGLOBINA
DESOXIGENADA : A) BOXEADORES

A) HIPEREMIA B) MAESTROS

B) ERITEMA C) INGENIEROS

C) OSMOSIS D) BUCEADORES

D) CONGESTIÓN E) NADADORES

287.-) ¿CUÁLES SON LOS COMPONENTES DE 291.-) SE CONSIDERAN FACTORES DE RIESGO


LA HEMOSTASIA ? PARA PRODUCIR COLELITIASIS, EXCEPTO:

A) ENDOTELIO, PLAQUETAS Y A) EDAD


ANTITROMBINA
B) ANTICONCEPTIVOS ORALES
B) ENDOTELIO, PLAQUETAS Y MONOCITOS
C) OBESIDAD
C) ENDOTELIO, PLAQUETAS Y CASCADA DE
D) DIETA RICA EN FIBRA
COAGULACIÓN
292.-) ES LA PRINCIPAL COMPLICACIÓN DE
D) LA PARED VASCULAR, MONOCITOS Y
LOS CÁLCULOS BILIARES POR OBSTRUCCIÓN
ERITROCITOS
DEL CUELLO DE LA VESÍCULA O DEL
288.-) ¿QUE FACILITA LA ADHERENCIA Y CONDUCTO CÍSTICO:
ACTIVACIÓN PLAQUETARIA?
A) COLECISTITIS LITIASICA AGUDA
A) LA AGREGACIÓN DE MICROORGANISMOS
B) COLECISTITIS CRÓNICA
B) LA EXPOSICIÓN DE LA MATRIZ
C) COLANGITIS
EXTRACELULAR MUY TROMBOGENICA
D) COLEDOCOLITIASIS
C) LA FORMACIÓN DE TROMBINA
E) NO EXISTE NINGUNA COMPLICACIÓN
D) LA AUSENCIA DE PLAQUETAS
293.-) LOS CASOS MÁS GRAVES DONDE LA
E) FORMACION DE PLASMINA
VESÍCULA BILIAR ESTÁ NECRÓTICA, VERDE
NEGRUZCO Y PUEDE PRESENTAR D) PRÓTESIS VALVULARES E
PERFORACIONES SE DENOMINA: HIPERESTROGENISMO

A) COLECISTITIS GANGRENOSA E) SÍNDROME NEFRÓTICO Y


MIOCARDIOPATÍAS
B) COLECISTITIS AGUDA CALCULOSA
297.-) LA LESIÓN CELULAR POR
C) COLECISTITS ENFISEMATOSA AGUDA
TETRACLORURO DE CARBONO Y TYLENOL SE
D) NO EXISTE SITUACIÓN CARACTERIZAN POR:

294.-) ENTRE LOS FACTORES DE RIESGO MÁS A) DAÑO RENAL GRAVE


IMPORTANTES PARA LA PRESENTACIÓN DE
B) SU CONVERSIÓN EN METABOLITOS
IAM DESTACAN:
ACTIVOS MUY TÓXICOS
A) HIPERLIPIDEMIAS FAMILIARES,
C) LESIÓN CELULAR REVERSIBLE
ENFERMEDADES PULMONARES
OBSTRUCTIVAS, SÍNDROMES D) ACTIVAR LOS MECANISMOS DE
PARANEOPLÁSICOS APOPTOSIS

B) INSUFICIENCIA RENAL Y PULMONAR, 298.-) ADAPTACIÓN CELULAR QUE CONSISTE


DISLIPIDEMIAS, DIABETES, SEDENTARISMO, EN EL AUMENTO EN EL NÚMERO DE CÉLULAS
DEPRESIÓN, DE UN ÓRGANO O TEJIDO CON EL
CONSECUENTE AUMENTO DE VOLÚMEN DEL
C) ESTRÉS, OBESIDAD, NÍVELES ELEVADOS DE
MISMO:
COLESTEROL Y ENZIMAS PANCREÁTICAS,
HIPERHIDROSIS A) HIPERPLASIA

D) TABAQUISMO OBESIDAD, HIPERTENSIÓN B) METAPLASIA


ARTERIAL, DIABETES MELLITUS,
C) HIPERTROFIA
HIPERCOLESTEROLEMIA Y ATEROESCLEROSIS
D) ATROFIA
E) ESTRÉS, INTOXICACIÓN POR ALCOHOL Y
TRASTORNOS DEL ESTADO DE ÁNIMO E) HIPOPLASIA
295.-) ¿CUÁL ES EL VIRUS ARN ONCÓGENO? 299.-) A LA ACUMULACIÓN ANORMAL DE
TRIGLICÉRIDOS DENTRO DE LAS CÉLULAS
A) VIRUS DE LA HEPATITIS B
PARENQUIMATOSAS SE LE LLAMA:
B) VIRUS DEL PAPILOMA HUMANO
A) GLUCOGENOSIS
C) VIRUS DE LA LEUCEMIA DE CÉLULAS T
B) COLAGENOSIS
D) VIRUS DE EPSTEIN-BARR
C) ESTEATOSIS
296.-) .- ES EJEMPLO DE TROMBOSIS POR
D) ANTRACOSIS
ESTADO DE HIPERCOAGULABILIDAD
PRIMARIA: E) HEMOSIDEROSIS
A) ANTICOAGULANTE Y LUPICO Y CÁNCER 300.-) A CÚAL DE LOS PIGMENTOS
ENDÓGENOS QUE SE PUEDEN ACUMULAR
B) TABAQUISMO Y ANTICONCEPTIVOS
INTRACELULARMENTE SE LE LLAMA
ORALES
“PIGMENTO DEL ENVEJECIMIENTO”?
C) DEF. DE ANTITROMBINA III Y DE PROTEÍNA
A) LIPOFUSCINA
CYS
B) BILIRRUBINA D) HAY PERDIDA DEL TAMAÑO DE LAS
CÉLULAS ANAPLÁSICAS
C) HEMOSIDERINA
E) HAY PERDIDA DE LA FORMA DE LAS
D) MELANINA
CÉLULAS ANAPLÁSICAS
301.-) ¿QUÉ TIPO DE BIOPSIA SE UTILIZA
305.-) EL RANGO PROMEDIO PARA LA
PARA PADECIMIENTOS EN PROSTATA?
FIJACIÓN ADECUADA DE UN
A) BIOPSIA ENDOSCOPICA ESPÉCIMENQUIRÚRGICO EN FORMOL ES:

B) BIOPSIA EN SACABOCADO A) DE 1 a 3 HORAS

C) BIOPSIA EN CUÑA B) DE 8 a 12 HORAS

D) BIOPSIA POR PUNCIÓN C) DE 72 a 78 HORAS

E) ENDOSCOPICA D) DE 4 a 6 DIAS

302.-) PROBLEMAS CLÍNICOS EN RIÑONES E) DE 48 A 72 HORAS


CAUSADOS POR UN SHOCK
306.-) LA TROMBOSIS ARTERIAL
A) OLIGURIA, ANURIA . GENERALMENTE ES SECUNDARIA A:

B) POLIURIA Y ACIDOSIS A) ESTASIS CIRCULATORIA

C) HIPERPLASIA RENAL B) ATEROESCLEROSIS

D) ISQUEMIA C)CÁNCER
D) DEFICIENCIAS ENZIMÁTICAS
E) DISURIA
307.-)UN TUMOR ES CLÍNICAMENTE
303.-) SI SE INMOVILIZA UNA EXTREMIDAD DETECTABLE CUANDO TIENE UN PESO DE 1
CON UN APARATO DE YESO POR TIEMPO GRAMO, EL CUAL CONTIENE:
PROLONGADO, LOS MUSCULOS
INMOVILIZADOS PRESENTARAN: A) 10 9 CELULAS

A) HIPERPLASIA B) B) 10 6 CELULAS

B) HIPERTROFIA C) C) 10 5 CELULAS

C) AGENESIA D) D) 10 7 CELULAS

D) ATROFIA E) E) 10 4 CELULAS

e) APLASIA 308.-) EL GRADO EN QUE LAS CÉLULAS


PARENQUIMATOSAS DE UNA NEOPLASIA SE
304.-) SE LLAMA PERDIDA DE LA POLARIDAD PARECEN A LA CONTRAPARTE NORMAL SE
CELULAR NORMAL CUANDO: DENOMINA:
A) HAY PERDIDA DE LA RELACIÓN NÚCLEO A) DIFERENCIACIÓN
CITOPLASMA CELULAR
B) INDIFERENCIACIÓN
B) HAY PERDIDA DE LA ORIENTACIÓN DE LAS
CÉLULAS ANAPLÁSICAS C) DESDIFERENCIACIÓN

C) HAY PERDIDA DE LAS FUNCIONES D) REGRESIÓN


CELULARES
E) PROGRESIÓN
309.-) CÓMO SE DENOMINAN A LOS A) RESPIRACIÓN ANAEROBICA
TROMBOS QUE PUEDEN TENER
B) FOSFORILACION OXIDATIVA
LAMINACIONES PRODUCIDOS POR
CAPAS PÁLIDAS ALTERNANTES DE C) GLUCÓLISIS ANAEROBIA
PLAQUETAS MEZCLADAS CON FIBRINA Y
HEMATÍES? D) SÍNTESIS DE CREATINCINASA

F) A) EMBOLO 312.-) ¿CUÁL DE LOS SIGUIENTES ES


ANTIOXIDANTES NO ENZIMÁTICO?
G) B) TROMBO MURAL
A) TRANSFERRINA
H) C) FLEBOTROMBO
B) CATALASAS
I) D) LÍNEAS DE ZAHN
C) GLUTATIÓN PEROXIDASA
310.-) ¿CÓMO SE CLASIFICAN LOS INFARTOS?
D) SUPERÓXIDO DISMUTASAS
A) ROJOS, BLANCOS, SÉPTICOS
313.-) LOS SIGUIENTES SON PIGMENTOS
B) CARDIOGÉNICO, HIPOVOLÉMICO, SÉPTICO. DERIVADOS DE LA HEMOGLOBINA:
C) DE PRIMERA INTENCIÓN, DE SEGUNDA A) LIPOFUCSINA Y MELANINA
INTENCIÓN
B) HEMOSIDERINA Y BILIRRUBINA
D) AGUDOS, CRÓNICOS
C) CARBON Y TATUAJE
E) CARDIACO, RENAL
D) PIGMENTO DE DESGASTE O
311.-) EL PRIMER PUNTO DE ATAQUE DE LA
HIPOXIA EN LA CELULA ES: ENVEJECIMIENTO

CORRELACIONAR LAS SIGUIENTES COLUMNAS CONTESTANDO EN SU HOJA DE RESPUESTA

314.) CONDROSARCOMA A) EPITELIAL BENIGNA

47.)CORIOCARCINOMA B) MESENQUIMATOSA

BENIGNA

48.) MOLA HIDATIFORME C) NO ES NEOPLASIA

49.) PAPILOMA D) EPITELIAL MALIGNA

50.) GRANULOMA

E) MESENQUIMATOSA

MALIGNA

B) ASPECTO ARREMOLINADO DE LOS HACES DEL


MÚSCULO LISO
315.-) CON RESPECTO A LA CARACTERÍSTICA
MICROSCÓPICO MÁS IMPORTANTE DEL C) ÁREAS DE REBLANDECIMIENTO COLOR
LEIOMIOMA: AMARILLENTO

A) COLOR ROSADO SIMILAR AL MÚSCULO LISO


D) TAMAÑO Y FORMA UNIFORMES, DE COLOR C) ESPACIOS PERISINUSOIDALES.
VIOLÁCEO
D) CONDUCTOS DE HERING.
316.-) OCURRE CUANDO EL NÚCLEO DE UN
E) CANALÍCULOS BILIARES.
FIBROBLASTO CUTÁNEO DE UN PACIENTE SE
INTRODUCE EN UN OVOCITO HUMANO 319.-) FACTOR DE CRECIMIENTO QUE
ENUCLEADO PARA FORMAR CME QUE SE CONTRIBUYE A LA REPARACIÓN DE HERIDAS,
CULTIVAN, SE INDUCE SU DIFERENCIACIÓN, Y SE ANGIOGENIA, HEMATOPOYESIS Y DESARROLLO
TRASPLANTAN AL PACIENTE: DE MÚSCULOS CARDIACO Y ESQUELÉTICO, LA
MADURACIÓN DEL PULMÓN, Y LA
A) CLONACIÓN REPRODUCTIVA.
ESPECIFICACIÓN DEL HÍGADO:
B) CLONACIÓN TERAPÉUTICA.
A) VEGF
C) TRANSDIFERENCIACIÓN.
B) FGF
D) PARTENOGÉNESIS.
C) HGF
E) PLASTICIDAD DEL DESARROLLO.
D) TGF-Α

E) PDGF

320 SUN UV RADIATIONS ARE ASSOCIATED WITH


THE FOLLOWING TYPES OF CANCER

A) GASTRIC AND COLONIC CARCINOMA

B) BASOCELULAR CARCINOMA

C) BONE AND INTESTINE SARCOMA

317.-) CARACTERÍSTICAS MORFOLÓGICAS DE LA D) BREAST ADENOCARCINOMA


APENDICITIS AGUDA SUPURADA:
E) LIPOSARCOMA AND LYMPHOMA

A) APARICIÓN DE ÁREAS DE ULCERACIÓN


321.-) .- EL NIVEL SERICO DE LA CPK-MB, SE
HEMORRÁGICA VERDOSA
ELEVA POR ENCIMA DE LOS VALORES BASALES A
B) NECROSIS GANGRENOSA QUE VA DE LA PARED LAS :
HASTA LA CAPA SEROSA
A) 2-4 HRS.
C) ABSCESO DE PARED ACOMPAÑADO POR
B) 16-20 HRS.
ULCERACIÓN
C) 72 HRS.
B) TRANSFORMACIÓN DE SEROSA NORMAL EN
MEMBRANA ROJA, GRANULOSA Y OPACA D) 48 HRS.
318.-) .-LAS CÉLULAS MADRE/PROGENITORAS DEL E) 1-2 HRS.
HÍGADO SE ALBERGAN EN:

A) SINUSOIDES HEPÁTICOS.
322-) SON EJEMPLOS DEL GRUPO
B) PLACAS HEPÁTICAS. FISIOPATÓLOGICO DE EDEMA POR
DISMINUCIÓN DE PRESIÓN OSMÓTICA: C) DE EPITELIO PLANO A EPITELIO CILÍNDRICO

A) PERICARDITIS CONSTRICTIVA Y TROMBOSIS D) DE TEJIDO MUSCULAR A TEJIDO ÓSEO

B) FILARIASIS Y NEOPLASIAS

C) DESNUTRICIÓN MARASMATICA Y TROMBOSIS 326.-) SON ANORMALIDADES ENCONTRADAS EN


UNA CITOLOGÍA EXFOLIATIVA EXCEPTO:
D) DESNUTRICIÓN KWASHORKIOR Y CIRROSIS
HEPÁTICA A) CÉLULAS BINUCLEADAS

E) DILATACIÓN ARTERIOLAR Y ASCITIS B) PERDIDA DE LA RELACIÓN NÚCLEO-


CITOPLASMA

C) CÉLULAS DE DIFERENTES TAMAÑOS


323.-) INTERACTUA CON UN RECEPTOR TIE 2
PARA RECLUTAR A LAS CÉLULAS D) PRESENCIA DE ANTICUERPOS
PERIENDOTELIALES:
E)PERDIDA DE LA HOMOGENEIDAD CELULAR
A) ANGIOPOYETINA 1

B) ANGIOPOYETINA 2
327.-) EL COLOR AZUL VERDOSO QUE ADQUIEREN
C) PDGF LOS HEMATOMAS EN LA FASE DE RESOLUCIÓN ES
PORQUE:
D) TGF-Β
A)LA HEMOGLOBINA ES FAGOCITADA POR LOS
MACRÓFAGOS

B)LA HEMOGLOBINA SE TRANSFORMA EN


BILIRRUBINA

C)LA HEMOGLOBINA SE TRANSFORMA EN


HEMOSIDERINA
324.-) EL PATRÓN MORFOLÓGICO DE NECROSIS
QUE PRESENTAN LAS CÉLULAS MUERTAS POR D)LA HEMOGLOBINA SE TRANSFORMA EN
ISQUEMIA (SALVO EN EL SNC) ES: LIPOFUGSINA

A) GANGRENOSA E) LA HEMOGLOBINA SE TRANSFORMA EN


MELANINA
B) COLICUATIVA

C) CASEOSA
328.-) ES UNA ALTERACIÓN DE ESCASA
D) COAGULATIVA FRECUENCIA QUE SE DEBE A UNA
PROLIFERACIÓN EXUBERANTE DE FIBROBLASTOS
Y OTROS ELEMENTOS DEL TEJIDO CONJUNTIVO

A) LESIONES DESMOIDES
325-) TIPO DE METAPLASIA MÁS FRECUENTE EN
B) HIPERPLASIA BENINGNA
TEJIDO EPITELIAL:
C) NEOPLASIAS
A) DE EPITELIO CILÍNDRICO A PLANO
ESTRATIFICADO D) CARNE ORGULLOSA
B) DE TEJIDO CONECTIVO A CARTÍLAGO E) CICATRIZ QUELOIDE
333.-) ADAPTACIÓN CELULAR DE LA
DIFERENCIACIÓN, QUE CONSISTE EN UN CAMBIO
329.- ) ADAPTACIÓN CELULAR DEL CRECIMIENTO
REVERSIBLE DE UNA CÉLULA ADULTA POR OTRA
QUE PUEDE SER TERRENO FÉRTIL EN DONDE SE
CÉLULA ADULTA DE DIFERENTE TIPO:
INICIA UNA PROLIFERACIÓN CANCEROSA:
A) METAPLASIA
A) ATROFIA
B) ATROFIA
B) HIPERTROFIA
C) HIPERPLASIA
C) DISPLASIA
D) HIPERTROFIA
D) HIPERPLASIA FISIOLÓGICA
E) HIPOPLASIA
E) APLASIA

334.-) PRINCIPAL QUIMIOTÁCTICO ENDÓGENO


330.-) WHICH ARN VIRUS HAS ONCOGEN
PARA NEUTROFILOS:
ACTIVITY?
A) C-C
A) HEPATITIS VIRUS TYPE B
B) C-X-C
B) HUMAN PAPILLOMA VIRUS
C) FRAGMENTOS FC DE IGG Y COLECTINAS
C) T-CELLS LEUKEMIA VIRUS
D) E-SELECTINA, P-SELECTINA Y L-SELECTINA
D) EPSTEIN-BARR VIRUS
E) COLECTINAS Y PROTEÍNAS TIPO MUCINA
331.-) ES EL COAGULO CONSTITUIDO POR
AGREGACION PLAQUETARIA Y DEPOSITO DE 335.-) PRINCIPAL QUIMIOCINA QUE INDUCE
FIBRINA: POTENTE ADHESION PARA MONOCITOS Y
LINFOCITOS T:
A) TAPON HEMOSTATICO PRIMARIO
A) CX3C
B) ANTITROMBINA
B) C
C) FIBRINOGENO
C) C-C
D) TAPON HEMOSTATICO SECUNDARIO
D) C-X-C
E) EMBOLIA DE LIQUIDO AMNIOTICO
E) SISTEMA DE LAS CININAS

336.-) COMPLICACION MAS FRECUENTE DEL


332.-) EN EL PROCESO DE REMODELACIÓN DE LA INFARTO DE MIOCARDIO:
REPARACIÓN DE LAS HERIDAS PARTICIPAN LAS
SIGUIENTES ENZIMAS: A) ARRITMIAS CARDIACAS

A) ELASTASAS, ENDONUCLEASAS B) DOLOR RETROESTERNAL

B) CATALASAS, GLUTATION PEROXIDASA C) EDEMA PULMONAR

C) SUPEROXIDO, DISMUTASA, FOSFATASA D) DISNEA PAROXISTICA NOCTURNA


ALCALINA
E) HIPERTENSION ARTERIAL
D) METALOPROTEÍNASAS, GELATINASAS

E) MIELOPEROXIDASAS
337.-) .- SELECTINAS INTEGRINAS Y 341.-) CRITERIO HISTOLOGICO PARA
GLUCOPROTEÍNAS TIPO MUCINA ACTÚAN COMO: DIAGNOSTICO DE APENDICITIS AGUDA:

A) SUSTANCIAS QUIMIOTÁCTICAS ENDÓGENAS A) INFILTRACION DE NEUTROFILOS EN LA


MUCOSA
B) OPSONINAS
B) INFILTRACION DE NEUTROFILOS EN LA CAPA
C) SUSTANCIAS QUE AUMENTAN LA
MUSCULAR
PERMEABILIDAD VASCULAR
C) INFILTRACION DE EOSINOFILOS EN LA CAPA
D) SUSTANCIAS VASODILATADORES
MUSCULAR
E) MOLÉCULAS DE ADHESIÓN
D) INFILTRACION DE EOSINOFILOS EN LA CAPA
338.-) LA LIZOSIMA, LACTOFERRINA, PROTEÍNA MUCOSA
BÁSICA PRINCIPAL Y LAS DEFENSINAS,
E) INFILTRACION DE NEUTROFILOS E LA
DESTRUYEN MICROORGANISMOS POR
SUBMUCOSA
MECANISMOS:

A) DESCONOCIDOS
342.-) SON SUSTANCIAS CON CAPACIDAD
B) DEPENDIENTES DEL OXÍGENO
ANTITROBOTICA:
C) RELACIONADOS CON LA APOPTOSIS
A) FACTOR DE VON WILEBRAND COLÁGENA
D) INDEPENDIENTES DEL OXÍGENO
B) ENDOTELINA Y FACTOR TISULAR
E) MEDIADOS POR LAS INMUNOGLOBULINAS
C) TROMBINA Y PLASMINA
339.-) ES LA CAPACIDAD DE UNA CÉLULA DE
D) FIBRINA Y TROBOXANO A2
TRANSDIFERENCIARSE A DISTINTOS ESTIRPES
CELULARES: E) PLASMINOGENO TISULAR Y PROSTACICLINA
ENDOTELIAL
A)TRANSMUTACIÓN.
343.-) ES EJEMPLO DE TROMBOSIS POR ESTADOS
B)PLASTICIDAD DEL DESARROLLO
DE HIPERCOAGULABILIDAD SECUNDARIA:
C)DIFERENCIACIÓN CELULAR.
A) DEFICIENCIA DE PROTEÍNA C O S
D)PLASTICIDAD CELULAR.
B) MUTACIONES EN EL GEN DE FACTOR V
E)MITOGÉNESIS.
C) DEFICIENCIA DE ANTITROBINA III
340.-) MANIFESTACION CLINICA INICIAL MÁS
D) SINDROME DE ANTICOAGULANTE LÚPICO
FRECUENTE DEL CARCINOMA RENAL:
E) MUTACIONES EN EL GEN DE PROTROMBINA
A) DISURIA Y CISTITIS
344.-) LOS INFARTOS ROJOS APARECEN EN LAS
B) DISPAUREMIA
SIGUIENTES CONDICIONES EXCEPTO:
C) NICTURIA
A) OCLUSIONES VENOSAS
D) HEMATURIA
B) TEJIDOS LAXOS (PULMON)
E) PERDIDA DE PESO
C) EN OCLUSIONES ARTERIALES

D) EN TEJIDOS CON DOBLE CIRCULACION


345.-) ES MITÓGENO PARA CÉLULAS EPITELIALES, 349.-) EN EL FIBROADENOMA MAMARIO SE
HEPATOCITOS Y FIBROBLASTOS, ES HOMOLOGO CARACTERIZA POR TODO LO SIGUIENTE EXCEPTO:
DEL TGF ALFA, AL TENER ACTIVIDADES
A) ES UN NODULO ESFERICO DE GRAN TAMAÑO
BIOLÓGICAS SIMILARES:
B) ESTA FIJO A PLANOS PROFUNDOS Y NO SE
A) PDGF
PUEDE LIMITAR SU TAMAÑO
B) FGF
C) ES MOVIL Y NO FIJO A PLANOS PROFUNDOS
C) EGF
D) GENERALMENTE ES UN HALLAZGO
D) TGFBETA
350 THE ABNORMAL TRIGLYCERIDE
E) VEGF ACCUMULATION INSIDE PARENCHYMA CELLS IS
CALLED
346.-) SON GRANDES COLECCIONES DE SANGRE
EN CAVIDADES EXCEPTO: A) GLYCOGENOSIS

A) HEMOSTASIA B) COLAGENOSIS

B) HEMOPERITONEO C) STEATOSIS OR CHANGE

C) HEMARTROSIS D) ANTRACOSIS

D) HEMOPERICARDIO E) HYALINE CHANGE

E) HEMOPERICARDIO 351.-) SE PUEDEN ENCONTRAR GRANULOMAS EN


LAS SIGUIENTES ENFERMEDADES:
347.-) EL TEJIDO DE GRANULACIÓN SE
CARACTERIZA POR: A) NEUMONIA Y BRONQUITIS

A) SER RICO EN CALCIO Y FIBRINA B) TIFOIDEA Y PARATIFOIDEA

B) CONTENER ABUNDANTES NEUTRÓFILOS Y C) TROMBOSIS Y EMBOLIA


HEMATÍES
D) ENFISEMA Y BRONQUIECTASIA
C) PROLIFERACIÓN FIBROBLASTOS Y DE
E) LEPRA LEPROMATOSA Y TUBERCULOIDE
PEQUEÑOS VASOS SANGUÍNEOS
352.-) SON METABOLITOS DEL ACIDO
D) PRESENTARSE EN LAS PRIMERAS 24 HORAS
ARAQUIDONICO POR LA VIA DE LA
DESPUÉS DE LA AGRESIÓN
CICLOOXIGENASA:
E) SER UNA REACCIÓN INMUNITARIA MEDIADA
A) LEUCOTRIENOS LTB4, LTD4
POR CÉLULAS T
B) PROSTAGLANDINAS Y TROMBOXANO
348.-) MOST CASES OF SEPTIC SHOCK ARE
CAUSED BY ENDOTOXIN-PRODUCING BACTERIA, C) IL1 Y FNT
WHICH ARE CLASSIFIED AS:
D) FAP
A) PYOGENIC
E) HISTAMINA Y SEROTONINA
B) GRAM-POSITIVE

C) GRAM-NEGATIVE

D) ENCAPSULATED

E) ACID FAST
353 SON TIPOS DE EMBOLIAS EXCEPTO: 357.-) LOS CUERPOS DE RUSSELL SON
ACUMULOS DE:
A) EMBOLIA GRASA
A) INMUNOGLOBULINAS EN CÉLULAS
B) EMBOLIA LINFATICA
PLASMÁTICAS
C) EMBOLIA GASEOSA
B) PROTEÍNAS EN LAS CÉLULAS TUBULARES
D) EMBOLIA DE LIQUIDO AMNIOTICO
C) GRASA EN HEPATOCITOS
354.-) TIPO DE CALCIFICACION QUE SE
D) COLESTEROL EN MACRÓFAGOS DE LA
ACOMPAÑA DE HIPERCALCEMIA Y
LAMINA PROPIA DE LA VESÍCULA BILIAR
TRASTORNOS EN EL METABOLISMO DEL
CALCIO: E) PROTEINAS ENCELULAS HEPATICAS

A) DISTROFICA 358.-) LOS CÁLCULOS BILIARES ESTÁN


FORMADOS POR:
B) METASTASICA
A) COLESTEROL, CALCIO, PIGMENTOS
C) CAMBIO HIALINO
BILIARES
D) FIBROPLASIA
B) AGUA, COLESTEROL
E) CALCITONINA
C) MAGNESIO, CALCIO, FÓSFORO
355 LOS LEIOMIOMAS DE ACUERDO A SU
D) PIGMENTOS BILIARES, HORMONAS
LOCALIZACIÓN EN EL CUERPO UTERINO SE
HEPÁTICAS
CLASIFICAN EN:
E) POTASIO, COLESTROL Y FERRITINA
A) INTERNOS, MURALES, EXTERNOS
359.-) COMPLICACIÓN MAS FRECUENTE DE
B) SUBMUCOSOS, SUBSEROSOS E
LA APENDICITIS:
INTRAMURALES
A) PANCREATITIS
C) PRIMARIOS, SECUNDARIOS, TERCIARIOS
B) HEPATITIS
D) HIPERTROFICOS, DISPLASICOS E
HIPERPLASICOS C) PERITONITIS

356.-) PIGMENTO ENDÓGENO, NO D) COLITIS


DERIVADO DE LA HEMOGLOBINA:
E) GASTRITIS
A) HEMOSIDERINA
360.-) LA ALFA FETO PROTEINA SE ELEVA
B) BILIRRUBINA ANORMALMENTE EN:

C) ANTRACOSIS A) CANCER RECTAL, PANCRETICO, GASTRICO

D) MELANINA B) CANCER PRIMARIO DE HIGADO, DE


CELULAS GERMINALES DE TESTICULO
E) TATUAJE
C) CANCER DE OVARIO, CANCER DE
PANCREAS
D) CANCER DE PROSTATA, CARCINOMA 364.-) EN CUAL DE LOS SIGUIENTES
MICROCITICO PULMONAR ÓRGANOS SE PUEDE DESARROLLAR UN
TERATOMA:
E) MIELOMA MULTIPLE Y OTRAS
GAMMAPATIAS A) RIÑÓN

361.-) LA CITOMETRIA DE FLUJO PUEDE B) MAMA


MEDIR:
C) ÚTERO
A) LAS ALTERACIONES DE LOS
D) OVARIO
CROMOSOMAS
E) PRÓSTATA
B) LA CAPACIDAD DE REPRODUCCIÓN DE LAS
CELULAS NEOPLASICAS

C) LOS ANTIGENOS DE MEMBRANA,


CONTENIDO DE ADN EN CELULAS
TUMORALES

D.) LA CAPACIDAD CELULAR DE FORMAR


METASTASIS

E) EL NUMERO DE MITOSIS POR CAMPO

362.-) LA INICIACIÓN DE LA CALCIFICACIÓN


DISTROFICA EXTRACELULAR SE INICIA EN:

A) LAS PROTEÍNAS DE LA MEMBRANA


CELULAR

B) LOS FOSFOLIPIDOS DE LA MEMBRANA


CELULAR

C) EN LOS CARBOHIDRATOS

D) EN LOS RECEPTORES DE MEMBRANA

E) EN LOS MICROTUBULOS

363-) FENÓMENOS QUE CONSIDERAN


DENTRO DEL DESTINO DEL TROMBO
EXCEPTO:

A) INCORPORACION A LA PARED

B) RECANALIZACIÓN

C) AGREGACIÓN

D) PROPAGACIÓN

E) EMBOLIZACION
1: Son neoplasias que tienen peor diferenciación o totalmente indiferenciadas, se rfiere al
termino de.

a. Nevo
b. Cortisoma
c. Hamartosoma
d. Anaplasia

2: Corresponde al evento principal de la hemostasia primaria.

a. Vasodilatación arteriolar.
b. Formación del tapón plaquetario.
c. Formación de fibrina.
d. Secreción de endotelina.

3: La deficiencia de la glucoproteína GPIB provoca la enfermedad de:

a. Von Willebrand
b. Tromboastenia de Glanzmann.
c. Nieman Pick.
d. Bernard Soulier.

4: Los principales factores de crecimiento implicados en la síntesis de tejido conjutivo.


Excpeto.

a. PGFD
b. FGF
c. IL-1
d. TGF-BETA

5: La estadificación de los cánceres sólidos se basa en que sistema

a. Sistema de Dukes
b. Ninguna de las anteriores
c. Sistema Bethesda
d. Sistema de TNM.

6: ¿Cuál de las siguientes causas es más probable que se acumule el líquido si existe
variación?

a. Permeabilidad vascular
b. Presión linfática.
c. Presión oncótica.
d. Diapedesis.

7: Neoplasia maligna de las células de los ductos seminales.

a. Coriocarcinoma
b. Carcinoma embrionario
c. Seminoma.
d. Tumor de senos endodermicos.
8: Un estado inflamatorio crónico puede causar anemia ferropénica en respuesta al.

a. Aumento de albumina
b. Aumento de la fibrina. ( un we le puso que era esta según)
c. Aumento de la hepcidina. ( acabo de leer un articulo dice que es por esto aumento
de hepcidina que según yo si aumenta las concentración de hierro)

d. Aumento del fibrinógeno.

FISIOPATOLOGÍA DE LA ANEMIA POR PROCESO INFLAMATORIO CRONICO • Compleja y


multifactorial • ACORTAMIENTO DE LA VIDA MEDIA DEL ERITROCITO •INHIBICION DE LA SINTESIS
DE ERITROPOYETINA •INHIBICION DE LA ERITROPOYESIS •AUMENTO DE LA CONCENTRACION DE
HEPCIDINA

9: Su crecimiento se acompaña de infiltración, invasión y destrucción progresiva del tejido,


están poco delimitados, y son de lenta expansión.

a. Neoplasia maligna.
b. Neoplasia preinvasiva.
c. Cancer in situ.
d. Neoplasia benigna.

10: Es la fase del shock en la que se activa el eje renina angiotensina.

a. Fase refractaria.
b. Fase no progresiva.
c. Fase irreversible.
d. Fase progresiva.

11: Estos trombos se forman en la circulación venosa, suelen contener más eritrocitos
incorporados y relativamente pocas plaquetas.

a. Trombos arteriales.
b. Trombos rojos.
c. Trombos murales.
d. Vegetaciones.

12: Es el colágeno que proporciona fibrillas de anclaje de las células epiteliales a la


membrana basal.

a. Colageno tipo I
b. Colageno tipo III
c. Colageno tipo IV
d. Colageno tipo VII

13: ¿Cuáles son las fases de la reparación de heridas?

a. Infiltración, remodelación, fibrosis y costra.


b. Sepsis, formación de tejido de granulación y angiogénesis.
c. Angiogenia, tejido de granulación y remodelación
d. Angiogenia, inflamación, contracción, fibrosis e hipoplasia

14: El resultado de múltiples mutaciones que se acumulan de forma independiente en


diferentes células y contribuyen así a la generación de subclones con características
diferentes a lo que se le conoce como.

a. Iniciación y promoción.
b. Progresión tumoral.
c. Acción directa e indirecta.
d. Heterogeneidad tumoral.

15: Fase del shock donde fracasan mecanismos reflejos de compensación disminuye la
perfusión de órganos vitales.

a. Fase latente
b. Fase progresiva.
c. Fase no progresiva.
d. Fase irreversible.

16: Patología donde las personas que sufren de mutaciones en el gen TP53 y que tiene
una posibilidad 25 veces mayor de experimentar una neoplasia maligna antes de los 50
años que la población general.

a. Síndrome de desgaste
b. Carcinoma
c. Neoplasia endocrina múltiple.
d. Síndrome de Li-Fraumeni.

17: Si la alteración displásica es intensa y afecta a todo el espesor del epitelio pero la
lesión no penetra la membrana basal, se habla de.

a. Carcinoma in situ.
b. Tumor infiltrante.
c. Tumor invasivo.
d. Metaplasia.

18: El gen APC, inhibidor de las vías de señalización mitógena, se ha relacionado con las
siguientes neoplasias.

a. Carcinoma basocelular y metaloblastoma.


b. Carcinoma de estómago, cólon, páncreas y melanoma.
c. Retinoblastomas.
d. Schwannoma y meningioma.

19: ¿Qué tan resistentes son las células musculares cardíacas a la hipoxia?

a. Muy resistentes, mueren después de muchas horas.


b. Extremadamente sensibles, mueren a los 5 minutos.
c. Bastante sensibles, mueren a los 20-30 minutos.
d. Poco sensibles, mueren a las pocas horas.

20: Son patologías que producen edema por aumento de la presión hidrostática del vaso.

a. Neoplasias y filarias
b. Aumento de la secreción renina-angiotensina-aldosterona.
c. Glomerulopatías y malnutrición.
d. Pericarditis constrictiva y trombosis.

21: Se le conoce así, cuando e tejido cicatrizal crece más allá de los límites de la herida
original, además no se contrae.

a. Fibromatosis agresivas.
b. Cicatriz queloide.
c. Granulació exuberante.
d. Dehiscencia.

22: Factores sistémicos que influyen en la curación de heridas.

a. Estrés oxidativo, perdida del tono vascular, anemia


b. Nutrición, situación circulatoria, estado metabólico y hormonas
c. Infecciones, nutrición, función renal y cardiaca deficiente
d. Tipo de herida, profundidad de la herida y estado metabólico.

23: Son neoplasias de las células germinales del testículo

a. Carcinomas
b. Teratomas
c. Cistoadenomas
d. Seminomas

24: Es una característica del Síndrome de Trousseau asociado a carcinomas de páncreas


y estomago se refiere a.

a. Émbolos
b. Déficit de glicoproteína 1B
c. Tromboflebitis migratoria
d. Infarto

25: Son neoplasias que tienen peor diferenciación o totalmente indiferenciadas, se refiere
al término de.

a. Nevo
b. Coristoma
c. Hamartoma
d. Anaplasia

26: Marcador tumoral relacionado con carcinoma de colon, páncreas, pulmón, mama.

a. Alfa fetoproteína
b. Antígeno prostático
c. Antígeno carcinoembrionario.
d. Enolasa neuronal específica

27: Se puede producir shock por.

a. Todas las anteriores


b. Sepsis
c. Infarto al miocardio
d. Quemaduras.

28: Marcador tumoral relacionado con el cáncer prostático.

a. Alfa fetoproteína
b. CA-125
c. CA-19-9
d. PSA

29: Morfológicamente las neoplasias malignas se caracterizan por.

a. Relación núcleo citoplasma normal.


b. Arquitectura tisular intacta
c. Pleomorfismo nuclear y celular
d. Figuras mitóticas normales

30: Se observa tras una destrucción sustancial de tejido, cuando la lesión inflamatoria
afecta a tejidos que no son capaces de regenerarse o cuando hay abundante exudación
de fibrina en tejidos o cavidades serosas. Que no es posible eliminar adecuadamente.

a. Inflamación crónica.
b. Remisión
c. Curación por reposición de tejido conjuntivo
d. Resolución completa.

31: El gen 9.12 se asocia con.

a. Von Hippel-Lindau
b. Trisomía 21
c. Cromosoma Filadelfia
d. Leucemia mieloide

32: Características de las señales sinápticas.

a. Secreción de neurotransmisores en uniones celulares especiales.


b. De difusión mínima y la señal se degrada rápido.
c. Se usa para amplificar una respuesta o inhibirla mediante retroalimentación.
d. Se libera al torrente circulatorio y actúa distancia sobre las células blanco.

33: Neoplasia maligna de la glándula tiroides.

a. Coriocarcinoma
b. Adenocarcinoma
c. Condrosarcoma
d. Linfangiosarcoma

34: Se considera como extensión de las neoplasias a sitios anatómicamente separados


del lugar de origen.

a. Metástasis
b. Micro invasión.
c. Son masas expansivas cohesivas.
d. Invasión.
35: Dentro de las funciones de p53 se encuentran, excepto.

a. Pausar el ciclo celular para reparación de ADN.


b. Inducir apoptosis.
c. Inducir proliferación celular
d. Inducir senescencia.

36: El cáncer es una enfermedad caracterizada por.

a. Daño genético letal.


b. Senescencia celular.
c. Lesión subletal al ADN.
d. Adquisición de mutaciones letales.

37: El gen RB, inhibidor de la progresión celular, se ha relacionado con las siguientes
neoplasias.

a. Retinoblastoma, osteosarcoma, carcinomas de mama, colon y pulmón.


b. Carcinoma basocelular y meduloblastoma.
c. Carcinoma basocelular.
d. Carcinoma de células renales y paraganglioma.

38: Los tipos de VPH de alto riesgo producen proteínas oncogenas que ocasionan lo
siguiente excepto.

a. Activan ciclinas.
b. Inhiben la apoptosis.
c. Combaten la senescencia celular.
d. Inmunodeficiencia

39: Las sustancias que inhiben la función plaquetaria.

a. Antitrombina
b. On, prostaciclina, ADPasa
c. Plasminogeno.
d. Tromboplastina.

40: Trombo formado en las cavidades cardíacas.

a. Trombo mural.
b. Aneurisma.
c. Petequia
d. Émbolo

41: Tienen mayores probabilidades de sufrir infarto rojo, excepto.

a. Intestino delgado.
b. Pulmón
c. Riñón
42: El edema que ocurre en la filariasis se debe a.

a. Disminución de la presión oncótica.


b. Bloqueo de la circulación linfática.
c. Aumento de la presión hidrostática venosa.
d. Aumento de la permeabilidad vascular.

43: Se caracterizan por degradar la MEC y permite la remodelación y la extensión del tubo
vascular.

a. Tirosina cinasa.
b. MPM metaloproteinasas de matriz
c. Factor transformante beta
d. Factor derivado de plaquetas.

44: Es la inmortalidad de las células malignas es considerado como factor decisivo.

a. Evasión de la senescencia. (también puede ser esta un we se la puso)


b. Mutación puntual
c. Reordenamiento de cromosomas.
d. Capacidad de regulación.

45: Cual enunciado acerca de las plaquetas es verdad.

a. Contiene cuerpos densos ricos en fibrinógeno.


b. Los precursores son monocitos
c. Contiene gránulos alfa ricos en difosfato de adenosina.
d. Contiene actina y miosina, microfilamentos y microtubulos.

46: Este tipo de células están presentes en la inflamación crónica, son abundantes en las
reacciones inmunitarias mediadas por inmunoglobulina E y en las infecciones parasitarias.

a. Neutrófilos
b. Linfocitos
c. Eosinófilos
d. Macrófagos.

47: Todo lo siguiente se puede esperar en la congestión hepática excepto.

a. Regiones centrolobulillares distendidas por sangre.


b. Infartos en forma de pirámide.
c. Patrón de nuez moscada
d. Incremento de peso.

48: Células que están presentes en el tejido conectivo contiguos a vasos sanguíneos por
debajo de superficies mucosas de vías respiratorias, la unión de antígeno alérgeno a
anticuerpo IgE, se refiere a.

a. Podocitos
b. Dentriticas.
c. Macrofagos
d. Mastocitos
49: Contienen una mayor cantidad de sangre oxigenada se caracteriza por el color rojo
brillante y dilatación de las arteriolas.

a. Hiperemia activa
b. Hiperemia pasiva
c. Congestión crónica
d. Congestión pasiva

50: Neoplasia maligna del epitelio placentario.

a. Carcinoma embrionario
b. Coriocarcinoma
c. Seminoma
d. Mesotelioma

51: Neoplasia maligna del tejido adiposo.

a. Liposarcoma
b. Astrocitoma
c. Neurofribroma
d. Lipoma

52: La extravasación de líquido al intersticio, cual es el término que se utilizaría.

a. Edema
b. Trasudado
c. Exudado
d. Infiltración

53: La estadificación de un tumor maligno se refiere a.

a. Grado de diferenciación de las células tumorales.


b. Tratamiento quirúrgico
c. Diagnóstico molecular.
d. Tamaño de la lesión primaria y si ha realizado metástasis

54:La congestión pasiva crónica del pulmón puede incluir todo lo siguiente excepto.

a. Edema pulmonar
b. Eritema
c. Macrófagos con hemosiderina
d. Induración café ( ya lo encontré esto si existe)

55: ¿Cuál es la definición de teratoma?

a. Que se origine de las células de Sertoli


b. Que se origine de más de 2 capas germinales.
c. Origen de linfocitos B
d. Que se origine de trofoblasto.

56: El shock cardiogénico se produce cuando existe una pérdida de la masa muscular del
ventrículo izquierdo del.

a. 25%
b. 60%
c. 45%
d. 35%

57: El efecto Warburg consiste en.

a. Incrementar el ciclo de Krebs


b. Disminuir la síntesis de biomoléculas
c. Promover la atrofia del tumor
d. La neoplasia maligna fija la glucólisis anaerobia a su metabolismo.

58: Neoplasia maligna de los vasos sanguíneos

a. Linfoma
b. Linfangiosarcoma
c. Leucemia
d. Angiosarcoma

59: Son estados de hipercoagulabilidad de bajo riesgo excepto.

a. Tabaquismo
b. Tabaquismo
c. Infarto agudo al miocardio.

60: Una de las razones por las cuales las células neoplásicas son inmortales.

a. No se autorrenuevan
b. Respetan las pausas del ciclo celular.
c. P53 funciona adecuadamente.
d. Expresan telomerasa.

61: Una lesión endotelial, tendrá como consecuencia.

a. No tiene relación con la coagulación.


b. Desarrollo de un ambiente procoagulante.
c. Produce la disolución de un coagulo
d. Desarrollo de un ambiente anticoagulante.

62: La inactivación o deleción de p16, adquirida por vía somática se ven en las familias
propensas a.

A. Carcinomas de páncreas, glioblastomas y cánceres de esófago.


B. Linfomas.
C. Adenocarcinoma pancreático y colangiocarcinomas.
D. Leucemia mieloide crónica y leucemias linfoblásticas agudas.

63: En la cicatrización por primera intención, ¿Cuándo se comienza a observar el tejido de


granulación?

a. Inmediatamente
b. Semanas después
c. A las 24 horas
d. 3-7 días después.

64: Son acontecimientos plaquetarios excepto.

a. Adhesión
b. Migración
c. Cambio de forma y secreción
d. Agregación

65: Especifique las variables que alteran la cicatrización.

a. Infección, diabetes, estado nutricional, factores mecánicos, mala perfusión, tipo,


alcance y localización de la lesión.
b. Estado nutricional, adecuado y esteroides
c. Baja presión local o la torsión de las heridas.
d. Infección y diabetes controlada

66: La mayor parte de las flebotrombosis se producen en.

a. Arterias de mediano calibre.


b. Corazón
c. Venas superficiales y profundas de las piernas.
d. Arterias cerebrales.

67: Es una causa de edema por disminución de la presión oncótica

a. Diabetes mellitus.
b. Desnutrición
c. Insuficiencia cardiaca
d. Cáncer

68: Es el término que se aplica a la neoplasia epitelial, benigna derivada de las glándulas,
se denomina como.

a. Papiloma
b. Pólipo
c. Angioma
d. Adenoma

69: Son procesos en la evolución de un trombo excepto.

a. Embolia recanalización
b. Propagación.
c. Trombolisis
d. Recanalización

70: El Virus de Epstein Barr produce.


a. Hepatomegalia
b. Leucemia de las células peludas
c. Linfoma de Burkitt
d. Sarcoma de Kaposi

71: La luz UV.

a. Es un carcinógeno químico
b. Hace dímeros en la guanina
c. No es un agente carcinógeno
d. Puede causar carcinoma basocelular.

72: ¿Cuál de los siguientes mediadores causan vasoconstricción y broncoconstricción?

a. Tromboxano A2
b. IL-2
c. LeucotrienosB4
d. Ninguna de las anteriores.

73: El edema por obstrucción linfática se da por las siguientes condiciones excepto.

a. Neoplásica
b. Postquirúrgica
c. Postirradiacion
d. Postquimioterapia

74:En el estado conocido como shock séptico

a. Es originado por perdida de líquidos


b. Es producido por el paso de bacterias, hongos al torrente circulatorio
c. Carece de respuesta inflamatoria
d. Se considera una respuesta inflamatoria local.

75: Es un factor de crecimiento con efectos mitógenos sobre los hepatocitos

a. Factor de crecimiento fibroblastico


b. Factor de crecimiento de dispersión
c. Factor de crecimiento epitelial
d. Factor de crecimiento derivado de plaquetas

76: La falta de diferenciación, es decir, implica una inversión de la diferenciación hacia un


plano más primitivo.

a. Displasia
b. Pleomorfismo
c. Anaplasia
d. Diferenciación

77: Es un ejemplo de edema localizado el que ocurre en casos de.


a. Síndrome nefrótico
b. Trichurasis
c. Cirrosis hepática
d. Insuficiencia cardiaca derecha

78: Se media por las interacciones con el factor de Von willebrand con la glicoproteína IB,
receptor de la superficie de las plaquetas y el colágeno expuesto.

a. Activación plaquetaria
b. Adhesión plaquetaria
c. Agregación plaquetaria
d. Fase de iniciación de la coagulación

79: ¿Qué gen supresor de tumores intervienen en la patogenia del carcinoma de mama?

a. Ninguna de las anteriores


b. APC1 APC2
c. BCRA1 BCRA2
d. NF1 NF2

80: En la cicatrización por primera intención, en qué tiempo la neovascularización alcanza


su máximo y el tejido de granulación llena la incisión.

a. 12 horas
b. 5 días
c. 48 horas
d. 24 horas

81: Son tumores constituidos por células inmaduras que se parecen a las que forman
tejido rudimentario fetal.

a. Papiloma
b. Blastoma
c. Hamartoma
d. Osteoma

82: Los tumores más comunes en los hombres se originan en.

a. Mama, pulmones, colon, recto


b. Piel, leucemia, páncreas
c. Piel, orofaringe y páncreas
d. Próstata, pulmones, colon y recto.

83: ¿Cuáles son las características del tejido de granulación?

a. Rico en colágeno y elastina


b. Rico en tejido conjuntivo provisional y nuevos capilares finos
c. Bajo en fibroblastos
d. Abundantes células inflamatorias
84: Es la vía más frecuente para la diseminación inicial de los carcinomas.

a. Siembre directa de cavidades o superficies corporales


b. Hematógena
c. Linfática
d. Continuidad

85:Los infartos pálidos se observan en.

a. Casos de obstrucción venosa


b. A y b son ciertas
c. Órganos sólidos
d. Casos de oclusión arterial

86: Un embolo gaseoso puede originarse.

a. Por trombosis venosa profunda


b. Por fracturas
c. Por cambios bruscos en la presión
d. Por células fetales.

87: En el choque séptico se puede producir.

a. Enteropatía hemorrágica
b. Daño alveolar difuso
c. Todas las anteriores
d. Encefalopatía isquémica

88: Mutaciones que contribuyen al fenotipo maligno se denomina.

a. Fenotipo mutador
b. Inestabilidad genómica
c. Ninguna de las anteriores
d. Mutación conductora

Segunda parte 7u7

89: La asbestosis y Silicosis está asociado con qué tipo de neoplasia

a. Carcinoma Pancreatico
b. Hepatocarcinoma
c. Mesotelioma y Carcinoma de Pulmón
d. Carcinoma Colorrectal

90: Son algunas causas de la alteración del retorno venoso

a. Sindrome nefrotico y cirrosis hepática


b. Obstruccion linfática y retención de sodio
c. Insuficiencia cardiaca congestiva, obstruccion y compresión venosa
d. Calor y frio

100: Es un cambio biológico necesario para la progresión de las neoplasias malignas


a. Evasión de la apoptosis
b. Metástasis
c. Potencial replicativo ilimitado
d. Angiogenia

101: Etapa caracterizada por la Hipoperfusion tisular y el inicio del empeoramiento


circulatorio y de los desequilibrios metabólicos incluyendo la acidosis láctica.

a. Fase progresiva
b. Fase no progresiva
c. Etapa reversible
d. Etapa irreversible

102: un cistoadenoma corresponde a un:

a. Neoplasia benigna quística glandular


b. Hiperplasia
c. Neoplasia maligna quística glandular
d. Neoplasia maligna

103: pertenece a una familia de factores de crecimiento con mas de 20 miembros:

a. FCDP
b. FCT-BETA
c. FCF
d. FCE

104: Esta via de diseminación es característica de los carcinomas de ovario. Ya que


frecuentemente se diseminan a la superficie peritoneal.

a. Iatrogénica
b. Hematogena
c. Cavidad y superficie corporal
d. Linfática

105: se produce por hipoxia y promueve la angiogenia

a. PDGF
b. TGF-BETA
c. FGF
d. VEGF

106: sufren una conmutación metabolica hacia la glucolisis aerobia (efecto warburg) que
facilita la síntesis de macromoléculas y orgánulos requeridos para un crecimiento celular
rápido

a. Evasion de la apoptosis
b. Replicación ilimitada
c. Alteración del metabolismo celular
d. Insensibilidad a las señales del crecimiento

107: cuál es la proteína más abundante en el ser humano

a. Vimentina
b. Miosina
c. Caderina
d. Colágeno

( esta madre es de práctica de lab igual hay que leer las practicas)

108: INTERNATIONAL NORMALIZED RATIO valora principalmente en el laboratorio:

a. Anticoagulacion con heparina


b. Acido acetilsalicilico
c. Factor VII y IX
d. Anticoagulacion oral

109: Responde al tratamiento contra anticuerpos que bloquean la actividad de HER2

a. Cáncer de mama
b. Adenocarcinoma de pulmon
c. Melanoma
d. Leucemia mielocitica aguda

110: la activación del plasminógeno genera su formación a plasmina para

a. Activar plaquetas
b. Iniciar la cascada fibrinolitica
c. Secreción y cambio de forma plaquetaria
d. Activación leucocitaria

111: Estos trombos son más frecuentes en las arterias coronarias, cerebrales y femorales,
conformados por plaquetas, fibrina, eritrocitos y leucocitos degenerados

a. Vegetaciones
b. Trombo mural
c. Trombo arterial
d. Trombos rojos

112: permiten determinar el diagnostico, el pronostico, la detección de enfermedad


residual minima y el diagnostico de la predisposición hereditaria.

a. Analisis microbiológico
b. Historia clínica
c. Análisis molecular
d. Analsis morfológico

113: via de la apoptosis mas involucrada en cáncer

a. Necroptosis
b. Via extrínseca
c. Via intrínseca
d. piropoptosis

114: clínicamente el proceso de fibrosis es común en

a. mucosas
b. epitelios
c. tejidos lábiles
d. órganos parenquimatosis ( hígado por ejemplo ya saben)

115: ¿Qué proteína fibrilar suene regenerar más en el tejido cicatrizal?

a. Integrina
b. Colageno
c. Elastina
d. Fibronectina

116: Factor de crecimiento quimiotactico para neutrófilos, macrófagos, células del musculo
liso. Estimula la proliferación de fibroblastos y células endoteliales.

a. Factor de crecimiento hepatocitario


b. Factor de crecimiento endotelial vascular
c. Factor de crecimiento derivado de las plaquetas
d. Factor de crecimiento Epidermico

117: Es el componente proteico encargado de dar resistencia a la tensión

a. Proteoglucanos
b. Laminina
c. Fibronectica
d. Colágeno

118: Es el termino aplicado a un resto Heterotopico de células, ejemplo: nódulos


pancreáticos en la submucosa del estomago.

a. Teratoma
b. Desmoplasias
c. Harmatoma
d. Coristoma

119: El factor de crecimiento placentario pertenece a la familia de:

a. FCE
b. FCH
c. FCVE
d. FCFD
120: Son áreas de necrosis isquémica causadas con más frecuencia por oclusión arterial,
derivada de una trombosis embolica.

a. Trombosis
b. Infarto
c. Isquemia
d. Coagulopatia por consumo (es lo mismo que decír coagulación intravascular
diseminada CID para los amigos)

121: En la regenaricion hepática la progresión de los hepatocitos del estado G1 a S


comienza con la formación del complejo: (esta si ni idea ni en el libro viene la busque y
nada)

a. Cliclinda D- CDK4
b. Ciclina A- CDK2
c. Ciclina B-CDK1
d. Ciclina D- CDK5

122: Son trastornos adquiridos predisponen a cáncer:

a. Mayor replicación celular


b. Todas las anteriores
c. Inmunodeficiencia
d. Inflamación crónica

123: se denomina sarcoma

a. Neoplasia maligna de tejido linfoide


b. Neoplasia maligna de la sangre
c. Neoplasia maligna epitelias
d. Neoplasia maligna del células mesenquimatosas

124: El gen CDHT inhibidor de invasión y metástasis se ha relacionado con las siguientes
neoplasias:

a. Carcinoma gástrico y carcinoma lobulillar de mama


b. Carcinoma basocelular y meduloblastoma
c. Carcinoma de estomago, colon, páncreas y melanoma
d. Schwanoma y meningioma

125: Factor que convierte el fibrinógeno en fibrina, activa al endotelio, linfocitos, monocitos
y produce agregación plaquetaria.

a. Proteína C
b. Trombina
c. Factor tisular
d. Factor activador del plasminógeno tisular
Es como la pregunta de la tromboflebitis solo que planteada de otra forma

126: sitio donde frecuentemente se originan trombosis venosas profundas:

a. En las extremidades superiores


b. En la vena cava
c. Circulación pulmonas
d. En las extremidades inferiores

127: principal familia de proteínas de adhesión encargadas de la interaccion con la MEC:

a. Cadherinas
b. Receptores FAS
c. Receptor FC
d. Integrinas

128: son neoplasias malignas derivadas del tejido conectivo:

a. neuroblatomas

b. sarcoma

c. osteoma

d. carcinoma

129: El tiempo parcial de tromboplastina evalua:

a. Via intrínseca
b. Via extrinseca
c. Via de properdin
d. Via alterna

130: se denomina a los hematomas subcutáneos 1 a 2 cm

a. Equimosis
b. Hematoma
c. Petequias
d. purpura

131: la técnica de inmunohistoquimica:

a. sirve para observar la morfología celular


b. carece de utilidad diagnostica
c. sirve para caracterizar tumores indiferenciados
d. es igual a la tinción por H Y E
132: molecula de adhesión que contienen los granulos alfa de las plaquetas:

a. selectina L
b. ICAM
c. PCAM
d. Selectina P

133: son ejemplos del grupo fisiopatologica por disminución de presión oncotica

a. Pericarditis constrictiva y trombosis


b. Filariasis y neoplasias
c. Desnutrición marasmatica y trombosis
d. Desnutrición kwashorkior y cirrosis hepatica

134: el factor de von willebrand

a. Participa en la hemostasia secundaria


b. Es un inhibidor de la coagulación
c. Se una a la glucoproteina IB de las plaquetas (según yo para dar paso a la adhesión
plaquetaria)
d. Se encuentra en el tejido endotelias ( marca según esta pero creo es la de arriba)

135: el edema de origen renal se caracteriza por:

a. Ser postural
b. Presentarse en tejido laxo
c. Presentarse la región sacra
d. Presentarse en miembros inferiores

tambien puede venir si la modifican para esta la respuesta seria retención de sodio esa
es.

136: las hemorragias diminutas de 1 a 2 MM se llaman:

a. Hematoma
b. Equimosis
c. Petequia
d. purpura

137: el tiempo de protrombina evalua la via intrínseca de la coagulación que factores


comprende:

a. XII,XI Y IX
b. VIII, X, Y V
c. XI, X Y FIBRINOGENO
d. VII, X, V Y FIBRONOGENO
138:no siempre responde a las moléculas que inhiben a la proliferación de las células
normales , debido a la inactivación de los genes supresor de tumores que modifican
componentes de estas vías de inhibitorias del crecimiento:

a. insensibilidad de las señales inhibitorios del crecimiento


b. potencial ilimitado de replicación
c. alteración del metabolismo celular
d. evasión de la apoptosis

139: una de las maneras por las que un tumor escapa de la respuesta inmune es: ( en
esta tengo duda de que creo esa no es la respuesta).

a. expresando proteínas inhibitorias de linfocitos T


b. expresando citocinas activadoras de linfocitos T
c. Sobre-expresando moléculas de CPH 1
d. Sobre-expresando antigenos

140: la edad influye de manera determinante en el riesgo de cáncer por lo tanto la mayor
parte de los carcinomas se presentan en:

a. Mayores de 20 años
b. Mayores de 55 años
c. 20 a 35 años
d. 10 a 20 años

141: Es un estado inflamatorio asociado a cáncer

a. Miositis
b. Enfermedad intestinal inflamatoria
c. condritis

142: la llamada induración parda es un ejemplo excepto:

a. congestión pasiva crónica de pulmón


b. congestión pasiva crónica esplénica
c. congestión pasiva crónica de hígado
d. Hemorragia de tubo digestivo

143: El carcinoma de mama, carcinoma de pulmon, carcinoma de células renales y la


leucemia de linfocitos T en el adulto se relación con el síndrome paraneoplasico:

a. Síndrome de cushing
b. Hupercalcemia
c. Hipocalcuria
d. Síndrome de secreción de hormona antidiuretica
144: Los estados hipercoagulativos se dividen en:

a. Fisiológicos y patológicos
b. Genéticos y adquiridos
c. De alto y bajo riesgo
d. Primarios y terciarios

145: afectan al proceso de cicatrización excepto:

a. Desnutrición
b. Tratamiento con clucocorticoides
c. Exceso de acido ascórbico
d. Diabetes mellitus

146: es un marcador tumoral de tipo hormonal:

a. CA-125
b. APS
c. CALCITONINA
d. ANTIGENO CARCINOGENOEMBRIONARIO

147: el restablecimiento de la estrucutra tisular normal solo se produce:

a. Cuando el tejido residual esta estructuralmente intacto (resección quirúrgica


parcial).
b. En nefrectomía unilateral
c. Cuando todo el tejido esta dañado por inflamación
d. Cuando todo el tejido esta dañado por infeccion

148: es considerado el guardian del genoma humano

a. TP53
b. K-RAS
c. MYC
d. JUN

149: sustancia generada por sistema de cininas, cuando hay daño vascular promueve
contracción de musculo liso, dolor y promueve la permeabilidad vascular.

a. BRADICININA
b. C5A DEL COMPLEMENTO
c. FOSFOLIPASA C
d. FACTOR DE NECROSIS TUMORAL
150: la característica histológica predominante de un infarto es:

a. Necrosis grasa
b. Necrosis fibrinoide
c. Necrosis caseosa
d. Necrosis coagulativa ( o isquémica)

151: es un carcinógeno químico de origen natural: (esta viene en la tabla del robins no en
texto).

a. Ciclosfosfamida
b. Cloruro de vinilo
c. Bencidina
d. griseofulvina

152: son funciones de la trombina excepto

a. conversión de fibrinogeno en fibrina


b. activación plaquetaria
c. secreción de prostaciclina
d. anticoagulante en el endotelio normal

153: en los tejidos lábiles el tipo de reparación de una lesión leve es:

a. cicatriz queloide
b. formación de cicatriz
c. senescencia
d. regeneración

154: en las células en repeso la proteica APC:

a. inhibe la proliferación por sobreexpresión de la beta- catenina


b. inhibe la proliferación por degradación de beta- catenina
c. induce apoptosis
d. induce proliferación por medio de beta- catenina

155: una de las características del shock es:

a. aumento del volumen sanguíneo circulante


b. aumento de la perfusión sanguínea
c. hipertensión
d. estado de Hipoperfusion tisular
156: consiste en aspirar las células y el liquido asociado con una aguja de pequeño
calibre

a. escisión
b. extensión para citología
c. trucut
d. punción aspiración con aguja fina

157: los agentes alquilantes son considerados:

a. carcinógenos que requieren activación metabolica


b. carcinógenos de acción directa
c. carcinógenos de acción indiretca
d. carcinógenos fisicos

158: cuál es la función de los macrófagos en la inflamación:

a. expresa receptores de superficie que se une a la porción FC


b. producir eotaxina
c. producir y secretar sustancias activas biológicas como citocinas, factores de
crecimiento
d. produce colagenasa

159: la trombocitopenia (plaquetopenia) y la uremia se pueden manifestar clínicamente


como:

a. hematomas
b. petequias
c. hipermenorrea
d. purpura

160: son características generales de las neoplasias malignas:

a. no presentar capsula, estar diferenciadas y crecimiento lento


b. presentar capsula, diferenciación y crecimiento relativamente lento
c. presentar capsula, estar diferenciadas y ser invasivas
d. no presentar capsula, poco diferenciadas y ser invasivas

161: en qué tiempo las neuronas sufren daños irreversibles ante la hipoxia:

a. 5 minutos
b. 1 minuto
c. 15 minutos
d. 30 minutos
162: son algunas causas de la alteración del retorno venoso:

a. Insuficiencia cardiaca congestiva, obstrucción y compresión venosa


b. Obstruccion linfática y retension de sodio
c. Calor y frio
d. Síndrome nefrotico y cirrosis hepatica

163: H.PYLORY ESTA RELACIONADO CON: (acuerden que igual a un MALToma por si
cambian la respuesta o ponen ambas juntas)

A. ADENOCARCINOMA GASTRICO
B. POLIPOS
C. LIPOMA
D. ADENOMA GASTRICO

164: son causas de edema de pulmon:

a. Insuficiencia del ventrículo izquierdo, insuficiencia renal e infección pulmonar


b. Insuficiencia del ventrículo derecho e infecciones pulmonares
c. Hipertensión portal y neuroinfeccion
d. Malnutrición y hipertensión arterial sistemica

165: los infartos rojos se producen en caso de:

a. Órganos con doble circulación


b. Casos con embolos sépticos
c. Oclusión arterial
d. Órganos solidos

166: son genes celulares normales cuyos productor fomentan la proliferación celular:

a. Onco proteínas
b. Aumento de expresión MYC
c. Monoclonalidad
d. protooncogenes

167: neoplasia maligna de hueso:

a. adenoma pleomorfico
b. osteosarcoma
c. leiomioma
d. condroma .
167: son tumores de origen germinal

a. glioma
b. mieloma multiple
c. tumor mixto
d. teratoma

168: los granulos densos de las plaquetas contienen excepto:

a. ADP
b. Calcio
c. Factor de von willebrand
d. Adrenalina y serotonina

169: la gastritis y las ulceras están relacionadas con qué tipo de neoplasia:

a. Adenocarcinoma gástrico y linfoma MALT


b. Hepatocarcinoma
c. Carcinoma pancreático
d. colangiocarcinoma

170: mecanismo derivado en el edema secundario a retorno venoso alterado:

a. retención de sodio
b. reducción de la presión osmótica del plasma
c. aumento de la presión hidrostática
d. inflamación

171: es un carcinógeno producido por el ASPERGELLUS FLAVUS: ( la que te decía


Erick)

a. ciclofosfamida
b. aflatoxina B
c. hidrocarburos aromaticos
d. cloruro de vinilo

172: el carcinoma microcitico de pulmón, carcinoma de páncreas y tumores neurales se


han relacionado con:

a. policitemia
b. hipercalcemia
c. síndrome de secreción inadecuada de hormona antidiuretica
d. síndrome de cushing
173: son ejemplos clínicos de choque asociado a inflamación sistémica:

a. traumatismo y quemaduras
b. hemorragias y vomitos
c. infecciones microbianas y pancreatitis
d. infarto al miocardio y rotura ventricular

174: neoplasia maligna del tejido linfoide:

a. Linfoma
b. Linfangiosarcoma (esta es de vaso linfático)
c. Leucemia
d. angiosarcoma
BANCO DE PREGUNTAS – ANATOMIA PATOLÓGICA
1. Funciones del factor de crecimiento transformador – Beta (TGF- B):
- Quimiotactico para leucocitos y fibroblastos
- Estimula la síntesis de proteínas de la MEC
- Inhibe la inflamación aguda

2. Se le conoce así, cuando el tejido cicatrizal crece más allá de los límites de la
herida original, además no se contrae.
a. Fibromatosis agresivas
b. Cicatriz queloide
c. Granulación exuberante
d. Dehiscencia de la herida

3. Neoplasia benigna de músculo Liso


Leiomioma

4. Neoplasia maligna de músculo liso:


Leiomiosarcoma.

5. Es la magnitud en que las células parenquimatosas neoplásicas se asemejan en


su función y forma a las células correspondientes del parénquima sano:
Diferenciación
6. Una lesión endotelial, tendrá como consecuencia.
a. No tiene relación con la coagulación.
b. Desarrollo de un ambiente procoagulante.
c. Produce la disolución de un coagulo
d. Desarrollo de un ambiente anticoagulante.

7. Es un grupo de proteínas grandes multifuncionales cuya principal funciones inhibir


a la angiogénesis:
Anticuerpos monoclonales

8. ¿Cómo se clasifican los infartos?


Infartos mucosos, murales y transmurales.

9. Masa neoplásica gelatinosa llamada pseudomixoma peritoneal, es formado a


veces por qué tipo de cáncer:
Carcinomas apendiculares secretores de moco
10. La congestión pasiva crónica del pulmón puede incluir todo lo siguiente excepto.
a. Edema pulmonar
b. Eritema
c. Macrófagos con hemosiderina
d. Induración café ( ya lo encontré esto si existe)
11. En la cicatrización por primera intención, ¿Cuándo se comienza a observar el
tejido de granulación?
a. Inmediatamente
b. Semanas después
c. A las 24 horas
d. 3-7 días después.

12. En el estado conocido como shock séptico


a. Es originado por perdida de líquidos
b. Es producido por el paso de bacterias, hongos al torrente circulatorio
c. Carece de respuesta inflamatoria
d. Se considera una respuesta inflamatoria local.

13. Son neoplásias mixtas:

Tumores mixtos de glándulas salivales, que contienen células epiteliales y estroma


mixoide.

14. ¿Cuál es la secuencia en la formación del tapón hemostático primario?


Las plaquetas se adhieren a la MEC expuesta a través del vWF y se activan, con
lo que sufren un cambio de forma y liberan sus gránulos; el ADP y el TxA2
liberados llevan a una mayor agregación plaquetaria hasta formar el tapón
hemostático primario.

15. ¿Cuál es la secuencia en la formación del tapón hemostático secundario?


La activación local de la cascada de la coagulación (en la que participan el factor
tisular y los fosfolípidos plaquetarios) da lugar a la polimerización de la fibrina, lo
que cementa las plaquetas en un tapón hemostático secundario definitivo.

16. A que gen induce TP53 para que dirigir a la célula hacia la apoptosis
Induce senescencia celular mediante E2F o genera apoptosis mediante aumento
de la transcripción de genes proapoptósicos

17. cuál es la proteína más abundante en el ser humano


a. Vimentina
b. Miosina
c. Caderina
d. Colágeno

18. El tiempo de protrombina (TP):


Es medido tras adición de factor tisular, fosfolípidos y calcio, estudia la función de
las proteínas en la vía extrínseca (VII, X, II, V y fibrinógeno).

19. El tiempo de protrombina parcial (TTP):


Es medido tras la adición de partículas de carga negativa (p, ej., de polvo de
vidrio), valora la función de las proteínas de la vía intrínseca (XII, XI, IX, VIII, X, V,
II y fibrinógeno).
20. El factor de crecimiento placentario pertenece a la familia de:
a. FCE
b. FCH
c. FCVE
d. FCFD

21. En un ateroma ulcerado, se produce:

22. El carcinoma ovárico, el fibrosarcoma y otros sarcomas mesenquimatodos, se han


relacionado con el siguiente síndrome paraneoplásico:
a. Síndrome de secreción inadecuada de hormona antidiurética
b. Síndrome de Cushing
c. Hipoglucemia
d. Hipercalemia

23. Permiten determinar el diagnostico, el pronóstico, la detección de enfermedad


residual mínima y el diagnóstico de la predisposición hereditaria al cáncer:
a. Análisis microbiológico
b. Historia clínica
c. Análisis molecular
d. Análisis morfológico

24. Es el término que describe la formación de tejido conectivo como respuesta a los
tumores:

25. Qué entidad se refiere a una forma de coagulopatía de consumo caracterizada


por trombosis microangiopatica de circulo vicioso trombosis y hemorragia:
Coagulopatía intravascular diseminada (CID)

26. El carcinoma de células renales, hemangioma, cerebeloso y el hepatocercinoma,


se ha relacionado con el siguiente síndroma paraneoplásico:
Policitemia
27. Son procesos en la evolución de un trombo excepto.
a. Embolia recanalización
b. Propagación.
c. Trombolisis
d. Recanalización

28. Son células madre que derivan de células adultas reprogramadas:


Células madre inducidas

29. Trombo formado en las cavidades cardíacas.


a. Trombo mural.
b. Aneurisma.
c. Petequia
d. Émbolo
30. Un trombo con líneas de Zahn:
Son trombos arteriales y parietales cardiacos, producidos por capas pálidas de
plaquetas y fibrina.

31. Contiene células o tejidos maduros e inmaduros reconocibles y pertenecientes a


más de una de las capas germinales (a veces, incluso de las tres):
a. Condroma
b. Linfoma
c. Teratoma
d. Fibroma

32. Una de las maneras por las que un tumor escapa de la respuesta inmune es:
a. expresando proteínas inhibitorias de linfocitos T
b. expresando citocinas activadoras de linfocitos T
c.Sobre-expresando moléculas de CPH 1
d. Sobre-expresando antígenos

33. Efectos de la trombina:


Escinde el fibrinógeno para producir fibrina, activa las plaquetas y las células
inflamatorias, la liberación de gránulos, favorece la anticoagulación.

34. ¿Cuáles son las células más importantes en la eliminación de agentes causales y
tejido muerto y promueven la migración y proliferación de los fibroblastos?
Macrófagos.

35. Con respecto a la coagulación intravascular diseminada, ¿cuál es el enunciado


correcto?
Se refleja en microtrombos generalizados de fibrina en la rnicrocirculación, es una
complicación de cualquier activación difusa de la trornbina. Los rnicrotrornbos
pueden causar una insuficiencia circulatoria difusa, en particular en el encéfalo,
los pulmones, el corazón y los riñones. Hay un consumo concurrente de plaquetas
y factores de la coagulación (coagulopatía de consumo)

36. Suelen ocasionar perdida de la función y, en la mayoría de los casos, para la


transformación se precisa el daño de los dos alelos.
Quemaduras
37. Molécula de adhesión que contienen los gránulos alfa de las plaquetas:
a. selectina L
b. ICAM
c. PCAM
d. Selectina P

38. La falta de diferenciación, es decir, implica una inversión de la diferenciación


hacia un plano más primitivo.
a. Displasia
b. Pleomorfismo
c. Anaplasia
d. Diferenciación
39. Neoplasia benigna mixta:
Adenoma pleomorfo, Tumores mixtos de glándulas salivales.

40. Son tumores constituidos por células inmaduras que se parecen a las que forman
tejido rudimentario fetal.
a. Papiloma
b. Blastoma
c. Hamartoma
d. Osteoma

41. Acumulación de cantidades excesivas de colágeno que crece más allá de los
bordes de la herida original y no regresa.
Cicatriz queloide.

42. Estos trombos se forman en la circulación venosa, suelen contener más eritrocitos
incorporados y relativamente pocas plaquetas.
a. Trombos arteriales.
b. Trombos rojos.
c. Trombos murales.
d. Vegetaciones.

43. Son agentes que estimulan el crecimiento de una neoplasia

44. Es ejemplo de edema por disminución de la presión oncótica:


Hipoalbuminemia, Síndrome nefrótico, Hepatopatía, Enteropatías con pérdida de
proteínas y Obstrucción linfática.
45. Son causas de edema de pulmón:
a. Insuficiencia del ventrículo izquierdo, insuficiencia renal e infección pulmonar
b. Insuficiencia del ventrículo derecho e infecciones pulmonares
c. Hipertensión portal y neuroinfeccion
d. Malnutrición e hipertensión arterial sistémica

46. Son características de las neoplasias benignas:


47. Se refiere así, a la tendencia perniciosa de los tumores a adquirir una conducta
cada vez más agresiva:

48. La deficiencia de GPIIB – IIIa produce la enfermedad de:


Tromboastenia de Glanzmann
49. En qué tiempo las neuronas sufren daños irreversibles ante la hipoxia:
a. 5 minutos
b. 1 minuto
c. 15 minutos
d. 30 minutos

50. Neoplasia maligna de células mesoteliales:


Mesotelioma

51. Morfológicamente las neoplasias malignas se caracterizan por:

52. Enfermedad granulomatosa no necrosante:


Neumonía por aspiración

53. El cromosoma filadelfia se ha relacionado con


a. Linfoma
b. Retinoblastoma
c. Leucemia de células peludas
d. Leucemia mieloide crónica

54. Fenómeno caracterizado por la dilatación de las arteriolas:


a. Hemorragia
b. Congestión pasiva
c. Hiperemia
d. Congestión crónica

55. Tienen una función esencial en muchos aspectos del fenotipo maligno, como la
expresión de los genes del cáncer, el control de la diferenciación y la
autorrenovación, incluso la sensibilidad de la resistencia a los medicamentos:
a. Cambios epigenéticos
b. Amplificación genética
c. Cambios cromosómicos
d. Deleción

56. Se refiere a la respuesta del cuerpo que reestablecerá el estado normal del tejido
en la localización de inflamación aguda:
a. Cicatrización
b. Curación por reposición del tejido conjuntivo
c. Fibrosis
d. Resolución o remisión

57. El origen más frecuente de los trombos en embolias pulmonares es:


a. Trombos in-situ del pulmón
b. Venas pélvicas
c. Venas profundas de las piernas
d. Venas periprostáticas

58. ¿Qué son los granulomas?


a. Está compuesto por macrófagos epiteliodes y linfocitos, fibroblastos.
b. Está compuesto por linfocitos T CD8
c. Están compuestos por células mononucleares, células gigantes y neutrófilos.
d. Están compuestos por neutrófilos, macrófagos y células gigantes.

59. La traslocación cromosómica de leucemia mieloide crónica es:


a. En el cromosoma 9 y 22
b. Cromosoma 14 y 18
c. En el cromosoma X
d. Del cromosoma 8 al 14

60. Causa shock neurogénico:


a. Sangrado genital normal
b. Antígenos
c. Sepsis por virus
d. Sección medular alta

61. Al líquido de edema ubicado en cavidad torácica se le denomina:


a. Trasudado
b. Exudado
c. Hidrotórax
d. Anasarca

62. la característica histológica predominante de un infarto es:


a. Necrosis grasa
b. Necrosis fibrinoide
c. Necrosis caseosa
d. Necrosis coagulativa (o isquémica)
63. Se considera como propagación de las neoplasias a sitios anatómicamente
separados del lugar de origen.
a. Metástasis
b. Micro invasión.
c. Son masas expansivas cohesivas.
d. Invasión.

64. Son acontecimientos plaquetarios excepto:


a. Adhesión
b. Migración
c. Cambio de forma y secreción
d. Agregación

65. Se le denomina así cuando las células parenquimatosas estimulan la síntesis de


un estroma con colágeno abundante:
a. Estroma reactivo
b. Pétreo
c. Desmoplasia
d. Parénquima

66. ¿Qué enzimas secreta la célula neoplásica para destruir la membrana basal del
endotelio y penetrar a la circulación?
a. Oxigenasa, metalasa, gelatinasa.
b. Condroitinsulfatasa, mostenectinasa, colagenasa.
c. Colagenasas, gelatinasas, estromelisinas.
d. Sulfatasa, colagenasa, condronectinasa.

67. Vía de la apoptosis que más se desregula en la situación del cáncer:


a. Vía intrínseca
b. Vía extrínseca
c. Piroptosis
d. Necroptosis

68. vía de la apoptosis más involucrada en cáncer


a. Necroptosis
b. Vía extrínseca
c. Vía intrínseca
d. Piroptosis

69. La lesión subletal por radiaciones ultravioleta causa:


a. Ca de pulmón
b. Ca de células basales
c. Ca de tiroides
d. Ca Renal

70. Son características de los tejidos lábiles:


a. Siempre se encuentran en G0
b. Meiosis
c. Proliferan toda la vida de la célula
d. Se presentan fuera del ciclo celular
71. Causa dilatación de las arteriolas y aumenta la permeabilidad de las vénulas.
a. TNF
b. Serotonina
c. Prostaglandinas
d. Histamina

72. Proceso de hemorragia en la cavidad corporal:


a. Equimosis
b. Petequias
c. Hemoperitone
d. DEBIÓ SER ESTA POR QUE NO ES NINGUNA DE LAS ANTERIORES.

73. Neoplasia benigna del cartílago:


a. Adenoma pleomorfo
b. Osteosarcoma
c. Condroma
d. Leimioma

74. La caquexia del cáncer se ha asociado a:


a. Ganancia de grasa y musculo magro.
b. Neutralización de los reactantes de la fase aguda.
c. Disminución del metabolismo basal.
d. Pérdida progresiva de grasa corporal y musculo magro.

75. Los diferentes estados de hipercoagulabilidad conocidos hasta ahora:


a. Siempre son adquiridos
b. Rara vez son adquiridos
c. Siempre son hereditarios
d. Pueden ser tanto hereditarios como adquiridos
e.

76. Son acontecimientos plaquetarios excepto.


a. Adhesión
b. Migración
c. Cambio de forma y secreción
d. Agregación

77. ¿Qué puede producir un exceso de factores estimulantes del crecimiento celular?
a. Trombosis
b. Necroptosis
c. Apoptosis
d. Cáncer o crecimiento incontrolado

78. Sustancia producida por las células endoteliales que promueve vasoconstricción
en caso de ruptura vascular:
a. Óxido nítrico
b. Prostaciclina
c. Endotelina
d. Óxido nítrico
79. Causa más frecuente de tromboembolia pulmonar:
a. Insuficiencia venosa
b. Deficiencia de proteína C
c. Trombosis venosa profunda
d. Trombos intracardiacos

80. Los patrones moleculares asociados a patógenos se unen a:


a. Receptor con actividad tirosin cinasa intrínseca
b. Receptores tipo señuelo
c. Receptores de siete proteínas
d. Receptores Jak Stat

81. Los factores que influyen en la curación de heridas se clasifican en:


a. Del huésped y el medio ambiente
b. Intrínsecos y extrínsecos
c. Locales y sistémicos
d. Internos y externos

82. Una de las características principales del shock es:


a. Hipertensión
b. Estado de hipoperfusión tisular
c. Aumento del volumen sanguíneo circulante
d. Aumento de la perfusión tisular

83. Tipos generales de señalización


a. Intrinseca – extrínseca
b. Intracelualr extracelular
c. Señal – receptor
d. Autocrina – paracrina – endocrina

84. La mayor parte de las flebotrombosis se producen en.


a. Arterias de mediano calibre.
b. Corazón
c. Venas superficiales y profundas de las piernas.
d. Arterias cerebrales.

85. Suele presentarse tras fracturas de huesos largos o rara vez, en traumatismos de
partes blandas y quemaduras:
a. Embolia grasa
b. Embolia pulmonar
c. Embolia de líquido amniótico
d. Embolia gaseosa

86. Tras una hepatectomía parcial:


a. Se produce cirrosis
b. Se produce fibrosis
c. Se produce una cicatriz
d. El hígado se regenera y los hepatocitos proliferan

87. Los principales componentes de la reparación con tejido conjuntivo son:


a. Lesión, inflamación, granulación, fibrosis, contracción
b. Inflamación, granulación, fibrosis
c. Angiogénesis, migración y proliferación de fibroblastos, síntesis de colágeno y
remodelación
d. Angiogénesis, depósito de la MEC, proliferación, fibrosis y reepitelizacion

88. Proceso activo donde se observa dilatación arteriolar con aumento del flujo
sanguíneo:
a. Trombosis
b. Congestión
c. Hiperemia
d. Hemorragia

89. El desarrollo de nuevos canales de sangre a través de trombos oclusivos se


llama:
a. Infarto
b. Propagación
c. Recanalización
d. Fibrinólisis

90. ¿Qué es incorrecto respecto a la triada de Virchow?


a. Trombocitopenia
b. Lesión endotelial
c. Estasis
d. Hipercoagulabilidad

91. Se refiere así, a la pérdida progresiva de la grasa corporal y de la masa corporal


magra, acompañada de una debilidad profunda, anorexia y anemia.
a. Caquexia cancerosa
b. Síndromes paraneoplásicos
c. Efectos hormonales del cáncer
d. Efectos locales del cáncer

92. Estado patológico que puede causar congestión venosa:


a. Trombosis
b. Estados de hipercoagulabilidad
c. Varices
d. Valvulopatías

93. ¿Qué función tiene el factor de crecimiento derivado de las plaquetas en la


angiogenia?
a. Induce depósito de colágeno tipo IV
b. Induce proliferación endotelial
c. Induce proliferación de musculo liso en las arteriolas
d. Induce la permeabilidad de los vasos de neoformacion

TENGO DUDA, EL PÁRRAFO DICE ASÍ: La neovascularización también estimula el


crecimiento tumoral a través de la producción por las células endoteliales de factores
como el factor de crecimiento similar a la insulina y el factor de crecimiento derivado de
las plaquetas (PDGF). Sin vasos nuevos, el tumor no accede al sistema vascular, de
manera que la angiogenia influye claramente en el potencial metastásico.
94. El restablecimiento de la estructura tisular normal solo se produce:
a. Cuando el tejido residual esta estructuralmente intacto (resección quirúrgica
parcial).
b. En nefrectomía unilateral
c. Cuando todo el tejido está dañado por inflamación
d. Cuando todo el tejido está dañado por infección

95. Las bacterias Gram positivas y Gram negativas pueden provocar:


a. Shock cardiogénico
b. Shock séptico
c. Shock anafiláctico
d. Shock hipovolémico

96. Son factores de mayores para aterosclerosis los siguientes, excepto:


a. Tabaquismo
b. Aumento de la edad
c. Hipertensión arterial
d. Dieta alta en grasas poliinsaturadas

97. Son protooncogenes excepto:


a. BRCA 1
b. Ciclinas
c. RAS
d. BRAF

98. En el cáncer, es el paso inicial que induce un cambio irreversible pero no letal en
el material genético de la célula afectada:
a. Iniciación
b. Progresión
c. Promoción
d. Reorganización

99. Es un tipo de tumor benigno:


a. Sarcoma
b. Adenoma
c. Leucemia
d. Linfoma

100. ¿Cuáles son los genes que fomentan el crecimiento celular autónomo?
a. P53
b. Genes superiores de tumores
c. Oncogenes
d. Genes involucrados en la reparación de ADN

101. Marcador tumoral relacionado con carcinomas de colon, páncreas, pulmón,


estómago y corazón.
a. Gonadotropina coriónica humana
b. Antígeno carcinoembrionario CEA pp.224
c. Calcitonina
d. Alfa fetoproteína

102. Es una característica exclusiva de las neoplasias malignas:


a. Necrosis
b. Metástasis
c. Mitosis
d. Angiogénesis

103. Todas las siguientes describen las formas de hemorragia, excepto:


a. Hemopericardio
b. Petequias
c. Hemostasia
d. Hemotorax

104. Son masas desorganizadas, pero benignas, de células propias del lugar afectado:
a. Teratomas
b. Condromas
c. Hamartomas
d. Lipomas

105. Son características de la congestión pulmonar crónica, excepto:


a. Presenta macrófagos en lipofuscina
b. Es causada por insuficiencia cardiaca congestiva
c. Tiene macrófagos con hemosiderina
d. Puede presentar fibrosis y edema septal

106. Neoplasia maligna de vasos linfáticos:


a. Linfoma
b. Linfangiosarcoma
c. Leucemia
d. Angiosarcoma

107. Consiste en aspirar las células y el líquido asociado con una aguja de pequeño
calibre
a. escisión
b. extensión para citología
c. trucut
d. punción aspiración con aguja fina

108. Está determinada por el aspecto citológico y se basa en la idea de que el


comportamiento y la diferenciación guardan relación entre sí, de tal suerte que los
tumores poco diferenciados presentan una conducta más agresiva:
a. Caquexia
b. Estadificacion
c. Gradación
d. Clasificación

109. La regeneración hepática, la progresión de los hepatocitos del estadio G1 a S


comienza con la formación del complejo:
a. Ciclina D – CDK4
b. Ciclina A – CDK2
c. Ciclina B – CDK1
d. Ciclina D – CDK6

110. Hace referencia al exceso de depósito de colágeno y otros componentes de la


matriz extracelular en un tejido:
a. Tejido de granulación
b. Regeneración
c. Cicatrización y fibrosis
d. Granuloma

111. Aumento de líquido en cavidad peritoneal se le conoce como:


a. Hidrotórax
b. Liquido ascitis
c. Hidropericardio
d. Anasarca

112. Son consideradas causa de shock cardiogénico:


a. Reaccionó inmunitaria severa
b. Hemorragias masivas
c. Infarto, arritmias ventriculares
d. Accidentes anestésicos

113. Al proceso inflamatorio que no se resuelve por sí solo, se puede considerar como:
a. Inflamación crónica
b. Inflamación aguda
c. Inflamación serosa
d. Inflamación fibrinosa

114. Genes alterados en las células neoplásicas a favor de la cascada RAS:


a. RB / P53
b. BAX / BCL2
c. MYC / JUN
d. BRAF / TOB

115. El edema que ocurre en la filariasis se debe a.


a. Disminución de la presión oncótica.
b. Bloqueo de la circulación linfática.
c. Aumento de la presión hidrostática venosa.
d. Aumento de la permeabilidad vascular.

116. La vía de propagación más común de los carcinomas es:


a. Vía hematógena
b. Siembra directa
c. Siembra quirúrgica
d. Vía linfática

117. Características de las señales sinápticas.


a. Secreción de neurotransmisores en uniones celulares especiales.
b. De difusión mínima y la señal se degrada rápido.
c. Se usa para amplificar una respuesta o inhibirla mediante retroalimentación.
d. Se libera al torrente circulatorio y actúa distancia sobre las células blanco.
118. ¿Cuál es el principal lugar de embolia arteriolar?
a. Riñones
b. Extremidades superiores 10%
c. Encéfalo
d. Extremidades inferiores 75%

119. Es un ejemplo de tejidos quiescentes:


a. Epitelio
b. Sangre
c. Musculo estriado
d. Hígado

120. Son funciones de la trombina excepto


a. conversión de fibrinógeno en fibrina
b. activación plaquetaria
c. secreción de Prostaciclina
d. anticoagulante en el endotelio normal

121. Es un protooncogen que pertenece a los inhibidores de las cinasas (INK)


a. P16
b. RB
c. P53
d. MYC

122. neoplasia maligna de hueso:


a. adenoma pleomorfo
b. osteosarcoma
c. leiomioma
d. condroma

123. ¿Cuál de las siguientes opciones es una causa de edema NO inflamatorio?


a. Coccidioidomicosis
b. Frío
c. Insuficiencia cardiaca congestiva
d. Tuberculosis

124. Analiza la aparición de tumores en poblaciones humanas:


a. Servicios coordinados
b. Biología molecular del cáncer
c. Salud publica
d. Epidemiología del cáncer

125. Es secundario a una insuficiencia de bomba miocárdica debido a las lesiones


intrínsecas del miocardio, compresión extrínseca u obstrucción al flujo de salida:
a. Choque toxico
b. Choque cardiogénico
c. Choque hipovolémico
d. Choque asociado a la inflamación sistémica
126. La formación excesiva de componentes que participan en el proceso de
reparación, en ocasiones dan lugar a:
a. Ulceras
b. Granulación exuberante
c. Fibromatosis agresiva
d. Cicatrización hipertrófica (puse eso porque el inciso D no venía en la foto)

El párrafo decía: Exceso de reparación. El exceso de tejido de granulación (excrecencias


carnosas) da lugar a una prominencia en la piel circundante, impidiendo la reepitelizacion. La
acumulación excesiva de colágeno forma una cicatriz hipertrófica elevada; la progresión más allá
del área original de & la lesión, sin la subsiguiente regresión, se denomina queloide.

127. La congestión pasiva crónica del pulmón puede incluir todo lo siguiente excepto.
a. Edema pulmonar
b. Eritema
c. Macrófagos con hemosiderina
d. Induración café (ya lo encontré esto si existe)

126. Si la alteración displásica es intensa y afecta a todo el espesor del epitelio pero la
lesión no penetra la membrana basal, se habla de.
a. Carcinoma in situ.
b. Tumor infiltrante.
c. Tumor invasivo.
d. Metaplasia.

127. El proceso de contracción de la herida se produce en casos de:


a. Curación por segunda intención
b. Organización del exudado
c. Curación por primera intención
d. Cicatrización queloide

128. Se identifica como: masa sólida, liquida o gaseosa transportada en la sangre a lugar
diferente de su lugar de origen:
a. Coagulo
b. Embolo
c. Trombo
d. Quiste

129. Al quinto día se observan los siguientes cambios:


a. Se observa en los bordes neutrófilos que migran hacia el coagulo
b. Neovascularización intensa y el tejido de granulación invade la herida
c. Las fibras de colágeno son más abundantes y establecen fuentes de incisión
d. Contracción de la herida

130. Al tercer día se observan los siguientes cambios:


a. Se observa en los bordes neutrófilos que migran hacia el coagulo
b. Neovascularización intensa y el tejido de granulación invade la herida
c. Las fibras de colágeno son más abundantes y establecen fuentes de incisión
d. Contracción de la herida
131. Marcador tumoral relacionado con el cáncer prostático:
a. APE
b. CA – 125+
c. Alfa Fetoproteína
d. CA – 19 – 9
e. PSA

132. En esta fase de choque se presenta insuficiencia renal por necrosis tubular aguda
que la supervivencia es imposible:
a. Fase de hipoperfusión tisular
b. Fase irreversible
c. Fase no progresiva
d. Fase progresiva

133. La estadificación de un tumor maligno se refiere a.


a. Grado de diferenciación de las células tumorales.
b. Tratamiento quirúrgico
c. Diagnóstico molecular.
d. Tamaño de la lesión primaria y si ha realizado metástasis

134. Las células de estos tejidos se destruyen y se reponen continuamente por


maduración, a partir de células madre adultas y por proliferación de células
maduras.
a. Tejidos lábiles
b. Tejidos estables
c. Tejidos permanentes
d. Tejidos proliferativos

135. Angiogenia se conoce como:


a. La información de nuevos vasos sanguíneos a partir de los ya existentes
b. Formación de nuevos vasos sanguíneos durante la vida fetal
c. Tejido nuevo que invade progresivamente el sitio de la lesión
d. Maduración y reorganización secuencial post lesión tisular

136. ¿En el hígado, donde se encuentran las células madre?


a. En los canales de hering
b. En el lobulillo hepático
c. Al final de la vena porta hepática
d. En los sinusoides

137. La embolia paradójica puede producirse en caso de:


a. Hipercoagulabilidad
b. Hipertensión arterial
c. Comunicación intercelular o interventricular
d. Trombos murales

138. Enfermedad que puede causar perdida hemática:


a. Sangrado genital anormal
b. Trombosis
c. Varices
d. Anemia

139. Las hemorragias diminutas de 1 a 2 MM se llaman:


a. Hematoma
b. Equimosis
c. Petequia
d. purpura

140. Estos genes se han relacionado con el carcinoma familiar de mama y ovárico;
carcinoma de mama masculino y leucemia linfocítica crónica:
a. BRCA 1 Y BRCA 2
b. TP53
c. CDH1
d. WT1 Y MEN 1

141. Los estados de hipercoagulabilidad se dividen en:


a. Fisiológicos y patológicos
b. Genéticos y adquiridos / primarios y secundarios
c. De alto y bajo riesgo
d.

142. Sitio donde frecuentemente se origina la trombosis venosas profundas:


a. En las extremidades superiores
b. En la vena cava
c. Circulación pulmones
d. En las extremidades inferiores

143. Deficiencia de GP18 provoca:


Bernard Soulier

144. Principales factores de crecimiento implicados en síntesis de tejido cognitivo:


IL - 1

145. Son neoplasias, tienen peor diferenciación o son totalmente indiferentes:


Anaplasias

146. Es el evento principal de la hemostasia primaria:


Formación del tapón plaquetario

147. La estadificación de los canceres solidos se basa en el sistema:


Sistema e TNM

148. ¿Cuál de las siguientes causas es más probable que se acumule el líquido, si
existe variación?
Presión oncótica

149. Neoplasia maligna de célula de los ductos Seminales:


Seminoma
150. Un estado inflamatorio crónico puede causar anémica ferropénica en respuesta al:
Aumento de la hepcidina

151. Su crecimiento se compaña de infiltración, invasión y destrucción progresiva del


tejido circundante, poco delimitado y son de lenta expansión:
Neoplasia maligna

152. Es la fase de shock en la que se activa el eje renina angiotensina (R A A):


Fase no progresiva

153. Corresponde al evento principal de la hemostasia primaria:


Formación del tapón plaquetario

154. Fase del shock donde fracasan mecanismos reflejos de compensación, disminuye
la perfusión de órganos vitales:
Fase progresiva

155. Características del tejido de granulación


Rico en tejido conjuntivo provisional y nuevos capilares finos

156. Vía más frecuente de diseminación de carcinomas


Linfática

157. Es el colágeno que proporciona fibrillas de anclaje de las células epiteliales a la


membrana basal
Colágeno tipo IV

158. La falta de diferenciación, es decir, implica una inversión de la diferenciación hacia


un plano más primitivo.
Anaplasia

159. Trombos que se forman en la circulación venosa o suelen contener más eritrocitos
incorporados y pocas plaquetas:
Trombos rojos

160. Fases de reparación en los heridas:


Angiogenia, tejido de granulación y remodelación

161. Patología donde se unen mutacines en el gen TP53 y tiene probabilidad 25 veces
mayor de experimentar una neoplasia maligna antes de los 50 años:
Sx de Li-fraumeni

162. Gen APC se ha relacionado con las sıguientes neoplasias:


Carcinoma de estómago, colon, páncreas y melanoma.

163. Neoplasias malignas de células germinales de testículo


Seminomas o carcinoma embrionario

164. Características de síndrome de Trousseau asociado a carcinoma de páncreas y


estómago se refiere a:
Tromboflebitis migratoria

165. Qué tan resistentes son las musculares cardiacas a la hipoxia:


Bastante sensible, mueren a los 20-30 min.

166. Son patologías que producen edema por aumento de la presión hidrostática del
vaso:
Pericarditis contrictiva y trombosis

167. Se le conoce así cuando el tejido cicatrizal crece más de los límites de la herida:
Cicatriz queloide

168. Factores sistémicos que influyen en la curación de heridas:


Nutrición, situación circulatorio estado metabólico y hormonal

169. Marcador tumoral de cáncer prostático:


PSA

170. Morfológicamente las neoplasias malignas se caracterizan por:


Pleomorfismo nuclear y celular

171. Dentro de las funciones de p53 se encuentran, excepto:


a. Pausar el ciclo celular para reparación de ADN.
b. Inducir apoptosis.
c. Inducir proliferación celular
d. Inducir senescencia

172. Cáncer es una enfermedad caracterizada por:


Adquisición de mutaciones letales

173. Se observa tras una destrucción sustancial de tejido cuando la lesión inflamatoria
afecta a tejidos que no son capaces de regenerarse o cuando hay abundante
exudación de fibrina en tejidos o cavidades serosas, que no es posible eliminar
adecuadamente:
Curación por reposición de tejido conjuntivo

174. Marcador tumoral Relacionado con carcinoma de colon, páncreas, pulmón y


mama:
Antígeno Carcinoembrionario

175. El gen 9.12 se asocia con:


Leucemia mieloide
176. Características de señales sinápticas:
Secreción de neurotransmisores en uniones celulares especiales.

177. Neoplasia maligna de la glándula tiroides:


Adenocarcinoma

178. Se caracteriza como extensión de las neoplasias a sitios anatómicamente


separados del lugar de origen:
Metástasis

179. Los tipos de VPH de alto riesgo producen proteína oncogena que ocasionan lo sig.
excepto:
a. Activan Ciclinas.
b. Inhiben la apoptosis.
c. Combaten la senescencia celular.
d. Inmunodeficiencia

180. Sustancias que inhiben la función plaquetaria:


On. Prostacidina y ADPasa

181. El gen 8, inhibidor de la progresion celular se ha relacionado con:


Retinoblastoma, osteosarcoma, C de mama, colon y pulmón

182. Trombos formados en cavidades cardiacas:


Trombo mural

183. Causas de la alteración de retorno venoso:


Insuficiencia Cardiaca congestiva, obstrucción y compresión venosa

184. La asbestosis y silicosis está asociada con qué tipo de neoplasia:


Mesotelioma y Cáncer de pulmón

185. Cual enunciado acerca de los plaquetas es verdad:


a. contiene cuerpos densos ricos en fibrinógeno.
b. Los precursores son monocitos
c. Contiene gránulos alfa ricos en difosfato de adenosina.
d. Contiene actina y miosina, microfilamentos y microtubulos.

186. El edema que ocurre en la filariasis se debe a:


Bloqueo de la circulación linfática

187. Contiene mayor cantidad de sangre oxigenada y se caracteriza por color rojo
brillante:
Hiperemia activa

188. Neoplasia Maligna del epitelio placentario:


Coriocarninoma
--------------------------------------------------------------------------------------------------------------
2° PARCIAL
1. Se utiliza para determinar el avance del cáncer mediante exploración
quirúrgica con técnicas de imagen y depende del tamaño, la propagación a
los ganglios linfáticos locales y regionales, y la presencia de metástasis
remota
A. Clasificación
B. Caquexia
C. Gradación
D. Estadificación

2. Sustancia producida por las células endoteliales que produce


vasoconstricción en caso de ruptura vascular
A. Óxido nítrico
B. Prostaciclina
C. Endotelina
D. Óxido Nítrico

3. La vía de propagación más común de los carcinomas es


A. Siembra quirúrgica
B. Vía hematógena
C. Siembra directa
D. Vía linfática

4. Es una causa de edema por disminución de la presión oncótica


A. Diabetes mellitus
B. Desnutrición
C. Insuficiencia cardiaca
D. Cáncer

5. Cuál es la característica que diferencia la curación por primera y segunda


intensión
A. La presencia de tejido de granulación
B. Filtración de neutrófilos y basófilos
C. Contracción de la herida
D. Presencia de factores de crecimiento derivados del endotelio

6. Neoplasia maligna de las células de los ductos seminales


A. Coriocarcinoma
B. Carcinoma embrinario
C. Seminoma
D. Tumor de senos endodermicos

7. Describe el deposito extenso de colágeno que se registra en pulmones,


hígado, riñon y otros órganos como consecuencia de la inflamación crónica
A. Proliferación
B. Fibrosis
C. Regeneración
D. Resolución

8. Un defecto en la expresión del factor de Von Willebrand


A. Es incompatible con la vida
B. Impide la hemostasia secundaria
C. Puede llevar a formación de petequias
D. Provoca hemorragias graves

9. Característica principal de las células madre adultas


A. Se encuentra en cerebro
B. Se encuentran en ciclo celular
C. Realizan división celular simétrica
D. Son pluripotenciales
Aquí me quedó duda porque dice:
Las Células Madre Embrionarias son pluripotenciales
Células madre adultas Realizan división asimétrica
Entonces quedaría la opción de se encuentran en cerebro

10. Se caracteriza por exudación del líquido con bajo contenido de células en
los espacios creados por la lesión celular o en las cavidades corporales
A. Inflamación granulomatosa
B. Inflamación fibrinosa
C. Inflamación serosa
D. Inflamación supurativa

11. La cicatriz de primera intención es dada por heridas con


A. Bordes con sepsis
B. Bordes necrosados
C. Bordes anfractuosos
D. Bordes alineados

12. De los componentes celulares básicos de un tumor, su nomenclatura se


basa en
A. En su tamaño
B. En su malignidad
C. En el comportamiento parenquimatoso
D. En el comportamiento estromal

13. Son células madre que derivan de células adultas reprogramadas


A. Células madre embrionarias
B. Células madre tisulares
C. Células madre inducidas
D. Células madre mesenquimatosas

14. Características clínicas de un paciente en shock séptico


A. Piel fría, palidez, alteraciones en el estado de conciencia
B. Piel caliente, enrojecimiento, anuria, taquicardia
C. Piel cianótica, hipotensión, taquipnea
D. Piel palida, diaforesis

15. En este tipo de shock hay insuficiencia de la bomba miocárdica debido a


lesiones intrínsecas de miocardio
A. Shock séptico
B. Shock hemorrágico
C. Shock hipovolémico
D. Shock cardiogeno

16. Los diferentes estados de hipercoagubilidad conocidos hasta ahora


A. Siempre son hereditarios
B. Siempre son adquiridos
C. Pueden ser tanto hereditarios como adquiridos
D. Rara vez son adquiridos

17. Son procesos en la evolución de un trombo


A. Embolia
B. Propagación
C. Trombolisis
D. Recanalización

18. Factor de riesgo para aterosclerosis


A. Estrés
B. Alcoholismo
C. Tabaquismo
D. Aumento VLDL

19. ¿Cuáles son los genes que fomentan el crecimiento celular autónomo?
A. Genes involucrados en la reparación de ADN
B. P53
C. Genes supresores de tumores
D. Oncogenes

20. Neoplasia benigna de los melanocitos


A. Verruga
B. Nevo
C. Angioma
D. Cistoadenoma

21. El virus de Epstein Barr produce


A. Hepatocarcinoma
B. Sarcoma de Kaposi
C. Leucemia de células peludas
D. Linfoma de Burkitt

22. El edema de origen cardíaco se caracteriza


A. Localizarse a nivel periorbitario
B. Localizarse en cavidades
C. Aparece por las mañanas
D. Ortostatico/subcutáneo

23. ¿Qué es incorrecto respecto de la triada de Virchow?


A. Lesión endotelial
B. Hipercoagubilidad
C. Estasis
D. Trombocitopenia

24. Proceso hemorrágico en cavidad corporal


A. Hemoperitoneo
B. Equimosis
C. Petequias
Las hemorragias extensas hacia las cavidades corporales reciben distintos
nombres en función de la localización —hemotórax, hemopericardio,
hemoperitoneo y hemartros (en las articulaciones). En algunos casos, las
hemorragias extensas pueden ocasionar ictericia por la degradación masiva de los
eritrocitos y la hemoglobina.

25. Es el coagulo constituido por agregación plaquetaria y depósito de fibrina


A. Tapón hemostático primario
B. Antitrombina
C. Hemostasia
D. Tapón hemostático secundario
Estos agregados de plaquetas rellenan el defecto vascular, formando el tapón
hemostático primario. C. La activación local de la cascada de la coagulación (en la
que participan el factor tisular y los fosfolípidos de las plaquetas) determina la
polimerización de la fibrina, lo que aporta un cemento para que las plaquetas den
lugar al tapón hemostático secundario definitivo, que es más grande y estable que
el primario y contiene eritrocitos atrapados y leucocitos.

26. Son patologías que producen edema por aumento de la presión hidrostática
del vaso
A. Neoplasias y filarias
B. Aumento de la secreción renina-angiotensina-aldosterona
C. Glomerulopatias y malnutrición
D. Pericarditis constrictiva y trombosis

27. La trombocitopenia (plaquetopenia) y la uremia se pueden manifestarse


clínicamente con
A. hipermenorrea
B. petequias
C. hematomas
D. purpura

28. neoplasia benigna del cartílago


A. adenoma pleomorfo
B. osteosarcoma
C. condroma
D. Leiomioma

29. Factor que convierte al fibrinógeno en fibrina, activa el endotelio a linfocitos,


a monocitos y produce agregación plaquetaria
A. Proteína c
B. Trombina
C. Factor tisular
D. TPA (Activador del plasminogeno tisular)

30. Es el término que describe la formación de tejido conectivo como respuesta


a los tumores
A. Anaplasia
B. Displasia
C. Desmoplasia
D. Metaplasia

31. Los granulos densos de las plaquetas contienen excepto


A. ADP
B. Calcio
C. Factor de Von Willebrand
D. Adrenalina y serotonina

32. Es una causa de edema por disminución de la presión oncótica


A. Diabetes mellitus
B. Desnutrición
C. Insuficiencia cardiaca
D. Cáncer

33. Se define como la propagación del tumor a sitios físicamente alejados al


tumor primario y marca de un modo inequívoco, dicho tumor como maligno
A. Tumor invasivo
B. Infiltrante
C. Metástasis
D. Neoplasia preinvasiva

34. Sustancias inductoras de fiebre se considera como


A. Neurotransmisores
B. Enzimas
C. AINES
D. Pirógenos

35. Es una proteína que estimula la degradación de P53


A. MPM
B. MDM2
C. INK
D. E2F

36. Como se clasifican los infartos


A. Rojos, blancos, sépticos
B. Cicatriz primera intención, de segunda intención
C. Cicatrización por primera intención y de segunda intención
D. Agudos, crónicos

37. Son las etapas de la carcinogenia química


A. Únicamente propagación
B. Nucleación y propagación
C. Únicamente iniciación
D. Iniciación y promoción
38. ¿Cuál es el agente biológico implicado en el linfoma de Burkitt?
A. Virus de EPSTEIN-BARR
B. Virus B y C de la hepatitis
C. Virus de papiloma humano
D. Helicobacter pylori

39. Son cambios que ocurren en el shock neurogénico


A. Fiebre
B. Vasodilatación periférica
C. Lesiones inducidas por leucocitos
D. De coagulación intravascular diseminada

40. Al líquido de edema generalizado se le conoce como:


A. Hidrotórax
B. Trasudado
C. Anasarca
D. Exudado

41. Localización más frecuente de lo trombos arteriales:


A. Arteria esplénica tiroidea
B. Arteria coronaria, cerebral y femoral
C. Arteria cerebral, braquial, torácica y epigástrica
D. Arteria aorta, pulmonar, la carótida y la subclavia

42. Son consideradas causas de shock cardiogeno


A. Hemorragias masivas
B. Accidentes anestésicos
C. Infarto, arritmias ventriculares
D. Reacción inmunitaria severa

43. Como se llaman los tejidos que están en la fase G0 y forman la mayor parte
de los órganos parenquimatosos
A. Tejidos estables
B. Tejidos en continua división
C. Tejidos permanentes
D. Tejidos lábiles

44. Del núcleo: características microscópicas de malignidad


A. Calcificación, lisis
B. Vacuolización, picnosis
C. Cariorrexis proteólisis
D. Mitosis atípicas

45. De las siguientes opciones cual no es una característica de la anaplasia


A. Pleomorfismo
B. Morfología nuclear anormal
C. Perdida de polaridad
D. Falta de mitosis

46. La hematopoyesis, la mayoría de los epitelios superficiales como la de los


epitelios escamosos de piel, cavidad oral, vaginal y cuello uterino son
ejemplos de:
A. Tejido del sistema nerviosos
B. Tejidos permanentes
C. Tejidos estables
D. Tejidos lábiles

47. El carcinoma ovárico, fibrosarcoma y otro sarcomas mesenquimatosos, se


han relacionado con el siguiente síndrome paraneoplasico
A. Síndrome de secreción inadecuada de hormona antidiurética
B. Hipercalcemia
C. Hipoglucemia
D. Sindrome de Cushing
48. Células responsables de la síntesis de nueva matriz extracelular
A. Linfocitos
B. Células endotelias
C. Mastocitos
D. Fibroblastos

49. El virus del Epstein-Barr está relacionado con la patogenia de


A. Linfoma de células T
B. Hepatitis C
C. Linfoma de Burkitt en su variante africana
D. Cáncer cervicouterino

50. Son trastornos adquiridos predisponentes a cáncer excepto


A. Inmunodeficiencia
B. Inflamación crónica
C. Mayor replicación celular
D. Inflamación aguda
2° parcial
1. El tiempo de tromboplastina que vía de coagulación evalúa
TTP
Vía intrínseca. (fac xii, xi, ix, viii, x, v, ii y fibrinógeno

2. Cuál es el papel de los macrófagos m2 en la reparación


Eliminación del agente causal y del te

3. Edema por obstrucción linfática.


Obstrucción localizada, postquirúrgico
excepto trombo
Neoplasia, postquimioterapia

4. Genes relacionados con neoplasia endocrina múltiple 1, tumores endocrinos


hipofisiarios, paratiroides y pancreáticos

MEN-1

5. Componente de la MEC que retiene agua y da resistencia a la comprensión


A. Hialuronico// colágeno

6. Fase del shock donde se activa mecanismos compensadores reflejos y se


mantiene la perfusión de los órganos vitales.

Fase no progresiva inicial

7. Los miofibroblastos son células


Que contribuyen a la contracción de la cicatriz

8. Gen APC inhibidor de las vías de señalización mitógena, se relaciona con


neoplasias.
Carcinomas de estómago, colon, páncreas y melanoma

9. Tras un episodio de síndrome coronario agudo (con isquemia de tejido


cardiaco) es más factible encontrar.
Infarto blanco
10. Neoplasia maligna de vasos linfáticos
Linfoma

11. Condiciones predisponentes para una trombosis vascular


Flujo sanguíneo turbulento
Lesión de barrera endotelial
Incremento de viscosidad sanguínea

12. Técnica inmunohistoquímica


Sirve para caracterizar tumores indiferenciados

13. Mutación de gen supresor de tumores más frecuente encontrado en cáncer


humano
P53

14. La formación de cantidades excesivas de tejido de granulación que sobresale


por encima del nivel de piel circundante y bloquea la reepitilización
Granulación exuberante

15. En su mayoría, los casos del shock endotóxico son causadas por
Sepsis de grampositivos, gramnegativos y hongos

16. Áreas de necrosis isquémica, causada por oclusión arterial debido a trombosis
o embolia
Infarto

17. Los infartos rojos se producen en casos de


Órganos de doble circulación

18. El líquido acumulado dentro de una cavidad corporal se denomina


Derrame// edema// hemorragia// hiperemia o congestión

19. Causas del edema de pulmón


Insuficiencia ventricular izquierdo, insuficiencia renal, sx dificultad respiratoria
aguda, proceso inflamatorio o infecciosos pulmonares

20. En que tiempo las neuronas sufren daños irreversibles ante la hipoxia
3-5min

21. Principal causa del exudado es


Reacción inflamatoria aguda

22. Causas del shock cardiogénico


Infarto, arritmias ventriculares

23. Características histológicas predominante del infarto


Necrosis coagulativa isquémica

24. Carcinógeno producido por aspergillus


Flavus aflatoxina

25. El carcinoma ovárico, el fibrosarcoma y otros sarcomas mesenquimatosos se


han relacionado con el siguiente síndrome paraneoplasico.
Hipoglucemia

26. Con el ganglio linfático centinela


El primer ganglio que recibe flujo del tumor primario
Se identidica para detectar diseminación de melanomas
Mapeo se realiza inyectando marcadores radiactivos
Colorantes azules
No se puede realizar biopsia

27. El efecto warbug consiste en la.


Neoplasia maligna la glucosis anaerobia en su metabolismo

28. Factores sistémicos que influyen en la curación de heridas


Nutrición, situación circulatoria, estado metabólico, hormonas

29. Las células madre pluripotenciales son capaces de.


Ser inducida en múltiples linajes celulares

30. Las infecciones por helicobacter pylori con que tipo de neoplasias.
Adenocarcinomas y linfomas gástricos

31. Neoplasia maligna de musculo liso.


Leiomiosarcoma

32. Estados de hipercoagubilidad se dividen


Primaria (genética) y secundaria (adquirida)

33. Marcador tumoral relacionado con carcinoma de colon pancreas pulmón .


Antígeno carcinoembrionario
34. Las hemorragias diminutas de 1 a 2 mm reciben el nombre de
Petequias

35. El proceso de regeneración se lleva a cabo por.


Cuando es promovida por factores de crecimiento y la integridad de la matriz

36. El factor de crecimiento de fibroblastos esta implicado en diferentes procesos.


Quimiotactico, mitógeno de fibroblastos
EXCEPTO mediador de la inflamación

Estimula angiogenia y síntesis de proteínas de MEC


37. Un trombo con líneas de Zahn
Se forma en la vida del paciente con sangre que fluía
Alternancia de capas de plaquetas y fibrina pálidas y las capas mas oscuro
que contienen eritrocitos

38. Son acontecimientos plaquetarios


Adhesión, migración, agregación

39. Células desempeñan un papel esencial en la reparación.


Linfocitos T

40. El estroma reactivo está compuesto por.


En algunos tipos de cáncer, las células tumorales están rodeadas de un tejido
denominado estroma, formado por células no tumorales que afectan la progresión de la
enfermedad

41. La aparición de hematomas puede deberse principalmente.

Deficiencia de factores de la coagulación

42. Características de las señales sinápticas.


Secreción de neurotramsmisores en uniones c/ especiales

43. Formas de hemorragia excepto.


Hemostasia

44. La metástasis por vía venosa afectan.


Hígado y pulmones
45. Células que se caracterizan por autorrenovación y proliferación asimétrica.
Células madres
46. Tienen mayor probabilidad de sufrir infarto rojo, excepto.
Circulación doble pulmones mas intesitno delgado.

Riñon/ inf. Blancos

47. Se media por las interaciones con el factor FvW con GpIB receptor de
superficie de las plaquetas de colágenos expuestos.
Adhesión plaquetaria

48. En la cicatrización por primera intención, cuando se comienza a observar el


tejido de granulación.
3-5 días

49. Como se clasifican los infartos.


Rojos, blancos, sépticos

50. El desarrollo de nuevos canales de sangre a través de trombos oclusivos.


Recanalización

51. Edema donde le liquido se acumula en tabiques alveolares y dificulta la


difusión de oxígeno.
Edema pulmonar

52. Causa mas frecuente de tromboembolia pulmonar.


Trombosis venosa profunda TVP

53. Resto heterotópico de células (nódulo de tejido pancreático perfectamente


desarrollado y con una organización normal en la submucosa del estómago)
Coristoma

54. Neoplasias malignas de células germinales del testículo.


Seminomas

55. Grupo de edad que tiene mayor riesgo de desarrollar cáncer.


Adultos mayores de 55 años

56. Causa de shock hipovolemico, excepto.


Anaplasia
1° parcial
1. La intoxicación por monóxido de carbono se relaciona con lo siguiente
A. Hipoxia
B. Radiación
C. Sustancias químicas
D. Reacciones inmunitarias

2. El término hiperplasia se refiere a


A. Aumento en el tamaño de las células
B. Aumento en el número de las células
C. Disminución en el número de células
D. Aumento en el tamaño y número de células

3. Cuáles son los metabolitos del ácido araquidónico (AA)


A. Eicosanoides
B. Prostaglandinas
C. Leucotrienos
D. TNF

4. Es un exceso de líquido extravascular con escaso contenido proteico


esencialmente es un ultrafiltrado del plasma sanguíneo debido a presiones
elevadas del líquido o a una reducción de las fuerzas osmóticas del plasma
A. Liquido inflamatorio crónico y agudo
B. Trasudado
C. Edema
D. Exudado

5. El músculo esquelético, responde a la inmovilización con


A. Atrofia por denervación
B. Atrofia por falta de nutrición
C. Atrofia por pérdida de estimulación endocrina
D. Atrofia por desuso

6. Reducen la inflamación inhibiendo el reclutamiento de leucocitos


A. Leucotrienos
B. Metabolitos del ácido araquidónico
C. Prostaglandinas
D. Lipoxinas
7. Se realiza colocando un arpón (guía metálica) en el sitio de la normalidad
mamográfica, después de la escisión se realiza mamografía en la biopsia y
el radiólogo confirma la presencia de la lesión
A. Biopsia excisional
B. Biopsia colposcopica
C. Biopsia aspiración
D. Biopsia trepanación

8. Las etapas secuenciales del reclutamiento de leucocitos para los sitios de


inflamación son
A. Marginación, rodamiento, adhesión al endotelio, quimiotaxis, diapédesis
B. Marginación, adhesión, rodamiento al endotelio, diapédesis y quimiotaxis
C. Marginación, rodamiento y adhesión al endotelio
D. Marginación, rodamiento, adhesión al endotelio, diapédesis y quimiotaxis

9. Cicatriz se define como


A. Estructuras de soporte tisular gravemente dañadas (lesión celular, úlcera)
B. Depósito extenso de colágeno en órganos lesionados (fibrosis)
C. Proliferación celular diversa sin interacción
D. Reparación por depósito de tejido fibroso

10. Estructura formada por un cúmulo de células epiteliodes rodeadas por un


collar de linfocitos y células plasmáticas
A. Foco necrótico
B. Granuloma
C. Zona de calcificación distrofica
D. Absceso

11. Una sobreestimulación del endometrio por estrógenos algunas veces puede
causar
A. Autofagia en el endometrio
B. Hiperplasia del endometrio
C. Hipertrofia del endometrio
D. Atrofia del endometrio

12. Tipo de necrosis en el pie diabético


A. Necrosis grasa
B. Licuefacción
C. Necrosis coagulativa
D. Necrosis caseosa
13. Es un acontecimiento necesario para iniciar el depósito de matriz
extracelular
A. Remodelación
B. Depósito de matriz extracelular
C. Contracción de la herida
D. Migración y proliferación de fibroblastos

14. Corazón anormalmente grande por obstrucción valvular mitral o aórtica


produce
A. Hiperplasia
B. Metaplasia
C. Displasia
D. Hipertrofia

15. La paciente que va a someterse a la toma de papanicolaou cervicovaginal


debe llenar el siguiente requisito
A. Ser mayor de 70 años
B. No estar menstruando
C. Ser gran multipara
D. Mujer nulípara

16. Cuál es el mecanismo de acción de la mieloperoxidasa


A. Conversión de peróxido a hipoclorito
B. Síntesis de óxido nítrico
C. Producción de peróxido de hidrógeno
D. Metabolismo de glutatión

17. A veces están presentes en la inflamación crónica y cuando se activan esta


puede ser persistente y grave aunque la función más destacada es la de
actuar como mediadores de la actividad adaptativa qué aporta defensa
contra los patógenos infecciosos
A. Eosinófilos
B. Neutrófilos
C. Linfocitos
D. Macrófagos
18. Son derivados del ácido araquidónico que participan en la patogenia del
dolor y la fiebre
A. Leucotrienos
B. Prostaglandinas
C. Citocinas
D. Quimiocinas

19. Acude a su consulta una mujer de 45 años con un bulto no palpable en la


mama izquierda, la mamografía indica un BI-RADS 4 (sospechosa de
malignidad) el ginecólogo decide operar sin hacer más pruebas y enviar
estudio transoperatorio para que el patólogo le confirme la sospecha de
carcinoma, ¿cómo tiene que enviar la muestra?
A. En fresco
B. El alcohol
C. En formol
D. En solución fisiológica

20. Se distribuyen ampliamente en los tejidos conjuntivos y participan en las


reacciones inflamatorias agudas y crónicas su acción se produce durante
las reacciones alérgicas frente alimentos, veneno de insectos o fármacos, a
veces con resultados catastróficos (ejemplo choque anafiláctico)
A. Neutrófilos
B. Macrófagos
C. Linfocitos
D. Mastocitos

21. Selectinas, integrinas y glucoproteínas tipo mucina actúan como


A. Sustancias quimiotácticas endógenas
B. Opsoninas
C. Moléculas de adhesión leucocitarias
D. Sustancias que aumentan la permeabilidad vascular

22. Para procesar la biopsia transoperatoria se utiliza la técnica de


A. Enfriamiento
B. fijación por calor
C. parafina
D. congelación
23. El útero atrofico en una mujer postmenopáusica es un ejemplo de
A. Atrofia por denervación
B. Atrofia por falta de nutrición
C. Atrofia por pérdida de estimulación endocrina
D. Atrofia por desuso

24. Qué acciones tomaría ante una mujer de 30 años de edad con factores de
riesgo para cáncer cervicouterino y que nunca ha tenido atención
ginecológica
A. realizarle una citología exfoliativa cervical inicial y dependiendo del
resultado se tomarán acciones específicas posteriores
B. Enviarla a clínica de displasias
C. Realizarle una conización
D. Nada, si no ha tenido síntomas, entonces no necesita intervención médica

25. Dentro de los ejemplos específicos de apoptosis está la inducida por lesión
del ADN celular, señale cuál es el gen involucrado en este mecanismo
A. TP53
B. RB
C. PCP
D. CED9

26. Estudio de gabinete de elección para el diagnóstico diferencial de


apendicitis aguda
A. Ultrasonido
B. Resonancia magnética
C. Tomografía, resonancia magnética
D. Rx simple de abdomen

27. Cuál es la principal modalidad de fijación tisular en caso de biopsia


A. Refrigeración a 10°
B. Formalina amortiguada al 10%
C. Alcohol
D. Gasa humedecida en solución fisiológica
28. Son mediadores químicos inhibidores de la respuesta inflamatoria
A. Leucotrienos
B. citocinas
C. lipoxinas
D. prostaglandinas

29. Se produce durante la formación de placa epifisiaria del hueso se asocia a


la esteatohepatitis, pancreatitis aguda, lesión por perfusión y enfermedades
neurodegenerativas
A. Autofagia
B. necroptosis
C. necrosis
D. apoptosis

30. Cuáles son los tipos de gránulos que contienen los neutrófilos
A. Específicos y azurofilos
B. Eosinófilos y rugosos
C. Azurofilos y eosinófilos
D. Lisos y rugosos

31. La fagocito oxidasa más óxido nítrico participa en el estallido respiratorio


formando
A. Anión superóxido
B. Peróxido de hidrógeno
C. Peroxinitrito
D. Hipoclorito

32. Es una antiproteasa que se encuentra en suero y fluidos


A. Gama 2 macroglobulina (en el libro dice alfa)
B. Transpeptidasa
C. Cininas
D. Defensinas

33. Las dos opsoninas más importantes en la inflamación son


A. Fragmento FC de la inmunoglobulina G y el factor c3b del sistema
del complemento
B. Factor C6A y factor C5A del sistema del complemento
C. Cadenas ligeras de IGM y de IGA
D. Cadenas pesadas de IGM y de IGA

34. La mayoría de los procesos inflamatorios producen leucocitosis con


elevaciones en el rango de
A. 100 a 200 mil células/ml
B. 2 a 3000 células/ml
C. 15 a 20 mil células/ml
D. 8 a 12 mil células/ml

35. Esta enfermedad se caracteriza por la alteración del complejo GpIIb-IIIa


A. Síndrome de la plaqueta gris
B. Enfermedad de Von Willebrand
C. Síndrome de Bernard-soulier
D. Trombastenia de Glanzmann

36. En donde tiene lugar principalmente la transmigración de leucocitos


A. Vénulas poscapilares
B. Arterias
C. Arteriolas
D. Arterias y arteriolas

37. Son metabolitos proinflamatorios del ácido araquidónico excepto


A. Lipoxina
B. Leucotrieno c4 d4 e4
C. Prostaglandina e2
D. Tromboxano a2

38. Mediadores relacionados con el dolor en la inflamación aguda


A. PAF
B. Prostaglandinas y bradicinina.
C. Quimiocinas y productos bacterianos
D. Leucotrienos y tromboxano a2

39. Cuál de las siguientes aseveraciones es falsa con respecto a la morfología


de la apoptosis
A. Se observa gran hinchazón celular
B. Hay constricción celular y picnosis
C. hay formación de vesículas intracitoplásmicas y formación de
cuerpos apoptóticos
D. hay fagocitosis de células y cuerpos apoptóticos
40. Es una respuesta adaptativa que existe una reducción en el tamaño y
función de las células
A. Hipertrofia
B. hiperplasia
C. atrofia
D. anaplasia

41. La adhesión firme de los leucocitos al endotelio se lleva a cabo a través de


A. Selectina e y selectina p
B. ICAM y V- CAM
C. Integrinas
D. Proteína de Lewis

42. Cuando la mitocondria pierde citocromo c desencadena el fenomeno de


A. Mitosis
B. apoptosis
C. necrosis
D. putrefacción

43. Es el resultado final de las alteraciones genéticas, bioquímicas y


estructurales de las células y tejidos

A. Cambios morfológicos
B. etiología
C. manifestaciones clínicas
D. patogenia

44. Son características de la apoptosis


A. Tamaño celular reducido, membrana plasmática intacta sin
inflamación adyacente y fragmentación celular
B. Edema, cariolisis con inflamación adyacente
C. Edema, membrana plasmática rota con digestión enzimática del
contenido celular
D. Edema, cariólisis, membrana plasmática rota con digestión
enzimática del contenido celular

45. Es una proteína que cuando se une a procaspasa 8 y es utilizada por virus
y células normales para protegerse de apoptosis por medio del receptor de
muerte fas

A. TNF
B. FLIP
C. BH3
D. FADD

46. Se refiere al reemplazo y recuperación del estado normal de algunos tejidos


A. Cicatrización
B. reparación
C. regeneración
D. Restitución

47. las neoplasias más frecuentemente encontrados en una citología exfoliativa


cervical son localizadas en
A. Útero
B. epitelio columnar
C. parametrios
D. epitelio escamoso

48. Hace referencia al aumento de tamaño de las células como consecuencia


del cual se registra un incremento del tamaño del órgano afectado
A. Metaplasia
B. Hiperplasia
C. atrofia
D. hipertrofia

49. Es exceso de líquido en los tejidos por tumefacción se refiere a

A. Linfedema
B. elefantiasis
C. anasarca
D. edema

50. Este tipo de inflamación se caracteriza por la formación de un exudado


fibrinoso
A. Inflamación fibrinosa
B. Inflamación aguda
C. Inflamación serosa
D. Inflamación crónica
2° PARCIAL FOTOS
1. Se refiere a la tendencia perniciosa de los tumores a adquirir una
conducta cada vez más agresiva.
R= Progresión tumoral

2. Son características de las neoplasias benignas.


R= Presentar cápsula, diferenciación, crecimiento relativamente lento.

3. La fibrosis es causada por.


Se mantiene la síntesis y secreción de factores de crecimiento y
citosinas fibrogénicas

4. Es la función de los macrófagos m2 en la inflamación crónica.


R=Producir y secretar sustancias activas biológicas como factores de
crecimiento, activan fibroblastos, favorecen la angiogenia

5. Es más frecuente la formación de émbolos a partir de.


R= Trombos (D)

6. Consiste en aspirar células y el líquido asociado con aguja de un


pequeño calibre.
R= Punción aspiración con aguja fina (B)

7. Cómo se clasifica el shock.


R= Cardiogénico, Hipovolemico, inflamación sistémica

8. Es considerado como el guardián del genoma, ya que es un gen


supresor de tumores.
R= P53

9. Se define como la propagación del tumor a sitios físicamente alejados


del tumor primario, y marca de un modo inequívoco, dicho tumor como
maligno.
R= Metástasis

10. El restablecimiento de la estructura tisular normal sólo se produce.


R= Cuando el tejido residual está estructuralmente intacto (resección
quirúrgica parcial).

11. Neoplasia benigna del nervio periférico.


R= Neurofibroma
12. Son características de la curación por segunda intención.
R= La reacción inflamatoria es mas intensa y mayor cantidad de tejido
de granulación.

13. Estos trombos se forman en la circulación venosa suelen contener más


eritrocitos incorporados y relativamente pocas plaquetas.
R=Trombos rojos

14. Tienen una función esencial en muchos aspectos del fenotipo maligno,
como la expresión de los genes del cáncer, el control de la
diferenciación y la autorrenovación, incluso la sensibilidad de la
resistencia a los medicamentos.

15. Mutaciones que contribuyen al fenotipo maligno se denomina.


Inestabilidad genómica

16. En su mayoría los casos de shock endotoxico son causados por.


R= Sepsis por gérmenes Gram+ - hongos

17. Proteína relacionada con el síndrome familiar del retinoblastoma


osteosarcoma.
R= RB

18. Es secundario a una insuficiencia de bomba micro cardíaca debido a las


lesiones intrínsecas del miocardio, compresión extrínseca u obstrucción
al flujo de salida.
R= Choque cardiogénico.

19. Principal causa que puede generar tromboembolia pulmonar.


R= Trombosis venosa profunda.

20. Son células responsables de la síntesis de la nueva matriz extracelular.


R= Células endoteliales

21. Son ejemplos de células estables.


R= Parenquima de órganos sólidos como hígado, riñón y páncreas

22. La trombosis venosa (fenómeno de Trousseau) se ha relacionado como


síndrome paraneoplásico para.
R= Carcinoma de páncreas, broncogeno y otros
23. El marcador tumoral CA-19-9 se relaciona principalmente con.
R= Cáncer de colon y páncreas, ovario.

24. La aparición de hematomas puede deberse principalmente.


R= Deficiencia de los factores de la coagulación.

25. Después de una lesión, un tejido permanente como el músculo cardíaco.


R= Se repara por depósito de tejido conjuntivo.
NO SE REPARA

26. El edema de origen cardíaco se caracteriza por


Localizarse en las cavidades

27. De las siguientes opciones cual no es una característica de la anaplasia.

R= Falta de mitosis.

28. Para el anclaje, la migración celular y mantener la polaridad del tejido se


requiere.
Control de la proliferación celular

29. Procesos activo en el que la dilatación arteriolar provoca aumento de


flujo sanguíneo.

R= Hiperemia.

30. La cicatriz de primera intención es dada por heridas con.

R= Bordes alineados.

31. Proceso activo en el que la dilatación arteriolar aumenta el flujo de


sangre.
R= Hiperemia

32. Neoplasia maligna de las células hematopoyéticas.


R= Leucemia.

33. Se refiere así a la pérdida progresiva de la grasa corporal y de la masa


corporal magra, acompañada de una debilidad profunda, anorexia y
anemia.
R= Caquexia cancerosa.
34. Es un protooncogen que pertenece a los inhibidores de las cinasas de
las ciclinas (INK).
R= p16

35. La tríada de Virchow incluye las siguientes características, excepto.


R= Incremento de la volemia

36. Es una causa de la reducción de la presión osmótica.


Síndrome nefrótico

37. Las células exhiben una variación en su tamaño y forma, es decir, las
células de un mismo tumor son uniformes.

R= Pleoformismo.

38. Cómo se clasifican los infartos.


R= Rojos, blancos, sépticos.

39. Corresponde el evento principal de la hemostasia secundaria.


R= Formación del coagulo de fibrina.

40. La congestión pasiva crónica del pulmón puede incluir todo lo siguiente,
excepto.
R= Induración café.

41. Defecto genético que predispone a sufrir trombosis o conocidas como


trombofilias y un 5 a 10% de las personas lo pueden presentar.
R= Deficiencia de antitrombina III, Proteina C-S y factor de Leiden

42. Acumulación de cantidades excesivas de colágeno que crece más allá


de los bordes de la herida original y no regresa.
R= Queloides.

43. Son neoplasias malignas, su origen es derivado del tejido conectivo o


conjuntivo.
R= Sarcoma.

44. Afirmación acerca de las plaquetas que es correcta.


R= Contiene granulos delta ricos en difosfato de adenosina.

45. Los tumores tienen capacidad de proliferar sin estímulos externos, en


general, como consecuencia de activación oncógena.
R= Autosuficiencia de las señales de crecimiento

46. Una vez que las células tumorales rompen la membrana basal, se habla
de.
R= Tumor infiltrante

47. Fase del shock en la que se presencia insuficiencia renal por necrosis
tubular aguda.
R= Irreversible.

48. La mayor parte de las flebotrombosis se producen en.


R= Venas superficiales y profundas de las piernas.

49. Son fases del shock hipovolémico y cardiógeno, excepto:


R= Fase de regeneración.

50. Son sustancias que inhiben la función plaquetaria.


R= Antitrombina.
INCOMPLETAS
44. Que son los granulomas
A Están compuestas por linfocitos CB
B Están compuestos por macrófagos epiteloides, linfocitos, fibroblastos
C Están compuestos por células mononucleares, células gigantes, neutrófilos
D Están compuestas por neutrófilos, células gigantes, macrófagos

46. Proonocogen que pertenece a los inhibidores de las cinasas de las ciclinas
(INK)
A. RB
B. P16
C. MYC
D. P54

50. La hemorragia, vómitos, diarrea y quemaduras pueden ocasionar


A Choque séptico
B Choque hipovolémico
C Choque cardiaco
D Choque anafiláctico

48. Patología donde las personas que sufren mutaciones en el gen TP53 y que
tienen una probabilidad 25 veces mayor de experimentar una anaplasia maligna
antes de los 50 años
A Síndrome de desgaste
B Carcinoma
C Síndrome de Liraumen
D Neoplasia endocrina múltiple
16. En una célula en reposo la proteína APC
A Induce la proliferación por medio de la betacatenina
B Inhibe la proliferación por sobreexpresión de la betacatenina
C Inhibe la proliferaicón por la degradación de la betacatenina
D Induce a la aptosis

17. Hemorragia diminuta en la piel, mucosa o serosa asociada a tromobicitopenia


A Purpura
C Equimosis
D Petequias
D Hematoma

45 El carcinoma microcitico de pulmón, carcinoma de páncreas y tumores


neutrales se han relacionado con el siguiente síndrome paraneoplasia
A Policitemia
B Hipercalcemia
C Síndrome de cushing
D Síndrome de Secreción

49. El carcinoma ovárico, el fibrosarcoma y otros sarcomas mesenquitomatoos se


han relacionado con el siguiente síndrome paraneoplásico
A Hipoglucemia
B Hipercalcemia
C Síndrome de secreción antidiurética
D Síndrome de Cushing

20 Herida
A Ocurre en forma lenta
B Bordes limpios y afrontables
C Dejan cicatrices largas y antiestéticas
D Se presentan después de inflamación crónica

. Síndrome paraneoplásico en cáncer de pulmón


A Cushing
B Policitemia
C
D

29. Contiene células o tejidos maduros e inmaduros reconocibles y pertenecientes


a mas de una de las capas germinales (a veces las tres)
Carcinoma, Lipoma, Tetaroma, Sarcoma, Carcinoma, Fibroma

Las células madre de encuentras en


NICHOS

30 En la cirrosis hepática se produce edema por la fibrosis del proceso isquémico


A Retención sodio y agua
B Espacio de la presión oncotica del plasma
C Espacio de la presión hidrostático
D Pérdida de sodio y agua

31 Proceso caracterizado por la dilatación de las arteriolas


LAS DE LAS FICHAS
51 Contiene una mayor cantidad de sangre oxigenada se caracteriza por el color
rojo brillante y dilatación de arteriolas
R= Hiperemia pasiva

52 Neoplasia maligna de epitelio placentario


R= Conocarcinoma

53 Neoplasia maligna del tejido adiposo


R=Liposarcoma

54 en cicatriz de primera intención en qué tiempola neovascularización alcanza


su máximo y el tejido de granulación llena la incisión
R=5 días

55 La bipedestación provoca edema en las partes declives del organismo.


¿cuál mecanismo explica mejor este fenómeno?
El edema subcutáneo puede ser difuso, pero, en general, se acumula
preferentemente en las regiones corporales que se encuentran más alejadas del
corazón, dado que en ellas la presión hidrostática es máxima. Por eso, el edema
suele ser más intenso en las piernas y en el sacro cuando se está en
bipedestación y en decúbito, respectivamente, relación que se denomina edema
en partes declives

56 se determina mediante la exploración quirúrgica o con técnicas de imagen y


depende del tamaño, la propagación a los ganglios linfáticos locales y regionales y
la presencia de metástasis remota
Cáncer/neoplasia

56 ejemplos de shock cardiogénico


 Infarto de miocardio
Rotura ventricular
 Arritmia
 Taponamiento cardiaco
 Embolia pulmonar

57 extravasación de líquido al intersticio, cuál es el término


R= edema

58 Describe el depósito de colágeno que se registra en pulmones hígado riñón y


otros órganos como consecuencia inflamación crónica

59 A qué deben los tumores su potencial replicativo ilimitado


R= actividad de telomerasas

60 una de las maneras en las que un tumor escapa de la respuesta inmune es


R=expresando proteínas inhibidoras de linfocitos T

61 La llamada induración parda es un ejemplo excepto


R=hemorragia de tubo digestivo

62 una de las razones por las cuales las células neoplásicas son inmortales son
R= Expresan telomerasas

63 fases del shock donde fracasan mecanismos reflejos de compensación,


disminuye la perfusión de órganos vitales
R= fase irreversible

64 característica histológica predominante de un inf es


R= Es necrosis coagulativa
65 sustancia generada por sistema de cininas cuando hay daño vascular,
promueve contracción de músculo liso, aparición de dolor y promueve
permeabilidad vascular
R=Bradicinina

66 Guardián del genoma


R= TP53

67 Carcinógenos químicos de origen natural


Aflatoxina B1
Griseofulvina
Cicasina
Safrol
Nueces de betel

68 Células responsables de síntesis de matriz extracelular

69 se refiere hacia la pérdida progresiva de masa corporal magra, acompañada de


una debilidad profunda anorexia y anemia
R= Caquexia

70 Proteína E6 y E7 del VPH de alto riesgo inhiben los genes


La E7 también inactiva las CDKI CDKN1A/p21 y CDNK1B/p27. La proteína
E6 muestra unos efectos complementarios. Se une y media en la
degradación de p53. Por analogía con E7, la proteína E6 de los tipos de
VPH de alto riesgo muestra una mayor afinidad para p53 que la proteína E6
de los tipos de VPH de bajo riesgo.

Ambas proteínas E6 y E7 se unen a dos supresores tumorales críticos, p53


y Rb, respectivamente, inactivándolos.
71 La gradación histopronóstica de una neoplasia maligna se refiere

72 marcador tumoral de tipo hormona


R= Calcitonina

73 Estados de hipercoagulabilidad se divide en


R= Geneticos y Adaptativos

74 Cáncer epidermoide de pulmón, cáncer de mama, cáncer células renales y la


leucemia/ linfoma de linfocitos T en el adulto sea relacionado con el siguiente
síndrome paraneoplásico
R= hipercalcemia

75 Características del tejido de granulación


Migración y proliferación de fibroblastos y depósito de tejido conjuntivo, que, junto
con los abundantes vasos y los leucocitos entremezclados, tienen un aspecto
granular rosado, por lo que se denominan tejido de granulación
El término tejido de granulación se debe a su aspecto macroscópico, similar al que
se encuentra por debajo de la costra de una herida cutánea. Su aspecto
histológico se caracteriza por la proliferación de fibroblastos y capilares de pared
delgada de nueva formación (angiogenia) en la MEC laxa, a menudo con células
inflamatorias entremezcladas, entre las que destacan los macrófagos

76 características de congestión pulmonar excepto


El estudio microscópico muestra que la congestión pulmonar aguda se caracteriza
por una marcada congestión por sangre en los capilares alveolares, con grados
variables de edema de los tabiques alveolares y hemorragia intraalveolar. En la
congestión pulmonar crónica, los tabiques aparecen engrosados y fibróticos, y los
espacios alveolares contienen numerosos macrófagos cargados de hemosiderina
(«células de la insuficiencia cardíaca») que se deben a la fagocitosis de los
eritrocitos.
77 Enfermedad por descompresión con aparición de burbujas de nitrógeno
R= Síndrome de descompresión

78 Estatificación de un tumor maligno se refiere a


R= tamaño de la lesión primaria y si ha realizado metástasis

79 Congestión pasiva crónica de pulmón puede incluir todo lo siguiente excepto


R= Induración café

80 El efecto Warburg consiste en


R= Neoplasia maligna fija la glucólisis anaerobia en su metabolismo

81 Inactivación o deleción de P16 se ve en familias propensas


R= Carcinoma de páncreas glioblastomas y cáncer de esófago

82 Cicatriz por primera intención se observa tejido de granulación


R= de 3 a7 días

83 Neoplasia maligna de vaso sanguíneo


R= Angiosarcoma

84 Estados de baja hipercoagulailidad excepto


R= infarto agudo

85 Definición de teratoma
R= El teratoma es un tipo especial de tumor mixto que contiene células maduras o
inmaduras o tejidos representativos derivados de más de una capa de células
germinales y, a veces, de las tres.
86 Shock cardiógenico que se produce cuando existe una pérdida de masa
muscular del ventrículo izquierdo del
R=45%

87 Acontecimientos plaquetarios excepto


R=migración
Esta unión activa una serie de acontecimientos que culminan en: 1) la adhesión de
las plaquetas; 2) la activación de las plaquetas, y 3) la agregación de las plaquetas

88 Lesión endotelial tendrá como consecuencia


R=Desarrollo de ambiente procoagulante

89 Mayor parte de flebotrombosis se produce en


R=Vena superior y superficiales y profundas de las piernas

90 Término que aplica la neoplasia epitelial benigna derivada de las glándulas


R= adenoma

91 la luz UV
R=causa carcinoma Basocelular

92 Causa edema por disminución de presión oncótica


R= desnutrición
93 proceso de evolución de trombo excepto
R= trombolisis

Fases:
Propagación
Embolización.
Disolución.
Organización y recanalización

94 Variables que alteran cicatrización


R= infección, diabetes, estadonutricional, factores mecánicos, mala perfusión, tipo,
alcance, local.

95 Se media por interacciones de VWB con la glucoproteína 1b receptor de


supresión de plaquetas y colágeno
R= Iniciación de coagulación

96 gen supresor de tumores de mama


R= BCRA1 y BCRA2

97. Estado conocido por shock séptico


R= producido por bacterias

98 virus de epstein-barr producen


R= sarcoma de Kaposi
Presencia del VEB en células de tumores sorprendentemente variados, como los
linfomas de linfocitos B en pacientes con inmunidad defectuosa de linfocitos T (p.
ej., en los infectados por el VIH), en un subgrupo de linfomas de Hodgkin, en el
carcinoma nasofaríngeo, en un subgrupo de linfomas de linfocitos T, en
carcinomas gástricos, en linfomas de linfocitos citolíticos naturales (células NK, del
inglés natural killer) e incluso, en algunos casos aislados, en sarcomas,
principalmente en pacientes inmunodeprimidos
99 edema por obstrucción linfática se da por excepto
R=post quimioterapia

100 Factor de crecimiento con efecto mitógeno sobre hepatocitos


R= Factor de crecimiento de dispersión

101 Cuál de los siguientes mediadores causan vasoconstricción


R= tromb A2

102 Infartos pálidos observan en


Los infartos secundarios a una oclusión venosa o que afectan a tejidos esponjosos
suelen ser hemorrágicos (rojos), mientras que los causados por oclusión arterial
en tejidos compactos son pálidos (blancos).

103 tumores constituidos por células inmaduras que se parecen a la que forman
tejidos rudimentario fetal
R= blastoma

104 tumores más comunes en hombres


R= próstata, pulmón, colon y recto

105 mutaciones que contribuyen al fenotipo maligno


R= Fenotipo mutador
106 ejemplo de edema localizado
R= trichuriasis

107 émbolo gaseoso


R=cambios de presión
--------------------------------------------------------------------------------------------------------------
ORDINARIO
1. Después de un infarto al miocardio cuánto tiempo se necesita para que
haya una lesión irreversible
A. 30 40 minutos
B. 15 a 20 minutos
C. 60 segundos
D. 10 a 15 minutos

2. El trasudado se caracteriza por


A. Tener polimorfonucleares
B. Tener gran cantidad de albúmina
C. Tener densidad baja
D. Tener gran cantidad de fibrina

3. Tienen un papel central en la hemostasia normal porque forman un tapón


hemostático que inicialmente se hallan los defectos vasculares y también
porque aportan una superficie sobre la cual se reclutan y concentran los
factores de la coagulación activados
A. Eosinófilos
B. Plaquetas
C. Megacariocitos
D. Células endoteliales

4. La esteatosis hepática es un claro ejemplo de


A. Acumulación exógena
B. Acumulación por metabolismo anómalo
C. Acumulación de pigmento
D. Acumulación de carbohidratos

5. Todos los siguientes son colorantes del tren de tinción excepto


A. Orange green
B. Hematoxilina ácida
C. Eosina alcalina
D. Violeta de Genciana
6. Es el resultado final de las alteraciones genéticas bioquímicas y
estructurales de las células y tejidos
A. Cambios morfológicos
B. Etiología
C. Manifestaciones clínicas
D. Patogenia

7. Biopsia de elección para lesiones cutáneas


A. Biopsia con pistola al vacío
B. Biopsia en huso
C. Biopsia en sacabocado
D. Biopsia en cuña

8. Es un AINE que inhibe de forma irreversible la ciclooxigenasa


A. Ibuprofeno
B. Paracetamol
C. Ácido acetilsalicílico
D. Naproxeno

9. Son redes fibrilares extracelulares


A. Trampas de colágeno
B. Trampas fibrilares
C. Trampas de fibrina
D. Trampas de neutrófilos

10. Son proteínas de la fase aguda excepto


A. Proteína C reactiva
B. Fibrinógeno
C. Sinaptofisina
D. Proteína del amiloide A
11. Consiste en la extirpación de un pequeño fragmento de tejido de la lesión
total
A. Biopsia incisional
B. Biopsia excisional
C. Biopsia trucut
D. Biopsia punción aspiración con aguja fina

12. La respuesta inmediata y transitoria está mediada por


A. Prostaglandinas y tromboxanos
B. Histamina bradicinina y leucotrienos
C. Proteasas plasmaticas
D. Inmunoglobulinas y complemento

13. Son aquellas capaces de inhibir la inflamación por tanto inhiben el


reclutamiento de los leucocitos y componentes celulares de la inflamación
(ANA)
A. ERO
B. LEUCOTRIENOS Y PROSTAGLANDINAS
C. Lipooxigenasas y leucotrienos
D. Lipoxinas

14. Principales quimiotácticos endógenos


A. C3 y C3BI
B. C5A y LTB4
C. Fragmento de FC de IGG y colectinas
D. E-selectina P-selectina L-selectina

15. De las adaptaciones celulares, ¿Cuál es terreno fértil sobre la cual puede
surgir un cáncer?
A. Atrofia
B. Hipertrofia patológica
C. Hiperplasia fisiológica
D. Hiperplasia patológica
16. Causas de la disminución del ATP
A. Reducción del aporte de oxígeno y nutrientes daño mitocondrial, acción de
tóxicos
B. Aumento del calcio citosólico
C. Estres oxidativo
D. Aumento de oxígeno y nutrientes

17. Es exceso de líquido en los tejidos por tumefacción se refiere a


A. Edema
B. Linfedema
C. Elefantiasis
D. Anasarca

18. En qué fases de la fagocitosis se constituye el fagosoma


A. Englobamiento ( no sé si sea lo mismo que ATRAPAMIENTO)
B. Exocitosis
C. Reconocimiento
D. Adherencia

19. Se ha demostrado que se presenta en el cáncer, trastornos


neurodegenerativos, enfermedades infecciosas y enfermedades intestinales
A. Autofagia
B. Necrosis
C. Apoptosis
D. Necroptosis

20. Biopsia indicada para valorar márgenes quirúrgicos sus resultados se


obtienen en minutos y por lo general es por congelación
A. Biopsia transoperatoria
B. Biopsia por aspiración
C. Citología exfoliativa
D. Pieza quirúrgica

21. qué acciones tomarías de una mujer de 30 años de edad con factores de
riesgo para cáncer cervicouterino y que nunca ha tenido atención
ginecológica
A. realizarle una citología exfoliativa cervical inicial y dependiendo del
resultado se tomarán acciones específicas posteriores
22. Cuáles son las fases del reclutamiento de leucocitos
A. Fijación adherencia migración
B. Reclutamiento adherencia migración
C. Reclutamiento migración adherencia marginación
D. Fijación rodamiento adherencia migración

23. es consecuencia del aumento en la permeabilidad de la membrana externa


mitocondrial y con esto la liberación de moléculas proapoptoticas en la
membrana interna de la mitocondria
A. APA 1
B. Vía extrínseca
C. Vía intrínseca
D. Blc 2

24. Todas las células tienen receptores citosolicos que reconocen una amplia
diversidad de moléculas liberadas o alteradas como consecuencia del daño
celular estos receptores activan un complejo citosólico multiproteínico
llamado

25. Es característica de la inflamación crónica


A. Infiltración de neutrófilos
B. Duración muy corta minutos horas o algunos días
C. Infiltración de células mononucleares macrófagos
D. Poca destrucción tisular
La inflamación crónica puede ser más insidiosa, dura más tiempo (días a años), y
se caracteriza por la presencia de linfocitos y macrófagos con proliferación
vascular y fibrosis (cicatriz) asociadas.

26. Es la forma de necrosis más frecuente


A. Necrosis grasa
B. necrosis caseosa
27. Biopsia qué se utiliza para vaciar cavidades
A. Curetaje
B. Cono
C. Cuña
D. Sacabocado

28. en la vía intrínseca mitocondrial las proteínas de la familia denominadas


como proteínas antiapoptóticas son
A. Bcl-2 bcl-xl mcl-1
B. Bid Bad SMAC
C. BAX
D. BAK

29. FENOTIPO DE ADAPTACIÓN COMO AUMENTO DEL NÚMERO DE


CÉLULAS
A. HIPERPLASIA
B. HIPERTROFIA
--------------------------------------------------------------------------------------------------------------
NEOPLASIAS
1. Neoplasia maligna de la glándula tiroides.
R= ADENOCARCINOMA

2. Se considera como extensión de las neoplasias a sitios anatómicamente


separados del lugar de origen.
R= METASTASIS

3. En la inmortalidad de las células malignas es considerada como factor decisivo.


R= EVASIÓN DE LA SENESCENCIA

4. Son neoplasias que tienen peor diferenciación o totalmente indiferenciadas. Se


refiere al termino de:
R= ANAPLASIA

5. Son neoplasias malignas de células germinales del testículo.


R= SEMINOMAS

6. Es una característica del Sx de Trousseau asociado a carcinoma de páncreas y


estomago se refiere a:
R= TROMBOFLEBITIS MIGRATORIA

7. Patología donde las personas que sufren mutaciones en el gen TP53 y que
tienen posibilidad 25 veces mayor de experimentar una neoplasia maligna antes
de los 50 años que la población general.
R= SÍNDROME DE LI-FRAUMENI
8. Son alteraciones diplásicas es intensa y afecta a todo el espesor del epitelio,
pero la lesión no penetra la membrana basal, se habla de:
R= CARCINOMA IN SITU
9. Dentro de las funciones de p53 se encuentran, excepto:
R= INDUCIR PROLIFERACIÓN CELULAR

10. El cáncer es una enfermedad caracterizada por:


R= ADQUISICIONE DE MUTACIONES LETALES.

11. El resultado de múltiples mutaciones que se acumulan de forma independiente


en diferentes células y contribuyen a la generación de subclones con
características diferentes a lo que se le conoce como:
R= PROGRESIÓN TUMORAL.

12. Los tipos de VPH de alto riesgo que producen proteínas oncogenas que
ocasionan lo siguiente excepto:
R= INMUNODEFICIENCIA

13. Neoplasia maligna de las células de los conductos seminales.


R= SEMINOMA

14. Marcador tumoral relacionado con el cáncer prostático.


R= PSA

15. Morfológicamente las neoplasias malignas se caracterizan por:


R= PLEOMORFISMO NUCLEAR Y CELULAR

16. El gen APC, inhibidor de las vías de señalización mitógena, se ha relacionado


con las siguientes neoplasias:
R= CARCINOMAS DE ESTOMAGO, COLÓN, PÁNCREAS Y MELANOMA.
17. El gen 9,12 se asocia con:
R= CROMOSOMA FILADELFIA
18. Marcador tumoral asociado con carcinoma de colon, páncreas y pulmón.
R= ANTÍGENO CARCINOEMBRIONANRIO

19. Neoplasia maligna de tejido adiposo


R= LIPOSARCOMA

20. Neoplasia maligna de epitelio placentario.


R= CORIOCARCINOMA

21. Neoplasia maligna de los vasos sanguíneos


R= ANGIOSARCOMA

22. La inactivación o deleción de p16, adquirida por vía somática se ve en las


familias propensas a:
R= CÁRCINOMA DE PÁNCREAS, GLIOBLASTOMA Y CÁNCERES DE
ESÓFAGO.

23. ¿Cuál es la definición de teratoma?


R= QUE SE ORIGINE DE MÁS DE 2 CAPAS GERMINALES

24. La estadificación de un tumor maligno se refiere a


R= TAMAÑO DE LA LESIÓN PRIMARIA Y SI HA REALIZADO METÁSTASIS

25. ¿Qué gen supresor de tumores intervienen en la patogenia del carcinoma de


mama?
R= BCR1 BCR2
26. El virus de Epstein Barr produce
R= SARCOMA DE KAPOSI

27. El efecto Warburg consiste en


R= LA NEOPLASIA MALIGNA FIJA LA GLUCOLISIS ANAEROBIA EN SU
METABOLISMO

28. Una de las razones por las cuales las células neoplásicas son inmortales
R= EXPRESESAN TELOMERASA

27. La luz UV
R= PUEDE CAUSAR CARCINOMA BASOCELULAR

28. Son tumores constituidos por células inmaduras que se parecen a las que
forman tejido rudimentario fetal.
R= BLASTOMA

29. Los tumores más comunes en los hombres se originan en


R= PROSTATA, PULMONES, COLON Y RECTO.

30. Es el término que se aplica a la neoplasia epitelial benigna derivadas de las


glándulas, se denomina como
R= ADENOMA

31. La falta de diferenciación, es decir, implica una inversión de la diferenciación


hacia un plano más primitivo
R= ANAPLASIA

32. Mutaciones que contribuyen al fenotipo maligno se denomina


R= FENOTIPO MUTADOR
33. La estadificación de los canceres solidos se basa en que sistema
R= SISTEMA DE TNM

33. Su crecimiento se acompaña de infiltración, invasión y destrucción progresiva


del tejido circundante, están poco delimitados y son de lenta expansión
R= NEOPLASIA MALIGNA

34. Es la vía frecuente para la diseminación inicial de los carcinomas


R= LINFÁTICA

35. Una vez que las células tumorales rompen la membrana basal, se habla de
R= TUMOR INFILTRANTE

36. Los tumores tienen capacidad para proliferar sin estímulos externos en
general, como consecuencia de la activación oncogena
R= AUTOSUFICIENCIA DE LAS SEÑALES DE CRECIMIENTO

37. Son neoplasias malignas, su origen es derivado del tejido conectivo o


conjuntivo
R= SARCOMA

38. Las células exhiben una variación en su tamaño y forma, es decir, las células
de un mismo tumor no son uniformes
R= PLEOMORFISMO

39. Es un protooncogen que pertenece a los inhibidores de las cinasas de las


ciclinas (INK) **
R= p16
40. Se refiere así, a la pérdida progresiva de la grasa corporal y de la masa
corporal magra, acompañada de una sensibilidad profunda, anorexia y anemia.}
R= CAQUEXIA CANCEROSA

41. Neoplasia maligna de células hematopoyéticas


R= LEUCEMIA

42. De las siguientes opciones, ¿Cuál no es una característica de anaplasia?


R= FALTA DE MITOSIS

43. El marcador tumoral CA 19-9 se relaciona principalmente con


R= CÁNCER DE COLON Y PÁNCREAS

44. Proteína relacionada con el síndrome familiar del retinoblastoma y


osteosarcoma
R= RB

45. Tienen una función esencial en muchos aspectos del fenotipo maligno, como
la expresión de los genes del cáncer, el control de la diferenciación y la
autorrenovación, incluso la sensibilidad de la resistencia a los medicamentos
R= CAMBIOS EPIGENÉTICOS

46. Mutaciones que contribuyen al fenotipo maligno se denomina *


R= MUTACIÓN CONDUCTORA

47. Neoplasia benigna de nervio periférico


R= NEUROFIBROMA

48. Es considerado como el guardián del genoma, ya que es un gen supresor de


tumores R= TP53
TRASTORNOS HEMODINÁMICOS.
1. Células que están presentes en el tejido conectivo contiguos a vasos
sanguíneos por debajo de superficies mucosas de vías respiratorias, la unión de
antígeno alérgeno a anticuerpo IgE, se refiere a:
R= MASTOCITOS

2. Cual enunciado acerca de las plaquetas es verdad.


R= PARTICIPAN EN LA HEMOSTASIA SECUNDARIA

3. Corresponde al evento principal de la hemostasia primaria.


R= FORMACIÓN DEL TAPÓN PLAQUETARIO

4. La deficiencia de la glucoproteína GPIB provoca la enfermedad de:


R= SÍNDROME DE BERNARD SOULIER.

5. ¿Qué tan resistentes son las células musculares cardiacas a la hipoxia?


R= BASTANTE SENSIBLES, MUEREN A LOS 20-30 MINUTOS.

6. Son patologías que producen edema por aumento de la presión hidrostática del
vaso.
R= PERICARDITIS CONTRICTIVA Y TROMBOSIS.

7. Un estado inflamatorio crónico puede causar anemia ferropénica en respuesta


al:
R= AUMENTO DE LA ALBÚMINA

8. Estos trombos se forman en la circulación venosa, suelen contener más


eritrocitos incorporados y relativamente pocas plaquetas
R= TROMBOS ROJOS
9. Se puede producir choque por:
R= TODAS LAS ANTERIORES (SEPSIS, INFARTO AL MIOCARDIO,
QUEMADURAS)

10. Son sustancias que inhiben la función plaquetaria.


R= ON (OXIDO NITRICO), PROSTACICLINA Y ADPasa.

11. El edema que ocurre en la filariasis se debe a:


R= BLOQUEO DE LA CIRCULACIÓN LINFÁTICA.

12. En la cicatrización de primera intención, en que tiempo la neovascularización


alcanza su máximo y el tejido de granulación llena la incisión.
R= 5 DÍAS

13. Contienen una mayor cantidad de sangre oxigenada se caracteriza por el color
rojo brillante y dilatación de las arteriolas.
R= HIPEREMIA PASIVA

14. Extravasación de líquido al intersticio, cual es el término que se utilizaría:


R= EDEMA

15. Son estados de hipercoagubilidad de bajo riesgo excepto:


R= INFARTO AGUDO AL MIOCARDIO

16. La cicatrización por primera intención ¿Cuándo se comienza a observar el


tejido de granulación?
R= 3 – 7 DÍAS DESPUÉS

17. Los estados de hipercoagubilidad se dividen en


R= GÉNETICOS Y ADQUIRIDOS
18. La congestión pasiva crónica del pulmón puede incluir todo lo siguiente
excepto
R= INDURACIÓN CAFÉ

19. La mayor parte de las flebotrombosis se producen en


R= VENAS SUPERCIALES Y PROFUNDAS DE LAS PIERNAS

20. El shock cardiogénico se produce cuando existe una pérdida de la masa del
ventrículo izquierdo del
R= 45%

21. Se media por las interacciones con el factor de Von Willedrand con la
glucoproteína IB, receptor de las superficies de plaquetas y el colágeno expuesto
R= FASE DE INICIACIÓN DE LA COAGULACIÓN

22. Son acontecimientos plaquetarios excepto


R= MIGRACIÓN

23. Son procesos de evolución de un trombo excepto


R= TROMBOLISIS

24. ¿Cuál de los siguientes mediadores causan vasoconstricción y


broncoconstricción?
R= TR COMBOXANO A2

25. Es una causa de edema por disminución de la presión oncotica


R= DESNUTRICIÓN

26. El estado conocido como shock séptico


R= ES PRODUCIDO POR EL PASO DE BACTERIAS, HONGOS AL TORRENTE
CIRCULATORIO

27. Los infartos pálidos se observan en


R= OCLUSIONES ARTERIALES Y TEJIDOS SÓLIDOS

28. El edema por obstrucción linfática se da por las siguientes condiciones excepto
R= POSQUIMIOTERAPIA

29. Es un ejemplo de edema localizado el que ocurre en casos de


R= TRICHIURASIS

30. Fase del shock donde fracasan mecanismos reflejos de compensación


disminuye la perfusión de los órganos vitales
R= FASE IRREVERSIBLE

31. Un embolo gaseosos puede formarse


R= POR CAMBIOS BRUSCOS DE LA PRESIÓN

32. En el choque séptico se puede producir


R= ENCEFALOPATÍA ISQUÉMICA

33. ¿Cuál de las siguientes causas es más probable que se acumule el líquido, si
existe variación?
R= PRESIÓN ONCOTICA

34. Es la fase del shock donde se activa el eje renina-angiotensina


R= FASE NO PROGRESIVA
35. Defecto genético que predispone a sufrir trombosis conocidas como
trombofilias y un 5% a 10% de las personas lo puede presentar
R= DEFICIENCIA DE ANTITROMBINA III, PROTEÍNA C-S Y FACTOR V LEIDEN

36. Corresponde al evento principal de la hemostasia secundaria


R= DEPOSICIÓN DE COAGULO DE FIBRINA

37. ¿Cómo se clasifican los infartos?


R= ROJOS, BLANCOS; SÉPTICOS

38. La triada de Virchow incluye las siguientes características excepto


R= INCREMENTO DE LA VOLEMIA

39. Es una causa de la disminución de la presión osmótica


R= SÍNDROME NEFRÓTICO

40. ¿Proceso activo en el que la dilatación arteriolar aumenta en el flujo de


sangre?
R= HIPEREMIA

41. El edema de origen cardiaco se caracteriza por


R= ORTOSTATICO/SUBCUTANEO

42. La aparición de hematomas puede deberse principalmente


R= ENFERMEDAD DE VON WILLEBRAND

43. La trombosis venosa (Fenómeno de Trousseau) se ha relacionado con


síndrome paraneoplásico
R= CARCINOMAS DE PÁNCREAS, BRONCÓGENO Y OTROS CÁNCERES
44. Principal causa que puede generar tromboembolia pulmonar
R= TROMBOSIS RPOFUNDA

45. Son células responsables de la síntesis de nueva matriz extracelular


R= CÉLULAS ENDOTELIALES

46. Es secundario a una insuficiencia de bomba miocárdica debido a las lesiones


intrínsecas del miocardio, comprensión extrínseca u obstrucción de flujo de salida
R= CHOQUE CARDIOGENICO

47. En su mayoría, los casos de shock endotoxico son causados por


R= SEPSIS POR GERMENES GRAM (-)

48. Es más frecuente la formación de émbolos a partir de


R= TROMBOS
REPARACIÓN.
1. Los principales factores de crecimiento implicados en la síntesis de tejido
conjuntivo, excepto: *
R= IL-1

2. Se les conoce así, cuando el tejido cicatrizal crece más allá de los límites de la
herida original, además no se contrae.
R= CICATRIZ QUELOIDE

3. Factores sistémicos que influyen en la curación de las heridas.


R= NUTRICIÓN, SITUACIÓN CIRCULATORIA, ESTADO METABOLICO Y
HORMONAS

4. ¿Cuáles son las fases de la reparación en las heridas?


R= ANGIOGENIA, TEJIDO DE GRANULACIÓN Y REMODELACIÓN.

5. Se observa una destrucción sustancial de tejido, cuando la lesión inflamatoria


afecta a tejidos que nos capaces de regenerarse o cuando hay abundante
exudación de fibrina en tejidos o cavidades serosas, que no es posible eliminar
adecuadamente.
R= CURACIÓN POR REPOSICIÓN DE TEJIDO CONJUNTIVO.

6. Característica de las señales sinápticas


R= SE USA PARA AMPLIFICAR UNA RESPUESTA O INHIBIRLA MEDIANTE
RETROALIMENTACIÓN

7. Todos los siguientes se pueden esperar en la congestión hepática excepto:


R= AUMENTO DE PESO
8. Una lesión endotelial tendrá como consecuencia
R= DESARROLLO DE UN AMBIENTE PROCOAGULANTE

9. Enfoque las variables que alteran la cicatrización.


R= INFECCIÓN, DIABETES, ESTADO NUTRICIONAL, FACTORES
MECÁNICOS. MALA PERFUSIÓN, TIPO, ALCANCE Y LOCALIZACIÓN DE LA
LESIÓN.

10. Es un factor de crecimiento con efectos mitógenos sobre los hepatocitos


R= FACTOR DE CRECIMIENTO DE DISPERSIÓN

11. Cuáles son las características del tejido de granulación


R= RICO EN TEJIDO CONJUNTIVO PROVISIONAL Y NUEVOS CAPILARES
FINOS

12. El colágeno que proporciona fibrillas de anclaje de las células epiteliales a la


membrana basal
R= COLAGENO TIPO IV

13. La cicatriz de primera intención es dada por heridas con


R= BORDES ALINEADOS

14. Para el anclaje, la migración de células y mantener la polaridad del tejido se


requiere
R= CONTROL DE LA PROLIFERACIÓN TISULAR

15. Después de una lesión en un tejido permanente como el músculo cardíaco


R= NO SE REPARA
16. Ejemplos de células estables
R= PARÉNQUIMA DE ORGANOS SÓLIDOS, COMO HIGADO, RIÑON Y
PÁNCREAS

16. Son características de la curación de segunda intención


R= LA REACCIÓN INFLAMATORIA ES MÁS INTENSA Y MAYOR CANTIDAD DE
TEJIDO DE GRANULACIÓN

17. El restablecimiento de la estructura tisular normal solo se produce


R= CUANDO EL TEJIDO RESIDUAL ESTÁ ESTRUCTURALEMTE INTACTO
(RESECCIÓN QUIRÚRGICA PARCIAL)

18. La fibrosis es causada por


R= SE MANTIENE LA SÍNTESIS Y SECRECIÓN DE FACTORES DE
CRECIMIENTO Y CITOCINAS FIBROGÉNICAS

19. ¿Cuál es la función de los macrófagos m2 en la inflamación crónica?


R= PRODUCIR Y SECRETAR SUSTANCIAS BIOLOGICAS COMO, FACTORES
DE CRECIMIENTO ACTIVAN FIBROBLASTOS Y FAVORECEN LA ANGIOGENIA
PREGUNTA 1
1. SUFREN UNA CONMUTACIÓN METABÓLICA HACIA LA GLUCÓLISIS AERÓBICA (EFECTO
WARBURG), QUE FACILITA LA SÍNTESIS DE MACROMOLÉCULAS Y ORGÁNULOS
REQUERIDOS PARA UN CRECIMIENTO CELULAR RÁPIDO.
A. POTENCIAL ILIMITADO DE LA REPLICACIÓN.

B. ALTERACIÓN DEL METABOLISMO CELULAR.

C. EVASIÓN DE LA APOPTOSIS.

D. INSENSIBILIDAD A LAS SEÑALES INHINITORIAS DEL CRECIMIENTO.

0.2 puntos
PREGUNTA 2
1. MORFOLÓGICAMENTE LAS NEOPLASIAS MALIGNAS SE CARACTERIZAN POR:
A. ARQUITECTURA TISULAR INTACTA

B. PLEOMORFISMO NUCLEAR Y CELULAR

C. RELACIÓN NUCLEO CITOPLASMA NORMAL

D. FIGURAS MITÓTICAS NORMALES

0.2 puntos
PREGUNTA 3
1. ES UN ESTADO DE HIPERCOAGULABILIDAD DE ALTO RIESGO
A. MIOCARDIOPATIA

B. TABAQUISMO
C. CANCER
D. DREPANOCITOSIS

0.2 puntos
PREGUNTA 4
1. HACE REFERENCIA AL EXCESO DE DEPÓSITO DE COLÁGENO Y OTROS COMPONENTES DE LA
MATRIZ EXTRACELULAR EN UN TEJIDO.
A. REGENERACIÓN.

B. GRANULOMA.

C. CICATRIZACIÓN Y FIBROSIS.

D. TEJIDO DE GRANULACIÓN.

0.2 puntos
PREGUNTA 5
1. SON NEOPLASIAS MALIGNAS DE CELULAS GERMINALES DEL TESTICULO:A
A. CARCINOMAS

B. TERATOMAS
C. SEMINOMAS
D. CISTOADENOMAS

0.2 puntos
PREGUNTA 6
1. EN LA EMBOLIA PULMONAR, LOS TROMBOS PROCEDEN PRINCIPALMENTE DE:
A. VALVULAS CARDIACAS

B. RANDES VASOS ARTERIALES

C. CAVIDADES CARDIACAS

D. VENAS PROFUNDAS DE LA PIERNA

0.2 puntos
PREGUNTA 7
1. DE LAS SIGUIENTES AFIRMACIONES SOBRE LA ISQUEMIA MESENTÉRICA AGUDA, SEÑALE
LA INCORRECTA
A. LA OCLUSION DEL TRONCO CELIACO TIENE EL MEJOR PRONÓSTICO

B. LA TROMBOSIS SUELE SER DISTAL AL ORIGEN DE LA ARTERIA MESENTÉRICA


SUPERIOR
C. LA CAUSA MÁS FRECUENTE ES ATEROESCLEROSIS

D. LA TROMBOSIS DE LA ARTERIA MESENTERICA SUPERIOR ES DE DIFÍCIL


DIAGNÓSTICO PRECOZ

0.2 puntos
PREGUNTA 8
1. ¿QUÉ TEJIDOS SE CONSIDERAN COMO PERMANENTES?
A. MÚSCULO LISO, HIGADO.

B. PIEL, CAVIDAD ORAL, CUELLO UTERINO.

C. CORAZÓN, ENCEFALO.

D. VEJIGA, GLÁNDULAS SALIVALES

0.2 puntos
PREGUNTA 9
1. ES EJEMPLO DE TEJIDOS QUIESCENTES:
A. EPITELIO
B. SANGRE

C. MUSCULO ESTRIADO

D. HÍGADO

0.2 puntos
PREGUNTA 10
1. SE LE DENOMINA ASÍ CUANDO LAS CÉLULAS PARENQUIMATOSAS ESTIMULAN LA SÍNTESIS
DE UN ESTROMA CON COLÁGENO ABUNDANTE:
A. PÉTREO

B. ESTROMA REACTIVO

C. DESMOPLASIA

D. PARÉNQUIMA

0.2 puntos
PREGUNTA 11
1. NEOPLASIA MALIGNA DE LAS CELULAS PLASMATICAS
A. PLASMOCITOMA

B. LEUCEMIA

C. LINFOMA

D. MIELOMA MULTIPLE

0.2 puntos
PREGUNTA 12
1. EL ORIGEN MÁS FRECUENTE DE LOS TROMBOS EN EMBOLIAS PULMONARES ES:
A. VENAS PERIPROSTÁTICAS

B. TROMBOS IN-SITU DEL PULMÓN

C. VENAS PROFUNDAS DE LAS PIERNAS

D. VENAS PÉLVICAS

0.2 puntos
PREGUNTA 13
1. NEOPLASIA MALIGNA DEL HUESO
A. ADENOMA PLEOMORFO

B. OSTEOSARCOMA
C. LEIOMIOMA
D. CONDROMA

0.2 puntos
PREGUNTA 14
1. <HEMORRAGIA DIMINUTA EN LA PIEL, MUCOSA O SEROSA ASOCIADA A
TROMBOCITOPENIA:
A. PURPURA

B. EQUINOSIS

C. HEMATOMA

D. PETEQUIAS

0.2 puntos
PREGUNTA 15
1. SON UN ELEMENTO IMPORTANTE EN LA CONTRACCION DE LAS HERIDAS:
FIBROBLASTOS

MIOCITOS

MIOFIBROBLASTOS

MACROFAGOS

0.2 puntos
PREGUNTA 16
1. SON CELULAS MADRE QUE DERIVAN DE CELULAS ADULTAS REPROGRAMADAS
A. CELULAS MADRE EMBRIONARIAS

B. CELULAS MADRE TISULARES


C. CELULAS MADRE INDUCIDAS
D. CELULAS MADRE MESENQUIMATOSAS

0.2 puntos
PREGUNTA 17
1. UN ESTADO INFLAMATORIO CRÓNICO PUEDE CAUSAR ANEMIA FERROPÉNICA EN
RESPUESTA AL:
A. AUMENTO DE LA FIBRINA

B. AUMENTO DE LA ALBÚMINA

C. AUMENTO DE LA HEPCIDINA

D. AUMENTO DEL FIBRINÓGENO

0.2 puntos
PREGUNTA 18
1. EL GEN RB, INHIBIDOR DE LA PROGRESIÓN CELULAR, SE HA RELACIONADO CON LA
SIGUIENTES NEOPLASIAS:
A. CARCINOMA DE CÉLULAS RENALES Y PARAGANGLIOMA.

B. CARCINOMA BASOCELULAR.

C. CARCINOMA BASOCELULAR Y MEDULOBLASTOMA.

D. RETINOBLASTOMA, OSTEOSARCOMA, CARCINOMAS DE MAMA, COLON Y PULMÓN.

0.2 puntos
PREGUNTA 19
1. PARA EL ANCLAJE, LA MIGRACION DE CELULAS Y MANTENER LA POLARIDAD DEL TEJIDO
SE REQUIERE:
A. EL MICROAMBIENTE TISULAR

B. CITOCINAS PROINFLAMARTORIAS

C. CONTROL DE LA PROLIFERACION TISULAR

D. SOPORTE MECANICO Y ANDAMIAJE TISULAR

0.2 puntos
PREGUNTA 20
1. LA FIBROSIS ES CAUSADA POR:
A. CARENCIA DE VITAMINA C

B. DESNUTRICIÓN Y DISMINUCIÓN DE LEUCOCITOS EN SANGRE

C. INHIBICIÓN DEL PROCESO DE REMODELACIÓN Y UNA ANGIOGÉNESIS AUMENTADA.

D. SE MANTIENE LA SÍNTESIS Y SECRECIÓN DE FACTORES DE CRECIMIENTO Y


CITOCINAS FIBROGÉNICAS

0.2 puntos
PREGUNTA 21
1. DE LAS SIGUIENTES OPCIONES, CUÁL NO ES UNA CARACTERÍSTICA DE LA ANAPLASIA?
A. PLEOMORFISMO

B. MORFOLOGÍA NUCLEAR ANORMAL

C. PÉRDIDA DE POLARIDAD

D. FALTA DE MITOSIS

0.2 puntos
PREGUNTA 22
1. AL PROCESO INFLAMATORIO QUE NO SE RESUELVE POR SI SOLO, SE PUEDE CONSIDERAR
COMO:
A. INFLAMACION AGUDA

B. INFLAMACION CRONICA

C. INFLAMACION SEROSA
D. INFLAMACION FIBRINOSA

0.2 puntos
PREGUNTA 23
1. TRAS UNA HEPATECTOMÍA PARCIAL:
A. SE PRODUCE FIBROSIS

B. EL HÍGADO SE REGENERA Y LOS HEPATOCITOS PROLIFERAN

C. SE PRODUCE CIRROSIS

D. SE PRODUCE UNA CICATRIZ

0.2 puntos
PREGUNTA 24
1. EN ESTE TIPO DE EDEMA, EL LÍQUIDO SE ACUMULA EN LOS TABIQUES ALVEOLARES Y
DIFICULTA LA DIFUSIÓN DE OXÍGENO:
A. EDEMA CEREBRAL

B. EDEMA SUBCUTÁNEO

C. ASCITIS

D. EDEMA PULMONAR

0.2 puntos
PREGUNTA 25
1. SON ÁREAS DE NECROSIS ISQUÉMICA CAUSADAS CON MÁS FRECUENCIA POR OCLUSIÓN
ARTERIAL DEBIDA TÍPICAMENTE A TROMBOSIS O EMBOLIA:
TROMBOSIS.

INFARTO.

ISQUEMIA.

COAGULOPATÍA POR COMSUMO.

0.2 puntos
PREGUNTA 26
1. EL DAÑO GENÉTICO NO LETAL, ESENCIAL EN LA CARCINOGENIA PUEDE DEBERSE A:
A. LAS ACCIONES HOMEOSTÁTICAS DE LOS GENES SUPRESORES.

B. EL BLOQUEO DE LAS EXPOSICIONES AMBIENTALES.

C. MUTACIONES.

D. EXPOSICIONES AMBIENTALES, HEREDARSE EN LA LÍNEA GERMINAL Y RESULTAR


ESPONTÁNEO Y ALEATORIO.

0.2 puntos
PREGUNTA 27
1. CAUSAN ALTERACIONES DEL FLUJO SANGUÍNEO:
A. MUTACIÓN DEL FACTOR V Y DÉFICIT DE PROTEÍNAS S Y C.

B. CÁNCER, OBESIDAD Y SÍNDROME DE ANTICUERPOS ANTIFOSFOLÍPIDOS.

C. DÉFICIT DE ANTITROMBINA II Y HOMOCISITINURIA.

D. ANEURISMAS, ESTENOSIS DE LA VÁLVULA MITRAL, FIBRILACIÓN AURICULAR,


HIPERVISCOSIS Y DREPANOCITOSIS.

0.2 puntos
PREGUNTA 28
1. SON EJEMPLOS DE SHOCK CARDIOGENICO EXCEPTO:
A. ARRITMIAS

B. TAPONAMIENTPO CARDIACO
C. TROMBOEMBOLIA PULMONAR
D. QUEMADURAS

0.2 puntos
PREGUNTA 29
1. ¿QUÉ MOLÉCULA ES EL FACTOR MÁS IMPORTANTE PARA LA SÍNTESIS DE TEJIDO
CONJUNTIVO?
A. HGF

B. TNF

C. TGF-B

D. VEGF

0.2 puntos
PREGUNTA 30
1. SE PRODUCEN EN CASO DE EMBOLIA DE VEGETACIONES INFECTADAS DE VÁLVULAS
CARDIACAS:
A. CHOQUE CARDIOGÉNICO.

B. INFARTOS SÉPTICOS.
C. NECROSIS COAGULATIVA.

D. NECROSIS ISQUÉMICA.

0.2 puntos
PREGUNTA 31
1. SON ACONTECIMIENTOS PLAQUETARIOS EXCEPTO:
A. ADHESION

B. MIGRACION
C. CAMBIO DE FORMA Y SECRECION
D. AGREGACION

0.2 puntos
PREGUNTA 32
1. FASE DEL SHOCK EN QUE SE PRESENTA INSUFICIENCIA RENAL POR NECROSIS TUBULAR
AGUDA:
A. FASE IRREVERSIBLE

B. FASE PROGRESIVA

C. FASE DE HIPOPERFUSIÓN TISULAR

D. FASE NO PROGRESIVA

0.2 puntos
PREGUNTA 33
1. SE REFIERE A LA RESPUESTA DEL CUERPO QUE REESTABLECERA EL ESTADO NORMAL DEL
TEJIDO EN LA LOCALIZACIÓN DE LA INFLAMACIÓN AGUDA.
A. RESOLUCIÓN O REMISIÓN

B. CICATRIZACIÓN

C. FIBROSIS

D. CURACIÓN POR REPOSICIÓN DE TEJIDO CONJUNTIVO

0.2 puntos
PREGUNTA 34
1. ¿QUÉ AFIRMACIÓN ACERCA DE LAS PLAQUETAS ES CORRECTA?
A. LOS PRECURSORES SON MONOCITOS

B. CONTIENE CUERPOS DENSOS RICOS EN FIBRINÓGENO

C. PARTICIPAN EN LA HEMOSTASIA SECUNDARIA

D. CONTIENE GRÁNULOS DELTA RICOS EN DIFOSFATO DE ADENOSINA

0.2 puntos
PREGUNTA 35
1. LA TRASLOCACIÓN CROMOSÓMICA EN LA LEUCEMIA MIELOIDE CRÓNICA ES:
A. CROMOSOMA 14 Y 8

B. DEL CROMOSOMA 8 AL 14
C. EN EL CROMOSOMA X

D. EN EL CROMOSOMA 9 Y 22

0.2 puntos
PREGUNTA 36
1. CUÁLES SON LOS COMPONENTES DE LA PLACA ATEROSCLERÓTICA?
A. CÉLULAS DE MUSCULO LISO, MACRÓFAGOS, MEC Y LÍPIDOS

B. FIBROSIS, LÍPIDOS, PMN

C. MEC, ATEROMA, CÉLULAS INFLAMATORIAS

D. MACRÓFAGOS Y LÍPIDOS

0.2 puntos
PREGUNTA 37
1. LOS TUMORES TIENEN CAPACIDAD DE PROLIFERAR SIN ESTÍMULOS EXTERNOS, EN
GENERAL, COMO CONSECUENCIA DE LA ACTIVACIÓN ONCÓGENA.
A. EVASIÓN DE LA APOPTOSIS.

B. ALTERACIÓN DEL METABOLISMO CELULAR.

C. INSENSIBILIDAD DE LAS SEÑALES INHIBIDORAS DEL CRECIMIENTO.

D. AUTOSUFICIENCIA DE LAS SEÑALES DE CRECIMIENTO,

0.2 puntos
PREGUNTA 38
1. LA CICATRIZACION DE SEGUNDA INTENCION:
A. OCURRE EN FORMA RAPIDA, SIN DEJAR CICATRIZ

B. SE OBSERVA EN HERIDAS DE OPERACIONES QUIRURGICAS

C. NO REQUIERE TRATAMIENTO

D. SE PRESENTA EN HERIDAS ANFRACTUOSAS CON GRAN PERDIDA DE TEJIDO

0.2 puntos
PREGUNTA 39
1. LA ENFERMEDAD POR DESCOMPRESION CON APARICION DE BURBUJAS DE NITROGENO...
A. ES LA ENFERMEDAD POR DESCOMPRESION CON APARICION DE BURBUJAS DE
NITROGENO:
B. ES TÍPICA DE LA SEPSIS.
C. ES TÍPICA DE BUCEADORES.

D. SE LE DENOMINA TROMBOEMBOLIA SÉPTICA.

0.2 puntos
PREGUNTA 40
1. AL LÍQUIDO ACUMULADO DENTRO DE UNA CAVIDAD CORPORAL SE LE DENOMINA:
A. DERRAME

B. EDEMA

C. PLASMA

D. EXUDADO

0.2 puntos
PREGUNTA 41
1. NEOPLASIA BENIGNA DEL NERVIO PERIFERICO:
A. PARAGANGLIOMA
B. NEUROFIBROMA
C. LINFOMA
D. NEUROMA

0.2 puntos
PREGUNTA 42
1. ¿LOS INFARTOS BLANCOS, SE PRESENTAN?
A. CUANDO UN TEJIDO SOLIDO CON CIRCULACION TERMINAL SE QUEDA SIN RIEGO
ARTERIAL
B. EN TEJIDOS LAXOS

C. EN OBSTRUCCION VENOSA

D. TEJIDOS CON DOBLE CIRCULACION

0.2 puntos
PREGUNTA 43
1. SE LE LLAMA ASÍ, A LOS TUMORES MALIGNOS QUE DERIVAN DE LOS VASOS SANGUÍNEOS.
A. LINFANGIOMA.

B. LINFANGIOSARCOMA.

C. ANGIOSARCOMA.

D. HEMANGIOMA.
0.2 puntos
PREGUNTA 44
1. SE PRODUCE CUANDO LA HERIDA SE ABRE PARCIALMENTE CICATRIZADA, EL TERMINO
CORRECTO ES:
A. ORGANIZACIÓN.

B. RESOLUCION.

C. DEHISCENCIA.

D. CICATRIZ QUELOIDE.

0.2 puntos
PREGUNTA 45
1. EL HÍGADO EN NUEZ MOSCADA ES UN PATRÓN MORFOLÓGICO CARÁCTERÍSTICO DE:
A. NECROSIS LICUEFACTIVA

B. CONGESTION AGUDA

C. CONGESTIÓN PASIVA CRÓNICA

D. EDEMA CELULAR

0.2 puntos
PREGUNTA 46
1. ¿CUÁL DE LAS SIGUIENTES SUSTANCIAS INHIBEN LA AGREGACION PLAQUETARIA?
A. ATPASA.

B. FACTOR TISULAR
C. OXIDO NITROSO
D. PROSTACICLINA

0.2 puntos
PREGUNTA 47
1. ES UNA CAUSA DE EDEMA POR DISMINUCION DE LA PRESION ONCOTICA
DIABETES MELLITUS

DESNUTRICIÓN
INSUFICIENCIA CARDIACA
CANCER

0.2 puntos
PREGUNTA 48
1. LAS CELULAS DE ESTOS TEJIDOS ESTAN DIFERENCIADAS TERMINALMENTE Y
NO SON PROLIFERATIVAS EN LA VIDA POSNATAL:
A. PERMANENTES
B. ESTABLES

C. SENESCENTES

D. LABILES

0.2 puntos
PREGUNTA 49
1. LOS GRANULOS DENSOS DE LAS PLAQUETAS CONTIENEN EXCEPTO
A. ADP

B. CALCIO
C. FACTOR DE VON WILLEBRAND
D. ADRENALINA Y SEROTONINA

0.2 puntos
PREGUNTA 50
1. EL TIEMPO DE TROMBOPLASTINA PARCIAL:
A. EVALÚA EL NÚMERO DE CÉLULAS SANGUÍNEAS

B. EVALÚA LA FIBRINÓLISIS

C. EVALÚA LOS FACTORES INTRÍNSECOS

D. EVALÚA LOS FACTORES EXTRÍNSECOS

0.2 puntos

Haga clic en Guardar y enviar para guardar y enviar. Haga clic en Guardar todas las respuestas
para guardar todas las respuestas.
PREGUNTA 1
1. ¿EN DÓNDE SE PRODUCEN LAS ESPECIES REACTIVAS DE OXÍGENO EN LA
FAGOCITOSIS?
A. NUCLEO
B. LISOSOMA Y FAGOLISOSOMA
C. REL Y MITOCONDRIA
D. RER Y LISOSOMA

0.2 puntos
PREGUNTA 2
1. LA LIPOFUSCINA SE CARACTERIZA POR:
A. SER UN PIGMENTO EXÓGENO

B. SER UNA ACUMULACIÓN DE TRIGLICÉRIDOS

C. SER UN PIGMENTO DE DESGASTE

D. SER UN PIGMENTO FISIOLÓGICO

0.2 puntos
PREGUNTA 3
1. LA ACUMULACIÓN INTRACELULAR DE POLVO DE CARBÓN RECIBE EL NOMBRE DE:
A. GLUCOGENOSIS

B. CUERPOS DE RUSSELL

C. CUERPOS DE COUNCILMAN

D. ANTRACOSIS

0.2 puntos
PREGUNTA 4
1. PEROXIDACIÓN DE LOS LÍPIDOS DE LA MEMBRANA, MODIFICACIÓN OXIDATIVA DE LAS
PROTEÍNAS Y LESIONES DEL DNA SON EFECTOS RELEVANTES PARA LA LESIÓN CELULAR
CAUSADAS POR:
A. HIPOXIA

B. ISQUEMIA

C. ACIDOSIS LÁCTICA

D. RADICALES LIBRES DERIVADOS DEL OXÍGENO

0.2 puntos
PREGUNTA 5
1. EL FRAGMENTO DE FC DE IgG, C3B Y LAS COLECTINAS FUNCIONAN COMO:
A. QUIMIOTÁCTICOS
B. OPSONINAS
C. VASODILATADORES
D. MOLÉCULAS DE ADHESIÓN

0.2 puntos
PREGUNTA 6
1. LA CELULA CARACTERÍSTICA DE LA INFLAMACION GRANULOMATOSA:
A. CELULAS DE KUPFFER

B. HISTIOCITOS SINUSALES

C. MACROFAGOS ALVEOLARES

D. CELULAS EPITELIOIDES

0.2 puntos
PREGUNTA 7
1. ¿CUAL ES EL EFECTO DE LA HISTAMINA SOBRE LOS VASOS SANGUÍNEOS?
A. VASODILATACIÓN

B. VASOCONSTRICCIÓN

C. QUIMIOTAXIS

D. LESIÓN ENDOTELIAL

0.2 puntos
PREGUNTA 8
1. LA BIOPSIA DE ELECCIÓN PARA SINUSITIS CRONICA ES:
A. LEGRADO.

B. EN CUÑA.

C. EXCISIONAL EN HUSO.

D. EN SACABOCADOS.

0.2 puntos
PREGUNTA 9
1. LOS TIPOS DE VIRUS DEL PAPILOMA HUMANO QUE PRODUCEN EN FORMA MÁS FRECUENTE
CA EPIDERMOIDE SON:
A. TIPOS 2, 4, 7

B. TIPOS 33, 35, 51

C. TIPOS 11, 42, 44


D. TIPOS 16, 18

0.2 puntos
PREGUNTA 10
1. TIPO DE CALCIFICACION QUE APARECE EN ZONAS DE NECROSIS COAGULATIVA CASEOSA O
LICUEFACTIVA
A. DISTROFICA

B. METASTASICA

C. CAMBIO HIALINO

D. METAPLASIA

0.2 puntos
PREGUNTA 11
1. PROTEINA DE LA FASE AGUDA QUE EN INFLAMACION CRONICA PUEDE CAUSAR
AMILOIDOSIS SECUNDARIA
A. PROTEINA C RECATIVA

B. PROTEINA AMILOIDE SERICA

C. FIBRINOGENO

D. PROTEINA S

0.2 puntos
PREGUNTA 12
1. EN LA MUERTE CELULAR DE TIPO NECROPTOSIS, LA CARACTERÍSTICA PRINCIPAL ES:
A. NO ACTIVA A LAS CASPASAS

B. ES IDÉNTICA A LA NECROSIS

C. ES IDÉNTICA A LA APOPTOSIS

D. ACTIVA LAS CASPASAS 6 Y 3

0.2 puntos
PREGUNTA 13
1. EN UN INFARTO CEREBRAL ES COMÚN ENCONTRAR EL PATRÓN DE NECROSIS:
A. CASEOSA

B. GRASA

C. LICUEFACTIVA

D. COAGULATIVA

0.2 puntos
PREGUNTA 14
1. TIPO DE NECROSIS EN EL PIE DIABÉTICO
A. NECROSIS CASEOSA

B. NECROSIS GRASA

C. LICUEFACCIÓN

D. NECROSIS COAGULATIVA

0.2 puntos
PREGUNTA 15
1. EN LOS PACIENTES CON HIPERTENSIÓN ARTERIAL EL MIOCARDIO SUFRE:
A. HIPERPLASIA
B. HIPERTROFIA
C. ATROFIA
D. METAPLASIA

0.2 puntos
PREGUNTA 16
1. ES UN CAMBIO MORFOLÓGICO CaRACTERIZADO POR CONDENSACIÓN Y
FRAGMENTACIÓN DE LA CROMATINA QUE SUCEDE EN:
A. HIPERTROFIA
B. NECROSIS
C. APOPTOSIS
D. ATROFIA

0.2 puntos
PREGUNTA 17
1. LA REGENERACIÓN HEPÁTICA QUE SE PRODUCE TRAS LA
HEPATECTOMÍA PARCIAL ES UN EJEMPLO DE:
A. METAPLASIA
B. ATROFIA
C. HIPERPLASIA
D. HIPERTROFIA

0.2 puntos
PREGUNTA 18
1. ES UNA BIOPSIA CUYO OBJETIVO ES UNICAMENTE DIAGNOSTICO:
A. BIOPSIA INCISIONAL
B. BIOPSIA EXCISIONAL
C. ESPÉCIMEN QUIRÚRGICO
D. BIOPSIA POR CURETAJE

0.2 puntos
PREGUNTA 19
1. ACUMULACIÓN INTRACELULAR PROPIA DEL ENVEJECIMIENTO:
A. LIPOFUSCINA
B. HEMOSIDERINA
C. FOSFOLÍPIDOS
D. LÍPIDOS

0.2 puntos
PREGUNTA 20
1. SON CAUSAS PATOLÓGICAS DE APOPTOSIS:
A. NUMERO CONSTANTE DE CÉLULAS
B. DURANTE LA EMBRIOGENESIS
C. PROTEÍNAS MAL PLEGADAS
D. CAMBIOS HORMONALES

0.2 puntos
PREGUNTA 21
1. LA MIGRACIÓN DE LOS LEUCOCITOS A TRAVÉS DEL ENDOTELIO SE LLAMA:
ADHESIÓN.

QUIMIOTAXIS.

DIAPÉDESIS.

RODAMIENTO.

0.2 puntos
PREGUNTA 22
1. EN PATOLOGÍAS COMO ENFERMEDAD DE ALZHEIMER Y COREA DE HUNTINGTON, LA
APOPTOSIS ES INDUCIDA POR:
A. DAÑO AL DNA

B. LIPOPEROXIDACIÓN

C. CARENCIA DE FACTORES DE CRECIMIENTO

D. MAL PLEGAMIENTO DE PROTEÍNAS

0.2 puntos
PREGUNTA 23
1. SON LLAMADAS TAMBIÉN HIPERSENSIBILIDAD. PUEDEN DIRIGIRSE CONTRA
AUTOANTÍGENOS O SER SUSTANCIAS ANÓMALAS CONTRA SUSTANCIAS AMBIENTALES O
CONTRA MICROBIOS.
INFECCIONES.

CUERPOS EXTRAÑOS.

REACCIONES INMUNITARIAS.
NECROSIS TISULAR.

0.2 puntos
PREGUNTA 24
1. EL MECANISMO DE LA ATROFIA COMIENZA CON:
A. REDUCCIÓN DE NECESIDADES METABÓLICAS

B. AUTOFAGIA

C. APOPTOSIS

D. AUMENTO DE SÍNTESIS DE PROTEÍNAS

0.2 puntos
PREGUNTA 25
1. CUÁLES SON LAS FORMAS DE EVOLUCIÓN DE LA INFLAMACIÓN AGUDA
A. REMISIÓN COMPLETA, CICATRIZACIÓN, FIBROSIS O PROGRESIÓN A
RESPUESTA CRÓNICA
B. CICATRIZACIÓN, PÉRDIDA DE LA FUNCIÓN Y PROGRESIÓN A
RESPUESTA CRÓNICA
C. RESOLUCIÓN COMPLETA, FORMACIÓN DE PUS Y PROGRESIÓN A
RESPUESTA CRÓNICA
D. RESOLUCIÓN COMPLETA, CURACIÓN POR REPOSICIÓN DE TEJIDO
CONJUNTIVO Y PROGRESIÓN A RESPUESTA CRÓNICA
0.2 puntos
PREGUNTA 26
1. LA INTOXICACIÓN POR MONÓXIDO DE CARBONO SE RELACIONA CON LO SIGUIENTE:
SUSTANCIAS QUÍMICAS

REACCIONES INMUNITARIAS

HIPOXIA

RADIACIÓN

0.2 puntos
PREGUNTA 27
1. ES UN FENOMENO PATOLOGICO QUE SUCEDE DURANTE LA FAGOCITOSIS
A. ENGLOBAMIENTO

B. UNION
C. REGURGITACION
D. ATRAPAMIENTO

0.2 puntos
PREGUNTA 28
1. TODAS LAS CÉLULAS TIENEN RECEPTORES CITOSÓLICOS QUE RECONOCEN UNA AMPLIA
DIVERSIDAD DE MOLÉCULAS LIBERADAS O ALTERADAS COMO CONSECUENCIA DEL DAÑO
CELULAR. ESTOS RECEPTORES ACTIVAN UN COMPLEJO CITOSÓLICO MULTIPROTEÍNICO
LLAMADO:
REACCIÓN ANTIGENO-ANTICUERPO.

INFLAMASOMA

LECTINA DE UNIÓN A MANOSA

SISTEMA DEL COMPLEMENTO

0.2 puntos
PREGUNTA 29
1. PROCEDIMIENTO DONDE, SE EXTIRPA PARTE DE LA LESIÓN, EXCLUSIVAMENTE CON UN
PROPÓSITO DIAGNÓSTICO.
A. BIOPSIA TREPANACIÓN

B. BIOPSIA ASPIRACIÓN

C. BIOPSIA INCISIONAL

D. BIOPSIA EXCISIONAL

0.2 puntos
PREGUNTA 30
1. SON MOLÉCULAS SENSORAS SOLO BH3 DE LA APOPTOSIS :
A. BCLX
B. BAD, BIM, BID

C. BAX BAK

D. BCL2

0.2 puntos
PREGUNTA 31
1. ¿CUAL ES LA PROTEINA QUE TIENE FUNCION ANTIAPOPTOSICO?:
AAPAF-1
.
B P53
.

CBLC2
.
DCASPASA 3
.
0.2 puntos
PREGUNTA 32
1. SE OBSERVA TRAS UNA DESTRUCCIÓN SUSTANCIAL DE TEJIDO, CUANDO LA LESIÓN
INFLAMATORIA AFECTA A TEJIDOS QUE NO SON CAPACES DE REGENERARSE O CUANDO
HAY ABUNDANTE EXUDACIÓN DE FIBRINA EN TEJIDOS O CAVIDADES SEROSAS, QUE NO ES
POSIBLE ELIMINAR ADECUADAMENTE.
REMISIÓN.

INFLAMACIÓN CRÓNICA.

RESOLUCIÓN COMPLETA.

FIBROSIS

0.2 puntos
PREGUNTA 33
1. CUÁLES SON LAS PRINCIPALES CELULAS
QUE GENERAN METABOLITOS DEL ÁCIDO ARAQUIDÓNICO EN LA
INFLAMACIÓN?
A. LEUCOCITOS, MASTOCITOS,CÉLULAS ENDOTELIALES Y PLAQUETAS
B. MASTOCITOS, NEUTRÓFILOS, EOSINÓFILOS Y BASÓFILOS
C. LINFOCITOS, LEUCOCITOS, NEUTRÓFILOS Y PLAQUETAS
D. PLAQUETAS, EOSINOFILOS, BASOFILOS

0.2 puntos
PREGUNTA 34
1. SON CAUSADOS POR CUERPOS RELATIVAMENTE INERTES, EN AUSENCIA DE REACCIONES
INMUNITARIAS MEDIADAS POR LINFOCITOS T. SUELEN FORMARSE EN TORNO A
MATERIALES COMO TALCO, SUTURAS O UTRAS FIBRAS.
GRANULOMAS DE CUERPO EXTRAÑO

HIPERSENSIBILIDAD

FAGOCITOSIS.

GRANULOMAS INMUNITARIOS.

0.2 puntos
PREGUNTA 35
1. ES UNA ALTERACIÓN EN EL DESARROLLO, LA MORFOLOGÍA Y MADURACIÓN DE LA
CÉLULA
A. DISPLASIA

B. HIPERPLASIA

C. NEOPLASIA

D. METAPLASIA

0.2 puntos
PREGUNTA 36
1. SON PROTEÍNAS DE FASE AGUDA QUE AUMENTAN EN RESPUESTA ESTÍMULOS
INFLAMATORIOS.
PROTEÍNA C REACTIVA Y FIBRINA

PROTEÍNA C REACTIVA, FIBRINÓGENO Y HEPCIDINA.

ALBUMINA Y TRANSFERRINA

AUMENTO DE VSG (VELOCIDAD DE SEDIMENTACIÓN GLOBULAR).

0.2 puntos
PREGUNTA 37
1. ES LA DEFICIENCIA DE OXÍGENO, INTERFIERE EN LA RESPIRACIÓN
OXIDATIVA AEROBIA Y ES UNA CAUSA IMPORTANTE Y COMÚN DE LESIÓN
Y MUERTE CELULARES
A. ISQUEMIA
B. APOPTOSIS
C. NECROSIS
D. HIPOXIA

0.2 puntos
PREGUNTA 38
1. LA METAPLASIA ES UNA ADAPTACIÓN CELULAR CORRESPONDIENTE A:
A. CAMBIO DE GENOTIPO

B. CAMBIO DE UNA CÉLULA MADURA POR OTRA CÉLULA MADURA

C. CAMBIO EN NÚMERO

D. CAMBIO EN TAMAÑO

0.2 puntos
PREGUNTA 39
1. EN UNA VÁLVULA CARDIACA DAÑADA ES COMÚN ENCONTRAR:
A. PIGMENTO INTRACELULAR

B. CALCIFICACIÓN DISTRÓFICA

C. CALCIFICACIÓN FISIOLÓGICA

D. CALCIFICACIÓN METASTÁSICA

0.2 puntos
PREGUNTA 40
1. LA CALCIFICACIÓN DISTRÓFICA SE ENCUENTRA EN:
A. ÁREAS DE NECROSIS.

B. LA REABSORCION ÓSEA.
C. LA INTOXICACIÓN CON VITAMINA D.

D. EL INCREMENTO DE LA SECRECIÓN DE HORMONA PARATIROIDEA.

0.2 puntos
PREGUNTA 41
1. ESTE TIPO DE LESIÓN PRESENTA BAJA RESPUESTA DE LINFOCITOS T:
A. GRANULOMA POR M. TUBERCULOSIS

B. GRANULOMA POR CUERPO EXTRAÑO

C. GRANULOMA INMUNITARIO

D. GRANULOMA POR BACILOS GRAMNEGATIVOS

0.2 puntos
PREGUNTA 42
1. SON CAUSAS DE INFLAMACION EXCEPTO
A. ANTIBIOTICOS
B. REACCIONES INMUNITARIAS
C. CUERPO EXTRAÑO
D. NFECCIONES CUERPO EXTRAÑO

0.2 puntos
PREGUNTA 43
1. LA FAGOCITO OXIDASA MAS OXIDO NITRICO PARTICIPA EN EL ESTALLIDO RESPIRATORIO
FORMANDO:
A. ANIÓN SUPEROXIDO

B. PEROXIDO DE HIDROGENO
C. PEROXINITRITO
D. HIPOCLORITO

0.2 puntos
PREGUNTA 44
1. TIPO DE NECROSIS QUE SE PRODUCE EN LA PANCREATITIS AGUDA:
A. NECROSIS CASEOSA
B. NECROSIS LICUEFACTIVA
C. NECROSIS COAGULATIVA
D. NECROSIS GRASA
0.2 puntos
PREGUNTA 45
1. PROCEDIMIENTO DONDE, SE EXTIRPA LA LESIÓN COMPLETA EN UN SOLO TIEMPO. SE
UTILIZA CUANDO SE DESEA RESECAR LA TOTALIDAD DE LA LESIÓN YA SEA BENIGNA O
MALIGNA, DEJANDO MÁRGENES DE SEGURIDAD PARA EVITAR LA RECIDIVA.
A. BIOPSIA ASPIRACIÓN

B. BIOPSIA INCISIONAL

C. BIOPSIA TREPANACIÓN

D. BIOPSIA EXCISIONAL

0.2 puntos
PREGUNTA 46
1. PUEDE DEBERSE A MUTACIONES, ENVEJECIMIENTO O FACTORES AMBIENTALES
DESCONOCIDOS, SE RECONOCE EN LA ACTUALIDAD COMO RASGO PROPIO DE DIVERSAS
ENFERMEDADES NEURODEGENERATIVAS, COMO ALZHEIMER, COREA DE HUNTINGTON O
EL PARKINSONISMO.
APOPTOSIS POR DAÑO AL ADN.

APOPTOSIS MEDIADA POR LINFOCITOS T CITOTÓXICOS.

APOPTOSIS POR MAL PLEGAMIENTO DE PROTEÍNAS.

APOPTOSIS POR CARENCIA DE FACTORES DE CRECIMIENTO.

0.2 puntos
PREGUNTA 47
1. LA DIGESTION EXZIMATICA ES EL EVENTO PREDOMINANTE EN EL SIGUIENTE TIPO DE
NECROSIS:
A. NECROSIS GRASA

B. N. DE COAGULACION

C. N. DE LICUEFACCION

D. NECROSIS FIBRINOSA

0.2 puntos
PREGUNTA 48
1. LOS RECEPTORES DE MANOSA, LOS RECEPTORES, BARREDORES (DEPURADORES) Y LOS
RECEPTORES DE DIVERSAS OPSONINAS PERTENECEN A LOS RECEPTORES DE:
RODAMIENTO.

ÚNICAMENTE RECEPTORES DE LA ACTIVACIÓN DEL COMPLEMENTO.

FAGOCITOSIS

DE LAS ESPECIES REACTIVAS DEL OXÍGENO.


0.2 puntos
PREGUNTA 49
1. LA PIROPTOSIS SE DEFINE COMO:
A. LESIÓN CELULAR

B. MUERTE CELULAR PROGRAMADA POR CASPASA 9

C. ADAPTACIÓN CELULAR

D. MUERTE CELULAR PROGRAMADA POR IL-1

0.2 puntos
PREGUNTA 50
1. ¿QUÉ EFECTO TIENE LA DISMINUCIÓN DE ATP EN EL PH CITOSÓLICO?
A. DISMINUYE POR AUMENTO DE PIRUVATO

B. DIMINUYE POR AUMENTO DE ÁCIDO LACTICO

C. AUMENTA POR INCREMENTO DE BICARBONATO

D. DISMINUYE POR INCREMENTO DE ÁCIDO CARBÓNICO


Edema
Edema: Aumento de líquidos en espacios intersticiales o en cavidades del organismo, cuando se presenta en tejido de extremidades inferiores, es detectable
clínicamente por el signo de Godet/de fóvea.

 Inflamatorio/No inflamatorio
 Localizado (Secundario a obstrucción venosa o linfática)/Sistémico (Por insuficiencia cardíaca)
 Agudo/Crónico
 Trasudado causando edema generalizado por disminución de la presión oncótica (por ejemplo, síndrome nefrótico, cirrosis hepática) o aumento
de la presión hidrostática (insuficiencia cardiaca) o favoreciendo aldosteronismo secundario que causa la retención de sodio y agua.
 Exudado por aumento en la permeabilidad del vaso sanguíneo a las proteínas (infección) u obstrucción en el sistema linfático (neoplasias)
expresando edema localizado en la región afectada.

Trastorno del Insuficiencia Pericarditis Ascitis Obstrucción Trombosis Presión Dilatación arteriolas por desregulación
retorno venoso cardiaca cnstrictiva o externa neurohumoral/Calor
congestiva compresión
venosa
Malnutrición Cirrosis Gastroenterop Glomerulopatías
(localiza en los hepatica (lleva a atía con
miembros ascitis) pérdida
pélvicos) proteica
Inflamatoria Neoplásica Postquirúrgica Secundaria a terapia con radiación
Aguda Crónica Angiogenia
Hipoperfusión Amento de la Aumento en secresión de renina-angiotensina-aldosterona.
renal reabsorión
tubular de sodio
Fuerzas de Starling
Diferentes fuerzas que resultan en movimientos netos de
líquidos intra y extracelulares, (regulan la distribución de
1. Aumento de presión hidrostática: Si la presión capilar media aumenta por encima de 17 mm Hg. líquidos entre los dos componentes del compartimiento
Causará un aumento de la presión de filtración neta, que cuando no puede ser manejada por el extracelular) expresadas en la Ley de Starling de
flujo linfático origina edema. intercambio capilar.
2. Disminución de la presión oncótica: La disminución de las proteínas plasmáticas, La presión hidrostática en los capilares y la presión
fundamentalmente la albúmina hace caer los valores de la presión coloidosmótica del plasma, oncótica de los coloides en el líquido intersticial favorecen
provocando así la salida de líquido hacia el espacio intersticial. el movimiento de líquido del espacio vascular al espacio
3. Daño capilar: El daño al endotelio capilar puede provocar un incremento de la permeabilidad que extravascular.
permite el pasaje de proteínas al espacio intersticial, incrementando así la salida de líquido del Pero la presión oncótica coloide (proteínas plasmáticas y
espacio vascular. El daño capilar es el responsable del edema inflamatorio, que es localizado, duro, la presión hidrostática en el líquido intersticial) favorecen
y se acompaña de rubor, calor, tumefacción y dolor. el desplazamiento de líquido al compartimiento vascular,
en consecuencia, hay un desplazamiento de agua y de
4. Obstrucción linfática: Al impedir el drenaje del exceso de líquido intersticial, se crea un importante
solutos difusibles del espacio vascular al extremo
incremento de la presión tisular, además de un aumento de la concentración de proteínas en el arteriolar de los capilares.
líquido, lo que hará posible la acumulación de líquido en el intersticio.
El líquido regresa del espacio intersticial hacia el sistema
vascular en el extremo venoso de los capilares y a través
de los linfáticos. Estos desplazamientos de líquidos
suelen equilibrarse de forma que existe un estado de
equilibrio en los tamaños de los compartimientos
intravascular e intersticial, pese al gran intercambio que
Insuficiencia cardíaca congestiva Síndrome nefrótico ocurre entre ellos.
Cirrosis hepática Síndrome nefrótico
Síndrome nefrótico Insuficiencia renal
Desnutrición severa
Bilateral Unilateral/Regional/Asimétrio
Generalizado
Insuficiencia cardiaca Insuficiencia venosa/ Obstrucción venosa
Desnutrición (kwashiorkor) Inmovilización de una parte del cuerpo
Insuficiencia hepática (cirrosis) Lesión de tejido blando (infección, quemaduras
Enfermedad renal (síndrome nefrótico) Linfedema secundario (filariasis)
En extremidades inferiores Lipedema: se observan con frecuencia marcas negras o azules, es sensible a
Síndrome de malabsorción la presión y al tacto, empeora al estar de pie durante periodos largos o en
Medicamentoso: nifedipina y amlodipina, antiinflamatorios no esteroides y un ambiente cálido, asociado a linfedema.
esteroides
Embarazo/ Preeclampsia ( inicio súbito después de la semana 20 de
embarazo (>900 g/ semana), presente en cara, manos y los miembros
pélvicos, es blando, blanco, indoloro, aumenta con la posición de pie)
Idiopático
Mixedema
Linferema 1º
Lipedema

- : edema sistémico extenso, as causas más comunes son la insuficiencia cardiaca congestiva y el síndrome
cirrótico. El diagnóstico es clínico por el examen físico, destacando el signo de la fóvea (Godet) positivo.

el edema es de inicio gradual, progresivo, localizado en las piernas, el dorso de los pies, se
observa el signo de Stemmer (incapacidad para plegar la piel del dorso de los dedos de la extremidad afectada),
predominio vespertino, duro, piel hiperqueratósica, papilomatosa (piel de naranja), indoloro, produce sensación
de pesantez, aumenta con el calor y con largos periodos de actividad, disminuye con la elevación de las piernas,
pero no desaparece, en 70% de los casos es unilateral y en 30% bilateral.

- inicio gradual, progresivo, en cara, párpados, el dorso de las manos (hipotiroidismo) y pretibial
(hipertiroidismo).

De consistencia es elástica, es blanco o pálido (hipotiroidismo), o amarillo marfil, el pretibial es pigmentado


(hipertiroidismo), indoloro (hipotiroidismo) y puede ser pruriginoso o doloroso, (hipertiroidismo).
Hemorragia
Extravasación de sangre tras la rotura de un vaso, puede ser externa o limitada a un tejido (hematoma), pérdidas mayores al 20% pueden dar
lugar a un shock hipovolémico (hemorrágico).

Hematoma Extravasación sanguínea, acompañada de tumefacción.

Equimosis Se producen por de la ruptura de vasos sanguíneos (principalmente capilares), que


conduce a extravasación de la sangre y acumulo en el tejido intradérmico, subcutáneo o
en ambos. Posterior a esto se inicia un proceso inflamatorio local que conlleva a la
degradación de la hemoglobina en subproductos cromóforos (hemosiderina, biliverdina,
bilirrubina, metahemoglobina) que dan la coloración inicial y sus cambios.
De 1 a 2 cm.

Petequia Si la lesión se presenta como un puntilleo sanguíneo, hasta del tamaño de la cabeza de un alfiler. 1 a 2 mm en piel, mucosas o serosas, por
aumento de la presión intravascular, trombocitopenia, defectos en la función plaquetaria o deficiencia en facotes de coagulación.
Derrame Si ocurre una extravasación de sangre muy grande o en lugar de sangre se extravasa linfa, se le conoce como derrame.

Púrpura Hemorragia mayor a 3mm, producidad bajo el mismo mecanismo de las petequias o bien, inflamación vascular local o fragilidad. vascular.

Grados de hemorragia
Sistema hemostático

Dentro de la hemostasia (conjunto de mecanismos fisiológicos mediante los cuales se consigue detener y cohibir
los procesos hemorrágicos y mantener la fluidez de la sangre circulante), se distingue una serie de fases enlazadas.

1) En un primer tiempo se forma el trombo plaquetario que tiene como función el cese inicial de la hemorragia
(hemostasia primaria). Ello ocurre en término de minutos y es de importancia básica para detener la hemorragia de
capilares, arteriolas pequeñas y vénulas. Requiere de dos fenómenos de importancia apital: el endotelio
vascular (fase vascular) y las plaquetas (fase plaquetar).

2) Con posterioridad se produce una serie de reacciones del sistema plasmático de coagulación que culminan en la
producción de trombina suficiente para convertir una pequeña porción plasmática de fibrinógeno en fibrina
(hemostasia secundaria). Precisa de más tiempo que la hemostasia primaria y, gracias a ella, los filamentos de
fibrina refuerzan el «tapón» hemostático primario. Tiene particular importancia en grandes vasos y evita la
hemorragia secundaria horas o días después de la lesión inicial. La conversión de la protrombina en trombina (fase
de los factores de coagulación) tiene lugar merced a diversas reacciones enzimáticas en las que intervienen los
factores de la coagulación que se agrupan en dos vías: la vía intrínseca y la vía extrínseca, que entre otros incluye
los factores vitamina K-dependientes (II, VII, IX, X).

3) Una vez transformado el fibrinógeno en fibrina, los coágulos formados se disuelven lentamente. Esto se consigue
gracias a la conversión de la fibrina insoluble en soluble mediante la acción de la plasmina, lo cual libera los
productos de degradación del fibrinógeno y fibrina (fibrinólisis). La plasmina se forma a partir de un precursor
plasmático inactivo llamado plasminógeno. En la figura 1 se muestra un esquema de las fases de la hemostasia.
Hiperemia y cogestión
fenómeno activo, consiste en un aumento en el flujo sanguíneo de los tejidos como resultado de dilatación arteriolar.

Se observa en general en órganos con aumento de actividad, como en las glándulas de secreción interna o en el músculo esquelético. En tejidos
que por su actividad tienen, en un momento dado, mayores necesidades metabólicas, siendo la concentración de oxígeno el regulador del más
importante del flujo sanguíneo local. Cuando baja la concentración de oxígeno, los esfínteres precapilares se abren y la sangre fluye por una
mayor cantidad de capilares. Además de la concentración de oxígeno influyen también sustancias vasodilatadoras como bióxido de carbono,
ácido láctico, adenosina y compuestos adenosinicos, histamina, iones de potasio e hidrógeno.

Fisiológico: Patológico:
 Ejercicio  Procesos inflamatorios y es causada por sustancias vasodilatadoras liberadas en los tejidos afectados
 Digestión  Irritación mecánica
 Hipertermia local
 Gestación  Radiación solar
Afectados en Insuficiencia ventricular derecha.

fenómeno pasivo, resulta de la


acumulación de la sangre en los capilares debido a un flujo lento en el lecho venoso u Afectado en insuficiencia de corazón derecho y obstrucción
obstrucción del flujo de salida de un tejido. La obstrucción aislada puede causar congestión de vena hepática o cava inferior, la congestión aguda lleva a
local, la sistémica se asocia a ICC. Macroscópicamente los tejidos son azul-rojizos (cianosis) estasis centrolobulillar que dilata la vena central y los
(acumulación de hemoxihemoglobina). sinusoides vecinos. Si la congestión se desarrolla muy rápido
y se retiene una gran cantidad de sangre en el hígado, los
En la los vasos están dilatados, la sangre se acumula en las venas y capilares, hepatocitos centrolobulillares sufren entonces necrosis
por lo regular no tiene consecuencias, pero si ocurre muy rápido, la hipoxia resultante y la isquémica.
compresión mecánica del tejido en contacto con los vasos dilatados puede causar necrosis. En la congestión pasiva crónica hepática los hepatocitos se
atrofian o mueren y se reemplazan por fibrosis.
En la la rotura de los capilares distendidos causa hemorragias focales,
con depósito secundario de hemosiderina en los macrófagos; también provoca isquemia, que
se acompaña de atrofia o pérdida de las células parenquimatosas que se reemplazan por tejido
conjuntivo, (macroscópicamente los órganos se ven café ocre, de consistencia aumentada y
fibróticos).
Hemostasia

El endotelio es un órgano que está involucrado en numerosos Las plaquetas son las principales implicadas
procesos fisiológicos, principalmente el de mantener la en alteraciones como la trombosis,
fluidez de la sangre. trastornos hemorrágicos y en eventos
Tienen propiedades antiplaquetarias, anticoagulantes y trombóticos hereditarios o adquiridos.
fibrinolíticas, pero tras su activación muestran una función Tras la lesión del endotelio vascular, entran
procoagulante. en contacto con constituyentes de la
Las células endoteliales no activadas expresan actividad ronectina, MEC (proteoglicanos,
anticoagulante, anti adhesiva y vasodilatadora, mientras que fibronectina, colágeno, glucoproteínas
la célula endotelial activada expresa actividad procoagulante, adhesivas)
proadhesiva y vasoconstrictora. Se activan, llevando a:
1.-Adhesión: Mediad por el vWF que actúa
como puente entre receptores de
plaquetas (principalmente glucoproteína I-
b) y el colágeno expuesto. (Enfermedad de
Von Wlebrand/Síndrome de Bernard-
Soulier=def de glucoproteína Ib).
2.-Cambio de forma: Transformándose de una célula discoide a una esférica, con la finalidad de tener
extensiones (pseudópodos) que faciliten la adhesión al endotelio y otras células, así como la interacción con
otras plaquetas y liberación del contenido de los gránulos en su interior.
3.-Reacción de liberación de gránulos: Los gránulos alfa expresan moléculas de adhesión tipo P-selectina y
contienen factores de coagulación y factores de crecimiento.
Gránulos densos/gránulos delta: Nucleótidos de adenosina (ADP (que es un ptentemediador de la agregasión)
y ATP), calcio y aminas vasoactivas como la histamina.
4.-Agregación: Promovida por *ADP cuya activación cambia la conformación de los receptores plaquetarios
GpIII-IIIa (receptor de la glucoproteína IIb/IIIa) permitiendo la unión al fibrinógeno, este hace que las
plaquetas conecten entre sí formando agregados. Y *tromboxano A2
El complejo GP Ib-IX-V es el segundo receptor plaquetario más abundante y participa principalmente en la
adhesión plaquetaria al subendotelio mediante la unión al factor de von Willebrand (FvW).También
promueve la unión a la trombina y facilita la activación de otras plaquetas.
Los estados de hiperactividad plaquetaria hereditarios o adquiridos son
causa de un número importante de eventos trombóticos arteriales y algunos
venosos.
Defecto Wein-Penzing; éste es poco frecuente, es una deficiencia en la vía
de la lipooxigenasa con incremento compensador de los productos de la
ciclooxigenasa (TXA2, PGE2 y PGD2), se asocia con infartos de miocardio en
gente joven.
Síndrome de la plaqueta pegajosa, causante de gran número de episodios
de trombosis arterial y venosa con morbilidad y mortalidad significativas
Se hereda con carácter autosómico dominante y las principales
manifestaciones clínicas son trombosis coronaria y vascular cerebral en
personas jóvenes sin otros factores de riesgo cardiovascular, aunque puede
haber episodios trombóticos arteriales y venosos en cualquier sitio.

Obstrucción local del flujo sanguíneo arterial o venoso que lleva a isquemia, la
Trombosis cual, en caso de prolongarse, produce una lesión celular irreversible (necrosis)
afectando cualquier tejido específico (infarto).
Un coágulo es una respuesta homeostática mientras que un trombo es un
fenómeno siempre patológico que se aloja en venas, arterias, capilares o
cavidades cardiacas.

Émbolo= masa intravascular capaz de obstruir un vaso, y que se transporta por la sangre hasta un lugar distante de su punto
Embolia de origen.
: es la más frecuente, en particular el émbolo trombótico, proveniente de un trombo. La embolia trombótica puede ser:
masiva, representada por un gran émbolo que incluso puede enclavarse en la bifurcación de la arteria pulmonar (émbolo
cabalgante); con trombos de pequeño tamaño, que ocluyen ramas de pequeño calibre y condicionan hemorragias,
hipertensión pulmonar e infarto pulmonar; y, por último, se pueden dar numerosas masas embólicas trombóticas.
dos tipos básicos de embolismo gaseoso: Venoso y arterial.
El embolismo venoso se produce al entrar aire en las venas del círculo mayor cercanas al corazón después de traumatismos, al
penetrar aire por las venas uterinas durante el parto, en cirugía torácica o en el neumotórax, donde la entrada de aire en las
venas rotas tiene lugar durante la inspiración, al disminuir la presión venosa. Se puede presentar en operaciones
neuroquirúrgicas, durante cirugías laparoscópicas, en la inserción o retirada de catéter venoso central estando el paciente
sentado (debe estar en Trendelenburg: tumbado con los pies más altos).
La entrada de 100-150 cc de aire es suficiente para causar la muerte.
la embolia grasa supone una obstrucción de los vasos por partículas grasas circulantes. Se produce tras fracturas
de hueso largo (con una médula ósea rica en grasa), tras masajes cardiacos prolongados, tras la libera
: 1 de cada 50.000 partos. Ocurre por la penetración de líquido amniótico (contiene escamas
epiteliales, lanugo, meconio, grasa, tromboplastina tisular y sustancias vasoactivas, como prostaciclina y tromboxano) o de
tejido fetal en la circulación materna a causa de un desgarro en las membranas placentarias o a través de una rotura de las
venas uterinas.
Las prostaglandinas causan una vasoconstricción pulmonar y alteran la contractibilidad cardiaca y la tromboplastina contribuye
al desarrollo del cuadro de coagulación intravascular diseminada.
Condición en que hay un suministro inadecuado y/o insuficiente de sangre a un tejido. Como consecuencia de la isquemia se
Isquemia produce una disminución de llegada de O2 a los tejidos. Si la hipoxia es mantenida en el tiempo se puede llegar a la anoxia. Los
tejidos no reciben la nutrición necesaria y hay un defecto en la eliminación de los productos de desecho.
Causas
 Intrínsecas: Entre ellas se encuentran los acontecimientos trombóticos o embólicos, casi todos como consecuencia de
una oclusión arterial, la arteriosclerosis, arteritis y shock.
 Extrínsecas: Debidas a ligaduras quirúrgicas, retorcimiento de los vasos (en la torsión testicular, ovárica o intestinal), la
compresión extrínseca de una vaso (tumores, quistes, abscesos) así como por la rotura traumática del riego sanguíneo.
Zona de necrosis isquémica por la oclusión del riego arterial o del retorno venoso. Tienden a ser cuneiformes, histológicamente
Infarto se caracterizan por necrosis coagulativa isquémica, donde se preservan las estructuras tisulares, a excepción de cerebro
(Licuefactiva).
 Trombosis o embolia
 Vasoespasmo
 Compresión extrínseca
 Torsión

Shock
El shock implica una hipoperfusión de los tejidos por una reducción del
volumen sanguíneo o del gasto cardíaco, esto es, disminuye el volumen de
eyección por lo que el corazón no bombea tanta sangre.
Puede deberse a una redistribución de la sangre, produciéndose una
vasodilatación periférica difusa (en el shock séptico, pacientes
politraumatizados): si aumenta el vaso, el contenido se reparte y por tanto
disminuye el volumen de sangre adecuado, puesto que ésta se tiene que
repartir.
7/10/2020 Realizar evaluación: PRIMER PARCIAL DEPARTAMENTAL – IDMD...
Cursos Mi institución Biblioteca virtual Centro
NANCY de Control
B. LÓPEZ LÓPEZ y Soporte

Tiempo restante: 02 minutos, 03 segundos.


?
IDMD 200 - Anatomía Patológica - 14 - Oto20 PRIMER PARCIAL DEPARTAMENTAL
Estado de nalización de la pregunta:
Realizar evaluación: PRIMER PARCIAL DEPARTAMENTAL
1 2 3 4 5 6 7 8 9 10 11 12 13 14 15 16 17 18
26 27 28 29 30 31 32 33 34 35 36 37 38 39 40 41 42 43

Realizar evaluación: PRIMER PARCIAL DEPARTAMENTAL

Información de la evaluación
Descripción
Instrucciones
Evaluación con Esta evaluación tiene un límite de tiempo de 1 hora.La evaluación se guardará y
limitación de se enviará automáticamente cuando el tiempo se agote.
tiempo Se mostrará una advertencia cuando falte la mitad del tiempo, 5 minutos, 1
minuto y 30 segundos.
Intentos Prohibido. Esta evaluación solo se puede realizar una vez.
múltiples
Forzar Esta evaluación se puede guardar y reanudar en cualquier momento hasta que
terminación el tiempo se haya agotado. El tiempo seguirá corriendo aunque salga de la
evaluación.
Your answers are saved automatically.

PREGUNTA 1 0.2 puntos Guardado

LA NECROSIS FIBRINOIDE SE CARACTERIZA POR

SE PRODUCE POR BACTERIAS PIÓGENAS COMO


A.
STAPHYLOCOCUS
Haga clicSE
enPRESENTA
Guardar yGRAN
enviarACUMULO
para guardar y enviar. Haga clic
DE NEUTRÓFILOS en Guardar todas las respuestas para guardar to
Y DETRITUS
B.
CELULARES EN EL ESPACIO CREADO POR LA NECROSIS

SE PRESENTA EN LA CAPA MEDIA DE LAS ARTERIAS POR G


C.
PROCESOS INMUNOLOGICOS
https://buap.blackboard.com/webapps/assessment/take/launch.jsp?course_assessment_id=_145473_1&course_id=_63286_1&content_id=_266160… 1/16
7/10/2020 Realizar evaluación: PRIMER PARCIAL DEPARTAMENTAL – IDMD...

SE PRESENTA EN ABSCESOS, APENDICITIS Y SALPINGITIS


D.
AGUDA 02 minutos, 03 segundos.
Tiempo restante:

Estado de nalización de la pregunta:

1 2 3
PREGUNTA 24 5 6 7 8 9 10 11 0.212puntos
13 14Guardado
15 16 17 18
26 27 28 29 30 31 32 33 34 35 36 37 38 39 40 41 42 43

FENOMENO DE ADAPTACION CON AUMENTO DEL NUMERO DE


CELULAS

A. HIPERPLASIA

B. HIPERTROFIA

C. METAPLASIA

D. ATROFIA

PREGUNTA 3 0.2 puntos Guardado

UN MECANISMO PARA EXPLICAR COMO PUEDE SER ATRAVESADO EL


ENDOTELIO EN LA INFLAMACION ES:

A LA EXISTENCIA DE POROS EN LA MEMBRANA PLASMÁTICA DE


A.
LAS CELULAS ENDOTELIALES

B. LA LIBERACIÓN DE PROTEASAS POR LAS PLAQUETAS

LA CONTRACCIÓN DE LAS CELULAS ENDOTELIALES CON


C.
ENSANCHAMIETO DE LAS UNIONES INTERCELULARES

DESORGANIZACION DEL ENDOTELIO POR PERDIDA DE


D.
ANCLAJE.

PREGUNTA 4 0.2 puntos Guardado

INHIBEN LA COX-1 Y COX-2, EN CONSECUENCIA, LA SÍNTESIS DE


PROSTAGLANDINAS:

AINES

ANTAGONISTAS DE RECEPTORES DE LEUCOTRIENOS.

CORTICOESTEROIDES.

AMINOPENICILINAS.
Haga clic en Guardar y enviar para guardar y enviar. Haga clic en Guardar todas las respuestas para guardar to

PREGUNTA 5 0.2 puntos Guardado


https://buap.blackboard.com/webapps/assessment/take/launch.jsp?course_assessment_id=_145473_1&course_id=_63286_1&content_id=_266160… 2/16
7/10/2020 Realizar evaluación: PRIMER PARCIAL DEPARTAMENTAL – IDMD...
p

LAS NEOPLASIAS MÁS FRECUENTEMENTE ENCONTRADAS EN UNA


Tiempo restante:
CITOLOGÍA 02 minutos,
ESFOLIATIVA 03 segundos.
CERVICAL SON LOCALIZADAS EN:

EPITELIO ESCAMOSO.
Estado de nalización de la pregunta:
EPITELIO COLUMNAR.
1 2 3 4 5 6 7 8 9 10 11 12 13 14 15 16 17 18
PARAMETRIOS.
26 27 28 29 30 31 32 33 34 35 36 37 38 39 40 41 42 43
ÚTERO.

PREGUNTA 6 0.2 puntos Guardado

SON REDES FIBRILARES EXTRACELULARES

A. TRAMPAS DE COLAGENO

B. TRAMPAS DE NEUTROFILOS

C. TRAMPAS FIBRILARES

TRAMPAS DE FIBRINA
D.

PREGUNTA 7 0.2 puntos Guardado

SON CARACTERÍSTICAS DE LA INFLAMACIÓN AGUDA:

INICIA EN DÍAS, PARTICIPAN PRICIPALMENTE LOS LINFOCITOS Y


LA LESIÓN TISULAR GENERALMENTE ES GRAVE Y PROGRESIVA.

INICIA EN MINUTOS U HORAS, PARTICIPAN PRICIPALMENTE LOS


LINFOCITOS Y LA LESIÓN TISULAR ES DE RESOLUCIÓN
ESPONTÁNEA.

INICIA EN MINUTOS U HORAS, PARTICIPAN PRICIPALMENTE LOS


NEUTRÓFILOS Y LA LESIÓN TISULAR GENERALMENTE ES DE
RESOLUCIÓN ESPONTÁNEA.

INICIA EN MINUTOS U HORAS, PARTICIPAN PRICIPALMENTE LOS


LINFOCITOS Y LA LESIÓN TISULAR GENERALMENTE ES GRAVE Y
PROGRESIVA.

PREGUNTA 8 0.2 puntos


Haga clic en Guardar y enviar para guardar y enviar. Haga clic en Guardar todas las Guardado
respuestas para guardar to

MEDIADORES RELACIONADOS CON EL DOLOR EN LA INFLAMACIÓN


AGUDA: G

A. LEUCOTRIENOS Y TROMBOXANO A 2
https://buap.blackboard.com/webapps/assessment/take/launch.jsp?course_assessment_id=_145473_1&course_id=_63286_1&content_id=_266160… 3/16
7/10/2020 Realizar evaluación: PRIMER PARCIAL DEPARTAMENTAL – IDMD...

B. QUIMIOCINAS Y PRODUCTOS BACTERIANOS


Tiempo restante: 02 minutos, 03 segundos.
C. PAF

PROSTAGLANDINAS
EstadoD.de Y BRADICININA
nalización de la pregunta:

1 2 3 4 5 6 7 8 9 10 11 12 13 14 15 16 17 18
26 27 28 29 30 31 32 33 34 35 36 37 38 39 40 41 42 43

PREGUNTA 9 0.2 puntos Guardado

SON CAUSAS GENERALES DE LESION CELULAR EXCEPTO:

A. AGENTES QUÍMICOS Y FARMACOS

B. REACCIONES INMUNOLÓGICAS

C. AGENTES FISICOS

D. ANTIOXIDANTES

PREGUNTA 10 0.2 puntos Guardado

ES UNA ANTIPROTEASA QUE SE ENCUENTRA EN SUERO Y FLUIDOS

A. CININAS

B. DEFENSINAS

GAMA 2 MACROGLOBULINA
C.

D. TRANSPEPTIDASA

PREGUNTA 11 0.2 puntos Guardado

ESTE TIPO DE NECROSIS SE CARACTERIZA PORQUE CONSERVA LA


ESTRUCTURA TISULAR DE LOS TEJIDOS INVOLUCRADOS

NECROSIS LICUEFATIVA

NECROSIS CASEOSA

NECROSIS GANGRENOSA

NECROSIS COAGULATIVA
Haga clic en Guardar y enviar para guardar y enviar. Haga clic en Guardar todas las respuestas para guardar to

G
PREGUNTA 12 0.2 puntos Guardado
https://buap.blackboard.com/webapps/assessment/take/launch.jsp?course_assessment_id=_145473_1&course_id=_63286_1&content_id=_266160… 4/16
7/10/2020 Realizar evaluación: PRIMER PARCIAL DEPARTAMENTAL – IDMD...

LA ESTIMULACIÓN DEL CRECIMIENTO DE LA PRÓSTATA POR


Tiempo restante: 02
ESTIMULACIÓN POR 03 segundos.
ANDRÓGENOS,
minutos, CORRESPONDE A:

A. HIPERPLASIA FISIOLÓGICA
Estado de nalización de la pregunta:
B. HIPERTROFIA PATOLÓGICA
1 C.2HIPERTROFIA
3 4 FISIOLÓGICA
5 6 7 8 9 10 11 12 13 14 15 16 17 18
26 27 28 29 30 31 32 33 34 35 36 37 38 39 40 41 42 43
D. HIPERPLASIA PATOLÓGICA

PREGUNTA 13 0.2 puntos Guardado

ES UN EJEMPLO DE APOPTOSIS DONDE LA EXPOSICIÓN DE CÉLULAS


A LA RADIACIÓN Y/O QUIMIOTERAPIA, POR UNA AGRESIÓN
GENOTÓXICA Y EN EL QUE INTERVIENE EL GEN SUPRESOR TUMORAL
TP53.

MAL PLEGAMIENTO DE PROTEÍNAS.

DAÑO AL ADN.

ENFERMEDADES NEURODEGENERATIVAS, COMO EL


ALZHEIMER, LA COREA DE HUNTINGTON O EL PARKINSONISMO.

CARENCIA DE FACTORES DE CRECIMIENTO.

PREGUNTA 14 0.2 puntos Guardado

¿CUÁL DE LOS SIGUIENTES MECANISMOS BIOQUÍMICOS NO


CONTRIBUYE A LA LESIÓN DE LA MEMBRANA CELULAR

A. DISFUNCIÓN MITOCONDRIAL

B. ALTERACIONES DEL CITOESQUELETO

C. ESPECIES REACTIVAS DE OXÍGENO

D. PRODUCCIÓN DE ATP Y BOMBA SODIO POTASIO NORMALES

PREGUNTA 15 0.2 puntos Guardado

EN LA MAYORÍA DE LOS CARCINOMAS DE CUELLO UTERINO, EL DNA


DEL VIRUS DEL PAPILOMA HUMANO SE ENCUENTRA:

A. INTEGRADO AL GENOMA
Haga clic en Guardar y enviar para guardar y enviar. Haga clic en Guardar todas las respuestas para guardar to
B. DENTRO DE LA CÁPSIDE VIRAL

C. EN FORMA DE EPISOMA G

D. DESTRUIDO
https://buap.blackboard.com/webapps/assessment/take/launch.jsp?course_assessment_id=_145473_1&course_id=_63286_1&content_id=_266160… 5/16
7/10/2020 Realizar evaluación: PRIMER PARCIAL DEPARTAMENTAL – IDMD...
. S U O

Tiempo restante: 02 minutos, 03 segundos.

PREGUNTA
Estado 16 de la pregunta:
de nalización 0.2 puntos Guardado

1 CAMBIO
2 REVERSIBLE
3 4 EN5 EL CUAL
6 UNA7 CELULA
8 ADULTA,
9 10 ES 11 12 13 14 15 16 17 18
SUSTITUIDA POR OTRA CELULA ADULTA DE UN TIPO DIFERENTE SE
26LLAMA
27 28 29 30 31 32 33 34 35 36 37 38 39 40 41 42 43

A. HIPERPLASIA

B. HIPERTROFIA

C. METAPLASIA

D. AGENESIA

PREGUNTA 17 0.2 puntos Guardado

SON ANORMALIDADES ENCONTRADAS EN UNA CITOLOGÍA


EXFOLIATIVA EXCEPTO:

A. CÉLULAS BINUCLEADAS

B. PERDIDA DE LA RELACIÓN NÚCLEO-CITOPLASMA

C. CÉLULAS DE DIFERENTES TAMAÑOS

D. PRESENCIA DE ANTICUERPOS

PREGUNTA 18 0.2 puntos Guardado

LA FAGOCITO OXIDASA MAS OXIDO NITRICO PARTICIPA EN EL


ESTALLIDO RESPIRATORIO FORMANDO:

ANIÓN SUPEROXIDO
A.

B. PEROXIDO DE HIDROGENO

C. PEROXINITRITO

D. HIPOCLORITO

HagaPREGUNTA
clic en Guardar
19 y enviar para guardar y enviar. Haga clic en Guardar todas las respuestas para guardar to
0.2 puntos Guardado

SE OBSERVA TRAS UNA DESTRUCCIÓN SUSTANCIAL DE TEJIDO,


G
CUANDO LA LESIÓN INFLAMATORIA AFECTA A TEJIDOS QUE NO SON
CAPACES DE REGENERARSE O CUANDO HAY ABUNDANTE
EXUDACIÓN DE FIBRINA EN TEJIDOS O CAVIDADES SEROSAS QUE
https://buap.blackboard.com/webapps/assessment/take/launch.jsp?course_assessment_id=_145473_1&course_id=_63286_1&content_id=_266160… 6/16
7/10/2020 Realizar evaluación: PRIMER PARCIAL DEPARTAMENTAL – IDMD...
EXUDACIÓN DE FIBRINA EN TEJIDOS O CAVIDADES SEROSAS, QUE
NO ES POSIBLE ELIMINAR ADECUADAMENTE.
Tiempo restante: 02 minutos, 03 segundos.
FIBROSIS

EstadoRESOLUCIÓN COMPLETA.
de nalización de la pregunta:
INFLAMACIÓN CRÓNICA.
1 2 3 4 5 6 7 8 9 10 11 12 13 14 15 16 17 18
26 REMISIÓN.
27 28 29 30 31 32 33 34 35 36 37 38 39 40 41 42 43

PREGUNTA 20 0.2 puntos Guardado

SON EFECTOS DE LA HIPOXIA SOBRE LA CÉLULA LESIONADA, SEÑALE


CUÁL NO LO ES:

A. DISMINUCIÓN EN LA PRODUCCIÓN DE ATP

B. REDUCCIÓN DE LA ACTIVIDAD DE LA BOMBA SODIO POTASIO

C. LA CÉLULA DAÑADA SUFRE ALCALOSIS METABÓLICA

D. ACIDOSIS LÁCTICA DE LA CÉLULA DAÑADA

PREGUNTA 21 0.2 puntos Guardado

REALIZO EL PRIMER ESTUDIO DE LA HISTAMINA EN LOS CAMBIOS


VASCULARES DE LA INFLAMACION

A. GEORGE BERNARD SHAW

SIR TOMAS LEWIS


B.

C. ELIE METCHNIKOFF

D. RUDOLF VIRCHOW

PREGUNTA 22 0.2 puntos Guardado

TODAS LAS CÉLULAS TIENEN RECEPTORES CITOSÓLICOS QUE


RECONOCEN UNA AMPLIA DIVERSIDAD DE MOLÉCULAS LIBERADAS
O ALTERADAS COMO CONSECUENCIA DEL DAÑO CELULAR. ESTOS
RECEPTORES ACTIVAN UN COMPLEJO CITOSÓLICO
MULTIPROTEÍNICO LLAMADO:

INFLAMASOMA
Haga clic en Guardar y enviar para guardar y enviar. Haga clic en Guardar todas las respuestas para guardar to

SISTEMA DEL COMPLEMENTO


G
REACCIÓN ANTIGENO-ANTICUERPO.

Ó
https://buap.blackboard.com/webapps/assessment/take/launch.jsp?course_assessment_id=_145473_1&course_id=_63286_1&content_id=_266160… 7/16
7/10/2020 Realizar evaluación: PRIMER PARCIAL DEPARTAMENTAL – IDMD...
LECTINA DE UNIÓN A MANOSA

Tiempo restante: 02 minutos, 03 segundos.

Estado de nalización de la pregunta:


PREGUNTA 23 0.2 puntos Guardado

1 2 3 4 5 6 7 8 9 10 11 12 13 14 15 16 17 18
LA ANGIOGENIA SE CONOCE COMO
26 27 28 29 30 31 32 33 34 35 36 37 38 39 40 41 42 43
A.

MIGRACIÓN Y PROLIFERACIÓN DE LINFOCITOS Y


FIBROBLASTOS

FORMACIÓN DE NUEVOS VASOS SANGUÍNEOS DURANTE LA


B.
REPARACIÓN

TEJIDO NUEVO QUE INVADE PROGRESIVAMENTE EL SITIO DE


C.
LA LESIÓN

MADURACIÓN Y REORGANIZACIÓN SECUENCIAL POTS LESIÓN


D.
TISULAR

PREGUNTA 24 0.2 puntos Guardado

UNA BIOPSIA ES:

A. ESPECÍMEN DERIVADO DE CÉLULAS DESPRENDIDAS

B. ÚNICAMENTE ESPÉCIMEN MICROSCÓPICO

C. ÚNICAMENTE ESPÉCIMEN MACROSCÓPICO

ESPÉCIMEN MACROSCÓPICO Y/O MICROSCÓPICO EXTRAÍDO


D.
DURANTE LA VIDA DEL PACIENTE

PREGUNTA 25 0.2 puntos Guardado

EL SISTEMA DEL COMPLEMENTO VÍA CLÁSICA SE ACTIVA POR:

A. ANTICUERPOS

B. OLIGOSACÁRIDOS COMPLEJOS COMO FUCOSA O MANOSA

C. LPS EN LA SUPERFICIE DE UN MICROBIO

D. GLUCOPROTEÍNAS

Haga clic en Guardar y enviar para guardar y enviar. Haga clic en Guardar todas las respuestas para guardar to

PREGUNTA 26 0.2 puntos Guardado G

ES UN CONJUNTO DE PROTEÍNAS SOLUBLES Y RECEPTORES DE


https://buap.blackboard.com/webapps/assessment/take/launch.jsp?course_assessment_id=_145473_1&course_id=_63286_1&content_id=_266160… 8/16
7/10/2020 Realizar evaluación: PRIMER PARCIAL DEPARTAMENTAL – IDMD...
ES UN CONJUNTO DE PROTEÍNAS SOLUBLES Y RECEPTORES DE
MEMBRANA QUE FUNCIONAN FUNDAMENTALMENTE EN DEFENSA
DEL ANFITRIÓN CONTRA MICROBIOS Y EN REACCIONES
Tiempo restante: 02
INFLAMATORIAS minutos, 03 segundos.
PATOLÓGICAS.

CITOCINAS.
Estado de nalización de la pregunta:
QUIMIOCINAS.
1 2 3 4 5 6 7 8 9 10 11 12 13 14 15 16 17 18
SISTEMA DEL COMPLEMENTO.
26 27 28 29 30 31 32 33 34 35 36 37 38 39 40 41 42 43
EICOSANOIDES.

PREGUNTA 27 0.2 puntos Guardado

EN LA HIPERPLASIA PROSTÁTICA BENIGNA, LA


DIHIDROTESTOSTERONA (DHT) RESPUESTA:

ES MENOS POTENTE QUE LA TESTOSTERONA, SE LOCALIZA EN


A.
HÍGADO

ES MÁS POTENTE QUE LA TESTOSTERONA, SE ENCUENTRA EN


B.
EL ESTRÓMA PROSTÁTICO

C. ES SINTETIZADA POR LA AROMATASA

D. ES IDÉNTICA A LA TESTOSTERONA

PREGUNTA 28 0.2 puntos Guardado

LA HISTAMINA ES SECRETADA POR:

A. LINFOCITOS

B. LEUCOCITOS

C. CÉLULAS DENDRÍTICAS

D. MASTOCITOS

PREGUNTA 29 0.2 puntos Guardado

SE CORRELACIONA CON EL ASPECTO BLANQUECINO Y


DISGREGABLE. GENERALMENTE SE PRESENTA EN FOCOS DE
INFECCIÓN TUBERCULOSA.

NECROSIS CASEOSA
Haga clic en Guardar
NECROSIS y enviar para guardar y enviar. Haga clic en Guardar todas las respuestas para guardar to
GANGRENOSA

NECROSIS COAGULATIVA
G
NECROSIS LICUEFATIVA

https://buap.blackboard.com/webapps/assessment/take/launch.jsp?course_assessment_id=_145473_1&course_id=_63286_1&content_id=_266160… 9/16
7/10/2020 Realizar evaluación: PRIMER PARCIAL DEPARTAMENTAL – IDMD...

Tiempo restante: 02 minutos, 03 segundos.

PREGUNTA 30 0.2 puntos Guardado


Estado de nalización de la pregunta:
REDUCEN LA INFLAMACIÓN INHIBIENDO EL RECLUTAMIENTO DE
1 LEUCOCITOS:
2 3 4 5 6 7 8 9 10 11 12 13 14 15 16 17 18
26 27 28
LIPOXINAS 29 30 31 32 33 34 35 36 37 38 39 40 41 42 43

PROSTAGLANDINAS.

LEUCOTRIENOS.

METABOLITOS DEL ÁCIDO ARAQUIDÓNICO.

PREGUNTA 31 0.2 puntos Guardado

LA LESIÓN POR ISQUEMIA- REPERFUSIÓN:

TIENE UN EFECTO PARADÓJICO YA QUE PUEDE SER BENÉFICA


A.
O PERJUDICIAL

NO CAUSA LESIÓN YA QUE AL HABER REPERFUSIÓN EL TEJIDO


B.
NO ES AFECTADO

C. SIEMPRE SE PUEDE REVERTIR

D. SIEMPRE INDUCE MUERTE CELULAR

PREGUNTA 32 0.2 puntos Guardado

LA LISOZIMA, LACTOFERRINA, PROTEÍNA BÁSICA PRINCIPAL Y LAS


DEFENSINAS, DESTRUYEN MICROORGANISMOS POR MECANISMOS

A. RELACIONADOS CON LA APOPTOSIS

B. DEPENDIENTES DEL OXÍGENO

C. INDEPENDIENTES DEL OXÍGENO

DESCONOCIDOS
D.

Haga clic en Guardar y enviar para guardar y enviar. Haga clic en Guardar todas las respuestas para guardar to
PREGUNTA 33 0.2 puntos Guardado

ES UN FENOMENO PATOLOGICOQUE SUCEDE DURANTE LA G


FAGOCITOSIS

https://buap.blackboard.com/webapps/assessment/take/launch.jsp?course_assessment_id=_145473_1&course_id=_63286_1&content_id=_26616… 10/16
7/10/2020 Realizar evaluación: PRIMER PARCIAL DEPARTAMENTAL – IDMD...
A. UNION

B. restante:
Tiempo 02 minutos, 03 segundos.
ATRAPAMIENTO

ENGLOBAMIENTO
EstadoC.de nalización de la pregunta:

1 2 3 4 5 6 7 8 9 10 11 12 13 14 15 16 17 18
D. REGURGITACION
26 27 28 29 30 31 32 33 34 35 36 37 38 39 40 41 42 43

PREGUNTA 34 0.2 puntos Guardado

SE REFIERE AL RESTABLECIMIENTO DE LA ARQUITECTURA Y


FUNCIÓN TISULAR TRAS UNA LESIÓN

RESTITUCIÓN
A.

B. CICATRIZACIÓN

C. REPARACIÓN

D. REGENERACIÓN

PREGUNTA 35 0.2 puntos Guardado

QUE ESTRUCTURAS ANATÓMICAS AFECTA LA


NECROSIS FIBRINOIDE:

A. CEREBRO

B. PÁNCREAS

C. ARTERIAS ARTERIOLASLINFATICOS
D. LINFATICOS

PREGUNTA 36 0.2 puntos Guardado

TIPO DE NECROSIS QUE SE PRODUCE EN LA


PANCREATITIS AGUDA:
A. NECROSIS GRASA
Haga clic en Guardar y enviar
B. NECROSIS para guardar y enviar. Haga clic en Guardar todas las respuestas para guardar to
LICUEFACTIVA
C. NECROSIS COAGULATIVA
G
D. NECROSIS CASEOSA
https://buap.blackboard.com/webapps/assessment/take/launch.jsp?course_assessment_id=_145473_1&course_id=_63286_1&content_id=_26616… 11/16
7/10/2020 Realizar evaluación: PRIMER PARCIAL DEPARTAMENTAL – IDMD...

Tiempo restante: 02 minutos, 03 segundos.

PREGUNTA 37 0.2 puntos Guardado


Estado de nalización de la pregunta:

EL ESTUDIO DE AUTOPSIA ANATOMO CLÍNICA QUIEN


1 LA REALIZA:
2 3 4 5 6 7 8 9 10 11 12 13 14 15 16 17 18
26 27 28 29 30 31 32 33 34 35 36 37 38 39 40 41 42 43
A. MÉDICO FORENSE

B. MÉDICO LEGISTA
C. ANATOMOPATÓLOGO
PERITO DEL MINISTERIO PÚBLICO
D.

PREGUNTA 38 0.2 puntos Guardado

TODOS LOS SIGUIENTES SON COLORANTES DEL TREN DE TINCIÓN


EXCEPTO:

A. ORANGE GREEN

B. HEMATOXILINA ACIDA

C. EOSINA ALCALINA

D. VIOLETA DE GENCIANA

PREGUNTA 39 0.2 puntos Guardado

LA FIJACIÓN DE LOS LEUCOCITOS A LAS CÉLULAS ENDOTELIALES


MEDIANTE UNA ADHESIÓN FIRME, ES MEDIADA POR:

INTEGRINAS.

SELECTINA P.

SELECTINA A Y SELECTINA E.

SELECTINA A, E Y P.

PREGUNTA 40 0.2 puntos Guardado

Haga clic en Guardar y enviar para guardar y enviar. Haga clic en Guardar todas las respuestas para guardar to
LA PRESENCIA DE ESTRIAS ROJAS EN LA PIEL CERCA DE UNA HERIDA
INDICA QUE:

HAY ANGIOGÉNESIS G

ES TEJIDO DE GRANULACIÓN
https://buap.blackboard.com/webapps/assessment/take/launch.jsp?course_assessment_id=_145473_1&course_id=_63286_1&content_id=_26616… 12/16
7/10/2020 Realizar evaluación: PRIMER PARCIAL DEPARTAMENTAL – IDMD...
ES TEJIDO DE GRANULACIÓN.

LA HERIDA TUVO DEHISCENCIA.


Tiempo restante: 02 minutos, 03 segundos.
LA HERIDA SE HA INFECTADO.
Estado de nalización de la pregunta:

1 2 3 4 5 6 7 8 9 10 11 12 13 14 15 16 17 18
26 PREGUNTA
27 28 41
29 30 31 32 33 34 35 36 0.237puntos
38 39Guardado
40 41 42 43

ES UN EXCESO DE LÍQUIDO EXTRAVASCULAR CON ESCASO


CONTENIDO PROTEICO; ESENCIALMENTE, ES UN ULTRAFILTRADO
DEL PLASMA SANGUÍNEO DEBIDO A PRESIONES ELEVADAS DEL
LÍQUIDO O A UNA REDUCCIÓN DE LAS FUERZAS OSMÓTICAS DEL
PLASMA.

LÍQUIDO INFLAMATORIO CRÓNICO Y AGUDO.

TRASUDADO.

EDEMA.

EXUDADO.

PREGUNTA 42 0.2 puntos Guardado

¿CUÁLES SON LOS TIPOS DE GRÁNULOS QUE


CONTIENEN LOS NEUTROFILOS?
A. ESPECÍFICOS Y AZURÓFILOS
B. LISOS Y RUGOSOS
C. AZURÓFILOS Y EOSINOFILOS
D. EOSINOFILOS Y RUGOSOS

PREGUNTA 43 0.2 puntos Guardado

LA LONGITUD DE LOS TELOMEROS SE


CORRELACIONA CON :
A. CALCIFICACIONES PATOLÓGICAS
B. SENESCENCIA REPLICATIVA

C. APOPTOSIS

D. NUMERO DE DIVISIONES CELULARES


Haga clic en Guardar y enviar para guardar y enviar. Haga clic en Guardar todas las respuestas para guardar to

PREGUNTA 44 0.2 puntos Guardado


https://buap.blackboard.com/webapps/assessment/take/launch.jsp?course_assessment_id=_145473_1&course_id=_63286_1&content_id=_26616… 13/16
7/10/2020 Realizar evaluación: PRIMER PARCIAL DEPARTAMENTAL – IDMD...
0. pu tos Gua dado

TAMBIÉN ES CONOCIDO COMO EL GUARDIÁN DEL


Tiempo restante: 02 minutos, 03 segundos.
GENOMA:
GEN
EstadoA.de DEL RETINOBLASTOMA
nalización de la pregunta: (RB)
B. RAS
1 2 3 4 5 6 7 8 9 10 11 12 13 14 15 16 17 18
26 27P5328
C. 29 30 31 32 33 34 35 36 37 38 39 40 41 42 43
D.

P21

PREGUNTA 45 0.2 puntos Guardado

EN EL AUMENTO DE LA PERMEABILIDAD VASCULAR LA HISTAMINA,


BRADICINA, LEUCOTRIENOS Y OTROS MEDIADORES QUÍMICOS
OCASIONAN:

AUMENTO DE LA TRANSCITOSIS
A.

B. LESIÓN ENDOTELIAL MEDIADO POR LEUCOCITOS

C. FILTRACIÓN A TRAVÉS DE VASOS NEOFORMADOS

D. RETRACCIÓN ENDOTELIAL

PREGUNTA 46 0.2 puntos Guardado

EL ESÓFAGO DE BARRET ES UN EJEMPLO DE:


A. ATROFIA.

B. HIPERPLASIA.

C. HIPERTROFIA.

D. METAPLASIA.

PREGUNTA 47 0.2 puntos Guardado

UNA CARACTERÍSTICA HISTOLÓGICA DE LOS ORGANOS QUE SUFREN


ATROFIA ES:

HIPERPLASIA
Haga clic en GuardarDEL RETÍCUL
y enviar ENDOPLASMICO
para RUGOSO
guardar y enviar. Haga clic en Guardar todas las respuestas para guardar to

FORMACIÓN DE VACUOLAS AUTOFAGICAS


G
PRESENCIA DE MITOCONDRIAS GIGANTES

https://buap.blackboard.com/webapps/assessment/take/launch.jsp?course_assessment_id=_145473_1&course_id=_63286_1&content_id=_26616… 14/16
7/10/2020 Realizar evaluación: PRIMER PARCIAL DEPARTAMENTAL – IDMD...
CARIORREXIS

Tiempo restante: 02 minutos, 03 segundos.

Estado de nalización de la pregunta:


PREGUNTA 48 0.2 puntos Guardado

1 2 3 4 5 6 7 8 9 10 11 12 13 14 15 16 17 18
SON PROTEÍNAS PROAPOPTÓSICAS QUE TRAS SU ACTIVACIÓN SE
26OLIGOMERIZAN
27 28 EN LA 30
29 MEMBRANA
31 MITOCONDRIAL
32 33 34 EXTERNA,
35 36 37 38 39 40 41 42 43
PERMITIENDO SU PERMEABILIDAD.

PUMA Y NOXA.

BAD, BIM Y BID.

BAX Y BAK.

BH3

PREGUNTA 49 0.2 puntos Guardado

LAS CELULAS MONONUCLEARES QUE SE OBSERVAN EN LA


INFLAMACIÓN CRÓNICA SON:

A. NEUTROFILOS Y EOSINOFILOS Y BASOFILOS

B. CELULAS CEBADAS Y PLAQUETAS

C. MACROFAGOS, LINFOCITOS Y CELULAS PLASMATICAS

D. ADIPOSITOS, CONDROCITOS Y OSTEOCITOS

PREGUNTA 50 0.2 puntos Guardado

SEÑALE LAS CIRCUNSTANCIAS ADVERSAS QUE PUEDEN OCASIONAR


LESIÓN CELULAR

A. AUMENTO EN APORTE DE NUTRIENTES

B. MAYOR APORTE DE OXÍGENO

C. ALTERACIONES METABÓLICAS

GLUCOLISIS AEROBIA
D.

Haga clic en Guardar y enviar para guardar y enviar. Haga clic en Guardar todas las respuestas para guardar to

https://buap.blackboard.com/webapps/assessment/take/launch.jsp?course_assessment_id=_145473_1&course_id=_63286_1&content_id=_26616… 15/16
7/10/2020 Realizar evaluación: PRIMER PARCIAL DEPARTAMENTAL – IDMD...

Tiempo restante: 02 minutos, 03 segundos.

Estado de nalización de la pregunta:

1 2 3 4 5 6 7 8 9 10 11 12 13 14 15 16 17 18
26 27 28 29 30 31 32 33 34 35 36 37 38 39 40 41 42 43

Haga clic en Guardar y enviar para guardar y enviar. Haga clic en Guardar todas las respuestas para guardar to

https://buap.blackboard.com/webapps/assessment/take/launch.jsp?course_assessment_id=_145473_1&course_id=_63286_1&content_id=_26616… 16/16
27/11/2020 Realizar evaluación: 2o PARCIAL DEPARTAMENTAL – IDMD 200...

Estado de nalización de la pregunta:


Realizar evaluación: 2o PARCIAL DEPARTAMENTAL

Información de la evaluación
Descripción
Instrucciones
Intentos múltiples Prohibido. Esta evaluación solo se puede realizar una vez.
Forzar terminación Esta evaluación se puede guardar y reanudar posteriormente.
Your answers are saved automatically.

PREGUNTA 1 0.2 puntos Guardado

LA GRADACIÓN HISTOPRONOSTICA DE UNA NEOPLASIA MALIGNA SE


REFIERE A :
A. TAMAÑO DE LA LESIÓN PRIMARIA

B. DIAGNÓSTICO MOLECULAR

C. TRATAMIENTO QUIRÚRGICO

D. GRADO DE DIFERENCIACIÓN DE LAS CÉLULAS TUMORALES

PREGUNTA 2 0.2 puntos Guardado

PROCESO ACTIVO DONDE SE OBSERVA DILATACIÓN ARTERIOLAR CON


AUMENTO DEL FLUJO SANGUÍNEO:
A. TROMBOSIS

B. HEMORRAGIA

C. CONGESTIÓN

D. HIPEREMIA

PREGUNTA 3 0.2 puntos Guardado

POTENTE FARMACO ANTICOAGULANTE QUE SE


UTILIZA EN LOS TRASTORNOS DE LA COAGULACION
https://buap.blackboard.com/webapps/assessment/take/launch.jsp?course_assessment_id=_145404_1&course_id=_63284_1&content_id=_266142… 1/15
27/11/2020 Realizar evaluación: 2o PARCIAL DEPARTAMENTAL – IDMD 200...
UTILIZA EN LOS TRASTORNOS DE LA COAGULACION
EstadoA.de
VIT. K
nalización de la pregunta:

B. WARFARINA

C. HEPARINA

D. CALCIO

PREGUNTA 4 0.2 puntos Guardado

SON TUMORES CONSTITUIDOS POR CELULAS


INMADURAS QUE SE PARECEN A LAS QUE FORMAN
TEJIDO RUDIMENTARIO FETAL:
A. PAPILOMA

B. OSTEOMA

C. HAMARTOMA

D. BLASTOMA

PREGUNTA 5 0.2 puntos Guardado

LOS PATRONES MOLECULARES ASOCIADOS A PATÓGENOS SE UNEN A:


A. RECEPTOR CON ACTIVIDAD TIROSIN CINASA INTRINSECA

B. RECEPTORES TIPO SEÑUELO

C. RECEPTORS DE SIETE PROTEINAS

D. RECEPTORES JAK STAT

PREGUNTA 6 0.2 puntos Guardado

ES UNA CARACTERISTICA DEL SINDROME DE


TROUSSEAU ASOCIADO A CARCINOMAS DE PANCREAS
Y ESTOMAGO SE REFIERE A:
A. EMBOLOS

B. INFARTO

C. DEFICIT DE GLUCOPROTEINA 1B
D. TROMBOFLEBITIS MIGRATORIA
https://buap.blackboard.com/webapps/assessment/take/launch.jsp?course_assessment_id=_145404_1&course_id=_63284_1&content_id=_266142… 2/15
27/11/2020 Realizar evaluación: 2o PARCIAL DEPARTAMENTAL – IDMD 200...

Estado de nalización de la pregunta:

PREGUNTA 7 0.2 puntos Guardado

LOS TROMBOS MURALES SON AQUELLOS QUE SE


LOCALIZAN:
A. EN CÁMARAS CARDIACAS
B. EN LAS VEGETACIONES
C. CAYADO AÓRTICO
D. EN LA COAGULACIÓN INTRAVASCULAR
DISEMINADA

PREGUNTA 8 0.2 puntos Guardado

ES LA ENFERMEDAD POR DESCOMPRESION CON


APARICION DE BURBUJAS DE NITROGENO:
A. ES TIPICA DE LA CID
B. TROMBOFLEBITIS

C. TROMBOEMBOLIA SEPTICA
D. ES TIPICA DE BUCEADORES

PREGUNTA 9 0.2 puntos Guardado

SON ÁREAS DE NECROSIS ISQUÉMICA CAUSADAS CON MÁS FRECUENCIA


POR OCLUSIÓN ARTERIAL DEBIDA TÍPICAMENTE A TROMBOSIS O
EMBOLIA:

TROMBOSIS.

INFARTO.

ISQUEMIA.

COAGULOPATÍA POR COMSUMO.

PREGUNTA 10 0.2 puntos Guardado

LOS DIFERENTES ESTADOS DE HIPERCOAGULABILIDAD CONOCIDOS


HASTA AHORA:
A. SIEMPRE SON ADQUIRIDOS

B SIEMPRE SON HEREDITARIOS


https://buap.blackboard.com/webapps/assessment/take/launch.jsp?course_assessment_id=_145404_1&course_id=_63284_1&content_id=_266142… 3/15
27/11/2020 Realizar evaluación: 2o PARCIAL DEPARTAMENTAL – IDMD 200...
B. SIEMPRE SON HEREDITARIOS
Estado de nalización de la pregunta:
C. PUEDEN SER TANTO HEREDITARIOS COMO ADQUIRIDOS

D. RARA VEZ SON ADQUIRIDOS

PREGUNTA 11 0.2 puntos Guardado

SE REFIERE A LOS CAMBIOS REVERSIBLES,


HEREDITARIOS EN LA EXPRESIÓN GÉNICA, QUE SE
PRODUCEN SIN MUTACIÓN:
A. DELECCIÓN

B. AMPLIFICACIÓN GÉNICA
C. CAMBIOS EPIGENETICOS
D. CAMBIOS CROMOSÓMICOS

PREGUNTA 12 0.2 puntos Guardado

¿CUÁL DE LAS SIGUIENTES OPCIONES ES UNA CAUSA DE EDEMA NO


INFLAMATORIO?
A. FRÍO

B. TUBERCULOSIS

C. COCCIDIOIDOMICOSIS

D. INSUFICIENCIA CARDIACA CONGESTIVA

PREGUNTA 13 0.2 puntos Guardado

EN LA FORMACIÓN DE UN TROMBO INTERVIENE:


A. LENTITUD Y TURBULENCIA DEL FLUJO

B. DAÑO ENDOTELIAL
C. ESTADOS DE HIPOCOAGULABILIDAD
D. SÓLO AYB

PREGUNTA 14 0.2 puntos Guardado

EL GEN VIRICO E7 DEL VPH INACTIVA:

https://buap.blackboard.com/webapps/assessment/take/launch.jsp?course_assessment_id=_145404_1&course_id=_63284_1&content_id=_266142… 4/15
27/11/2020 Realizar evaluación: 2o PARCIAL DEPARTAMENTAL – IDMD 200...
A.
Estado de nalización de la pregunta:

P53 RB

B. MYC

C. RAS

D. RB

PREGUNTA 15 0.2 puntos Guardado

EL VIRUS DE EPSTEIN BARR PRODUCE:


A. LINFOMA DE BURKITT

B. SARCOMA DE KAPOSI

C. LEUCEMIA DE CELULAS PELUDAS

D. HEPATOCARCINOMA

PREGUNTA 16 0.2 puntos Guardado

ESTA MUTACIÓN ONCÓGENA SE HA DETECTADO EN CASI LA TOTALIDAD


DE LAS LEUCEMIAS DE CÉLULAS PELUDAS, MÁS DEL 60 % DE LOS
MELANOMAS Y EL 80% DE LOS NEVOS BENIGNOS.
A. MUTACIONES DE BRAF.

B. MUTACIONES PI3K

C. MUTACIONES ERBB1.

D. MUTACIONES RAF.

PREGUNTA 17 0.2 puntos Guardado

CONTRIBUYEN AL FENOTIPO MALIGNO DE LAS NEOPLASIAS:


A. MUTACIONES CONDUCTORAS.

B. MUTACIONES INICIADORAS.

C. MUTACIONES PASAJERAS.

D MUTACIONES CON PÉRDIDA DE LA FUNCIÓN DE GENES


https://buap.blackboard.com/webapps/assessment/take/launch.jsp?course_assessment_id=_145404_1&course_id=_63284_1&content_id=_266142… 5/15
27/11/2020 Realizar evaluación: 2o PARCIAL DEPARTAMENTAL – IDMD 200...
D. MUTACIONES CON PÉRDIDA DE LA FUNCIÓN DE GENES.
Estado de nalización de la pregunta:

PREGUNTA 18 0.2 puntos Guardado

¿CUALES SON LAS CARACTERÍSTICAS DEL TEJIDO DE GRANULACIÓN?


A. RICO EN COLÁGENO Y ELASTINA

B. RICO EN TEJIDO CONJUNTIVO PROVISIONAL Y NUEVOS CAPILARES


FINOS
C. BAJO EN FIBROBLASTOS

D. ABUNDANTES CÉLULAS INFLAMATORIAS

PREGUNTA 19 0.2 puntos Guardado

EL EDEMA SE DEFINE COMO:


A. AUMENTO DE VOLUMEN SANGUÍNEO

B. AUMENTO DEL LÍQUIDO EN EL ESPACIO INTERSTICIAL

C. AUMENTO DE LÍQUIDOS INTRACELULARES

D. DESCENSO EN LOS LÍQUIDOS INTERSTICIALES

PREGUNTA 20 0.2 puntos Guardado

ES UN FACTOR DE CRECIMIENTO QUE INTERVIENE EN LA ANGIOGENIA


A. FACTOR DE CRECIMIENTO FIBROBLASTICO

B. FACTOR DE CRECIMIENTO DE DISPERSION

C. FACTOR DE CRECIMIENTO TRANSFORMANTE ALFA

D. FACTOR DE CRECIMIENTO DERIVADO DE LAS PLAQUETAS

PREGUNTA 21 0.2 puntos Guardado

ES EL PASO INCIAL QUE INDUCE UN CAMBIO


IRREVERSIBLE PERO NO LETAL EN EL MATERIAL
GENETICO DE LA CELULA AFECTADA:
A. PROGRESION

B INICIACION
https://buap.blackboard.com/webapps/assessment/take/launch.jsp?course_assessment_id=_145404_1&course_id=_63284_1&content_id=_266142… 6/15
27/11/2020 Realizar evaluación: 2o PARCIAL DEPARTAMENTAL – IDMD 200...
B. INICIACION
Estado de nalización de la pregunta:
C. PROMOCION

D. REORGANIZACION

PREGUNTA 22 0.2 puntos Guardado

LOS CAMBIOS EN LAS INTERACCIONES CELULA-CELULA DE LA


NEOPLASIA Y LA DEGRADACION DE LA MEC PARTICIPAN EN EL PROCESO
DE:
A. ANGIOGENIA.

B. CARCINOGENIA

C. INVASION Y METASTASIS

D. INESTABILIDAD GENOMICA

PREGUNTA 23 0.2 puntos Guardado

NEOPLASIA BENIGNA DE LOS ASTROCITOS


A. LIPOSARCOMA

B. ASTROCITOMA

C. NEUROFIBROMA

D. LIPOMA

PREGUNTA 24 0.2 puntos Guardado

TROMBO FORMADO EN LAS CAVIDADES CARDIACAS:


A. ÉMBOLO

B. PETEQUIA

C. ANEURISMA

D. TROMBO MURAL

PREGUNTA 25 0.2 puntos Guardado

ESTAS CÉLULAS DESEMPEÑAN UN PAPEL ESENCIAL EN LA REPARACIÓN:


A. CÉLULAS DENDRÍTICAS
https://buap.blackboard.com/webapps/assessment/take/launch.jsp?course_assessment_id=_145404_1&course_id=_63284_1&content_id=_266142… 7/15
27/11/2020 Realizar evaluación: 2o PARCIAL DEPARTAMENTAL – IDMD 200...

EstadoB.de
MACRÓFAGO M1la pregunta:
nalización de

C. LINFOCITO T

D. MACRÓFAGO M2

PREGUNTA 26 0.2 puntos Guardado

EN ESTAS NEOPLASIAS, EL GEN ABL SE TRANSLOCA DESDE SU


LOCALIZACIÓN HABITUAL EN EL CROMOSOMA 9 HASTA EL
CROMOSOMA 22:
A. ADENOCARCINOMA DE CÉRVIX.

B. LEUCEMIA MIELOIDE CRÓNICA

C. ADENOCARCINOMA PANCREÁTICO Y COLANGIOCARCINOMAS

D. CÁNCER DE PÁNCREAS.

PREGUNTA 27 0.2 puntos Guardado

NEOPLASIA MALIGNA DE LAS CELULAS DE LOS DUCTOS SEMINALES


A. CORIOCARCINOMA

B. CARCINOMA EMBRIONARIO

C. SEMINOMA

D. TUMOR DE SENOS ENDODERMICOS

PREGUNTA 28 0.2 puntos Guardado

ESTE TIPO DE CÉLULAS ESTÁN PRESENTES EN LA INFLAMACIÓN


CRÓNICA, SON ABUNDANTES EN LAS REACCIONES INMUNITARIAS
MEDIADAS POR INMUNOGLOBULINA E Y EN LAS INFECCIONES
PARASITARIAS.
A. EOSINÓFILOS

B. LINFOCITOS

C. NEUTRÓFILOS

D. MACRÓFAGOS

https://buap.blackboard.com/webapps/assessment/take/launch.jsp?course_assessment_id=_145404_1&course_id=_63284_1&content_id=_266142… 8/15
27/11/2020 Realizar evaluación: 2o PARCIAL DEPARTAMENTAL – IDMD 200...

Estado de nalización de la pregunta:

PREGUNTA 29
0.2 puntos Guardado

EL FACTOR DE CRECIMIENTO PLACENTARIO PERTENECE A LA FAMILIA


DE:
A. FCE

B. FCH

C. FCVE

D. FCFD

PREGUNTA 30 0.2 puntos Guardado

LAS TURBULENCIAS PRODUCEN:


A. ATEROESCLEROSIS

B. TROMBOS ARTERIALES

C. TROMBOSIS VENOSA PROFUNDA

D. EDEMA

PREGUNTA 31 0.2 puntos Guardado

QUE ENTIDAD SE REFIERE A UNA FORMA DE


COAGULOPATIA DE CONSUMO CARATERIZADA POR
TROMBOSIS MICROANGIOPATICA DE CÍRCULO VICIOSO
TROMBOSIS Y HEMORRAGIA :
A. EMBOLIA DE LÍQUIDO
B. COAGULACION INTRAVASCULAR DISEMINADA
C. TROMBOS MURALES
D. EMBOLIA TUMORAL

PREGUNTA 32 0.2 puntos Guardado

LA TRIADA DE VIRCHOW INCLUYE LAS SIGUIENTES CARACTERÍSTICAS,


EXCEPTO:
https://buap.blackboard.com/webapps/assessment/take/launch.jsp?course_assessment_id=_145404_1&course_id=_63284_1&content_id=_266142… 9/15
27/11/2020 Realizar evaluación: 2o PARCIAL DEPARTAMENTAL – IDMD 200...

A. HIPERCOAGULABILIDAD
Estado de nalización de la pregunta:
B.
INCREMENTO DE LA VOLEMIA
C. LESIÓN ENDOTELIAL

D. ALTERACIONES EN EL FLUJO SANGUÍNEO

PREGUNTA 33 0.2 puntos Guardado

¿CUAL ES LA DEFINICIÓN DE TERATOMA?


A. QUE SE ORIGINE DEL TROFOBLASTO
B. ORIGEN EN LINFOCITOS B
C. QUE SE ORIGINE DE LAS CÉLULAS DE SERTOLI
D. QUE SE ORIGINE DE MÁS DE 2 CAPAS
GERMINALES

PREGUNTA 34 0.2 puntos Guardado

ENFERMEDAD GRANULOMATOSA NO NECROSANTE:


A. SARCOIDOSIS

B. SIFILIS

C. TUBERCULOSIS

D. NINGUNA DE LAS ANTERIORES

PREGUNTA 35 0.2 puntos Guardado

LAS CÉLULAS QUE CONTIENEN LÍPIDOS EN LOS


ATEROMAS SON:
A. FIBROBLASTOS

B. MACRÓFAGOS

C. CÉLULAS ENDOTELIALES
D. CÉLULAS DE MÚSCULO LISO

PREGUNTA 36 0 2 puntos Guardado


https://buap.blackboard.com/webapps/assessment/take/launch.jsp?course_assessment_id=_145404_1&course_id=_63284_1&content_id=_26614… 10/15
27/11/2020 Realizar evaluación: 2o PARCIAL DEPARTAMENTAL – IDMD 200...
PREGUNTA 36 0.2 puntos Guardado

Estado de nalización de la pregunta:

ES UN CAMBIO CROMOSOMICO QUE SE CARACTERIZA POR


HIPERMETILACION DEL ADN:
A. CAMBIO EPIGENETICO

B. DELECION

C. TRASLOCACION

D. AMPLIFICACION GENICa

PREGUNTA 37 0.2 puntos Guardado

CLÍNICAMENTE, EL PROCESO DE FIBROSIS ES COMÚN EN:


A. TEJIDOS LÁBILES

B. MUCOSAS

C. ÓRGANOS PARENQUIMATOSOS

D. EPITELIOS

PREGUNTA 38 0.2 puntos Guardado

NEOPLASIA BENIGNA DEL NERVIO PERIFERICO


A. NERUROMA

B. NEURILEMMOMA

C. NEUROFIBROMA

D. SCHWANOMA

PREGUNTA 39 0.2 puntos Guardado

LA LESION SUBLETAL POR RADIACIONES ULTRAVIOLETA CAUSA:


A. CA DE PULMON

B. CA DE CELULAS BASALES

C. CA DE TIROIDES

D. CA RENAL

https://buap.blackboard.com/webapps/assessment/take/launch.jsp?course_assessment_id=_145404_1&course_id=_63284_1&content_id=_26614… 11/15
27/11/2020 Realizar evaluación: 2o PARCIAL DEPARTAMENTAL – IDMD 200...

Estado de nalización de la pregunta:

PREGUNTA 40
0.2 puntos Guardado

SUSTANCIAS QUE INHIBEN LA FUNCION PLAQUETARIA


A. ON Y PROSTACICLINA Y ADPASA

B. ANTITROMBINA

C. PLASMINOGENO

D. TROMBOPLASTINA

PREGUNTA 41 0.2 puntos Guardado

EN ESTA FASE DEL CHOQUE SE PRESENTA INSUFICIENCIA RENAL POR


NECROSIS TUBULAR AGUDA QUE LA SUPERVIVENCIA ES IMPOSIBLE.
A. FASE IRREVERSIBLE

B. FASE NO PROGRESIVA

C. FASE PROGRESIVA

D. FASE DE HIPOPERFUSIÓN TISULAR

PREGUNTA 42 0.2 puntos Guardado

SON EJEMPLOS CLÍNICOS DE CHOQUE ASOCIADO A INFLAMACIÓN


SISTÉMICA:
A. HEMORRAGIA Y VÓMITOS.

B. INFARTO AL MIOCARDIO Y ROTURA VENTRICULAR.

C. TRAUMATISMOS Y QUEMADURAS.

D. INFECCIONES MICROBIANAS DEVASTADORAS Y PANCREATITIS.

PREGUNTA 43 0.2 puntos Guardado

DILATACIÓN DE LA PARED DE LA AORTA QUE PRODUCE ESTASIS LOCAL


SUSCEPTIBLE A TROMBOSIS:
A. ÉMBOLO

B. TROMBO AURICULAR

C
https://buap.blackboard.com/webapps/assessment/take/launch.jsp?course_assessment_id=_145404_1&course_id=_63284_1&content_id=_26614… 12/15
27/11/2020 Realizar evaluación: 2o PARCIAL DEPARTAMENTAL – IDMD 200...
C. ANEURISMA
Estado de nalización de la pregunta:
D. TROMBO MURAL

PREGUNTA 44 0.2 puntos Guardado

ES EL COLAGENO QUE PROPORCIONA FIBRILLAS DE ANCLAJE DE LAS


CELULAS EPITELIALES A LA MEMBRANA BASAL
A. COLAGENO TIPO I

B. COLAGENO TIPO III

C. COLAGENO TIPO IV

D. COLAGENO TIPO VII

PREGUNTA 45 0.2 puntos Guardado

ES UNA PROTEINA DE ADHESION CELULAR GRANDE QUE LIGA MUCHAS


MOLECULAS DE LA MEC:
A. CADHERINA

B. FIBRONECTINA

C. OSTEONECTINA

D. TROMBOSPONDINA

PREGUNTA 46 0.2 puntos Guardado

SON FASES DEL SHOCK HIPOVOLÉMICO Y/O CARDIÓGENO, EXCEPTO:


A. FASE DE REGENERACIÓN

B. FASE NO PROGRESIVA

C. FASE IRREVERSIBLE

D. FASE PROGRESIVA

PREGUNTA 47 0.2 puntos Guardado

É
https://buap.blackboard.com/webapps/assessment/take/launch.jsp?course_assessment_id=_145404_1&course_id=_63284_1&content_id=_26614… 13/15
27/11/2020 Realizar evaluación: 2o PARCIAL DEPARTAMENTAL – IDMD 200...
UNA VEZ QUE LAS CÉLULAS TUMORALES ROMPEN LA MEMBRANA
BASAL,
Estado de SE HABLA DE:
nalización de la pregunta:
A.
NEOPLASIA PREINVASIVA.
B. METAPLASIA

C. CARCINOMA IN SITU.

D. TUMOR INFILTRANTE.

PREGUNTA 48 0.2 puntos Guardado

EL RESTABLECIMIENTO DE LA ESTRUCTURA TISULAR NORMAL SOLO SE


PRODUCE:
A. CUANDO EL TEJIDO RESIDUAL ESTÁ ESTRUCTURALMENTE INTACTO
(RESECCIÓN QUIRÚRGICA PARCIAL).
B. EN NEFRECTOMÍA UNILATERAL

C. CUANDO TODO EL TEJIDO ESTÁ DAÑADO POR INFLAMACIÓN.

D. CUANDO TODO EL TEJIDO ESTÁ DAÑADO POR INFECCIÓN.

PREGUNTA 49 0.2 puntos Guardado

ES LA FASE DEL SHOCK EN LA QUE SE ACTIVA EL EJE RENINA-


ANGIOTENSINA
A. FASE REFRACTARIA

B. FASE NO PROGRESIVA

C. FASE IRREVERSIBLE

D. FASE PROGRESIVA

PREGUNTA 50 0.2 puntos Guardado

ES EL COMPONENTE DE LA MEC QUE RETIENE AGUA Y DA RESISTENCIA A


LA COMPRESIÓN:
A. ÁCIDO HIALURÓNICO

B. LAMININA

C. FIBRONECTINA

D. COLÁGENO
https://buap.blackboard.com/webapps/assessment/take/launch.jsp?course_assessment_id=_145404_1&course_id=_63284_1&content_id=_26614… 14/15
27/11/2020 Realizar evaluación: 2o PARCIAL DEPARTAMENTAL – IDMD 200...
COLÁGENO
Estado de nalización de la pregunta:

Haga clic en Guardar y enviar para guardar y enviar. Haga clic en Guardar todas las respuestas para
guardar todas las respuestas.

Guardar todas las respuestas Cerrar ventana Guardar y enviar

https://buap.blackboard.com/webapps/assessment/take/launch.jsp?course_assessment_id=_145404_1&course_id=_63284_1&content_id=_26614… 15/15
1. Causa predisponente de la calcificación metastásica
R: Medio alcalino

2. La biopsia de elección para una lesión hiperpigmentada localizada en la piel y que mide un
centímetro de diámetro es:
R: Excisional

3. Quimiocina, linfotactina para:

4. Es causa de la calcificación distrófica.


R: Focos de necrosis

5. Características de necrosis fibrinoide


R: Depósitos de inmunocomplejos en pared de vasos sanguíneos

6. 2:55 histamina, prostaglandina


7. ¿Cómo la telomerasa inhibe el acortamiendto de los telómeros?
8.
9. Son sensores de lesión o estrés celular en la apoptosis. R: Puma y Bid
10.
11. La vía extrínseca de la apoptosis se activa por: caspasa 8
12. De las causas de lesión celular cuál es la que afecta de manera directa a la fosforilación
oxidativa aeróbica. R: Isquemia o hipoxia.
13. La telomerasa es un complejo. R: RNA proteína
14. En la lesión celular reversible debida a isquemia/hipoxia la primera activación bioquímica
se presenta en: disminución de la fosforilación
15. Fenómenos que caracterizan que la lesión celular es irreversible.
16. Proteínas de transporte y almacenamiento con capacidad antioxidante: Lactoferrina,
transferrina, ferritina.
17. La calcificación distrófica se presenta en: ateroesclerosis
18. Metaplasia
19. Efectos locales provocados por IL-1, FNT e IL-6 en la reacción inflamatoria: aumento de la
adhesión y activación leucocitaria.
20. Moléculas antiapoptóticas: BCL2, BCL-XL
21. Son características morfológicas de la apoptosis excepto: Fallo de funciones dependientes
de energía
22. Tipo de metaplasia en el esófago de Barret: de epitelio plano a epitelio cilíndrico.
23. La acumulación intracelular de polvo de carbón produce reacción fibroblástica recibe el
nombre: neumoconiosis
24. Pigmento del envejecimiento: Lipocromo o Lipofuscina
25. Componentes de la inflamación aguda: Modificación en el calibre de los vasos y en la
estructura de la microvasculatura.
26. El aumento de la permeabilidad vascular da como resultado:
27. Selectinas, integrinas y glicoproteínas actúan como: moléculas de adhesión.
28. La aspirina inhibe la PGE2 para disminuir la fiebre.
29. Peroxidación de los lípidos de la membrana, lesiones del DNA son causados por: Radicales
libres derivados de oxígeno.
30. IgG e C3b funcionan como: opsoninas
31. Los principales quimiotácticos endógenos: C5a, C3a
32. Tipo de necrosis en encéfalo: necrosis licuefactiva.
33. La fractalquina es una molécula de adhesión para: monocitos y Linfocitos T
34. Los cuerpos de Russell son acúmulos de: Inmunoglobulinas en las células plasmáticas.
35. Productos del metabolismo del ácido araquidónico por la vía de la ciclooxigenasa:
Prostaglandinas y protaciclina, Txa2
36. Grupo de mediadores químicos que producen inhibición de la quimiotaxis y de la
adherencia de neutrófilos: Lipoxinas
37. Calicreina
38. Enfermedad de Crohn
39. Quimiocina que da lugar activación y quimiotaxis de los neutrófilos, con activación
limitada sobre monocitos y eosinófilos:
40. Prostaglandina que produce hiperalgesia, que hace que la piel presente hipersensibilidad,
dolor y fiebre: PGE. (Revísala)
41. Proteína que puede unirse a la proteína BCL2, BCL-XL y realiza funciones antiapoptóticas:
BH3
42. Dependiente de calcio
43. En inflamación son pirógenos endógenos: IL-1 y FNT
44. Causas más frecuentes de esteatosis hepática: alcoholismo y desnutrición.
45. Ejemplos de inflamación serosa: infecciones virales y quemaduras de segundo grado.
46. 10 veces más potente que la histamina: FAP
47. En la apoptosis ambas moléculas se unen a la membrana de la mitocondria, hacen poros y
liberan citocromo c: BAX y BAK
48. Factor de crecimiento relacionado con la hipertrófia relacionado con el aumento de la
carga de trabajo: TGF-B
49. La atrofia se produce por:
50. El receptor de muerte más conocido en la apoptosis se llama: FAS
51. Proteínas plasmáticas de fase aguda de la inflamación: Proteína C reactiva, fibrinógeno,
52. Inhibe el apetito: FNT

También podría gustarte